Está en la página 1de 259

National Licence

2008

Thx Everyones
1. ผู้ป่วยหญิงไทยอายุ 32 ปี ปวดข้อเป็นๆหายๆ และมีผื่นขึ้นที่หน้า ผลการตรวจห้องปฏิบัติการณ์พบ urine protein 1+, red blood cell
cast การส่งตรวจเพิ่มเติมในข้อใดต่อไปนี้ specific สาหรับการวินิจฉัยในผู้ป่วยรายนี้มากที่สุด
a. Rheumatoid factor
b. ANA
c. Anti- histone antibody
d. Anti ds-DNA antibody
ตอบ ในผู้ป่วยรายนี้นึกถึงว่าเป็น SLE มีเกณฑ์ในการวินิจฉัยที่เรียก ARA criteria เพื่อใช้เป็นเกณฑ์ในการวินิจฉัยโรคได้อย่างรวดเร็วและ
ถูกต้องแม่นยายิ่งขึ้นดังนี้ คือ
1. ผื่นที่หน้ารูปปีกผีเสื้อ บริเวณโหนกแก้มสองข้างจมูก (malar rash) โดยเว้นร่องระหว่างจมูกกับริมฝีปาก (nasolabial fold)
2. ผื่น discoid ลักษณะเดียวกับที่พบใน discoid lupus
3. อาการแพ้แดด (photosensitivity)
4. แผลในปาก (oral ulcer)
5. ข้ออักเสบชนิดที่ไม่มีกัดกร่อนของข้อ (nonerosive arthritis)
6. เยื่อหุ้มปอดหรือเยื่อหุ้มหัวใจอักเสบ (serositis)
7. ความผิดปกติทางไต ได้แก่ การมีโปรตีนรั่วออกมาในปัสสาวะมากกว่า 0.5 กรัมต่อวัน หรือเกิน 3+ โดยการตรวจคร่าว ๆ
8. ความผิดปกติทางระบบประสาทซึ่งหาสาเหตุไม่ได้ โดยอาจเป็นอาการชักหรืออาการโรคจิต ( psychosis) ก็ได้
9. ความผิดปกติของระบบโลหิตซึ่งอาจเป็นภาวะซีดจากเม็ดเลือดแดงแตก หรือจานวนเม็ดเลือดขาวต่ากว่า 4,000/ลบ.มม. (ตรวจพบ
2 ครั้งขึ้นไป) หรือจานวนเม็ดเลือดขาวชนิดลิมบ์โฟซัยท์น้อยกว่า 1,500/ลบ.มม. (ตรวจพบ 2 ครั้งขึ้นไป) หรือจานวนเกร็ดเลือดน้อยกว่า
100,000/ลบ.มม. โดยไม่ได้เป็นผลจากยา
10. ความผิดปกติทางภูมิคุ้มกันจากการตรวจเลือดซึ่งได้แก่ การตรวจพบ anti double stranded DNA หรือ anti-Sm antibodies หรือ
ตรวจพบ antiphospholipid antibodies หรือพบผลบวกปลอมของการตรวจทางซีโรโลยี่สาหรับโรคซิฟิลิส
11. การตรวจเลือดเกี่ยวกับโรคแพ้ภูมิพบ antinuclear antibodies (ANA) ให้ผลบวกโดยไม่ได้เป็นผลจากยา
ถ้ามีความผิดปกติตามเกณฑ์ตั้งแต่ 4 ข้อขึ้นไป ผู้ป่วยนั้นจะมีโอกาสเป็นโรคลูปัสถึงร้อยละ 96
ซึ่ง lab ที่จะช่วยในการวินิจฉัยได้ดีที่สุดคือ Anti ds-DNA antibody เนื่องจากมีความจาเพาะต่อโรคนี้ค่อนข้างมาก พบค่า titerที่สูงได้ถึง 60-
70% ในผู้ป่วย SLE ในขณะที่โรคautoimmune อื่นหรือคนปกติจะพบได้น้อยหรือมีค่า titerที่น้อยกว่ามาก
Antihistone antibodies (AHAs) พบว่ามีความเกี่ยวข้องกับ drug-induced lupus erythematosus และอาจพบได้ในโรค
autoimmune อื่นๆ เช่น SLE unrelated to medications, rheumatoid arthritis, juvenile rheumatoid arthritis, primary biliary cirrhosis,
autoimmune hepatitis, dermatomyositis/polymyositis และscleroderma
Rheumatoid factor เป็น immunoglobulin (antibody) ที่ใช้ในการวินิจฉัย rheumatoid arthritis พบได้บ้างในjuvenile
rheumatoid arthritis โรค autoimmune อื่นๆ ที่พบว่ามี rheumatoid factor เป็น positive เช่น
Sjogren's syndrome, Systemic lupus erythematosus, Scleroderma, Polymyositis, Dermatomyositis, Mixed Connective Tissue
Disease รวมถึงโรคติดเชื้อ เช่น Bacterial endocarditis, Osteomyelitis
Anti-nuclear antibodies (ANAs) จะพบว่ามีค่าสูงขึ้นในโรค autoimmune
ค่า titer ปกติของ ANA คือ 1:40 หากค่าสูงกว่านี้จะนึกถึงว่าเป็นโรค autoimmune โดยที่นึกถึงมากคือ lupus erythematosus (พบ 80-90%
ของผู้ป่วยSLE) นอกจากนี้ยังพบใน Sjögren's syndrome (60%), rheumatoid arthritis, autoimmune hepatitis, scleroderma and
polymyositis & dermatomyositis (30%),
ข้อบ่งชี้ในการส่งตรวจ (indication) :
1.) Systemic lupus erythematosus
2.) Connective tissue disease อื่น ๆ
3.) ตรวจคัดกรองเบื้องต้น ก่อนพิจารณาส่งตรวจ specific antibody

2. ผู้ป่วยหญิงอายุ 35 ปี มีไข้ต่าๆ ปวดข้อเข่าและข้อมือมา 4 วันก่อนมาโรงพยาบาล มาพบแพทย์เจาะน้าข้อเข่าตรวจพบ WBC = 89000,


PMN = 90%, G/S: gram negative intracellular diplococci การรักษาข้อใดต่อไปนี้เหมาะสมที่สุด
a. Cloxacillin
b. Gentamicin
c. Ceftriazone
d. Doxycycline
e. Metronidazole
ตอบ ผู้ป่วยรายนี้นึกถึงว่าเป็น gonococcal arthritis
Gonococcal arthritis เป็นการติดเชื้อแบคทีเรียในข้อ โดยพบในผู้ป่วยที่เป็น gonorrhea พบในเพศหญิงมากกว่าเพศชาย และมักพบในช่วง
วัยรุ่น
Two forms of gonococcal arthritis exist:
 One involves skin rashes and multiple joints, usually large joints such as the knee, wrist, and ankle
 The second, less common form involves disseminated gonococcemia, which leads to infection of a single joint
อาการ
 Fever
 Lower abdominal pain
 Migrating joint pain for 1 to 4 days
 Pain in the hands or wrists due to tendon inflammation
 Pain or burning on urination
 Single joint pain
 Skin rash (lesions are flat, pink to red, may later contain pus or appear purple)
Exams and Tests
ในกรณีที่สงสัย gonococcal arthritis ควรส่ง hemoculture ทุกราย เพื่อหา gonorrhea infection
นอกจากนี้ยังหาการติดเชื้อได้จาก
 Cervical gram stain
 Culture of joint aspirate
 Joint fluid gram stain
 Throat culture
Treatment
การรักษาเหมือนกับ Gonorrhea โดยแต่ก่อนจะให้เป็น Penicillin แต่ปัจจุบันนี้ไม่ใช้แล้วเนื่องจากมีการดื้อยามากขึ้น
ปัจจุบันนี้เลยนิยมใช้ยากลุ่ม cephalosporins เช่น Ceftriaxone (Rocephin) ส่วน Azithromycin (Zithromax) ก็อาจให้ได้เช่นกันในกรณี
uncomplicated infections.
การ follow-up จะดูที่ 7 วันหลังได้รับการรักษา
3. ผู้ป่วยชายวัยกลางคน ชักแบบ Generalized Tonic clonic ปฏิเสธประวัติอุบัติเหตุหรือการใช้สารเสพย์ติด ผู้ป่วยไม่เคยมีอาการชักแบบนี้
มาก่อน ไม่มีไข้ จากการตรวจร่างกายเบื้องต้นไม่พบ neurological deficitใดๆ แต่พบว่ามี papilledema ผล CT scan : multiple calcified
cystic lesion เชื้อก่อโรคในข้อใดต่อไปนี้น่าจะทาให้เกิดอาการในชายผู้นี้มากที่สุด
a. Taenia solium
b. Taenia saginata
c. Toxocara
d. Toxoplasma
ตอบ จากอาการและผล CT ทาให้นึกถึง cysticercosis จาก Taenia solium หรือพยาธิตืดหมู ซึ่งอันตรายจาก cysticercosis ขึ้นกับอวัยวะ
ที่ติดเชื้อและจานวนตัวอ่อน,หากมีจานวนน้อยผู้ป่วยจะไม่แสดงอาการ แต่ปกติจะมีจานวนมาก อาจถึงหลายพันตัว ตัวอ่อนสามารถเจริญใน
เนื้อเยื่อและอวัยวะทุกชนิด พบบ่อยในกล้ามเนื้อลาย , สมอง รองลงมาคือ subcutaneous tissue ,ตา ,หัวใจ , ปอดและเยื่อบุช่อง
ท้อง ในขณะที่ตัวอ่อนไชไปตามส่วนต่างๆของร่างกาย จะทาให้ผู้ป่วยปวดตามกล้ามเนื้อและมีไข้ เนื้อเยื่อที่อยู่รอบๆตัวอ่อน จะเสื่อมและฝ่อ
ตาย ตัวอ่อนจะถูกล้อมรอบด้วย fibrous tissue และในที่สุดจะมีหินปูนมาห่อหุ้ม อาการที่รุนแรงจะเกิดเมื่อ cerebral cysticercosis คือตัว
อ่อนจะเจริญอยู่ในส่วนต่างๆ ของสมอง ทาให้เกิดอาการบวมน้า และมีความดันสูง ปกติอาการที่แน่ชัด ยังไม่แสดงภายใน 5 -8 ปี หรืออาจ
นานถึง 20 ปี นอกจากว่าตัวอ่อนจะตาย และกระตุ้น ให้เกิดการอักเสบอย่างรุนแรงขึ้น , อาการระยะต้นๆ อาจมีสาเหตุจากการกดดันของ cyst
และการอุดตันของน้าไขสันหลัง ถ้าตัวอ่อนอยู่ในสมองส่วนที่ควบคุมการทางานของกล้ามเนื้อต่างๆ อาการที่เด่นชัดที่สุดคือชักแบบลมบ้าหมู ,
อาการอื่นๆได้แก่ ปวดศีรษะอย่างกะทันหัน ,อาเจียนมีอาการทางจิต ,กระสับกระส่าย กังวลใจ และไม่มีสมาธิ เป็นต้น ที่ตามักจะมีตัวอ่อน
เพียงตัวเดียวและสามารถทาให้ นัยน์ตาอักเสบ ปวดและมองไม่ชัด การติเชื้อที่กล้ามเนื้อหัวใจ ทาให้หายใจลาบาก
ส่วน Taenia saginata หรือพยาธิตืดวัวนั้น ก็มีอาการคล้ายกับพยาธิตืดหมู คือมีอาการเบื่ออาหาร ปวดท้อง แน่นท้อง อาจมีถ่าย
ท้อง ถ้าบางส่วนของปล้องเข้าไปในช่องว่างของไส้ติ่งทาให้เกิด appendicitisได้ และถ้าพันกันในลาไส้ ทาให้เกิด intestinal obstruction แต่
ไม่พบว่าเกิด cysticercus bovis ในกล้ามเนื้อหรืออวัยวะต่างๆ ในคน
Toxocara spp. ทาให้เกิดการติดเชื้อเรียกว่า Toxocariasis ซึ่ง Toxocara เป็นพยาธิตัวกลมที่มักพบในลาไส้ของสุนัข (Toxocara
canis)และแมว(T. cati)
รูปแบบของ toxocariasis แบ่งได้เป็น 2 อย่างคือ
1) Ocular larva migrans (OLM):
สามารถทาให้ตาบอดได้ โดยเมื่อตัวพยาธิเข้าไปในตา จะทาให้เกิดการอักเสบและเกิด scarที่ retina เกิด permanent partial loss of vision
ได้
2) Visceral larva migrans (VLM):
Toxocara infections อย่างรุนแรงหรือเกิดขึ้นซ้า ทาให้เกิด VLM ได้แต่พบได้น้อยมาก การติดเชื้อในรูปแบบนี้ทาให้อวัยวะต่างๆ รวมถึง CNS
บวม อาการของVLM เกิดขึ้นจากการเคลื่อนไหวของพยาธิไปยังส่วนต่างๆของ ร่างกาย เกิดอาการเช่น fever, coughing, asthma,หรือ
pneumonia
Toxoplasma ทาให้เกิด toxoplasmosis
เชื้อ Toxoplasmosis gondii เป็นปาราสิตชนิดหนึ่งซึ่งเกิดในแมวและสัตว์เลือดอุ่นอื่นๆ เชื้อนี้จะเจริญในผนังลาไส้ของลาไส้เล็ก ไข่จะออกมา
กับอุจาระ คนได้เชื้อนี้อย่างไรไม่ทราบ ทางเข้ามายังคนอาจจะเข้าได้โดยทางหายใจจากคนที่มีเชื้อหรือจากการกินอาหารที่ไม่สุก ผู้ป่วย HIV
มักจะติดเชื้อชนิดนี้โดยเฉพาะที่สมอง
สามารถแบ่งอาการออกตามกลุ่มเสี่ยงได้เป็น 3 กลุ่ม
1.คนปกติ
- อาการไม่รุนแรง อาจพบเพียงอาการคล้ายหวัด หรือปวดตามกล้ามเนื้อ 2 - 3 วันจนถึง 2 - 3 สัปดาห์อาการเหล่านี้จะหายไป ซึ่งปกติ
แล้วคนส่วนใหญ่จะมีอาการแบบไม่รุนแรง
- อาการรุนแรง อาจพบ อาการปวดศรีษะอย่างรุนแรงซึ่งไม่ตอบสนองต่อยาแก้ปวด ร่างกายด้านหนึ่งอ่อนแรงชัก มีปัญหาด้านการ
มองเห็น การพูดและการเดิน
2.หญิงตั้งครรภ์
- การติดเชื้อในช่วงแรกของการตั้งครรภ์( 3 เดือนแรก ) มีโอกาสประมาณ 45% ที่จะติดต่อสู่ทารก ทั้งนี้พบว่าทารกที่ติดเชื้อ 60%ไม่
พบอาการผิดปกติ 10% จะเสียชีวิต อีก30%จะมีอาการรุนแรงเช่น สมองบวมน้า ( hydrocephalus) มีการสะสมของแคลเซียมในสมอง (
intracerebral calcification ) การอักเสบของจอตา และประสาทตา อารมณ์ผิดปกติ
- การติดเชื้อก่อนตั้งครรภ์หรือเลยช่วง 3 เดือนแรกไปแล้วมักไม่พบการติดต่อสู่ทารกในครรภ์
3.ผู้ที่มีภาวะภูมิคุ้มกันต่่า
ในผู้ป่วยโรคเอดส์ ผู้ที่เปลี่ยนอวัยวะ ผู้ป่วยที่รักษาด้วยเคมีบาบัด จะพบอาการรุนแรงกว่าคนปกติ มักมี อาการเจ็บป่วยแบบเรื้อรัง
การ วินิจฉัย ควรตรวจหาภูมิคุ้มกันต่อเชื้อ TOXO หากผลภูมิให้ผลบวกและการตรวจ MRI scan มีรอยโรคที่สมองก็ให้การวินิจฉัยได้
อาจจะต้องเพาะเชื้อจากน้าไขสันหลัง
ถึงแม้จะทาให้ชักได้เหมือนกันทาให้เกิด lesion เป็น calcification เหมือนกันแต่ไม่เป็น cystic นะ

4. ผู้ป่วยชายอายุ 55 ปีไอแห้งๆ บางครั้งมีเลือดปนออกมาด้วย น้าหนักตัวลดลง 4 กิโลกรัมใน 1 เดือน ผลการตรวจ AFB ได้ผล negative 3 ครั้งติดต่อกัน ผล
CXR พบว่ามี reticulonodular infiltration with thickening cavity การปฏิบัติในข้อใดต่อไปนี้น่าจะเหมาะสมกับผู้ป่วยรายนี้มากที่สุด
a. รอผล culture for TB
b. Start anti-TB drug
c. Bronchoscopy
d. Aspiration
e. PCR for TB
ตอบ c. Bronchoscopy
จากโจทย์ พบว่า เป็น Case an old man with chronic non-productive cough,
อาการเหล่านี้ทาให้นึกถึงโรคในระบบหายใจ และเมื่อ approach จาก non productive cough ร่วมกับอาการร่วม คือ hemoptysis, weight loss และผล
CXR พบว่ามี reticulonodular infiltration with thickening cavity ทาให้นึกถึง Pulmonary TB มากที่สุด แต่จากโจทย์ ได้ส่ง Sputum AFB ได้ผล Negative 3
ครั้งติดต่อกัน เมื่อพิจารณาตามเกณฑ์การวินิจฉัย ผู้ป่วยรายนี้อยู่ในระดับ Probable คือ ภาพรังสีทรวงอกพบลักษณะเข้าได้กับวัณโรคปอด แต่ถ้าระดับ definite
จะต้องตรวจพบเชื้อวัณโรคจากตัวผู้ป่วย ซึ่งจากการตรวจเสมหะยังไม่พบเชื้อ ดังนั้น เพื่อให้ได้ definite diagnosis จึงควรทาการส่งตรวจเพิ่มเติมเพื่อหาเชื้อให้ได้
คือ การทา Bronchoscopy และในผู้ป่วยรายนี้ ยังไม่สามารถ R/O Lung cancer ออกไปได้ เพราะจากประวัติ ผู้ป่วยมีอายุค่อนข้างมาก อาการก็สามารถเข้าได้
และผล chest X-ray ก็เข้าได้ คือ thickening cavity ซึ่งพบได้ใน necrotizing lung cancer
แต่ในทางปฏิบัติ ถ้าไม่สามารถส่งทา Bronchoscopy ได้ ก็สามารถให้การรักษาแบบผู้ป่วย Smear Negative ได้เลย เพราะผู้ป่วยรายนี้ ย้อมเสมหะไม่พบ
เชื้อ มีอาการและภาพรังสีทรวงอกพบลักษณะเข้าได้กับวัณโรคปอด
c(‘O’)nt ‘010 Med X
5. ผู้ป่วยชายอายุ 18 ปี ไปเยี่ยมสถานรับเลี้ยงเด็กมาเมื่อ 2 สัปดาห์ก่อนมาโรงพยาบาล ต่อมามีอาการอ่อนแรงของแขนขา เป็นมากขึ้นเรื่อยๆ แพทย์ทาการ
ตรวจร่างกายพบว่ามี Upper motor power 4/5, lower 1/5, absent DTR ข้อใดคือการวินิจฉัยในผู้ป่วยรายนี้ที่เหมาะสมที่สุด
a. Transverse myelitis
b. GBS
c. MG
d. Neurosyphilis
ตอบ b.GBS
จากโจทย์ ผู้ป่วยมี muscle weakness แบบ Lower motor neuron
เนื่องจากมีอาการเข้าได้ ดังตาราง 
และจากประวัติ ผู้ป่วยมีอาการค่อนข้างเร็ว (ไม่รู้เวลาที่แน่นอน แต่ไม่ถึง 2
สัปดาห์) จึงทาการวินิจฉัยแยกโรคของ Acute neuromuscular weakness
(LMN.) โดยโรคกลุ่มนี้พบผู้ป่วยได้บ่อย มักมาด้วยอาการของ LMN ในเวลา
อันรวดเร็วเป็นวันหรือสัปดาห์ โดยโรคที่พบได้บ่อยที่สุด คือ Guillain Barre’
syndrome และ Myasthenia gravis โดยสามารถแบ่งสาเหตุตามกลุ่มโรคได้ดังนี้

Diagnostic Criteria for Guillain-Barré Syndrome


 REQUIRED
จากประวัติและการตรวจร่างกายพบว่า ผู้ป่วยมีการอ่อนแรงแบบ 1. Progressive weakness of 2 or more limbs due to neuropathya
2. Areflexia
Ascending paralysis คือ อ่อนแรงขามากกว่าแขน( rubbery legs)และ 3. Disease course <4 weeks
Exclusion of other causes [e.g., vasculitis (polyarteritis
เป็นมากขึ้นเรื่อยๆ และมี absent deep tendon reflex ซึ่งเป็นอาการแสดง 4. nodosa, systemic lupus erythematosus, Churg-Strauss
ของ GBS และอาการอ่อนแรงนี้จะเกิดขึ้นเร็ว(over hours to a few days) syndrome), toxins (organophosphates, lead), botulism,
diphtheria, porphyria, localized spinal cord or cauda equina
นอกจากนี้มักเกิดตามหลัง Acute infectios process ประมาณ 1-3 สัปดาห์ syndrome]
และมักเป็นการติดเชื้อในระบบ respiratory & gastrointestinal ซึ่งในผู้ป่วย  SUPPORTIVE
1. Relatively symmetric weakness
รายนี้ก็มีประวัติไปเยี่ยมสถานรับเลี้ยงเด็กมาเมื่อ 2 สัปดาห์ก่อน จึงทาให้มี 2. Mild sensory involvement
Facial nerve or other cranial nerve involvement
โอกาสรับเชื้อได้ ดังนั้น จึงทาให้นึกถึงภาวะนี้มากที่สุด 3.
4. Absence of fever
ส่วน Transverse myelitis นั้น เกิดจาก inflammation of the 5. Typical CSF profile (acellular, increase in protein level)
6. Electrophysiologic evidence of demyelination
spinal cord โดยมีอาการและอาการแสดง คือ lower back pain,
weakness in the legs and arms, sensory disturbance, spasms leading to gradual paralysis, and bowel or bladder dysfunction. โดยอาการเหล่านี้จะพบ
ตามระดับของ lesion ที่ spinal cord
Myasthenia gravis (MG) : relatively rare autoimmune disorder of peripheral nerves in which antibodies form against acetylcholine (ACh)
nicotinic postsynaptic receptors at the myoneural junction. A reduction in the number of ACh receptors results in a characteristic pattern of
progressively reduced muscle strength with repeated use of the muscle and recovery of muscle strength following a period of rest.
Neurosyphilis : Symptoms : Personality change (including cognitive and/or behavioral impairment), Ataxia, Stroke, Ophthalmic
symptoms (eg, blurred vision, reduced color perception, impaired acuity, visual dimming, photophobia),Urinary symptoms (eg, bladder
incontinence),Lightning pains (larynx, abdomen, various organs), Headache, Dizziness, Hearing loss and Seizures

Signs of neurosyphilis : Hyporeflexia, Sensory impairment (eg, decreased proprioception, loss of vibratory sense), Pupillary changes
(anisocoria, Argyll Robertson pupils),Cranial neuropathy, Dementia, mania, or paranoia, Romberg sign, Charcot joint, Hypotonia and Optic atrophy
……. c(‘O’)nt ‘010 Med X

6. ผู้ชายอายุ 30 ปี เหนื่อยมา 1 เดือน แพทย์ทาการตรวจร่างกายพบว่ามี neck vein


engorgement, Hepatomegaly ฟังปอดปกติ ผลจากการตรวจ EKG พบว่ามี diffuse
ST-T change, low voltage ข้อใดคือการวินิจฉัยในผู้ป่วยรายนี้ที่เหมาะสมที่สุด
a. Restrictive pericarditis
b. Beri beri
c. Cardiac tamponade
d. Dilated cardiomyopathy
ตอบ c. Cardiac tamponade
(ตัวเลือกผิด ข้อ a. อาจเป็นระหว่าง Constrictive pericarditis กับ Restrictive
cardiomyopathy )
จากโจทย์ผู้ป่วยมาด้วยอาการเหนื่อยและจากการฟังปอดก็ปกติทาให้นึกถึงสาเหตุ จาก
ระบบทางเดินหายใจน้อยลง ซึ่งในผู้ป่วยรายนี้นึกถึงสาเหตุจากระบบ CVS มากที่สุด ดังนั้น จึง
สามารถวินิจฉัยแยกโรคระหว่าง Cardiac tamponade, Constrictive pericarditis และ
Restrictive cardiomyopathy ดังตารางข้างล่าง

Features That Distinguish Cardiac Tamponade from Constrictive Pericarditis and Similar Clinical Disorders
Characteristic Tamponade Constrictive Restrictive RVMI
Pericarditis Cardiomyopathy
Clinical
Pulsus paradoxus Common Usually absent Rare Rare
Jugular veins
Prominent y descent Absent Usually present Rare Rare
Prominent x descent Present Usually present Present Rare
Kussmaul's sign Absent Present Absent Absent
Third heart sound Absent Absent Rare May be present
Pericardial knock Absent Often present Absent Absent

Electrocardiogram
Low ECG voltage May be present May be present May be present Absent
Electrical alternans May be present Absent Absent Absent

Echocardiography
Thickened pericardium Absent Present Absent Absent
Pericardial calcification Absent Often present Absent Absent
Pericardial effusion Present Absent Absent Absent
RV size Usually small Usually normal Usually normal Enlarged
Myocardial thickness Normal Normal Usually increased Normal
Right atrial collapse and RVDC Present Absent Absent Absent
Increased early filling, ↑ mitral flow velocity Absent Present Present May be present
Exaggerated respiratory variation in flow velocity Present Present Absent Absent

CT/MRI
Thickened/calcific pericardium Absent Present Absent Absent

Cardiac catheterization
Equalization of diastolic pressures Usually present Usually present Usually absent Absent or present
Cardiac biopsy helpful? No No Sometimes No

Abbreviations: RV, right ventricle; RVMI, right ventricular myocardial infarction; RVDC, right ventricular diastolic collapse; ECG, electrocardiograph.
Source: From GM Brockington et al, Cardiol Clin 8:645, 1990, with permission. ( Reference : HARRISON )

 จากตารางและจากโจทย์ ก็ยังไม่สามารถแยกได้ชัดเจนนัก จึงได้ไปปรึกษาอาจารย์ พบว่า


จากอาการแล้ว typical กับ cardiac tamponade และจาก EKG ที่พบ low voltage นั้น ช่วยยืนยันมากขึ้น เพราะการที่จะมี low voltage ได้นั้น จะต้องมีอะไรมา
หัวใจเอาไว้ ซึ่งนั่นก็คือ cardiac tamponade เพราะมี pericardial effusion มาคั่นไว้ ส่วนใน Constrictive pericarditis นั้น อาจพบได้ ในกรณีที่เป็น effusive-
constrictive pericarditis เพราะจึงจะมี effusion มาคั่น (เพิ่มเติม http://emedicine.medscape.com/article/152083-overview)

Constrictive pericarditis เป็นการอักเสบเรื้อรังของ pericardium ทาให้มีการสร้างและ


สะสมของ fibrin กลายเป็น chronic fibrotic scarring และอาจมี calcification จึงทาให้อาการค่อยเป็นค่อยไป ใช้เวลานานเป็นปี อาการที่พบบ่อยที่สุดคือ dyspnea
ร่วมกับอาจพบ fatigue และ orthopnea ได้บ่อย อาการอื่นๆได้แก่ Lower-extremity edema ,abdominal swelling ,Nausea, vomiting, and right upper
quadrant pain,(จาก hepatic congestion หรือ bowel congestion) (เพิ่มเติม http://emedicine.medscape.com/article/157096-overview)

 Restrictive cardiomyopathy : gradually worsening shortness of breath, progressive


exercise intolerance, fatigue,abdominal distension, chest pain, palpitation and syncope (เพิ่มเติม http://emedicine.medscape.com/article/153062-
overview)

 Beri-Beri : Wet beri-beri จึงจะเกี่ยวกับCVS (ถ้า dry


beri-beri จะเกี่ยวกับ neuromuscular
system) โดยถ้าเป็นแบบ acute wet beri-beri  tachycardia, low diastolic pressure,
cardiomegaly, pulmonary edema, and cyanosis แต่ถ้าเป็นแบบ chronic wet beri-beri จะเป็น
อาการของ high-output cardiac failure เช่น peripheral edema and pulmonary
effusions จึงทาให้ไม่นึกถึงภาวะนี้ในผู้ป่วยรายนี้

 Dilated cardiomyopathy :
Symptoms :Swelling of the feet,ankle and
abdomen ,Pronounced neck veins ,Loss of appetite,Shortness of breath, especially with
activity , Shortness of breath which occurs after lying down for a while , Fatigue,
weakness, faintness ,Sensation of feeling the heart beat (palpitations), Pulse may feel irregular or rapid , Decreased alertness or concentration ,
Cough ,Low urine production and Need to urinate at night …………………………c(‘O’)nt ‘010 Med X
7. เด็กชายอายุ 9 ขวบ มาโรงพยาบาลด้วยไข้สูง หายใจหอบเหนื่อย ไอ แพทย์ทาการตรวจร่างกายพบว่ามี BT= 39.0 oC, PR= 120 /min, RR
= 40 /min, crepitation both lungs, decreased breath sound left lung ผู้ป่วยมีแผลตุ่มหนองและ cellulitis ที่ขาซ้าย เชื้อก่อโรคในข้อใด
ต่อไปนี้น่าจะทาให้เกิดอาการในชายผู้นี้มากที่สุดเฉลย
a. Salmonella spp.
b. S. aureus
c. H. influenzae
d. Mycoplasma pneumonia
e. Mycoplasma tuberculosis
s. aureus
ถ้าโจทย์บอกเวลาเป็น acute onsetอีกก็ยิ่งconfirmว่าเป็น s.aureus เพราะประวัติผู้ป่วยไข้สูง หอบเหนื่อย ตรวจร่างกายก็บอกว่าหอบแห่กๆ
แล้ว ...น่าจะบอกอาการมาอีกว่าเป็นในกี่วัน แถมท้ายโจทย์ยังบอกsource infection ที่skinอีกก็คือ s.aureusนี่ล่ะ (ไม่รู้ว่าตั้งใจมาใบ้หรือขุด
หลุมให้เรากระโจนลงไป) โดยในระยะ2-3วันแรกผู้ป่วยจะมีอาการแย่ลงอย่างมาก ทาให้ปอดเป็นโพรงหนอง โดยที่ film chest x-rayจะยังไม่
เปลี่ยนแปลงใน2-3วันแรก ต้องอาศัยการติดตามอาการอย่างใกล้ชิด และติดตามผล x-ray จะมีลักษณะx-ray เป็นฝ้าทั้งกลับปอด
-samonella : มาได้ยังไงเนี่ย สาหรับ clinical manifestation samonellosis มีดังนี้
- acute asymptomatic infection
+acute gastroenteritis
-bacteremia with or without metastatic focal infection
-enteric fever
-asymptomatic chronic carrier state
Choice ข้อนี้ทวนความรู้ enteric feverละกัน
-1st wk ผู้ป่วยมีอาการสาคัญ : pea soup stool , fever increase
-2nd wk fever high and sustained , complication : PUP
Physical finding ที่สาคัญในโรคนี้ bradycardia ทั้งๆที่high grade fever , hepatosplenomegaly, distended abd , rose spot ,
rhonchi and scattered rales
-H.influenza ไปเปิดจากNelson มาแล้ว บอกว่า clinicalไม่สามารถแยกได้จากother microorganismซึ่งอาจจะแสดงอาการร่วมของการติด
เชื้อหลายๆตัว
Nelson บอกว่า เด็กอายุน้อยกว่า12ปี ควรได้รับ parenteral ATBเพราะมี risk สูงที่จะเกิดbacteremia ขณะที่อายุ>12ปี ให้ oral
ATB
- Mycoplasma pneumoniae เป็นกลุ่ม atypical pneumonia ผู้ป่วยมักมีไข้ต่าๆ ไม่เกิน 38.5C ,ไอมาก, เป็นในschool age(>8yrs) ,
subacute onset (5-6วัน) , CXR more severe than clinical
การรักษาต้องใช้ยากลุ่ม macrolide ex erythromycin
- Mycoplasma tuberculosis อันนี้ไกลสุดโต่งเลย น่าจะมีประวัติ exposeต่อโรค, ไข้ต่าตอนบ่ายๆ , อ่อนเพลีย,น้าหนักลด, เหงื่อออก
ตอนกลางคืน ,ไอ
NATTAPAT MED014

8.ผู้ชายอายุ 50 ปี กินเหล้ามานาน 5 วัน มาด้วยปวดท้องลิ้นปี่ ร้าวไปหลัง นอนงอตัวดีขึ้น แพทย์ทาการตรวจร่างกายพบว่ามี tenderness


and guarding at epigastrium, liver dullness +ve ข้อใดคือการวินิจฉัยในผู้ป่วยรายนี้ที่เหมาะสมที่สุด
a. acute pancreatitis
เฉลย ประวัติข้อนี้typicalเลย Hx กินเหล้า, ปวดท้องลิ้นปี่ร้าวไปหลัง , งอตัวแล้วดีขึ้น(ไม่เหมือนPUPที่ต้องนอนนิ่ง) ตรวจร่างกาย liver
dullness positive (confirm อีกว่าไม่loss of liver dullness ) ตอบได้เลยว่า acute pancreatitis มาดูความรู้เพิ่มเติมละกัน
-clinical feature เพิ่มเติม : grey-turner’s sign เป็นรอยช้าที่สีข้าง เห็นเมื่อเป็นมาหลายวัน ถ้าเป็นรอบสะดือ เรียกว่า cullen’s sign
-investigation ในacute pancreatitis :
-serum amylase >500 somogyi unit (ปกติ 80-180) สูงอยู่ประมาณ3วัน
-urine amylase สูงอยู่ได้7-10วัน
-x-ray : colon cut off sign คือ transverse colon หายไปตรงตาแหน่งตับอ่อน
-treatment:
-NPO
-i.v. fluid
-ยาแก้ปวด ควรเลี่ยงmorphine เพราะทาให้sphincter of oddi เกร็งมาก
- Antibiotic ให้จาเพาะในรายที่มีติดเชื้อร่วม เช่น มี cholangitis ร่วม
- calcium หาก calcium ต่าจนมีอาการ

-Complication
-pseudocyst สงสัยในผู้ป่วยที่ปวดท้องหลัง2สัปดาห์และ serum amylaseยังสูง ไข้สูง WBCสูง investigation: U/S ,Upper GI
study
-Pancreatitis abscess (พบร้อยละ5) ผู้ป่วยที่อาการไม่ดีขึ้น ไข้สูง WBCสูง แต่serum amylase ไม่สูง investigation: U/S , Upper
GI study ,CT
-DDx จากโรคที่ปวดท้องรุนแรงทาให้อาการเลวลงได้รวดเร็ว : PUP ,dissecting aneurysm, rupture aneurysm ,strangulation obstruction
ในรายที่อาการไม่รุนแรง ควรDDx acute cholecystitis, cholangitis
-มาดูแต่ละโรคที่ควรรู้...ถือว่าทวนละกันสั้นๆ
-PUP : ปวดท้องกะทันหัน รุนแรง ผู้ป่วยมักจะนอนนิ่ง
P.E.: loss of liver dullness , board-like rigidity
Investigation : film acute abdomen series พบ free-air ใต้diaphragm 80%
Treatment: simple closure with omental patch or definitive: truncal vagotomy and pyroloplasty
-Acute cholecystitis : ปวดท้อง จุกแน่นใต้ชายโครงขวาและลิ้นปี่ ร้าวไปสะบักขวาหรือหลัง คลื่นไส้อาเจียน มีไข้
P.E.: murphy’s sign positive
Investigation : U/S
-Cholangitis : charcot’s triad ปวดท้อง,ตัวเหลืองตาเหลือง,ไข้หนาวสั่น นอกจากนี้ ปัสสาวะสีเข้ม คันตามตัว
Investigation : Bilirubin SGOT SGPT สูง , U/S
NATTAPAT MD014

9 มี rash เป็นแบบ vesicle ตามแนวซี่โครง มีแสบๆ เจ็บๆ ข้อใดคือการวินิจฉัยในผู้ป่วยรายนี้ที่เหมาะสมที่สุด


a. VZV
โจทย์บอกเป็นvesicleตามแนวซี่โครง พูดง่ายๆคือ dermatome คาตอบก็คงรู้อยู่แล้วว่าเป็น VZV เอาเป็นว่าเพิ่มเติมที่ควรรู้ล่ะกัน
-VZV(herpes type 5) เป็น DNA virus
Herpes viruses known to cause human disease:

 Herpes simplex virus Type 1 (HSV-1)


 Herpes simplex virus Type 2 (HSV-2)
 Varicella Zoster virus (VZV) herpes type3
 Epstein Barr virus (EBV) herpes type4
 Cytomegalovirus (CMV) herpes type5
 Human herpes virus 6 (exanthum subitum or roseola infantum)
 Human herpes virus 7
 Human herpes virus 8 (Kaposi’s sarcoma)

-หากเป็นในหญิงตั้งครรภ์ โดยเฉพาะ1st trimester ไวรัสสามารถผ่านไปสู่ทารกทาให้เกิดความผิดปกติแต่กาเนิด (ร้อยละ1.2) เรียก fetal


or congenital varicella syndrome มีlesion เรียกว่า cicatrical skin lesion

-อาการและอาการแสดง ประวัติสัมผัสโรค10-20วัน ไข้ต่าๆ ผื่นกระจายลักษณะcentripetal คือ มากตามลาตัว หน้าและขามีน้อย ลักษณะ


ที่สาคัญคือ multistage(ผื่นหลายระยะในบริเวณเดียวกัน )
-หากขูดก้นฝาvesicleไปตรวจ จะพบ giant cell ที่มีintranuclear inclusion body
-DDX…Dermatitis herpetiformis ซึ่งมักเป็นเท่ากัน2ข้าง เป็นpapulovesicular มักได้ประวัติเป็นเรื้อรัง เมื่อหายผิวจะดา
-ภาวะแทรกซ้อน-ปอดบวม มักเป็นในผู้ใหญ่ เกิดใน5วันแรกหลังผื่นขึ้น
- bacterial superimposed จากการไม่รักษาความสะอาด
-สมองอักเสบ มักพบในเด็ก ปวดหัว คอแข็ง ชัก หมดสติ สามารถหายเป็นปกติได้
ตายร้อยละ5-30
- hemorrhagic chicken pox เป็นthrombocytopenia เลือดออกผิวหนัง ปาก จมูก เกิดใน2-3วันหลังผื่นขึ้น
- disseminated varicella พบในเด็กที่มีภูมิคุ้มกันต่า
- reye syndrome เกิดหลังหายป่วยVZV 2-3วัน
-อื่นๆ ไตอักเสบ ตับอักเสบ ข้ออักเสบ ลูกอัณฑะอักเสบ
Treatment เอาเฉพาะที่มีใช้ในบ้านเราละกันนะ
-ในimmuncompromised host : acyclovir 1500mg/ตร.ม./day แบ่งให้tid 7-10วัน
-คนที่มีอัตราเสี่ยงสูงจะเกิดโรครุนแรง เช่น เด็กอายุมากกว่า12ปี ,เป็นโรคผิวหนังเรื้อรัง , เป็นโรคปอดเรื้อรัง , ทานยา salicylate เป็นเวลานาน :
acyclovir 80mg/kg/day oral แบ่งให้qid
NATTAPAT MD014
10. ผู้ป่วยหญิงไทยอายุ 40 ปี มีอาการปวดบวมต้นขาซ้ายและมีไข้ แพทย์ทาการตรวจร่างกายพบว่ามี BT = 39 c, erythema and tender at
left upper thigh เจาะแล้วได้หนอง เชื้อก่อโรคในข้อใดต่อไปนี้น่าจะทาให้เกิดอาการในผู้ป่วยรายนี้มากที่สุด

a. S. aureus
b. S.pyogenes
c. H. influenzae
d. P. aeruginosa

ans. a. S. aureus – Pyomyositis ระยะ Suppurative stage


โรคนี้คือ Pyomyositis เป็น acute bacterial infection และลงทายดวย deep abscess ในสวนลึกของกลามเนื้อสวนใหญพบที่ thigh โดยพบ
มากสุดที่ Quadricepsเชื้อที่พบมากที่สุดคือ Staphylococcus aureus 66-90%
อาการ ใน typical case จะแบงเปน 3 ระยะ ไดแก
1. Invasive stage ในระยะนี้การอักเสบจะอยู เฉพาะในชั้นกลามเนื้อไมกระจายมายัง subcutaneous และ skin ทาใหไมมี sign of local
inflammation จะมีเพียงปวดกลามเนื้อ, ตะคริว , รวมกับไขต่าๆ, ครั่นเนื้อครั่นตัว , เมื่อ aspiration จะไมได pus
2. Suppurative stage อยูในชวงสัปดาหที่ 2-3 จะมี muscle abscess formation ผูปวยจะมีไขสูง กลามเนื้อบวมและเจ็บมาก อาจคลาไดก
อน มีอาการบวมแดงรอนอักเสบ ผูปวยรอยละ 90 จะมาพบในระยะนี้ การทา needle aspiration จะได pus ออกมา
3. Late stage ถาไมไดรับการรักษาในระยะที่ 2 ผูปวยจะแยลงมากมี sign ของ toxicity และ septic shock ไดในที่สุด
อาการของ pyomyositis อาจสับสนไดกับ hematoma, cellulitis, deep vain thrombosis, septic arthritis, osteomyelitis, soft
tissue sarcoma ได gold standard ในการวินิจฉัยคือ pus aspiration หรือ muscle biopsy รวมกับ culture และผลการตรวจทางหอง
ปฏิบัติการจะพบมี leukocytosis, ESR มากกวา 20-100 ถาพบeosinophil เดนมักจะสัมพันธกับ parasite infection เชน trichinosis หรือ
cysticercosis
S.pyogenes และ H. influenzae เป็นเชื้อที่สามารถก่อโรคนี้ได้เช่นกัน แต่พบได้น้อยกว่า ต้องนาไป culture แยกเชื้อเพื่อการวินิจฉัยที่ถูกต้อง
Streptococcus pyogenes – เป็นเชื้อที่ทาให้เกิดโรค Scarlet fever, pharyngitis, erysipelas, severe invasive infections เป็นส่วนใหญ่
แต่อาจพบในโรค pharyngitis (strep throat), scarlet fever (rash), impetigo (infection of the superficial layers of the skin) or cellulitis
(infection of the deep layers of the skin). Invasive, toxigenic infections can result in necrotizing fasciitis, myositis and
streptococcal toxic shock syndrome. Patients may also develop immune-mediated post-streptococcal sequelae, such as acute
rheumatic fever and acute glomerulonephritis, following acute infections caused by Streptococcus pyogenes.
H. influenzae – Meningitis, Cellulitis, Epiglottitis, Pneumonia, Septic arthritis, Pericarditis, Occult bacteremia
-- ApplE_BlooM med 026 –

11. ผู้ป่วยชายไทยสูงอายุ ประวัติโรคประจาตัวเดิมเป็น DM, HT ครั้งนี้มาด้วยอาการชาและอ่อนแรง 1 ข้าง นาน 10 นาที จากนั้นหายเป็น


ปกติ ข้อใดคือการวินิจฉัยในผู้ป่วยรายนี้ที่เหมาะสมที่สุด

a. TIA
b. Temporal lobe ischemia
c. cerebella ischemia
d. Bell’s palsy
Ans.
Transient ischemic attack (TIA) หรือ mini stroke เป็นภาวะที่เกิดชั่วคราวไม่ถึง 1 วัน ไม่มี brain damage อาจจะมีอาการชาหรืออ่อนแรง
ของกล้ามเนื้อหรือระดับความรู้ตัวเปลี่ยนไป
Sign & symptom :
- Visual loss in one or both eyes
- Double vision
- Vestibular dysfunction : True vertigo is likely to be described as a spinning sensation rather than nonspecific lightheadedness.
- Trouble swallowing may indicate brainstem involvement; if the swallowing problem is severe, there may be an increased risk
of aspiration.
- Unilateral or bilateral: either decreased sensation (numbness) or increased sensation (tingling, pain) in the face, arm, leg, or
trunk
- Slurring of words or reduced verbal output; language difficulty pronouncing, comprehending, or "finding" words
- Clumsy arms, legs, or trunk; loss of balance or falling (particularly to one side) with standing or walking
Bell's palsy is a unilateral, peripheral facial paresis or paralysis that has an abrupt onset and no detectable cause. Bell palsy is
one of the most common neurologic disorders affecting the cranial nerves, and it is certainly the most common cause of facial
paralysis – Clinical The most common complaint is of weakness on one side of the face, Postauricular pain, Tear flow This is
due to the reduced function of the orbicularis oculi in transporting the tears. Fewer tears arrive at the lacrimal sac and overflow
occurs. The production of tears is not accelerated. Altered taste, Dry eyes, Hyperacusis
Temporal lobe ischemia และ cerebella ischemia
ทั้ง 2 โรคนี้ จะเป็นถาวร อาการจะไม่หายไปภายใน 24 ชม. อาการจะแสดงตามรอยโรคบริเวณนั้นๆ เช่น อาการทาง cerebellum หรือ
temporal lobe อย่างชัดเจน
-- ApplE_BlooM med 026 –

12. ผู้ป่วยชายอายุ 18 ปี ก่อนหน้านี้สุขภาพร่างกายดี ไม่มีโรคประจาตัว ครั้งนี้มีการ upper respiratory tract infection ผู้ป่วยมาด้วยอาการ
ไข้ ไอ หอบ ผลจากการตรวจ CXR พบว่ามี interstitial infiltration ยาปฎิชีวนะใจเหมาะสมที่สุดในผู้ป่วยรายนี้
a. Ampicillin
b. Ciprofloxacin
c. Roxithromycin
d. Gentamycin
e. Augmentin
Ans. c. Roxithromycin
ผู้ป่วยรายนี้เป็น pneumonia จากประวัติแล้วร่างกายแข็งแรงดี ไม่มีโรคประจาตัวอะไร จัดเป็นกลุ่มที่ 1 คือ รักษาแบบผู้ป่วยนอก ไม่มีโรคปอด
เรื้อรัง หรือโรคหัวใจ เชื้อโรคที่เป็นสาเหตุหลักของโรคปอดบวมกลุ่มนี้คือ S. pneumonia, C. pneumonia, M. pneumonia และ ไวรัส
ยาปฎิชีวนะที่ใช้ oral macrorides เช่น roxithromycin, clarithromycin, azithromycin
รับประทาน 7 – 10 วัน หรือ doxycycline 100 mg. oral bid. 7-10 day ควรนัดผู้ป่วยมาดูอาการ ใน 1 สัปดาห์ด้วย
Ampicillin – เป็นยากลุ่ม Aminopenicillins ใช้รักษาในกลุ่ม gram + ve (like penG) and gram – ve : E. coli, P. mirabilis, H. influenzae
Ciprofloxacin – เป็นยากลุ่ม fluoro-quinolone ใช้รักษา Lower Respiratory Tract Infections caused by Escherichia coli, Klebsiella
pneumoniae, Enterobacter cloacae, Proteus mirabilis, Pseudomonas aeruginosa, Haemophilus influenzae, Haemophilus
parainfluenzae, or penicillin-susceptible Streptococcus pneumoniae. Also, Moraxella catarrhalis for the treatment of acute
exacerbations of chronic bronchitis.
Roxithromycin เป็นยากลุ่ม macrolide
AUGMENTIN is an oral antibacterial combination consisting of the semisynthetic antibiotic amoxicillin and the β-lactamase
inhibitor, clavulanate potassium (the potassium salt of clavulanic acid)
indication : Lower Respiratory Tract Infections – caused by β-lactamase–producing strains of H. influenzae and M. catarrhalis.
Otitis Media– caused by β-lactamase–producing strains of H. influenzae and M. catarrhalis.
Sinusitis – caused by β-lactamase–producing strains of H. influenzae and M. catarrhalis.
Skin and Skin Structure Infections – caused by β-lactamase–producing strains of S. aureus, E. coli, and Klebsiella spp.
Urinary Tract Infections – caused by β-lactamase–producing strains of E. coli, Klebsiella spp., and Enterobacter spp.

-- ApplE_BlooM med 026 –

13. ในการรับน้องที่มหาวิทยาลัยแห่งหนึ่ง รุ่นพี่ได้ให้รุ่นน้องให้กินน้าเปล่าปริมาณมาก หลังจากนั้นผู้ป่วยมีอาการซึม ปวดศีรษะและ คลื่นไส้


อาเจียน ข้อใดต่อไปนี้น่าจะทาให้เกิดอาการในผู้ป่วยรายนี้มากที่สุด
ตอบ
Hyponatremia จากการดื่มน้ามากเกินไป จนเกิด water intoxication (Pna < 135 mmol/L)

Approach to hyponatremia

อาการ : เริ่มจาก N/V แล้วจึงเกิดอาการทางสมอง ผู้ป่วยมักมาด้วยปวดศีรษะ alteration of consciousness


Treatment : ในกรณีนี้เป็น hypotonic hyponatremia ( euvolemic ) ให้การรักษา โดยให้ Diureticและให้Saline เพื่อเพิ่ม Na

Reference
http://medbhumibhol.com/Emed2008/13%20Hyponatremia.pdf
14. สารพิษที่พบในสีทาบ้าน ทาให้เกิดอาการ peripheral neuropathy (wrist drop) และซีดคืออะไร
ตอบ
สารพิษที่พบในสีทาบ้าน คือ lead (ตะกั่ว)
อาการ : ซีด เบื่ออาหาร เหนื่อยง่าย ปวดหัว อาเจียน ปวดท้อง กระสับกระส่าย ความจาไม่ดี ท้องเดิน/ท้องผูกเดินเซ โคม่า ชัก
Sign :
- Peripheral neuropathy : wrist drop, foot drop
- Bruton’s lines : purple line on gums
- Renal tubular acidosis
- ปัสสาวะสีน้าตาลแดง
Lab investigate
PBS : basophilic stippling , hypochromic microcytic anemia
X-ray : lead line on bone (มักพบในเด็กอายุไม่เกิน 6 ปี) , สารทึบแสงในท้อง
Blood lead level > 25 microgram/dl.
Treatment
- อาการไม่มาก ให้เฉพาะ EDTA อย่างเดียว
หรือให้ oral penicillamine 25-40 mg/kg/day x 5 day
(ในเด็กให้ไม่เกิน 1 g/day)
- ในรายที่มีอาการทางสมอง หรือมี Blood lead level > 70 microgram/dl. จะให้
o BAL 4mg/kg/dose IM q 4 hr x 5 day
o หลังให้ BAL 1 ครั้ง ให้ตามด้วย EDTA 5 mg/kg/dose IM q 4 hr x 5 day
- ในผู้ป่วยที่มีอาการทางสมอง ต้องรักษาภาวะชักและโคม่า

Reference
- First aid for the wards หน้า 352-353
- เวชศาสตร์ฉุกเฉิน หน้า 876-878

15. ผู้ป่วย CKD มี K = 7 EKG change จงบอกการรักษาที่สาคัญที่สุดในผู้ป่วยรายนี้


ตอบ
Treatment ใน acute hyperkalemia ใช้ในกรณีที่ระดับ Pk > 6 mEq/L หรือ EKG change หรือมีกล้ามเนื้ออ่อนแรง

EKG change ใน hyperkalemia จะเป็นแบบ peaked T waves, QRS widening

Evaluate : ECG, electrolytes, BUN, Cr, and glucose.


Treatment
Emergency treatment (guideline CPR 2005)
หลัก : shift K เข้า cell,ขับ K ออก,stabilizing myocardial cell membrane
1. ***ให้ 10% calcium gluconate 10-20 ml. IV ใน 2-3 min ( ร่วมกับ monitor EKG) อาการจะดีขึ้นใน 5-10 min (ถ้าไม่
ดีขึ้น ให้ซ้าได้อีก)
ระวัง ผู้ป่วยที่ได้ digitalis อยู่  ให้แบบ 5% D/W 100 ml. in 30 min
2. Sodium bicarbonate 50 mEq IV over 5 minutes.
3. Glucose plus insulin (10 units of regular insulin and an amp of D50)
4. Nebulized albuterol (10 to 20 mg over 15 minutes)
5. Furosemide 40 to 80 mg IV

16. ผู้ป่วยชายไทย 40 ปีมีไข้และปวดเมื่อยตามตัวมา 3 วัน ผู้ป่วยเล่าว่าละแวกบ้านมีน้าและต้องลุยน้าออกจากบ้านทุกวัน แพทย์ตรวจ


ร่างกายพบวามี BP = 120/80 , PR = 110/min , RR = 24/min , BT = 39 C , mild jaundice , lung clear , liver 2 cm BRCM ข้อใดคือการ
วินิจฉัยผู้ป่วยรายนี้ที่เหมาะสมที่สุด
a. Enteric fever
b. Dengue fever
c. Leptospirosis
d. Scrub typhus
e. Murine typhus
เฉลย c ข้อนี้ดูจากประวัติ ชัดเจนว่ามีการลุยน้าออกจากบ้านทุกวัน ก็นึกถึง lepto มากที่สุด เพราะ enteric ติดทาง GI ไข้เลือดออกก็
สามารถนึกถึงได้จากน้าขัง สครับน่าจะมีประวัติเข้าป่าเข้าพุ่มไม้ ส่วน murine เกิดจากตัวหมัดมากัดแล้วมีประวัติเกาจนเป็นแผล ซึ่งจากแค่
ประวัติแล้ว สามารถแยกออกได้ 3 ข้อเลยทีเดียว จะเหลือแค่ Dengue กับ Lepto
Leptospirosis
เกิดจากเชื้อ leptospira interrogans ผ่านทางแผลที่ผิวหนัง หรือ mucous มันเกิดช่วงหน้าฝนมีน้าท่วม หรือทุ่งนาที่มีน้าขัง
Clinical manifestation มักจะมีไข้สูง ตับม้านโตได้ ต่อน้าเหลืองโต ตาแดง ปวดศีรษะจะเจอบ่อยมาก
Musculoskeletal มักมีการปวดกล้ามเนื้อโดยเฉพาะบริเวณ claves, back, neck increase CPK
GI มี N/V ได้ บางรายเหลืองได้
U/A จะผิดปกติ พบ albumin สูงขึ้น อาจพบ WBC หรือ RBC ส่วน BUN/Cr จะปกติ
Respiratory อาจพบว่าหอบเหนื่อยได้ CXR มักเจอ patchy alveolar infiltration BLL
CBC พบ Lymphocytosis 74% , PMN predominate, Plt < 100000
Dengue fever
เกิดจากเชื้อไวรัสในยุงลาย มักเกิดในหน้าฝน น้าขังเยอะๆเหมือนกัน มักมีประวัติคนในชุมชนเป็นโรค
Clinical manifestation มักจะมีไข้สูง ตับโต ม้ามไม่โต ต่อน้าเหลืองโต ตาแดงได้
Musculoskeletal ไม่มีอาการปวดเมื่อยกล้ามเนื้อ
GI มีเบื่ออาหาร กินข้าวได้น้อยลง
U/A ปกติ
Respiratory อาจพบว่าหอบเหนื่อยได้ จาก plasma leakage มักเป็น Rt. Pleural effusion
CBC พบ Lymphopenia WBC<5000 , Plt < 100000 , L Predominate พบ atypical L
จะเห็นได้ว่าจาก review โรคคร่าวๆ ก็ได้คาตอบว่า LEPTOSPIROSIS นั่นใช่เลย !!!
17. ผู้ป่วยหญิงอายุ 50 ปี มาด้วยปวดแขนขา ขึ้นบันไดลาบาก ลุกจากเก้าอี้ลาบาก 4 wkPTA PE : tenderness of quadriceps muscle,
purple-red discoloration over upper forehead eyelids and cheeks ข้อใดคือ Dx
a. Polymyositis
b. Dermatomyositis
c. Psoriasis
d. Mixed connective tissue disease
e. SLE
เฉลย a
Polymyositis เป็นโรค autoimmune ชนิดหนึ่งที่มีการอักเสบของกล้ามเนื้อต่างๆ ทั่วร่างกาย จัดเป็นหนึ่งในกลุ่มโรคเนื้อเยื่อเกี่ยวพัน
ปัจจุบันยังไม่ทราบสาเหตุที่แน่ชัด พบได้บ่อยในเพศหญิงมากกว่าเพศชาย PM อายุที่พบอยู่ในช่วง 30-50 ปี แต่พวก dermatomyositis มัก
เจอในกลุ่มอายุน้อยๆ
History

 Symmetric proximal muscle weakness with insidious onset


 Muscles usually painless (Myalgias occur in fewer than 30% of patients.)
 Dysphagia (30%) and aspiration, if pharyngeal and esophageal muscles are involved
 Arthralgias may be associated
 Difficulty kneeling, climbing or descending stairs, raising arms, and arising from a seated position; weak neck
extensors cause difficulty holding the head up; involvement of pelvic girdle usually greater than upper body weakness
 Family history and medication history are important in excluding other causes of myopathy.

Physical

 Muscle tenderness on palpation


 Normal sensory test results and reflexes (Reflexes may be abnormal with advanced disease.)
 Muscle atrophy
 In DM, a characteristic heliotrope rash preceding or accompanying muscle weakness
o The heliotrope rash is a symmetric, confluent, purple-red, macular eruption of the eyelids and periorbital
tissue. Edema may also be present.
o Other rashes seen with DM include erythematous nail beds and a scaly, purple erythematous papular
eruption over the dorsal metacarpophalangeal and interphalangeal joints (Gottron sign).
o Violaceous erythematous confluent macular eruption over the deltoids, posterior part of the shoulders, and the
neck is known as the shawl sign. A similar eruption of the V area of the anterior part of the neck and the upper
part of the chest is known as the V sign.
 Extramuscular manifestations
o Cardiac - Congestive heart failure (CHF), arrhythmia
o Lung - Interstitial lung disease, pneumonia/aspiration
o Gastrointestinal - Dysphagia
o Joints - Arthralgias, symmetric arthritis
Dermatomyositis เป็นกลุ่ม musculoskeleton disease เหมือน PM พบในผู้หญิงที่อายุน้อยๆ มาด้วยเรื่อง skin เด่นกว่า

 Patients often present with skin disease (eruption on exposed surfaces. The rash is often pruritic, and intense pruritus
may disturb sleep patterns)
 Muscle involvement คล้ายๆ PM แต่มักไม่มี ปวดกล้ามเนื้อ
 arthralgia, arthritis, dyspnea, dysphagia, arrhythmia, and dysphonia. ได้

จะเห็นได้ว่า PM เข้ามากกว่า DM จึงวินิจฉัยว่าเป็น DM ส่วน Psoriasis ตัดไปได้เลย และกลุ่ม connective tissue disease ก็ตัดไปได้เพราะ
ไม่มีอาการที่เด่นชัดของกลุ่มนี้เลย

18. ผู้ป่วยชาย 40 ปี ไอแห้งๆมา 1 เดือนก่อนมารพ. เป็นมากตอนอากาศหนาวและตอนเย็น ไอมีเสมหะสีขาว ตรวจร่างกายปกติ ผล CXR


ปกติ การปฏิบัติการต่อไปนี้เหมาะสมที่สุด
a. skin test
b. CT chest
c. sputum AFB
d. bronchoscopy
e. Pulmonary function test
เฉลย คิดว่า e
เนื่องจากประวัติมาด้วยเหมือน Lung hypersensitivity มากกว่า ไม่เหมือน TB ซึ่งมักจะไม่มีอาการ หรือถ้ามีไอเรื้อรังมักจะไอตลอด
ทั้งวัน และเสมหะมักเป็นสีเหลืองเขียว จึงคิดว่าน่าจะตรวจ fuction ของ lung มากกว่า

19. ผู้ป่วยชายอายุ 50 ปี กินเหล้าขาวมานาน 25 ปี ครั้งนี้มีอาการเหนื่อย ขาบวม 1 สัปดาห์ก่อนมาโรงพยาบาล แพทย์ทาการตรวจร่างกาย


พบว่ามี BP = 120/70 mmHg, fine crepitation both lower lungs, PMI 6th ICS 1 cm lateral to MCL, S3 gallop การรักษาที่
เหมาะสมที่สุดในผู้ป่วยรายนี้คือข้อใด
f. Digoxin
g. Thiamine
h. Atropine
ตอบ Digoxin (รักษาโดยการเพิ่ม contractility)
จากโจทย์ผู้ป่วยเป็น both sides CHF ก็ให้การรักษา CHF
จุดประสงค์หลักของการรักษาcongestive heart failure ได้แก่ ลดอาการ,ลดการเข้ารับการรักษาตัวในโรงพยาบาลและลดอัตรา
ตาย โดยทั่วไปวิธีการรักษาประกอบด้วย
1. การลดการทา งานของหัวใจ (เช่น จากัดการออกกาลังกาย, ลดน้าหนักตัว และควบคุมความดันเลือด ฯลฯ)
2. การควบคุมอาหาร ควรแนะนาให้ผู้ป่วยจากัดการบริโภคเกลือเพื่อป้องกันหรือลด salt-water retention
3. Pharmacologic therapy การใช้ยาชนิดต่างๆ ใน congestive heart failure มีจุดประสงค์หลัก 2 ประการ คือ
3.1 เพื่อบรรเทาอาการ ซึ่งอาการจากcongestive heart failure เป็นผลสืบเนื่องมาจากการเพิ่มปริมาณน้าในร่างกาย และ
preload, การเพิ่ม afterload และการลดลงของ cardiac output ดังนั้นยาที่มีผลบรรเทาอาการจึงต้องสามารถเปลี่ยนแปลง
hemodynamics เหล่านี้ไปสู่ทิศทางที่ดีขึ้นซึ่งผลของยาชนิดต่างๆ ต่อ hemodynamic effects มีดังต่อไปนี้ คือ
Hemodynamic effects
1. ลด preload
◊ Diuretics
◊ Venodilators (เช่น organic nitrates)
◊ ACEIs** หรือ ARBs**
◊ Sodium nitroprusside**
5. ลด afterload
◊ Arterial vasodilators (เช่น hydralazine)
◊ ACEIs** หรือ ARBs**
◊ Sodium nitroprusside**
6. เพิ่ม contractility
◊ Cardiac glycosides (digitalis)
◊ Beta1 adrenergic agonists
◊ Phosphodiesterase enzyme inhibitors
* ACEIs, ARBs, venodilators และ arterial vasodilators รวมไว้ในกลุ่ม vasodilators
**มีฤทธิ์ขยายได้ทั้งหลอดเลือดดาและหลอดเลือดแดง จึงมีผลลดได้ทั้ง preload และ afterload
3.2 เพื่อลดการเกิด cardiac remodeling
ได้แก่ ยาที่มีฤทธิ์ยับยั้ง RAS เช่น angiotensin converting enzyme inhibitors (ACEIs) หรือ angiotensin receptor blockers
(ARBs) และยาที่มีฤทธิ์ยับยั้งการทางานของ sympathetic nervous system (เช่น beta-blockers) นอกจากนี้ยาที่มีผลลด
ventricular wall stresses (เช่น vasodilators) ก็อาจมีส่วนลด cardiac remodeling ได้เช่นเดียวกัน
By BuM

20. ผู้ป่วยชายหลังจากกลับจากน้าตก มีไข้และแผลดังแสดงในรูป เชื้อก่อโรคในข้อใดต่อไปนี้น่าจะทาให้เกิดอาการใน ผู้ป่วยรายนี้มากที่สุด


a. Chigger mite
b. Louse
c. Aedes egypti
d. Tick
e. Flea
ตอบ Chigger mite (ถ้าแผลในรูปที่ว่านี้เป็น eschar นะ เพราะมันไม่มีรูป )

Scrub Typhus เป็นโรคติดต่อที่เกิดจากเชื้อริกเกตเซีย (Rickettsia) ตามธรรมชาติเป็นโรคติดต่อของสัตว์ป่าโดยเฉพาะสัตว์ตระกูล


ฟันแทะ เชื้อนี้ติดต่อจากสัตว์ตัวหนึ่งไปอีกตัวหนึ่งได้โดยถูกไรอ่อน ( Chigger) ที่มีเชื้อกัด สาหรับคนบังเอิญเข้าไปใน enzootic cycle หรือเข้า
ไปในแหล่งที่อาศัยของไรและถูกไรอ่อนกัด แหล่งเกิดโรคตามธรรมชาติ ( Endemic foci) จะต้องมีปัจจัยประกอบกัน 4 ประการ ได้แก่

1. ต้องมีสัตว์ฟันแทะ ( Rodent) โดยเฉพาะหนูป่าในกลุ่ม Rattus spp.


2. ต้องมีไรอ่อนในตระกูล Leptortrombidium สาหรับประเทศไทยและ ประเทศในแถบตะวันออกเฉียงใต้จะมี L.deliense เป็นพาหะ
หลัก
3. พื้นที่ที่เป็นแหล่งอาศัยของไร ( Transitional vegetation) เช่น ป่าโปร่ง Fringe vegetation พื้นที่เพาะปลูกหรือพื้นที่เกษตรกรรม
4. มีเชื้อ Orientia tsutsugamushi

แหล่งที่มีเชื้อแพร่กระจายอยู่ได้แก่ เขตร้อน (Tropical zones) จนถึงเขตอบอุ่น ( Temperate zones) ซึ่งจะครอบคลุมถึงป่าร้อนชื้น (


rain forest) ป่าสน พื้นที่กึ่งทะเลทรายและพื้นที่ชายทะเล

อาการของโรค : สครับไทฟัส เป็นโรคที่เกิดจากเชื้อ Rickettsia genus ใหม่ คือ Orientia


tsutsugamushi ( R.tsutsugamushi ); คนเป็น accidental host รับเชื้อโดย ถูกพาหะคือตัวอ่อน
ของไรแดงกัด (Chigger) ลักษณะเฉพาะของโรค คือ ผิวหนังที่ถูกตัวไรกัด มัก เป็นแผลบุ๋มสีดา
รูปร่างกลมออกรี ขอบนูนเรียบ ไม่เจ็บ ขนาดประมาณ 0.5- 1.5 เซนติเมตร ลักษณะคล้ายแผล
ถูกบุหรี่จี้ (Eschar) ซึ่งพบอยู่นานประมาณ 6-18 วัน พบได้ประมาณ 30-40% ต่อมาผู้ป่วยจะมี
อาการป่วย มีไข้สูง ปวดศีรษะมาก , คลื่นไส้ , อาเจียน , หูอื้อ , เหงื่อออก , หนาวสั่น , ปวดเมื่อยตามตัว บางรายมีปวดน่อง ตาแดง
(Conjunctival injection), มีอาการต่อมน้าเหลืองอักเสบ (Lymphadenopathy) โดยเฉพาะต่อมน้าเหลืองที่อยู่ใกล้ eschar (Watt and
Olson,2000), ตับโต , ม้ามโต , หลังมีไข้ 4-5 วัน บางรายปรากฏผื่นนูนแดง (Maculopapular rash) ตามลาตัวซึ่งกระจายไปยังแขน ขา ผื่น
เหล่านี้จะหายไปในเวลา 2-3 วัน อาการที่พบบ่อยคือ ไอ เมื่อเอกซ์เรย์ปอดพบการอักเสบของเนื้อปอด ส่วนใหญ่เป็นแบบ Interstitial
infiltration แต่มีบางรายเป็นแบบ Alveolar infiltration, คอแข็ง (Stiff neck) ดีซ่าน (Jaundice)

การรักษา :

การให้ยาปฏิชีวนะที่สามารถฆ่าเชื้อ O. tsutsugamushi เป็นวิธีเดียวที่ช่วยลดอาการของโรค ลดอัตราเจ็บป่วย อัตราตาย และการ


แพร่กระจายของโรคในผู้ป่วย โดยทั่วไปรักษาด้วยยา Tetracycline 500 mg วันละ 2 ครั้ง Doxycycline 100 mg วันละ 2 ครั้ง เป็นเวลา 7 วัน
หรือ Chloramphenicol 50-75 mg/ น้าหนักตัว 1 kg/ วัน ให้ผลการรักษาดีพอกัน หรืออาจจะใช้ยาแบบฉีดถ้าผู้ป่วยคลื่นไส้อาเจียนมาก หรือ
ในรายที่รุนแรง ส่วนใหญ่ไข้จะลงในเวลา 24-36 ชั่วโมงหลังได้รับการรักษา สาหรับในผู้หญิงที่ตั้งครรภ์ หรือในเด็ก ไม่สามารถใช้ยา กลุ่ม
Tetracycline หรือ Chloramphenicol อาจให้ยา Azithromycin แทน

By BuM

21. ผู้ป่วยชายไทย มาโรงพยาบาลด้วยอาการชาที่บริเวณต้นคอ แพทย์ทาการตรวจร่างกายพบว่ามี BP 60/30 mmHg ไม่มี sacral reflex ข้อ
ใดคือการวินิจฉัยในผู้ป่วยรายนี้ที่เหมาะสมที่สุด
a. Cardiogenic shock
b. Spinal shock
c. Neurogenic shock
d. Hypovolemic shock
e. Obstructive shock
ตอบ Neurogenic shock
เพราะว่า ไม่มี sacral reflex รวมทั้งมีอาการชาที่บริเวณต้นคอ ซึ่ง
Neurogenic shock คือ ภาวะ cardiovascular instability ที่เกิดขึ้นในราย complete injury ของ cervical / upper thoracic cord
หรือเกิดอันตรายกระทบกระเทือนต่อสมองรุนแรง
pathophysiology
เนื่องจากเกิด disruption ของ descending sympathetic pathways ทาให้ parasympathetic innervation จาก vagus nerve ไม่
มี sympathetic มา counteraction ด้วย
sign and symptoms
Generalize vasodilatation with increase vessel capacitance
Decrease central venous return
Lower extremity venous stasis
Loss of body heat regulation below lesion เนื่องจาก denervation ของ sweat gland
Bradycardia with hypotension เป็นตัวแยกออกจาก hypovolemic shock (ใน hypobolemic shock เป็น tachycardia)
การรักษา
- Trenderlenburg position - ให้ intravenous fluid ด้วยความระมัดระวัง
- อาจต้องให้ atropine - อาการจะค่อยดีขึ้นจากหลายวัน ถึง เดือน
pharmacologic treatment
วัตถุประสงค์เพื่อลด second injury cascade ที่ตามหลัง spinal cord injury ยาเหล่านี้ได้แก่
- corticosteroid
- 21-aminosteroid
- antioxidants
- gangliosides
ส่วน spinal shock คือสภาวะที่มี transient loss of all motor,sensory,reflex function distal ต่อ level of injury ถ้าเราตรวจพบ
bulbocarvernosus reflex,anal wink reflex กลับคืนมา หมายความว่าคนไข้พ้นภาวะ spinal shock แล้ว (เป็นภาวะหนึ่งที่เกี่ยวข้องกับการ
เกิด spinal cord injury ไม่มีผลต่อ cardiovascular collapse ซึ่งมักจะกินเวลาไม่เกิน 24 ชม.แต่อาจนาน เป็นสัปดาห์)
By BuM

22. วัยรุ่นชายอายุ 18 ปี แข็งแรงดี มีอาการหอบ หายใจลาบาก ตอนออกกาลังกาย และตอนกลางคืน มีอาการ 2-3 ครั้งในเดือนที่ผ่านมา
แพทย์ทาการตรวจร่างกายเบื้องต้นพบว่าไม่มีความผิดปรกติใดๆ การรักษาที่เหมาะสมที่สุดในผู้ป่วยรายนี้คือข้อใด
a. inhale salbutamol prn + inhale corticosteroid
b. alpha-agonist as need
c. steroid ตลอด
d. beta-agonist as need + steroid ตลอด
e. Oral bronchodilator
f. Salbutamol inhalation as needed
g. Long acting beta2 agonist + corticosteroid
h. Inhaled corticosteroid
เฉลย
โจทย์ข้อนี้ผู้ป่วยก็เป็น Asthma ก็พิจารณาตาม อันข้างๆ ดังนี้   
Characteristic Controlled Partly controlled Uncontrolled
(All of the following) (Any present in any week)
Daytime symptoms None (2 or less/wk) More than twice/wk 3 or more features
of partly controlled
Limitations of activities None Any asthma present in
Nocturnal symptoms / None Any any week
awakening
Need for rescue / “reliever” None (2 or less/wk) More than twice/wk
treatment
Lung function Normal < 80% predicted or personal
(PEF or FEV1) best (if known) on any day
Exacerbation None One or more/year 1 in any week

Reduce Increase
หลังจากพิจารณาตามตาราง ในผู้ป่วยรายนี้มีมีอาการ หอบ หายใจลาบาก ตอนออกกาลังกาย และตอนกลางคืน มีอาการ 2-3 ครั้งใน
เดือน แสดงว่าอยู่ในกลุ่ม partly controlled จึงให้การรักษาตั้งแต่ Step2 ขึ้นไป ให้ Rapid acting Beta2-agonist prn. + Low dose inhaled
corticosteroid or Leukotriene
## ดังนั้นจึงตอบข้อ a. inhale salbutamol prn + inhaled corticosteroid ##
แล้วจะ highlight ข้อ B,C,D เพื่อ? หรือว่าเฉลยผิดเนี่ย
--------------------------------------------------------
23. Motor Cycle Accident มี lesion ที่ T2 คาสาย Foley Catheter มีอาการแน่นหน้าอก เหงื่อออกมากขึ้น BP 180/110 RR เพิ่ม ควรทา
อย่างไร
a. Off F/C
b. Hydralazine
c. ลุกนั่งปลดเสื้อผ้า
d. นอนหัวต่าวัด BP ซ้า
e. ADVICE
เฉลย
จากโจทย์ข้อนี้บอกผู้ป่วยมีอาการที่เข้ากับโรค “Autonomic dysreflexia“ โดยมีลักษณะต่างๆ ดังต่อไปนี้
Definition : เป็นกลุ่มอาการที่เกิดแบบ Acute onset ในผู้ป่วยที่มีรอยโรคที่ไขสันหลังตั้งแต่ T6 ขึ้นไป (จากโจทย์ T2) จะมีการตอบสนองของ
ระบบ Autonomic ที่มีต่อตัวกระตุ้นบางชนิดที่มากผิดปกติโดยที่ร่างกายไม่สามารถใช้กลไกของสมองและไขสันหลังเพื่อปรับร่างกายให้สมดุล
ได้
Pathophysiology : เกิดจากการกระตุ้นบางอย่าง ได้แก่ ขยายตัวโป่งพองของอวัยวะในช่องเชิงกราน ได้แก่ อวัยวะโป่งพองเนื่องจากการอุด
กลั้นของสายสวนหรือมีอุจจาระอุดกั้นจากท้องผูก จะทาให้เกิดการกระตุ้นในส่วนของ sympathetic ของ Spinal cord ทาให้เกิดการบีบตัว
ของ VV. ในส่วนต่ากว่ารอยโรคทาให้ BP สูงและอื่นๆ ตามมา ในคนปกติสามารถควบคุมได้ ในผู้ป่วยที่มีรอยโรคที่ไขสันหลังจะไม่สามารถ
ควบคุมได้ที่ระดับต่ากว่ารอยโรค
ตัวกระตุ้นที่ทาให้เกิด AD : ตัวกระตุ้นที่ทาให้เกิดอันตรายต่อเนื้อเยื่อของร่างกาย ที่พบบ่อยที่สุดก็คือ Full bladder, Impact feces
Clinical : ปวดศีรษะ (มักพบใน Pt.ที่มี BP>180 mmHg.) N/V , agitatiate, หายใจลาบาก ตามัว ชัก BP สูง (Sys. = 190-250, Dias. = 130-
150 mmHg.) PR ลดลงจากกระตุ้น Baroreceptor
Complication : เกิดจาก BP ที่สูงมากทาให้เกิด Hypertensive enphalopathy, subarachnoid hemorrhage, intracerebral hemorrhage
Treatment : หาสาเหตุและกาจัดตัวกระตุ้น ขั้นแรกจัดท่ายกศีรษะสูง ( ดังนั้นข้อ D ผิดชัวร์ ) หากพบกระเพาะปัสสาวะโป่งพองให้รีบแก้ไข
หากใส่ F/C ก็ให้เปลี่ยนใหม่เพราะข้างในอาจมีการอุดตัน ( ข้อ A คงจะผิดเพราะให้ off F/C เฉยๆ )
Medication : ส่วนใหญ่รักษาโดยไม่ใช้ยา จะใช้เมื่อการรักษาทั่วไปไม่ได้ผลหรือหาตัวกระตุ้นไม่ชัดเจน จาเป็นต้องใช้ยาลดความดัน ยาที่ใช้
ได้แก่ Nifedipine 5-10 mg. oral, Nitroprusside 100 microgram/ml, Hydralazine 10 mg IV ( มีใน Choice ก็น่าจะตอบข้อนี้ ? ) ## สรุป
ตอบข้อ B. Hydralazine มั้ง ? ^^
## จาก choice ข้อ C ไม่เกี่ยว ข้อ E จะหลอกเปล่าว่ะไม่แน่ใจ
--------------------------------------------------------
24. หญิงไทย 70 ปี no Underlying Disease สบายดีมาตลอด มีอาการแขนซ้ายอ่อนแรง 5 นาที EKG เป็น AF ใช้ยาอะไรป้องกันอาการ
อ่อนแรง
a. ASA
b. Warfarin
c. Clopidogrel

เฉลย
ให้ยาในกลุ่ม Anticoagulant to prevent thrombus formation and stroke โดยให้การรักษาตาม Guideline ดังนี้

weaker risk Moderate risk High risk


 Female gender  Age ≥ 75 years  Previous stroke, TIA or
 Age 65-74 years  Hypertension embolism
 CAD  Heart failure  Mitral stenosis
 Thyrotoxicosis  LV ejection fraction ≤ 35%  Prosthetic heart valve
 Diabetes mellitus

Risk Category Recommended Therapy


No risk factors Aspirin : 81-325 mg daily
One moderate risk factor Aspirin : 81-325 mg daily or Warfarin (INR 2.0 - 3.0, target 2.5)
Any high risk factor Warfarin (INR 2.0 to 3.0, target 2.5)
or > 1 moderate risk

 จากโจทย์ผู้ป่วยมี 1 moderate risk (Age ≥ 75 years) + 1 high risk (stroke) ก็เลยให้การรักษาด้วย Warfarin ตามตารางข้างบน ##
สรุปตอบข้อ B. warfarin ##
--------------------------------------------------------
By นสพ.กาญจน์ ยศธรสวัสดิ์ 47460647
ปล. ถ้าผิดก็บอกด้วย ห้ามด่า กลัวแล้ว

25. ผู้หญิง อายุ.. PE : HT + Cushing appearance ถามว่า Lab จะเจออะไร


Ans. Increase cortisal level

Cushing syndrome
เป็นความผิดปกติของฮอร์โมนร่างกาย เกิดจากการที่ระดับฮอร์โมน glucocorticoids สูงขึ้นเป็นเวลานานไม่ว่าจะเป็นฮอร์โมนที่ผลิต
ในร่างกาย หรือได้รับฮอร์โมนจากภายนอก
มักเกิดกับผู้ป่วยอายุระหว่าง 20-50 ปี

สาเหตุของโรค

1. สาเหตุที่สาคัญที่สุดคือ การได้รับฮอร์โมนกลูโคคอติคอยด์จากภายนอกร่างกาย เช่น ผู้ป่วยโรคหอบหืดที่กินยาสเตียรอยด์เป็นประจา


หรือผู้ป่วยโรคข้ออักเสบรูมาตอยด์ โรคลูปุส เป็นต้น รวมทั้งผู้ป่วยที่ได้รับการเปลี่ยนถ่ายอวัยวะที่ต้องกินยากดภูมิคุ้มกัน
2. ผู้ป่วยโรคเนื้องอกของต่อมใต้สมองจะสร้างฮอร์โมนคอร์ติซอลในร่างกายมากเกินไป ทาให้เกิดกลุ่มอาการคุชชิ่ง เรียกว่า Pituitary
adenoma ซึ่งเป็นเนื้องอกชนิดไม่ร้ายแรง ไม่ใช่มะเร็ง ส่วนใหญ่จะเป็นก้อนเดียว ผู้หญิงเป็นมากกว่าผู้ชาย 5 เท่า
3. Ectopic ACTH Syndrome เกิดจากเนื้องอกหรือมะเร็งที่อยู่นอกต่อมใต้สมอง ที่พบบ่อยคือมะเร็งปอด โดยพบมากกว่าครึ่งหนึ่งของ
ทั้งหมด มักเป็นเซลล์มะเร็งชนิดโอ๊ดเซลล์หรือชนิดตัวเล็ก
4. เนื้องอกของต่อมหมวกไต เป็นเนื้องอกชนิดที่สร้างฮอร์โมนคอร์ติซอลออกมาในระดับที่สูงเกินปกติ
5. มะเร็งของต่อมหมวกไตเป็นสาเหตุของ Cushing's syndrome ที่พบได้น้อย เซลล์มะเร็งหลั่งฮอร์โมนออกมาเป็นจานวนมาก ทั้งคอร์
ติซอลและแอนโดรเจน ผู้ป่วยจะมีอาการที่รุนแรงภายในเวลาอันรวดเร็ว และตรวจพบระดับฮอร์โมนในเลือดสูงมาก
6. ชนิดที่เกิดขึ้นในครอบครัวเดียวกัน เรียกว่า Familial Cushing's Syndrome พบได้น้อย

อาการของโรค

ผู้ป่วยแต่ละรายอาจมีอาการแตกต่างกันไป ลักษณะที่สังเกตได้ง่าย พบว่าผู้ป่วยจะลักษณะตัวอ้วนแขนขาเล็ก ใบหน้ากลมฉุ คอหนา โหนก


หลังหนา มีอาการกล้ามเนื้ออ่อนแรง โดยเฉพาะอย่างยิ่งกล้ามเนื้อต้นแขน ต้นขา สังเกตุได้ว่าเลือดออกง่ายผิดปกติ ผู้ป่วยอ้วนมาก น้าหนัก
เพิ่มขึ้นเรื่อยๆ และมีขนงอกขึ้นมาก
ผิดปกติมักพบว่าเป็นเบาหวานร่วมด้วย มีความดันโลหิตสูง เนื้อกระดูกบาง ภูมิคุ้มกันโรคบกพร่อง บางรายมีอาการซึมเศร้าหรือขี้หงุดหงิด
บางรายมีอาการของโรคประสาทกังวล

การวินิจฉัยโรค

วินิจฉัยจากประวัติอาการ ลักษณะการเปลี่ยนแปลงของร่างกาย การตรวจพิเศษเพิ่มเติมช่วยในการ


วินิจฉัยหาสาเหตุ ที่สาคัญได้แก่ เอ็กซเรย์ต่อมใต้สมองและต่อมหมวกไต เพื่อดูขนาดและรูปร่างของต่อมทั้งสอง และตรวจหาว่ามีเนื้องอก
หรือไม่ ส่วนใหญ่นิยมทา CT scan หรือ MRI, การตรวจปัสสาวะ 24 ชั่วโมงเพื่อวัดระดับของฮอร์โมนคอร์ติซอล เป็นการทดสอบที่มี
ความเฉพาะมากที่สุดถ้ามากกว่า 50–100 ไมโครกรัม ถือว่าเป็นโรค Cushing's syndrome การตรวจทางฮอร์โมนอื่นๆที่อาจจาเป็นต้องทา
การทดสอบในกรณีที่มีข้อบ่งชี้ ได้แก่Dexamethasone Suppression Test, CRH Stimulation Test,Dexamethasone-CRH Test แต่ไม่
จาเป็นต้องทาทุกราย

Diagram สรุปอ่ะ...ดีนะ
26. ผู้หญิง Subacute thyroiditis ให้ยาใดเพื่อลดอาการ
Ans. Propanolol

Subacute thyroiditis หรือ "De Quervain's Thyroiditis"


ผู้ป่วยมาด้วยต่อมธัยรอยด์โต และกดเจ็บ (painful) ร่วมกับมีอาการของ thyrotoxicosis และมีไข้(Classic presentation of a viral
illness. onset is sudden, with high fever, myalgia, and neck pain.)
การรักษา: รักษาตามอาการ (เพราะส่วนใหญ่เกิดจากการติดเชื้อไวรัส หายได้เอง)
ให้ Beta blocker (เพื่อลดอาการใจสั่น) เช่น Propanolol 40-120 mg./day หรือ atenolol 25-50 mg./day เป็นเวลาประมาณ 3
เดือน ร่วมกับการรักษาสาเหตุด้วย (สาคัญที่สุด) เช่น เป็นเนื้องอก ให้กาจัดเนื้องอก, หากเกิดจากการรับประทานยาก็ให้หยุดยา เป็น
ต้น

27. T4 ปกติ TSH ปกติ จะให้การวินิจฉัยอย่างไร


A. Simple Goiter
B. Iodine def.
C. Thyroid CA
D. Subacute thyroiditis
E. Hashimoto thyroiditis

การแปลผล Thyroid function test

FT3 FT4 TSH Diagnosis


เพิ่ม เพิ่ม ลด Hyperthyroid
เพิ่ม ปกติ ลด T3 toxicosis
ปกติ เพิ่ม ลด T4 toxicosis
เพิ่ม เพิ่ม ปกติหรือเพิ่ม Inappropriate TSH
ลด ปกติหรือลด ปกติหรือลด Non thyroidal illness
ปกติหรือลด ลด เพิ่ม Primary hypothyroid
ปกติหรือลด ลด ปกติหรือลด Secondary hypothyroid
ปกติ ปกติ เพิ่ม Subclinical hypothyroid
ปกติ ปกติ ลด Subclinical hyperthyroid
เฉลย ข้อ B. Iodine def. จ้า

28.ผู้หญิงอายุ 28 ปี ฉี่ไม่ออกต้องใส่สายสวนปัสสาวะ 2 ครั้ง ต่อมา ขาอ่อนแรงชาถึงระดับสะดือ PE: DTR 3+ , loss of sphincter tone
ผู้ป่วยมี lesion อยู่ที่ใด
a.parasagital sinus
b.basal pons
c.thoracic area
d.lumbosarcal area
e.clonus medularis
ในผู้ป่วยรายนี้มาด้วยฉี่ไม่ออกนึกถึงสาเหตุที่ทาให้เกิดพยาธิสภาพที่ lower urinary tract ส่วนที่เป็น bladder มากกว่าตาแหน่งอื่นในระบบ
นี้ เพราะอายุน้อยความเสี่ยงในการเกิดพยาธิสภาพตาแหน่งอื่นน้อย และไม่ได้บอกตาแหน่งการ trauma มาเฉพาะ โดย bladder นี้ก็เลี้ยงด้วย
autonomic system ซึ่งตาแหน่งที่อยู่ของระบบนี้คือ T1-L2 และ S2-S4โดยจากประวัติและการตรวจร่างกายนั้น เราต้องแยกว่าเป็นความผิด
ปกติของระบบประสาทระดับใดคือ UMN or LMN ซึ่งมีความแตกต่างกันดังนี้คือ
Clinical features UMN LMN
Pattern of weakness Pyramidal(arm extension,leg variable
flexion)
Tone Spastic(initial flaccid) Flaccid,hypotonic
DTRs hyperreflex Hyporeflex/areflex
Miscellaneous sign Babinski’s ,other CNS Atriophy,fasciculation,Babinski
sign,clonus flexor response
โดยผู้ป่วยคนนี้นึกถึงเกิดพยาธิสภาพที่ UMN มากที่สุด ซึ่งเรานึกถึงที่ spinal cord มากที่สุดเพราะเป็นทางผ่านของ tract ที่สามารถอธิบาย
พยาธิสภาพของผู้ป่วยรายนี้ได้ โดย tract ที่เกี่ยวข้องคิดว่ามี
- lateral corticospinal = movement of contralateral limb
- dorsal column medial lemnicus = tactile ,vibration,sensation
- spinothamic = pain ,temperature
โดยระดับการชาสูงสุดในขณะนี้ของผู้ป่วยตรวจได้ที่ ระดับสะดือซึ่งตาม dermatome ของ sensory คือระดับ T10 ,ผู้ป่วยมี paraplegia
ของขาทั้งสองข้างโดย The lower urinary tract นี้ innervated by 3 sets of peripheral nerves involving the parasympathetic,
sympathetic, and somatic nervous systems:

 Pelvic parasympathetic nerves: arise at the sacral level of the spinal cord, excite the bladder, and relax the urethra
 Lumbar sympathetic nerves: inhibit the bladder body and excite the bladder base and urethra
 Pudendal nerves: excite the external urethral sphincter

ดังนั้นตาแหน่ง lesion ที่ควบคุมอาการแสดงที่ผิดปกติของผู้ป่วยรายนี้น่าจะอยู่ในตาแหน่งที่สูงพอที่จะอธิบายอาการชาถึงระดับสะดือ


คือ ระดับ thoracic area

29. ผู้ป่วยชายไทยเป็น bronchial asthma พ่นยาไม่ดีขึ้น lung: epiratory wheezing การรักษาที่เหมาะสมที่สุดของผู้รายนี้คือข้อใด


Cause:
bronchial hyperreactive,airway inflammation,mucous plugging,smooth muscke hypertrophy
History/PE:
มาด้วย cough,dyspnea,wheezing,chest tightness อากการเป็นมากตอนเช้า หรือดึกๆ
PE พบ tachypnea,tachycardia,decrease breath sound,prolong epiratory wheezing,decrease oxygen satuation,use accessory
muscle,possible pulsus paradoxus
Diagnosis:
ABGs : mild hypoxia and respiratory alkalosis
Spirometry/PETs: decrease FEV1/FVC, increase residual volume and TLC
CBC: อาจมี eosinophil
CXR: hyperinflation
Methacholine challenge : test bronchial hyperreponsiveness
Treatment:
Acute: oxygen,bronchialdilating agent ( first line > bata2 >>>agonist,ipratropium,>>>>systemic steroid,>>>intubation in
severe case)
ในผู้ป่วยรายนี้รักษาด้วย inhalation drug ไม่ได้ผลแล้วก็เปลี่ยนมาเป็น systemic steroid นะ จากโจทย์ก็มีแค่ IV Dexamathasone
นะ Ha เธอ

30.EKG : bradycardia ต่อมาตรวจ V/S พบว่าผู้ป่วยมีความดันโลหิตตก (80/50 mmHg, HP 30 )การรรักษาที่เหมาะสมที่สุดของผู้ป่วยรายน


นี้คือข้อใด
a. atropine
b. dobutamine

จากโจทย์ EKG นี้จัดเป็น arrhymia ที่ทาให้เกิดภาวะ bradycardia ซึ่งสาเหตุการเกิดก็มีอยู่ 2 ประเภทคือ


1.bradyarrhythmias
2.conduction abnormalities
Type etiology Sign and symptom EKG treatment
sinus bradycardia เป็นการตองสนอง -อาจไม่มีอาการเลย Ventricular rate<60 -ไม่ต้องรักษาถ้าไม่มี
- คนปกติ ที่ปกติอยู่แล้วของ - หรืออาจมาด้วย bpm มี normal p อากการ
- คนแก่ ระบบ lightheadadedness,syncope,c wave เกิดก่อนทุก -ผู้ป่วยคนนนี้มีอาการ
- โรคหัวใจ cardiovascular hest pain,hypotention QRS complex ยาที่ใช้ในการรักษาก็มี
( MI ,sick system ซึ่งอาจ atropine เพื่อ เพิ่ม HR
sinus เกิดจาก sinus or ทา pacemacker
syndrome) node replacement ซึ่งเป็น
- ยาที่ว่าไป dysfunction or definite treatment in
- Myxedema ผลของยา beta – severe case
- Mental blocker or
depression calcium channel
- Hyperventril blocker(CCB) ที่
ation ใช้มากเกินไป
syndrome

First-degree AV block -เกิดเองได้ asymptomatic PR interval >200 None necessary


-เกิดจากการใช้ยา
bata blocker or
CCB or คนนั้นมี
vagal tone เพิ่ม
มากขึ้น
Secondary –degree Drug มักไม่มีอาการ -PR interval ค่อยเพิ่ม - หยุดใช้ยา
AV block(Mobitz I) effect(digoxin,ba แล้วก็ เริม reset PR
ta block,CCB) interval ใฟม่อีกรอบ
หรือ มีเพิ่ม vagal
tone
Secondary –degree เป็นผลจาก มักมีอาการ syncope or -unexpect beat โดย pacemacker
AV block(Mobitz II) fibrotic disease progression เป็น third –degree ไม่มีการเปลีย่ แปลง PR replacement
ที่เกิดในระบบ AV block inteeval
conductive
system or
previous MI
Third –degree AV No electrical Syncope,dizziness,acute ว่า หาความสัมพัน pacemacker
block(Complete) communication heart ระหว่าง P wave and replacement
ระหว่าง atrial failure,hypotension,cannon A QRS complex ไม่ได้
and ventricle wave

รูปเพื่อนๆดูในหนังสือนะ Ha

นสพ.SomO-lucky>>>>>Odin-Mosa 12-1-52
ถ้าผิดพลาดประการใดอภัยให้ด้วยนะ Ha

31. ผู้ป่วยหญิงอายุ 27 ปี เจ็บคอมา 2 สัปดาห์ PE : white patch เต็มคอและลิ้น


ถามว่าจะส่งตรวจอะไรเพื่อให้ได้เหตุผลของโรคที่ดีที่สุด
จากคาถาม พบ white patch เต็มคอและลิ้น น่าจะเกิดจาก candida หากไม่แน่ใจให้ทา KOH จะพบลักษณะ budding yeast
and/or pseudohyphae
Candidal Thrush : soft white plaques that can be rubbed off, with an erythematous base and possible mucosal burning.
The differential includes oral hairy leukoplakia (lateral border of tongue, not easily rubbed off)
Risk factor : xerostomia, antibiotic use, denture use and immunosuppressed states (e.g. HIV, leukemia, lymphoma,
cancer, diabetes, corticosteroid inhaler use, immunosuppressive treatment)
โจทย์ข้อนี้น่าจะถามว่านึกถึงสาเหตุจากโรคใด และจะตรวจเพิ่มเติมอย่างไร ... เมื่อพบ oral thrush เต็มคอและลิ้นในคนอายุน้อย
น่าจะเกิดจาก opportunistic infection โดยมีสาเหตุจาก HIV ซึ่งวิธีการวินิจฉัยมีดังนี้
1. ELISA test : [ high sensitivity, moderate specificity]  screen test

2. Western blot : [low sensitivity, high specificity]  comfirmator , after ELISA +ve

3. Rapid HIV tests

4. Baseline evaluation should include HIV RNA PCR(viral load) , CD4+ cell count , CXR, PPD skin test, Pap smear,
VDRL and serologies for CMV, hepatitis, toxoplasmosisand VZV.

32. ผู้ป่วยชาย (ให้ EKG อ่านเอง เป็น Mobitz II) จะให้ยาอะไร


First degree AV block
- PR interval > 0.20 sec และมีค่าคงที่ตลอด
Second degree AV block
- ลักษณะสาคัญคือ มี P wave บางตัวไม่มี QRS complex ตามหลัง เนื่องจากกระแสไฟฟ้าไม่สามารถผ่านลงมา depolarize
ventricle ได้ครบทุกครั้ง

Mobitz I (Wenckebach)
 PR interval ค่อยๆยาวขึ้นจนถึง P wave ที่ไม่มี QRS complex ตามหลัง จากนั้นจึงเริ่มวงจรใหม่ (ไฟฟ้าที่
ส่งผ่าน AV node จะถูกกักให้เสียเวลาระดับหนึ่ง และไฟฟ้าที่ส่งมาในจังหวะถัดไปก็จะยิ่งเสียเวลาที่ AV
node มากขึ้นกว่าเดิมตามลาดับ จนกระทั่งจังหวะสุดท้ายจะไม่สามารถลงมา depolarize ventricle ได้
เลย)

 การรักษา : Atropine , Often paroxysmal/asymptomatic , no Rx required

Mobitz II
 EKG เหมือนกับ Mobitz I ต่างกันที่ Mobitz II จะมี PR interval คงที่ ขณะที่ Mobitz I จะยาวขึ้นเรื่อยๆ

 การรักษา : Atropine , Pacemaker often require.

Third degree AV block (Complete block)


 P wave กับ QRS complex ไม่สัมพันธ์กัน เกิดจากไฟฟ้าผ่านลงมาสู้ ventricle ไม่ได้เลย ventricle จะถูก
depolarize โดย pacemaker ส่วนที่อยู่ต่ากว่าบริเวณที่ถูก block

 รักษา : permanent pacemaker


33. ผู้ป่วยหญิงอายุ 68 ปี แพทย์ทาการตรวจร่างกายเบื้องต้นทุกอย่างปกติดี จะให้วัคซีนใด
a. Rabies vaccine
b. Tetanus vaccine
c. HBV vaccine
d. Influenza vaccine
e. HPV vaccine

Rabies vaccine : ถูกหมากัดค่อยมาฉีด


Tetanus vaccine : 1 dose Td booster ทุก 10 ปี
HBV vaccine : จะให้ในผู้ใหญ่ที่เป็น high risk โดยดูจาก clinical, behavioral or travel exposure เช่น คนที่ต้องสัมผัสกับเลือดหรือส้วน
ประกอบของเลือด ,ต้อ$งเดินทางไป endemic country, ผู้ป่วยที่ทา hemodialysis
Influenza vaccine : routine annual administration to individual with chronic illness at any age, and to all adults > 50 yrs of age.
HPV vaccine : recommended for all woman aged <= 26 yrs , และควรได้รับวัคซีนก่อนสัมผัสเชื้อทางเพศสัมพันธ์
34.ผู้ป่วยหญิงอายุ 18 ปีมีอาการเจ็บหน้าอกโน้มตัวไปข้างหน้าอาการดีขึ้น PE lung:clear มีเสียง rub ที่ chest คิดว่าเป็น
อะไร
a. Endocarditis c. Myocarditis
b. Pancarditis d. Pneumonia
ตอบ B. pancarditis ref. medicinenet.comและอาจารย์วันชาติ
( ขออธิบาย pericarditis นะ )
Acute pericarditis is an inflammation of the sac surrounding the heart --- the pericardium --- usually lasting < 6
weeks
สาเหตุ
(35%) Neoplastic ( 6%) Bacterial (other than tuberculosis)
(23%) Autoimmune ( 6%) Uremia
(21%) Viral - adenovirus, enterovirus, ( 4%) Tuberculosis
cytomegalovirus, influenza virus, hepatitis B virus, ( 4%) Idiopathic
and herpes simplex virus, etc
(remaining) trauma, drugs, post-AMI, myocarditis, dissecting aortic aneurysm, radiation
อาการpericarditis โดยมีอาการดังนี้
Chest pain เป็นmost common มักเจ็บลักษณะ sharp and stabbing
อาการปวดจะเป็น arise slowly or suddenly and can radiate directly to the back, to the neck or to the arm.
อาการปวดจะแย่ถ้าทาท่า lying flat ,หายใจเข้าสุด (pleuritic)และดีขึ้นเมื่อ leaning forward.โน้มตัวไปข้างหน้า
ตรวจร่างกายของpericarditis
pericardial rub very specific sign of acute pericarditis แต่ไม่ได้ยินไม่ได้ rule out โรคนี้นะพี่น้อง เสียงมันเกิดจากการ frictio
generated by the two inflamed layers of the pericardium

a. Infectious Endocarditis
เมื่อดูตามDuke Criteria แล้วไม่ตรงกับ Major CriteriaและMinor Criteria Clinical criteria for infective endocarditis
requires:
“ Two major criteria, or
“ One major and three minor criteria, or
“ Five minor criteria จึงคิดถึงน้อย
c.myocarditis
1.อาการจะมาด้วยเจ็บที่หน้าอก stabbing
2.เป็นหนักheart failure ,edema,breathlessness,hepatic congestion
3.Palpitations (due to arrhythmias)
4. Sudden death (in young adults, myocarditis causes up to 20% of all cases of sudden death
5.Fever (especially when infectious, e.g. in rheumatic
d.Pneumonia ref.สมาคมอุรเวชช์แห่งประเทศไทย
clinical
1.New pulmonary infiltration -Fever,cough , +/- productive sputum , Dyspnea ,
2.Acute onset(duration <= 2 weeks) pleuritic chest pain , consolidation or crackles on PE.
3.symptoms and signs of LRI(3 in 5)
35. ผู้ป่วยชาย อายุ 16 ปี เป็น Homozygous Beta-thal ต้องรับเลือดเป็นประจา มีอาการอ่อนเพลียและบวมมา 1 เดือน แพทย์
ทาการตรวจร่างกายเบื้องต้น BP drop , irregular heart rate & rhythm , Hepatosplenomegaly ถามว่า ผู้ป่วยมี heart
complication จากอะไร
A. cardiac siderosis
B. cardiac hypertrophy
C. immune cardiomyosotis
D. Extramedullary hematopoiesis at heart

คาตอบ A. cardiac siderosis


แนวคิด
ผู้ป่วยเป็น Homozygous Beta-thal ต้องรับเลือดเป็นประจา เป็นระยะเวลานาน = Chronic blood transfusion และ
มีการสร้างเม็ดเลือดแดงที่มากขึ้นมีการดูดซึมธาตุเหล็กจากระบบทางเดินอาหารเพิ่มขึ้น ทาให้เกิดมี iron overload มีการสะสม
ตามอวัยวะต่างๆ นาไปสู่ภาวะของ Hemochromatosis ได้

ลักษณะที่พบ
1. Hepatomegaly พบได้ 95% ไม่มีอาการผิดปกติอื่นๆ , LFT normal
2. Splenomegaly
3. Excessive skin pigmentation พบได้ 90%
4. DM พบได้ 65% เชื่อว่าเกิดจาก Direct damage islets by iron deposit
5. Arthropathy มักพบในผู้ป่วยอายุมากกว่า 50 ปี
6. Cardiac involvement พบได้ 15% : Congestive heart failure
Cardiac arrthymia : PVC , AF, A flutter

ลักษณะที่พบ insidious onset  Pericarditis  dysarrthymia  Pump failure

ในผู้ป่วยรายนี้อาการค่อยๆเป็นไปในช่วงเวลา 1 เดือน (อาการอ่อนเพลีย+บวม)


ตรวจร่างกายพบ, irregular heart rate & rhythm , BP drop เข้าได้กับมีภาวะ dysarrthymia และ Pump failure
Hepatosplenomegaly สามารถอธิบายได้ด้วย Hemochromatosis ข้างต้น
36.ผู้ป่วยชายอายุ 20 ปี motorcycle accident ทา CT พบ crescent shape at frontl region ในผู้ป่วยรายนี้น่าจะคิดถึงโรคใด
มากที่สุด
a. acute epidural hemorrhage
b. acute subdural hemorrhage
c. subacute epidural hemorrhage
เฉลย.b. acute subdural hemorrhage
Subdural hematoma
 subdural space = potential space bet .pia - arachnoid mb. + dura matter
 freely extending across suture lines
 limited by interhemispheric fissure + tentorium
 CT : extra - axial peripheral crescentic fluid collection
: hyperdense (< 1 wk) / isodense(1-2 wks) /hypodense (3-4 wks)
Epidural hematoma
 within potential space bet. inner table + calvarial periosteum
 bound down at suture margins
 Associated with skull fracture in 75 - 95 %
 CT : lens - shaped, smoothly demarcated high density ( over the surface of the hemisphere )

Ref. เอกสารประกอบการเรียนรังสีวิทยา อ.พญ.วาณีรัตน์ กาฬสีห์


2. clinical rheumatoid arthritis ยาใดลดปวดเร็วสุด
จ้าเล่าเกี่วกับ RA
a. Ibuprofen**
ข้าวๆนะ เจี๊ยกๆๆ
b. Paracetamol
c. MTX
d. Sulfazalazine
e. Chloroquine Film x-ray
 jt spaces narrowing แบบ diffuse เท่ากัน
ทั้ง jt line
คิดถึงเมื่อ
 marginal erosion แยกกะ OA เป็น
1. มี sign ของ inflammation
central erosion
2. คลาพบ synovial thickening 
3. เจอ rheumatoid nodule
4. Morning stiffness
5. chronic

Dx โดย 1-2-3+specific
1 ชั่วโมง= morning stiffness>1 hr
2 ข้าง symmetrical polyarthritis
3 ข้อ
Spec= rheumatoid nodule Tx
(นิ้ว,extensor area) 1. ให้ early intensive Tx ลด permanent

= rheumatois factor+ 2. ให้ NSIAD ไว้ก่อน รอ ยาออกฤทธิ์


3. Drug of choice=DMARDS(combine)
= radio change
=อาการที่มือ > 6 mo DMARDS

1. Hydrochloroquine
S/E retinal toxicity & diarrhea
ข้อที่พบ บ่อย ข้อที่ขยับมากๆ 2. Sulfasalazine
มือ+> MCP,PIP spare TL-spine,DIP S/E bone marrow suppression
3. Metrotrexate
คอ +>C 1-2subluxation ได้บ่อย S/E BM, liver , lung
4. Gold salt & azathiopine
S/E BW , renal toxic

Note (ไม่ต้องจำก็ได้ )
ยาอื่น ก็พวก immune suppressive:
cyclophosphamine, TNF
,adalimumab,etanercept,infiximab

===== ข้อนี้เลยตอบ NSAIDs ========


39. ชายอายุ 22 ปี หมดสติในห้องน้า ไม่มีไข้ ตรวจร่างกาย PR ต่า RR ต่า miosis การรักษาที่เหมาะสมที่สุดในผู้ป่วยรายนี้คือ
ข้อใด
a. atropine
b. naloxone
c. diazepam
เฉลย ข้อ b. aof_pa!!
วิเคราะห์โจทย์ : ชาย วัยรุ่น  มาด้วยหมดสติ PR ต่า RR ต่า miosis ซึ่งมีอาการคล้ายกลุ่ม opioids มีอาการ euphoria, ซึม
ลง หมดสติ N/V ท้องผูก miosis (ม่านตาเล็กมาก) กดการหายใจ
 ไม่รู้สึกตัว ความดันต่า flaccidity  benzodiazepine,barbiturate,ethnal

 ไม่รู้สึกตัว หายใจช้ามาก pinpoint pupils  opiate

 น้าตาไหลพราก น้าลายฟูม N/V miosis  organophosphate/carbamate

Antidose ของสารพิษ
1. Acetaminophen  N-acetylcysteine

2. Benzodiazepine  flumazenil

3. Carbamate(not stable bond)/organophosphate( stable bond) atropine(muscarinic receptor bloker) and


2-PAM ( ใช้- organophosphate  reactivation ของ acetylcholinesterase)

4. Iron  deferoxamine

5. Narcotics  naloxone

40. หญิง 40 ปี หนัก 80 kg สูง 160 cm เพิ่ง Dx DM,


FBG 180 การรักษาที่เหมาะสมที่สุดในผู้ป่วยรายนี้คือ
ข้อใด
a. ควบคุมอาหาร
b. Metformin
c. Acarbose
d. Glibenclamide

ตอบ a. ควบคุมอาหาร
ข้อนี้ pt. เพิ่ง Dx DM ก็เลยตอบว่า a.
ควบคุมอาหาร ไป 2 เดือนก่อน
Evidence-based clinical practice guideline 2548
หน้า 452
ข้อ 41) ตอบ c.
เนื่องจากผู้ป่วยรายนี้มีอาการของภาวะ Grave’s disease ซึ่งมีระดับอาการดังนี้
อาการรุนแรง มีโรคแทรกซ้อนที่อันตราย เช่น โรคหัวใจล้มเหลว, หัวใจเต้นผิดจังหวะ ชนิด AF, myopathy, thyrotoxic
periodic paralysis,cachexia
อาการปานกลาง น้าหนักลดมากกว่า 5 kg, pulse rate > 120 bpm, มีอาการเหนื่อยมากไม่สามารถทางานได้ หรือไม่
สามารถออกกาลังกายมากกว่าที่ทาตามปกติได้
อาการน้อย ไม่มีอาการที่ระบุไว้ข้างต้น
 การเลือกการรักษาดังนี้
1.ควรเลือก long term medical treatment:
- อายุน้อยกว่า 30 ปี
- อาการไม่มาก
- เป็นมาน้อยกว่า 6 เดือน
- ต่อมขนาดโตไม่มาก (น้อยกว่า 3 เท่า, 45 g)
2.ควรเลือกการผ่าตัด :
- อายุน้อยกว่า 20 ปี
- ต่อมขนาดโตมาก(มากกว่า 5 เท่า, 75 g)
- มีอาการระดับปานกลางหรือรุนแรง
- แพ้ยาต้านthyroid
3.ควรเลือกให้ radioactive iodine:
- อายุมากกว่า 30 ปี
- มีอาการระดับปานกลางหรือรุนแรง
- มีโรคที่เป็นอุปสรรคต่อการผ่าตัด เช่นโรคหัวใจ โรคปอด
- แพ้ยาต้านthyroid
- มีการกลับเป็นซ้าของโรคเมื่อให้การรักษาด้วย long term medical treatment หรือ
การผ่าตัด
 การรักษา
1. long term medical treatment
methimazole นัดติดตามอาการ 1 เดือน ตรวจระดับ FT3, FT4 หลังให้ยา 2 เดือน ปรับยาตามระดับอาการและ
ฮอร์โมน แนะนาให้ยาจนครบ 2 ปีและตรวจระดับ FT3, FT4 ก่อนหยุดยา
B-blocker ถ้าผู้ป่วยมีอาการใจสั่น ใจเต้นเร็ว
ในผู้ป่วยที่ตั้งครรภ์ให้ PTU แทนและเลี่ยง B-blocker
2. RAI
ให้ iodine 131 ขนาด 100 ไมโครคูรี่/กรัม ของต่อม แต่ต้องระวังในข้อห้าม คือ ภาวะตั้งครรภ์ และให้นมบุตร
3. การผ่าตัดแนะนาให้ทา subtotal thyroidectmy
>>>> ในผู้ป่วยรายนี้มีอายุน้อย และมีอาการไม่มากน่าจะให้การรักษาด้วยยาก่อน ซึ่ง PTU สามารถให้แทน methimazole
ได้ ...........ref: evidence-base CPG ของจุฬา ฯ by ReaL……
42. ชายอายุ 32ปี มีอาการใจสั่น น้าหนักลด BP 150/100 mmHg PR 120 /min PE: neuro sensory normal, muscle weak
gr.1/5 both upper and lower , reflex 2+all
Na 136 mmol/l K 2.5 mmol/l
Cl 102 mmol/l CO2 22 mmol/l
ต้องส่งตรวจอะไรเพื่อเป็นการหาสาเหตุของการอ่อนแรง
a. Free T4
b. plasma glucose
c.CT brain
d.CSF profile

ในผู้ป่วยรายนี้มาด้วยเรื่องแขนขาอ่อนแรงทั้งสองข้าง สามารถ DDx.ได้ดังนี้


Differential diagnosis
Electrolyte imbalance : K+ , Ca+(hypo ,hyper)
Muscle disorder : polymyositis , alcoholic myopathy parasitic polymyositis Neuromuscular : myasthenia gravis
Polyneuropathies : Guillain-Barre syndrome
- จากผลelectrolyte พบว่ามีภาวะ Hypokalemia
Hypokalemia
Cause:
Inadequate intake ” at least 10-30 mEq/day
Excessive renal losses ” diuretic use, metabolic alkalosis, trauma and stress, ↑aldosterone level
Excessive losses from GI tract and Skin
Redistribution: ECF→ICF
- Excess insulin
- Excess -adrenagic catecholamines
- ในผู้ป่วยรายนี้อธิบายได้ว่าน่าจะมี Hypokalemic Paralysis ซึ่งสาเหตุของการเกิด Hypokalemic Paralysis ได้แก่
Thyrotoxic periodic paralysis Hyperaldosteronism
Insulin excess Infectious enteritis
Renal tubular acidosis Short bowel syndrome
จากที่ผู้ป่วยรายนี้ไม่มีประวัติ DM ,โรคไต , การติดเชื้อ หรือได้รับการผ่าตัดมาก่อน จึงทาให้นึกถึง thyrotoxic periodic
paralysis มากที่สุด

ผู้ป่วยที่มีภาวะthyrotoxocosis ผลของthyroid hormone ทาให้กระตุ้นระบบ sympathetic จึงทาให้มี PR&BP สูงได้ ผู้ป่วยโรค


นี้จะมีภาวะอยากอาหารมากขึ้น เมื่อกินอาหารที่ high carbohydrate จะกระตุ้นการหลั่ง insulinออกมามาก insulinที่หลั่ง
ออกมาจะกระตุ้นการทางานของNa+K-ATPase ทาให้K+ ถูกปั๊มเข้าเซลล์ จึงทาให้เกิด hypokalemiaมีผลให้กล้ามเนื้ออ่อนแรง

ดังนั้น ในการยืนยันการวินิจฉัย ควรจะส่ง investigation คือ thyroid function test (ในโจทย์ตอบ free T4)

เฉลย bY : ต้อง MeD X


43. ผู้ป่วยชายมีอาการใจสั่น มี muscle weakness ตรวจ DTR 1+, กล้ามเนื้อแขนขาอ่อนแรง 1+, ลุกไม่ขึ้น K 2.5 ถามว่า
ตรวจอะไรเพิ่มเติม
a. Free T4

คนไข้มาด้วย proximal muscle weakness ให้นึกถึงพยาธิสภาพ ดังนี้


- Myopathy
- NMJ disease
- Neuropathy
- Anterior horn cell disease
และจากผล lab พบว่า K ต่า ดังนั้น สาเหตุของการ weak น่าจะเกิดจาก Hypokalemia (ทาให้เกิด myopathy) และในผู้ป่วย
รายนี้มีอาการใจสั่น ซึ่งทาให้คิดได้ว่า Hypokalemia อาจเกิดจากภาวะ thyrotoxic hypokalemic periodic paralysis ได้
ดังนั้นจึงควรส่งดู thyroid function test
อาการกล้ามเนื้ออ่อนแรง อันเนื่องมาจาก hypokalemia เป็นอาการแสดงหนึ่งของ thyrotoxicosis ซึ่งพบได้บ่อยในเอเชีย โดย
ผู้ป่วยจะมีอาการกล้ามเนื้ออ่อนแรงร่วมกับอาการแสดงอื่นๆของภาวะ hyperthyroidism ดังนั้นในผู้ป่วยรายนี้มีอาการดังกล่าว
จึงสงสัยภาวะนี้มากกที่สุด

44.หญิง 15 ปี มี webbed neck ไม่มีเต้านม ข้อใดถูก


a. Insulin ต่า
b. GH ต่า
c. Estrogen ต่า
d. Gonadrotropin สูง
e. Parathyroid hormone สูง
ตอบ c. Estrogen ต่่า
ผู้ป่วยรายนี้เป็น Turner’ syndrome พบได้ 1:2,500 เพราะมีลักษณะที่เข้าได้ดังนี้
webbed neck,หน้าอกนูนขึ้นเหมือนโล่ (shield chest ) ,มุมของข้อศอกกางออกมากกว่าขึ้น (increased carrying angle ) ขาด
ระดูแบบปฐมภูมิ ไม่มีการพัฒนาลักษณะเพศขั้นที่สอง มีความผิดปกติของการพัฒนาเต้านมเมื่อเข้าสู่วัยสาว , ส่วนขนที่
รักแร้และหัวหน่าวอาจมีได้ขึ้นกับในแต่ละ mosaic นั้นๆ, แต่มีสติปัญญาปกติ pigmented nevi , lymphedema เมื่อคลอด ,
ความผิดปกติของหัวใจ , ความผิดปกติของไตที่พบบ่อย คือ horseshoe kidneys , อาจพบความผิดปกติของหลอดเลือดใหญ่ ที่
พบบ่อยได้แก่ coarctation of the aorta , อาจพบ DM , thyroid , HT , autoimmune ,
turner syndrome ส่งผลทาให้ ovaries เกิด non-function ซึ่งผลดังกล่าวทาให้ ovaries ไม่สามารถ ผลิต sex
hormones(estrogen and progesterone) ออกมาได้
ดังนั้นผู้ป่วยรายนี้จะขาด Estrogen
ส่วนการรักษา
1.เนื่องจาก turner’ syndrome 45/X0 ซึ่ง gene X นี้เป็นส่วนสาคัญใน long bone growth เมื่อ ตัว gene X หายไป
จึงเป็นปัญหาต่อ skeletal features จะพบshort fingers and toes, and irregular rotations of the wrist and elbow joints ได้
ดังนั้นจึงให้ growth hormone (GH) treatment ช่วย in childhood.
2. Estrogen-progesterone treatment to maintain their secondary sexual development and to protect their
bones from osteoporosis until ~ the usual age of menopause (50 years) เหมือนเพื่อนๆคณะเรา คงขาดestrogen กัน
เยอะเลย เฮยๆ ทาไงดีหล่ะ
45. ชาย 30 ปี ไอมา 1 เดือน เสมหะขาว มีเลือดปนบางครั้ง ไข้ต่าๆ น้าหนักลด 3 กก. ตรวจร่างกาย : ปกติ
CXR : Reticulo-patchy infiltration with thick wall cavity, AFB sputum negative x 3 ครั้ง ควรทาอะไรต่อ
A. bronchoscope
B. FNA
C. Anti ” TB drug
D. รอผล culture for TB

คาตอบ D.รอผล culture for TB


แนวคิด
การวินิจฉัยวัณโรคปอด
1) อาการและอาการแสดง อาการที่น่าสงสัยว่าเป็นวัณโรคปอด ได้แก่อาการไอเรื้อรัง โดย
เฉพาะหากไอนานเกิน 3 สัปดาห์ หรือไอเป็นเลือดสาหรับอาการอื่นๆ เช่น อาการเหนื่อย อ่อนเพลีย
เบื่ออาหาร น้าหนักลดโดยไม่ทราบสาเหตุ เหงื่อออกตอนกลางคืนหรือเจ็บหน้าอก อาจเป็นอาการของ
วัณโรคได้ แต่มีความเฉพาะเจาะจงน้อยกว่าอาการไอเป็นเลือด
2) การถ่ายภาพรังสีทรวงอก เป็นการตรวจที่มีความเฉพาะเจาะจงต่า คือความผิดปกติที่
เห็นอาจจะไม่ใช่เกิดจากวัณโรคก็ได้ โดยอาจจะเป็นเงาเปรอะเปื้อนบนฟิล์ม เป็นเนื้องอกหรือมะเร็ง หรือ
เกิดจากโรคติดเชื้ออื่นๆ การที่จะวินิจฉัยว่าผู้ป่วยเป็นวัณโรคปอดจึงต้องการทาร่วมกับการตรวจเสมหะหาเชื้อวัณโรคด้วยเสมอ
3) การตรวจเสมหะหาเชื้อวัณโรค
3.1 การย้อมเสมหะและตรวจด้วยกล้องจุลทรรศน์
ในทางปฏิบัติให้ตรวจเสมหะที่ผู้ป่วยไอออกมาในโอกาสแรกที่พบแพทย์ และวันต่อๆมารวมเป็น 3
ครั้ง วิธีเก็บเสมหะที่ถูกต้องมีความสาคัญมากจะต้องอธิบายให้ผู้ป่วยเข้าใจว่าต้องเป็นเสมหะที่ไอจากส่วนลึกของหลอดลมจริงๆ
ไม่ใช่นาน้าลายมาตรวจ
3.2 การเพาะเชื้อวัณโรค
ทาการเพาะเชื้อในรายที่อยู่โรงพยาบาลที่สามารถจะทาการเพาะเชื้อได้หรือในรายสงสัยว่าจะเป็นวัณโรคแต่เสมหะไม่พบเชื้อเพื่อ
ยืนยันการวินิจฉัย
ในผู้ป่วยรายนี้ อาการและอาการแสดง การถ่ายภาพทางรังสีทรวงอก ทาให้คิดถึงโรควัณโรคมากที่สุด แต่การตรวจ
เสหมหะไม่พบเชื้อวัณโรค ซึ่งอยู่โรงพยาบาลที่สามารถจะทาการเพาะเชื้อได้ก็ควรทา เพื่อยืนยันการวินิจฉัย ก่อนที่จะให้ยารักษา
วัณโรค
โดยจากโจทย์คิดว่าผู้ป่วยส่ง culture for TB ไปแล้วจึงควรรอผลก่อน
ในกรณีที่อยู่ในโรงพยาบาลที่ไม่สามารถในการเพาะเชื้อได้แต่ผู้ป่วยมีอาการและภาพถ่ายรังสีทรวงอกที่เข้าได้อาจ
พิจารณาให้ยารักษาวัณโรค
46. ชายไทย30 ปี อาชีพขับรถ bus มีไข้ต่า ไอแห้ง น้าหนักตัวลดลง ตรวจพบ AFB 3+ แพทย์ให้ยา TB ควรเขียนให้ลาหยุดงาน
นานเท่าใด
a. 2 wk
b. 1 mo
c. 2 mo
d. ทีละ 1 wk จนกว่า AFB จะ neg
e. ไม่จาเป็นต้องหยุดงาน
เฉลย. c. 2 mo
47. ชาย อายุ 40 ปี โรคประจาตัวเป็น Asthma เป็นหวัด 2 วัน PTA พ่นยาแล้วอาการไม่ดีขึ้นแพทย์ทาการตรวจร่างกายเบื้องต้น
T 37.4 oC BP 110/90 mmHg PR 100 bpm RR 28 /min Lung: wheezing at both lower lung with accessory muscle
used การรักษาที่เหมาะสมที่สุดในผู้ป่วยรายนี้คือข้อใด
b. Terbutaline Sc
c. IV steroid
d. IV dexamethasone
e. IV Theophyline
f. NB salbutamol

>>ในช่วง acute ให้ O2 mask พ่น B2 agonist ก่อน ก็ได้แก่


terbutaline , salbutamol ทุก 15-30 min ก่อน อาจให้ dexa iv or
prednisolone oral ไม่ดีคุยให้ anticholinergic , ipratropium
bromide ตารางข้างบนนะ

>> ทีนี้มาดูการรักษาและให้ยาใน asthma ธรรมดากันดีกว่าออก


ข้อสอบเยอะอยู่นะ ส่วนใหญ่เห็นถามเรื่องการให้ยา ไม่ค่อยถาม DX
>เริ่มจากแบ่งกลุ่มเป็น control / partial /uncontrol ให้ยา
ตามภาพเลยอ่าต้องใช้ความจากันนิดนึงนะ น่าจะคุ้มนะออกตรงๆ
มั้ง (reference GINA 2006)

จะจาได้ได้มั้ยเนี่ย!!!!!!!!!!!!!!!!!
หนัก อก หนักใจจริงๆ (o^o)
49. ผู้ป่วยหญิงมาด้วยอาการผื่น มี สะเก็ด เป็นทั่วศีรษะ ข้อเข่า ข้อศอก เกาแล้วมีผื่นขึ้นแกะสะเก็ดออกมีเลือดออกเล็กน้อย ใน
ผู้ป่วยรายนี้น่าจะคิดถึงโรคใดมากที่สุด
a. Psoriasis
b. Lichen planus เป็นกลุ่ม papulosquamous – skin lesion
c. Discoid lupus erythematosus ลักษณะเด่น 4 อย่าง คือ
เฉลย ข้อ a. aof_pa!! 1. ผื่นสีแดงนูนมีขอบเขตชัดเจน
Psoriasis
2. มี non-coherent silvery scale ปกคลุม

3. Erythematous skin ใต้สะเก็ดมีสีแดงเข้ม

4. แกะสะเก็ดพบจุดเลือดออกเล็กๆๆ

ตาแหน่งที่พบบ่อย- ศีรษะ,ข้อศอก,ข้อเข่า
Tongue lesion : geographic tongue
Nail lesion : pits , onychodystrophy
Lab: increase uric acid , high ESR ,
hypocalcium
Tx. : tar / antralin /salicylic acid +steroid

Lichen planus
ลักษณะเด่น 4P คือ 1. Purple ผื่นสีแดงม่วง
2. Polygonal ผื่นมีลักษณะหลายเหลี่ยม
3. Pruritic คันมาก
4. Papule ตุ่มนูนแบนๆ บริเวณผิวด้านหน้ามีสะเก็ดบางๆ
ติดแน่น Wickham striae
ตาแหน่ง- ขามากกว่าแขน ,หลัง,ลาตัว
Nail lesion :
brown discoloration , pterigium

เป็น autoimmume disease ชนิดที่มีแสงแดดเป็นตัวกระตุ้น


Discoid lupus erythematosus
ชนิด chronic cutaneous LE มีลักษณะเป็นตุ่มแดง/ปื้นแดง
หนาตัวเพิ่มขึ้น ขอบเขตชัด ตรงกลางบางมีสะเก็ดติดแน่น (
adherent scale) มีการอุดตันของรูขุมขน (follicular
plugging) ดึงสะเก็ดออก พบขุยหนาติดใต้สะเก็ด (
carpettack sign)
ตาแหน่งที่พบ- หน้า,หนังศีรษะ,คอ,แขนด้านนอก
50. Pt 50 yr มาด้วย ตัวเหลืองตาเหลือง 1wk แพทย์ทาการตรวจร่างกายเบื้องต้น, cholesterol 350, TG 250 อาจจะพบอาการ
แสดงทางผิวหนังอะไร
a. Eczema
b. Lipoma
c. Xanthelasma
d. Seborrheic dermatitis
ตอบ c. Xanthelasma
ผู้ป่วย Dyslipidemia ตรวจร่างกายหา tendon xanthoma, xanthelasma, corneal arcus, palmar xanthoma,
eruptive xanthoma, อาการแสดงของ hypothyroid, edema, ตรวจ reflex
Evidence-based clinical practice guideline 2548 หน้า 181, 184

คนปกติควรมีระดับไขมัน เกณ์ระดับไขมันสูง
Cholesterol < 200 > 240
TG < 150 > 200
LDL < 100 > 130
HDL > 40 < 40

ข้อ 51) ตอบ ข้อ e.A,C (propranolol,flunarizine)


Indication for Migraine Prophylaxis
1. อาการปวดรุนแรงจนทาให้ไม่สามารถปฏิบัติภาระกิจได้ > 2 ครั้งต่อเดือน
2. มีข้อห้ามหรือใช้ยาในการรักษาระยะเฉียบพลันไม่ได้ผล
3. ต้องใช้ยาหยุดยั้งอาการปวดมากกว่า 2 ครั้งต่อสัปดาห์
4. มีโรคไมเกรนบางอย่าง เช่น Hemiplegic migraine ,ไมเกรนที่มี aura นาน,migrainous infarction

ยากลุ่มแรกที่พิจารณา : Propranolol, Timolol, Amitriptyline, Divalproex sodium , Sodium valproate

ยากลุ่มอื่น : Flunarizine, lissuride, pizotifen, time-released dihydroergotamine, methysergide

ส่วนยาที่ใช้บรรเทาอาการ (abortive therapy):ASA, acetaminophen, caffeine, high-dose NAIDs, metoclopramide,


5-HT1 agonists(triptans), ergotamine

Ref:evidence-base CPG, pocket medicine…………………………….by ReaL


52. ผู้ป่วยชาย 30 ปี ปวดศีรษะมากทันที เวียนหัว คลื่นไส้อาเจียน drowsinsess, BP 150/110 , PR 80 , RR 22 , stiffness of
neck ,Babinski sign ”neg , retinal a. absent of pulse ,
no papilledema ในผู้ป่วยรายนี้นึกถึงโรคใดมากที่สุด
a. migraine
b. viral encephalitis
c. subarachnoid hemorrhage
d. bacterial meningitis

.ในผู้ป่วยรายนี้มาด้วยอาการที่ typical ของSAH คือ


- sudden onset of severe headache >>ปวดมากแบบที่ไม่เคยเป็นมาก่อน
- บริเวณที่ปวดจะเป็นแบบ localized หรือ generalizedก็ได้ อาการอาจจะเบาลงหรือหายไปได้เองหรือดีขึ้นเมื่อใช้ non-narcotic
analgesic
-อาการอื่นๆ : คลื่นไส้ อาเจียน , alteration of consciousness , seizure, neck pain
-ตรวจร่างกายพบ : BPสูง , stiff neck +ve

สาเหตุของการเกิด SAH ที่ most common คือ intracranial aneurysm พบได้60-70% (spontaneous SAH)
รองลงมาคือ Peri-mesencephalic hemorrhage พบได้ 15-20% (non-(spontaneous SAH) รองลงมาอีกคือ Ateriovenous
malformations and associated aneurysms(AVM) พบได้ 5%
Lab investigation
1. CT scan >>ขึ้นอยู่กับปริมาณเลือดที่ออก , ระยะเวลา (senseสูงสุดใน 24 ชม.แรก)
- NECT จะพบว่ามีเลือดเซาะไปตาม sulci , gyri หรือตกอยู่ใน ventricle
2. Lumbar puncture >>
- elevate opening pressure
- ต้องแยกกับ traumatic tap โดยต้องเจาะ 4 tube ซึ่งถ้าเป็นSAH เลือดจะไม่จางลงทั้ง
4 หลอด
- Xanthochromia >> สีจะจางลงเป็นสีเหลืองๆเมื่อเทียบกับน้าเปล่า ต้องดูที่เวลาอย่างน้อย 12 ชม.หลังเจาะ
peak ที่ 7 วัน อยู่ได้ประมาณ 14 วัน

ส่วนตัวเลือกข้ออื่นๆ นึกถึงได้น้อย
- ไม่นึกถึง migraine เพราะไม่ได้มีประวัติปวดศีรษะแบบตุบๆ ที่เป็น typicalของmigrain ไม่มีประวัติ auraมาก่อน
-ไม่นึกถึง viral encephalitis เพราะไม่focal neuro deficit
-อาจนึกถึง bacterial meningitisได้ อาจต้องอาศัย LP&CSF profile ช่วยในการวินิจฉัยแยกโรค แต่ผู้ป่วยรายนี้ ไม่มีไข้มาก่อน
และอาการที่ typical ของ SAHมากกว่า

เฉลย bY : ต้อง MeD X


53. หญิง 35 yr หนังตาตกมา 4 wk เป็นมากขึ้นตอนบ่ายๆ ตรวจตาซ้ายพบ ptosis มี mild proximal muscle weakness,
normal DTR, normal sensory จง Dx
a. Polymyositis
b. Horner’s syndrome
c. Hyperthyroidism
d. Myasthenia gravis
e. CN III palsy
คนไข้มาด้วย proximal muscle weakness ให้นึกถึงพยาธิสภาพ ดังนี้
- Myopathy
- NMJ disease
- Neuropathy
- Anterior horn cell disease
ลักษณะ pattern of weakness ของผู้ป่วยรายนี้ เข้าได้กับ NMJ lesion และเข้าได้กับ MG มากที่สุด แต่อย่างไรก็
ตาม ก็ควรแยกจากภาวะ Lambert-Eaton myasthenic syndrome (LEMS) ด้วยดังนี้
- มี proximal muscle weakness involve ที่ extraocular and bulbar muscle เป็นอันดับแรก ซึ่งต่างจาก LEMS ที่จะ
มีลักษณะ proximal muscle weakness with ocular sparing
- มีลักษณะ Fluctuating แบบ fatiquable (เป็นมากขึ้นภายหลังการใช้กล้ามเนื้อมัดนั้น หรือ อาจเป็นมากขึ้นในช่วง
บ่าย) เนื่องจาก Ach ถูกใช้ไป ต่างจาก LEMS ซึ่งจะมีกาลังกล้ามเนื้อดีขึ้นภายหลังการออกกาลังกายของกล้ามเนื้อ
มัดนั้น
- กลไกการเกิด MG : acetylcholine receptor antibodies ส่วน LEMS : voltage gated calcium channel
antibodies
- ใน LEMS อาจพบ hyporeflexia and autonomic dysfunction ได้

54.ผู้ป่วยชายอายุ 21 ปี อ่อนแรงขาทั้ง 2 ข้าง ต่อมาอ่อนแรงไปทั้งตัว มา 3 วัน , Total ophthalmoplegia ,bilateral


facial palsy , paradoxical breathing , generalized decrease motor power gr II-III/V, areflexia,loss of vibration sense
all toe and fingers ข้อใดคือการรักษาที่เหมาะสมที่สุด
a. Azatiopine
b. Interferon beta 1A
c. Cyclophosphamide
d. Methlprednisolone
e. Intravenous Immunoglobulin
ตอบ E. Intravenous Immunoglobulin
Ref.ซีต อ.พญ.ปวรมน ปี5.เรื่องspinal cord diseaseผู้ป่วยรายนี้มีขอapproach อาการไปในกลุ่มโรค peripheral
nerve disease เนื่องจาก peripheral nerve disease มี nature of symptom ดังนี้
1. Motor
2. sensory
3. autonomic(sympathetic and parasympathetic)
Motor phenomena
Negative symptom : motor weakness LMN lesion flaccid , atrophy , hyporeflexia , areflexia , distal muscle
weakness
Positive symptom
Fasciculation:sign of lower motor neuron lesion
Fibrillation:spontaneous discharge
Sensory phenomena
Negative symptom
Numbness,loss or absence of feeling , loss of propioception
Positive symptom
Paresthesia
Neuropathic pain
Autonomic phenomena
Orthostatic hypotension
Bowel and bladder dysfunction
Sexual dunction
โรคที่พบบ่อยในdemyelination แยกเป็น
Acute : Guillain barre syndrome
Chronic: chronic inflammatory demyelinatig polyneuropathy CIDP slow progressive or relapsing
Guillain barre syndrome
Onset < 2 wk
Immune process
Demyelination process and axonal degeneration (GBS-varient)
Classic syndrome
1. Proximal muscle weakness
2. loss propioception sene
3. areflexia
4. may have autonomic involvement (autostatic hypotension , urinary incontinence)
รักษา
No role of steroid
high-dose intravenous immunoglobulins (IVIg) at 400mg/kg for 5 days or plasmapheresis can be
administere (Wikipedia)
55. ผู้ป่วยหญิง 70 ปี มีแขนขาซ้ายอ่อนแรงและ EKG พบว่ามี ST elevation ให้ยาใดป้องกันอาการดังกล่าว
A. aspirin
B. Enoxaparin
C. Clopidogrel
D.warfarin
E.Dipyridamole

คาตอบ aspirin
แนวคิด
แนวทางในการดูแลรักษาผู้ป่วย STEMI
1. เมื่อได้รับการวินิจฉัยว่าเป็น STEMI การรักษาในช่วงแรกที่ผู้ป่วยควรได้รับ คือ
- Aspirin 160 ” 325 mg เคี้ยวและกลืนทันที
- Nitrate อมใต้ลิ้นหรือ IV ในกรณีที่มีอาการเจ็บหน้าอกมาก
- ให้นอนพัก และออกซิเจน
2.STEMI ที่เกิดอาการภายใน 3 ชั่วโมงควรได้รับการรักษาโดย Thrombolytic agent หรือ การ
สวนหัวใจหรือขยายหลอดเลือดอย่างเร่งด่วน โดยต้องไม่มีข้อห้ามในการใช้การให้ยาละลายลิ่ม
เลือด

3. หลังจากที่ผู้ป่วยรักษาอาการ stable ดีแล้ว ควรได้รับการรักษาเพื่อไม่ให้เกิด โรคซ้าอีก ได้แก่การ รับประทานยาต้าน


เกร็ดเลือด และลดปัจจัยเสี่ยงต่อภาวะ atherosclerosis

Antiplatelet agents
1. Antiplatelet drugs แบ่งออกเป็นกลุ่มใหญ่ๆ ได้ดังนี้
1.1. Platelet cyclooxygenase inhibitor
1.2. Thienopyridine group
2. GP ( Glycoprotein) IIb/IIIa receptor antagonist
1. Antiplatelet drugs
1.1. Platelet cyclooxygenase inhibitor
ยาในกลุ่มนี้จะยับยั้งการออกฤทธิ์ของเอ็นไซม์ Prostaglandin H-synthase ซึ่งเป็นเอ็นไซม์
ที่สาคัญที่ใช้ในการสังเคราะห์ PGG2 และ PGH2 Arachidonic acid, PGH2 จะถูกสังเคราะห์ต่อไปเป็น
Thromboxane A2 มีบทบาทสาคัมีบทบาทสาคัญของกลไกของการเกิดการจับตัวของเกล็ดเลือด
ยาที่สาคัญในกลุ่มนี้คือ Aspirin (แอสไพริน)
ข้อบ่งชี้ของ Aspirin
1) ใช้ป้องกันกล้ามเนื้อหัวใจตายเหตุขาดเลือดขั้นปฐมภูมิ (primary MI prevention) เฉพาะใน :ผู้ที่มีความเสี่ยงต่อโรค
หลอดเลือดเลี้ยงหัวใจภายใน 10 ปี (10-year CAD risk) ตั้งแต่ร้อยละ 10 ขึ้นไป (American Heart Association 2002)
” ผู้ป่วยโรคเบาหวาน อายุ 40 ปีขึ้นไป (American Diabetes Association 2004)
- ผู้ที่มีอายุ 50 ปีขึ้นไป ‚และ‛ มีปัจจัยเสี่ยงตั้งแต่ 1 ข้อขึ้นไป ได้แก่ สูบบุหรี่ , โรคความดันเลือดสูง , โคเลสเตอรอลในเลือดสูง ,
ประวัติคนในครอบครัวเป็นโรคกล้ามเนื้อหัวใจตายเหตุขาดเลือด , ภาวะปัสสาวะมีแอลบูมิน
2) ใช้ป้องกันขั้นทุติยภูมิ (secondary prevention) เพื่อลดโอกาสเกิดกล้ามเนื้อหัวใจตายเหตุขาดเลือดซ้้า โรค
หลอดเลือดสมองซ้้า หรือการตาย ในผู้ป่วยโรคหลอดเลือดเลี้ยงหัวใจ ( Acute Coronary Syndrome, Coronary Artery
Disease), ผู้ป่วยโรคหลอดเลือดสมอง (Cerebral Vascular Disease), ผู้ป่วยท้าบอลลูน (Angioplasty), ผู้ป่วยผ่าตัด
ท้าทางเบี่ยงหลอดเลือดหัวใจ (Coronary Artery Bypass Graft Surgery; CABG)

1.2 Thienopyridine groups


มี 2 ตัวที่สาคัญคือ Ticlopidine และ Clopidogrel ใช้เป็นตัวเลือกในกรณีที่ผู้ป่วยมีปัญหา
ไม่สามารถใช้ Aspirin ได้ หรือใช้ร่วมกับ Aspirin ในกรณีของการทา Coronary stenting ซึ่ง
ข้อบ่งชี้
1. ใช้ในผู้ป่วยที่แพ้หรือไม่สามารถรับประทาน Aspirin ได้ เช่น -เสี่ยงต่อภาวะเลือดออกง่าย, แพ้ ASA, โรคหืด
2. ในผู้ป่วยที่ได้รับการรักษาด้วยการใส่ขดลวดในหลอดเลือดหัวใจมาไม่เกิน 6 เดือน

การลดปัจจัยเสี่ยงการเกิด atherosclerosois
- งดสูบบุหรี่
- LDL < 70 mg/dl
- HA1C < 7%
- BP < 140/90 mmHg
- Exercise > 30 min 3x4 per wk

ในผู้ป่วยรายนี้มี Cardiovascular และ Cerebrovascular disease ยาที่ใช้ป้องกันอาการดังกล่าวคือ antiplatelet โดยเลือก


ให้ ASA เนื่องจากไม่มีข้อห้ามใช้ ในผู้ป่วย

56. male วัยกลางคน มีอาการปวดบวมเข่าและนิ้วโป้งเท้าขวา เจาะข้อได้ WBC 20,000 และ needle shaped crystal จะเจอ
x-ray finding อะไร
a. osteoporosis
b. osteophyte
c. chondrocalcinosis
d. bone ankylosis
e. lumpy soft tissue

เฉลย e. lumpy soft tissue

Gouty arthritis ลักษณะทางรังสีที่พบจะพบความผิดปกติทางรังสีได้หลังจากเกิด gouty attacks ,kมาหลายปี


- preservation of joint space ในระยะแรก joint space จะไม่ถูกทาลาย
- periarticular marginal erosion แบบ punch out lesion ขอบเขตการทาลายชัดเจนมีขอบ sclerosis ด้วย
- eccentric periarticula soft tissue ซึ่งคืก tophi และอาจจะมีหรือไม่มี calcium สะสมภายในก้อนก็ได้ , พบการ
ทาลายกระดูกที่อยู่ติดกับก้อน tophi ได้ โดยไม่จากัดอยู่ในบริเวณข้อเท่านั้น
- ไม่มีภาวะ osteoporosis ของกระดูกรอบข้อ ยกเว้นในรายที่เป็นมานาน
- Asymmetrical Ref. เอกสารประกอบการเรียนรังสีวิทยา อ.พญ.จันทร์เพ็ญ บูรณโชคไฟศาล
57.ผู้ป่วยชายอายุ 50 ปี ปวดเข่ามา 2 วัน มีไข้ ปวดบวมแดงเข่าขวา ขยับข้อเข่า
ทาได้กันมั๊ยคะ อิอิ
ไม่ได้ เนื่องจากมีอาการปวดมาก PE: BT 38.5oC, Rt. knee swelling and
สู่ๆ น้า
marked tender. Lab ใดจะมีประโยชน์มากที่สุด
a. Serum uric acid ….
b. Rheumatoid factor
c. Joint fluid analysis
d. ESR
e. Knee plain film X-ray

ผู้ป่วยมาด้วย ปวดข้อ ก็จะแยกอยู่ไม่กี่โรคที่เราควรรู้ จากอาการเราจะยังไม่สามารถแยก Septic


1. Septic arthriris arthriris,Rheumatoid arthritis,Gout
2. Rheumatoid arthritis ออกจากกันยังไม่ได้(ประวัติเท่านี้)
3. Gout แยกกันด้วย joint fluid analysis ตามตารางอ่า
4. OA Non inflame++++ OA
จากประวัติคิดถึง OA ได้น้อย เนื่องจาก Inflame+++++++ RA
OA จะมาด้วย ปวดเป็นๆหายๆ ไม่ค่อยมีบวมแดงไม่มีไข้ Septic++++++++ Septic
นะ มี crepitation เลยพอตัดออกไปได้ ข้อ 37 พูด RA ไป ละ ข้อนี้ OA ละกัน
ตอบ Joint fluid analysis OA
คนแก่ cartilage เสื่อม
บ่อยที่ DIP,PIP,1ST CMP ,Knee, hip,spine(C5-6,L3-4)
ปวดเป็นๆหายๆ , crepitus , ข้อโตจาก new bone
formation
Node= bouchard & Heberden ‘s node
Film
o Jt spaces asymmetrical narrowing
o Subchondral bone sclerosis
o Marginal osteophyte
o Bone cyst
TX
 ลดน้าหนัก
 กายภาพ
Non inflamatory
Inflamatory  Medication
Osteoarthritis RA
SpA o Paracetamol ก่อน ไม่ดีเพิม
Traumatic arthritis Crystal-induced arthritis
Avascular necrosis Septic arthritis o Weak opeiod ;tamadol,capsaicin
Rheumatic fever
Subsiding crystal-induced arthritis Reactive arthritis
o NSAIDs
Charcot’s/neuropathic joint CNT diseases o Intraarticular steroid(เป็นน้อยข้อ)
SLE ทั้งหมดไม่รักษาแค่ ลดปวดเว้น hyaluronate ช่วยสร้างน้า
Hypothyroidism
ลดปวดลดรุนแรง
Amyloidosis
59. ชายอายุ 49 ปี มีไข้ ไม่มีหนาวสั่น เจ็บที่ชายโครงข้างขวาร้าวไปที่ไหล่ขวา 3 วันก่นมาโรงพยาบาล อุจจาระปกติ ไม่เหลือง มี
เบื่ออาหาร น้าหนักลด PE: right lobeliver enlarged with mild tenderness , HBsAg positive
LAB: LFT: TB/DB = 1.0/0.2
AST = 60
ALT = 70
ALP = 112
Total protein = 5.0
Albumin = 2.3
U/S : Hyperechoic mass at right lobe liver diameter 6 cm. , no intrahepatic duct dilatation
a. metastatic CA
b. Amoebic liver abscess
c. Hepatocellular carcinoma
d. peripheral cholangiocarcinoma
e. postnecrotic cirrhosis with hepatic adenoma
เฉลย ข้อ b. aof_pa!!
metastatic CA
อาการ เบื่ออาหาร น้าหนักลด อืดแน่นท้อง ตัวเหลือง ตับโต มีประวัติ มี CA ที่อื่นด้วย
Amoebic liver abscess เชื้อ Entameba histolytica
 กิน cyst แล้วปล่อย trophozoites ” มักเป็นก้อนเดียว กลีบขวาบ่อยกว่าซ้าย
อาการ = ไข้ตามด้วยปวดท้อง ปวดใต้ชายโครงขวาร้าวไปหัวไหล่ขวา(กลีบขวา)
ปวดที่ epigastium (กลีบซ้าย)
ตรวจร่างกาย = ตับโต เจ็บบริเวณที่ฝี
Lab = leukocytosis , AST, ALT , alkaline phosphatase อาจจะสูงกว่าปกติ , bilirubin ปกติ
U/S = วินิจฉัยเป็นหลัก ลักษณะของอัลตราซาวด์ ของ amebic liver abscess
ลักษณะที่สาคัญของ amebic liver abscess คือ
 ไม่มี echo wall
 รูปร่างกลมหรือวงรี
 เป็นก้อนที่มี hypoecho ขอบเขตชัดเจน
และมี low internal echo
 อยู่ใกล้หรือติดกับขอบตับ
 มี posterior enhancement(distal sonic
enhancement)
treatment = metronidazole 400 mg. tid นาน 7-10 วัน
60. ชายวัย 52 ปี มีอาการเจ็บใต้ชายโครงซ้ายเฉียบพลัน หลังจากอาเจียนมาก PE: profused sweating, decrease breath
sound Lt. lung, Abd: tender with guarding Dx?
a. Hiatal hernia
b. Ruptured esophagus
c. Mallotry-weiss syndrome
d. Tension pneumothorax
e. Acute MI
ตอบ b. Ruptured esophagus
(Deja review USMILE Step2 หน้า 126, 154)
Boerhaave’s syndrome: characterized by esophageal perforation following severe vomiting
Tension pneumothorax: +/- trauma, pleuritic chest pain, dyspnea, tachypnea PE: BP drop, +ve jugular
venous distension, absent breath sound & hyperresonance, trachea debiation
Mallory-weiss syndrome: มักจะมาด้วย UGIB ตามหลังอาเจียนรุนแรง หรือ มี melena
Hiatal hernia: no symptom, heartburn, regurgitation
Acute MI: chest pain, ไม่น่าจะมี decrease breath sound

ข้อ 61) ตอบ ข้อ c.


เนื่องจากผู้ป่วยมีถ่ายอุจจาระเป็นเลือด แต่เลือดไม่ปนเป็นเนื้อเดียวกับอุจจาระ เป็นลักษณะของเลือดที่เกิดจาก
anorectal lesion ถ้าเลือดออกสูงขึ้นไปเลือดจะคลุกเคล้ากับอุจจาระเป็นมูกเลือด เป็นสีแดงคล้า หรืออาจเป็นสีดาแดงได้
DDx. Common causes of anorectal lesion.
 Hemorrhoid  Proctits
 Fissures  Rectal trauma
 Fistulas
จากประวัติผู้ป่วยดื่มเหล้าเป็นประจา อาจทาให้เกิด cirrhosis ซึ่งมีความสัมพันธ์กับภาวะ hemorrhoid จาก portal
hypertension หรืออาจเกิดจากการเบ่งถ่ายรุนแรงและเรื้อรัง(ท้องผูกหรือท้องเสีย) ซึ่งในผู้ป่วยรายนี้มีอาการปวด
หน่วงท้องน้อยร่วม
Internal hemorrhoid
Stage 1: ริดสีดวงอยู่ในทวารหนัก มีอาการเลือดออกอย่างเดียว
Stage 2: ริดสีดวงออกนอกทวารหนักเวลาเบ่ง และกลับเข้าไปได้เองขณะเบ่ง
Stage 3: ริดสีดวงย้อยออกมาเวลาเบ่ง และต้องดันกลับจึงจะเข้า
Stage 4: ริดสีดวงย้อยออกมานอกทวารหนักตลอดเวลา
การรักษาhemorrhoid
1. การรักษาทั่วไป: ดูแลให้ขับถ่ายตามปกติ อย่าให้ท้องผูกหรือท้องเสีย ห้ามเบ่งมาก หมั่นออกกาลังกาย กินhigh
fiber dietหรือกินยาเพิ่มปริมาตรอุจจาระ เช่น branหรือpsyllium ดื่มน้าให้เพียงพอ
2. การฉีดยา: ใช้กับระยะ 1 และ 2
3. barron’s ligation: ใช้ในระยะ 2 และระยะ 3 บางราย
4. hemorrhoidectomy: ใช้ในระยะ 3 ขึ้นไป
5. อื่นๆ: anal dilatation, cryosurgery, infrared and bipolar diathermy
Ref: อาการวิทยาศิริราช, ตาราศัลยศาสคร์จุฬาฯ ...............................................................by ReaL
62. ผู้ป่วยอายุ 45 ปี ดื่มเหล้าขาววันละ 1 ขวด มา 25ปี ถ่ายเหลวมา 3-4 ครั้ง/วัน เป็นเวลา 2 วัน มาด้วยอาการสับสน จาญาติ
ไม่ได้ ตรวจร่างกาย v/s ปกติ พบscaling lesion บริเวณหน้า คอ และแขน 2 ข้าง ถามDx
a. Pellagra
b. subdural hematoma
c. Wernicke’s encephalopathy
d. electrolyte imbalance
e. delirium tremens

Pellagra เป็นโรคที่เกิดจากการขาด vitamin B3 (niacin) ซึ่งเป็นองค์ประกอบที่สาคัญของ coenzymes และยังเป็นตัว


precursor ในการสังเคระห์ tryptophan จะพบใน

 Compromised intake of niacin or tryptophan


o Poverty , Poor nutrition
o Chronic alcoholism
o Anorexia nervosa
 Compromised ability to absorb ingested niacin and tryptophan
o Malabsorptive states
o Prolonged diarrhea
 Altered intermediary metabolism impacting niacin synthesis
o Hartnup disease
o Fad diets
o Isoniazid therapy: Treatment with the antituberculosis drug
o Carcinoid tumors
o Medications: 5-flurouracil, pyrazinamide, 6-mercaptopurine, hydantoins, ethionamide,
phenobarbital, azathioprine, and chloramphenicol.
 Multifactorial, miscellaneous, or unknown mechanism
o Liver cirrhosis
o Diabetes mellitus
o Prolonged febrile illness, possibly leading to increased energy hence niacin requirements
o Human immunodeficiency virus (HIV) disease

อาการที่พบ "the 3 Ds" (ปัจจุบันเป็น 4 Ds แล้ว เพิ่ม deathเข้าไปอีกตัว)

D=diarrhea (GI findings)

o Patients with pellagra tend to suffer from poor appetite, nausea, epigastric discomfort, abdominal
pain, and increased salivation.
o Gastritis can be present and may result in achlorhydria.
o Glossitis typically causes soreness of the mouth and dysphagia.
o Diarrhea is the manifestation of intestinal inflammation. Diarrhea is typically watery (enteritis) but
is occasionally bloody and mucoid (colitis).
D=dermatitis (Skin findings)

o Affected skin lesions are initially erythematous and are associated with a burning sensation.
o The distribution of the cutaneous eruption is typically symmetrical and bilateral in parts of the
body exposed to sun.
o As the dermatitis progresses, the affected skin becomes hyperpigmented and thickened.

D=dementia (Neuropsychiatric findings)

 Early neuropsychiatric symptoms of pellagra include lethargy, apathy, depression, anxiety,


irritability, and poor concentration.
 As the disease advances, patients become disoriented, confused, and delirious. Eventually, the
patient becomes stuporous and comatose.

D=Death

o Death is the result of the depletion of the coenzyme required to generate sufficient energy to
support vital body functions.
เฉลย bY : ต้อง MeD X

63. ผู้ป่วยชาย อายุ 33 ปี dyspnea, engorged neck vein, no murmur, EKG: low voltage, generalized ST change with
inverted T ในผู้ป่วยรายนี้น่าจะคิดถึงโรคใดมากที่สุด
a. Constrictive pericarditis
b. Cardiac tamponade
c. RBBB
d. Lateral wall infarction
e. Pulmonary embolism
ตอบ Cardiac temponade ข้อนี้ต้องพิจารณาแต่ละโรคดังนี้

Constrictive pericarditis ภาวะนี้เกิดเนื่องจากมีการหนาตัวของเยื่อหุ้มหัวใจทั้งสองชั้น ถ้ายังมีช่องว่างของเยื่อหุ้มหัวใจ


เหลืออยู่ก็จะมีเยื่อไฟบริน และน้าปริมาณไม่มากอยู่ในช่องดังกล่าว ทาให้เกิดผลบีบรัดหัวใจ ถ้าการอักเสบเป็นมากขึ้น เยื่อหุ้ม
หัวใจจะติดกันแข็งมากขึ้น ทาให้การขยายตัวของ ventricle ทั้งสองข้างมีความดันเพิ่มขึ้น ร่วมกับปริมาณเลือดที่ออกจากหัวใจ
ลดลง เนื่องจาก preload ที่ลดลง
Clinical features
มักมาด้วยอาการของ right side heart failure มากกว่า left side heart failure เช่น neck vein engorge , dyspnea ,
orthopnea , PND , hepatomegaly , peripheral edema เป็นต้น และมีอาการของ pericarditis เช่น เจ็บหน้าอก ตรวจพบ
pericardial friction rub นอกจากนี้บางรายยังตรวจพบ Kussmaul ‘s sign (หลอดเลือดดาที่คอโป่งมากขึ้นขณะหายใจเข้า ) ,
pulsus paradoxus พบได้ประมาณ 25 % ซึ่งต่างจาก cardiac temponade ที่พบได้มากกว่า
EKG
- low voltage พบ 62 % - Inverted T พบ 19%
- ST-T change พบ 22% - AF พบ 12%
- *** การพบ generalized ST-T change เป็นลักษณะที่อาจนึกถึง pericarditis กล่าวคือ จะพบ ST elevation แบบ
concave upward ( ST segment ยก แต่เว้าลงตรงกลาง) ในทุก lead ยกเว้น V1 และ aVR ซึ่งอยู่ที่ base ของหัวใจที่
ไม่มี pericardium หุ้ม

Cardiac tamponade เป็นภาวะที่หัวใจโดนกดจากการมี pressure ที่ pericardium เพิ่มขึ้น มักเกิดตามหลังการบาดเจ็บต่อ


หัวใจหรือการผ่าตัดหัวใจ (กล่าวคือ เกิดแบบรวดเร็วจนร่างกายปรับตัวไม่ทัน) ถ้าเกิดแบบค่อยเป็นค่อยไปเป็น pericardial
effusion จนกระทั่งมีปริมาณน้ามาก ก็กลายเป็น temponade ได้
Clinical features
Beck’s triad : Hypotension , neck vein engorge , distance heart sound
Dyspnea and tachypnea with lung clear
pulsus paradoxus พบได้มากกว่า 75% ส่วน pericardial friction rub พบได้น้อย
EKG
- low voltage , electrical alternan +_ generalized ST-T change

ข้อนี้ที่เฉลย Cardiac temponade เพราะไปถาม อ.


อ. บอกว่า low voltage ให้นึกถึง กลุ่มของโรคที่มีอะไรมากั้น เช่น pericardial effusion , cardiac temponade , COPD ,
restrictive cardiomyopathy ที่เกิดจากการ infiltrative เช่น amyloidosis

และที่สาคัญ อ.บอกว่าถ้าข้อนี้ constrictive pericarditis ก็จะยากไปสาหรับ นสพ.ปี 5 เอาไว้ให้ resident ตอบ แป่ว!!
64. ผูป้ ่ วยชายไทยอายุ 18 ปี มีอาการเหนื่อยมากขึ้นทันที ไอเป็ นฟองสี ชมพู แพทย์ทาการตรวจร่ างกายพบว่ามี
Pink frothy sputum , Crepitation both lower lungs, BP 160/100 mmHg ได้
ให้ Furosemide และ O2 ไป การรักษาเพิ่มเติมที่เหมาะสมที่สุดในผูป้ ่ วยรายนี้คือข้อใด
a. Diogoxin
b. Propanolol
c. Dobutamine
d. Nitroprusside
e. Mophine
Ans c. Dobutamine
รายนี้เพื่อนๆน่าจะรู ้นะว่าเป็ น CHF with volume overload and Pulmonary
edema การ Treatment คือ
 Restrict salt/fluid
 O2 therapy [ ผูป้ ่ วยรายนี้ได้แล้ว ]
 ACE-I [ decrease mortality and sympyom ] โดยเริ่ ม low dose ก่อนและต้อง
ระวังภาวะ Hypotension กับ Renal insuf ด้วยนะเพื่อนๆ ซึ่ งบางงานวิจยั ก็จะบอกให้ใช้ใน
ผูป้ ่ วยที่ EF<40 %
 Diuretic [ ผูป้ ่ วยรายนี้ได้แล้ว ] มักให้ในรายที่มี volume overload
 B-block ไม่ใช้เนื่องจากจะเลือกใช้ในรายที่ HF NYHA class II หรื อ III
 Dobutamine [Drug of choice] เป็ นยาที่เหมาะสมที่สุดที่ควรเลือกใช้ในรายนี้ ( อืม
เพิ่มเติมอีกนิดว่าถ้าโจทย์มี NTG ให้เลือกด้วยก็เป็ นยาอีกด้วยนะที่ควรเลือกใช้พอๆกับ
obutamine)
ส่ วนยาบางตัวไม่พิจารณาเลือกใช้เนื่องจาก
 Digitalis ไม่เลือกใช้เนื่องจากจะให้ในรายที่มี AF, severe CHF, EF<30% ซึ่ งในรายนี้
ก็ไม่พบภาวะดังที่กล่าวมา บางงานวิจยั ใหม่ก็อาจเลือกใช้ในรายที่มีประวัติ readmission บ่อยๆ แต่
จากประวัติก็ไม่ได้บอกว่ามีประวัติดงั กล่าว
 Nitropusside ไม่เลือกใช้ก็เนื่องจาก BP ก็ไม่สูง และอาจมีขอ้ เสี ยจาก Reflex
tachycardia

65. ผูป้ ่ วยหญิงอายุ 32 ปี Underlying Rheumatic heart disease eith mitral valve
stenosisขณะนี้ได้ยา Digoxin(0.25mg) 1 tab OD, Furosemide(40 mg) 1 tab OD
โดย 3 วันก่อนมาโรงพยาบาลผูป้ ่ วยมีอาการเหนื่อยมาก คลื่นไส้อาเจีย น เบื่ออาหาร เห็นแสงสี ผดิ ปกติ PE:
BP 100/70 mmHg, HR 50 bpm, Lung fine crepitation both lungs, Heart
loud S1, Diastolic murmur at apex อะไรเป็ นสาเหตุที่ทาให้เกิดอาการมากขึ้น
a. Hypokalemia
b. Hyponatremia
c. Drug interaction
d. Volume depletion
e. Metabolic alkalosis

Ans a. Hypokalemia
โจทย์ขอ้ นี้ก็เป็ นอาการของ Digitalis intoxication อ่ะเนอะ และถ้าฟังๆดูร่วมกับ choice ที่
มีอ่ะคงจะถามว่ามี condition ไรบ้างทาให้เกิด worsen digitalis toxicity ในผุป้ ่ วยที่เป็ น Digitalis
intox ซึ่ งถ้าเจอคาถามประมาณนี้นะหาคาตอบเกี่ยวกับ E’lyte เลยนะ(ถามบ่อยมัก่ ๆ)
E’lyte imbalance ที่เป็ นสาเหตุได้ก็มี3 ตัวนะ คือ K, Ca, Mg ( คงตัด choice ตอบข้อนี้
ได้แล้วเนอะว่าHypokalemia ..55) และ condition ที่เป็ นสาเหตุได้คือ
 Hypo/Hyperkalemia
 HypoMg
 HyperCa
โดยในรายนี้มีประวัติ on Furosemide ซึ่ งก็มี toxic ที่สาคัญคือเกิด HypoK อยูแ่ ล้วจึงน่าจะ
เป็ นสาเหตุที่สาคัญที่กระตุน้ ให้เกิด worsen digitalis toxicity ส่ วน HyperK มักพบในผูป้ ่ วยเด็ก, เพิ่งรับยา
digitalis มาใหม่ๆ ซึ่ งไม่เหมือนในผูป้ ่ วยรายนี้

66. ผูป้ ่ วยหญิงอายุ 32 ปี Underlying Rheumatic heart disease eith mitral valve
stenosisขณะนี้ได้ยา Digoxin(0.25mg) 1 tab OD, Fuaosemide(40 mg) 1 tab OD
โดย 3 วันก่อนมาโรงพยาบาลผูป้ ่ วยมีอาการเหนื่อยมาก คลื่นไส้อาเจียน เบื่ออาหาร เห็นแสงสี ผดิ ปกติ PE:
BP 100/70 mmHg, HR 50 bpm, Lung fine crepitation both lungs, Heart
loud S1, Diastolic murmur at apex การรักษาที่เหมาะสมที่สุดในผูป้ ่ วยรายนี้คือ
a. เพิ่มยา Atropine
b. เพิม่ ยา Beta blocker
c. หยุดทานยา Digoxin
d. เพิม่ ยา Steroid
e. เพิ่มยา Furosemide

Ans c. หยุดทานยา Digoxin


อันนี้ก็ต่อเนื่องมาจากข้อที่แล้ว ก็แน่นอนอ่ะก็ตอ้ งตอบ off digoxin อ่ะ ส่ วนการรักษาอื่นๆ เช่น
 supportive care -> hydration with intravenous (IV) fluids,
oxygenation and support of ventilatory function
 correction of electrolyte imbalances -> ในรายนี้ก็ควร replace K ซึ่ งก็มี
recomment ให้ replaceเลยแม้ K ไม่ต่าอ่ะ โดยปกติก็ให้เลยเมื่อ K < 4 mmol/L.
 จริ งๆแล้วในรายนี้ก็ควรหยุดยา Diuretic ด้วย แต่ก็ให้ต่อเนื่องจากผูป้ ่ วยมีความจาเป็ นต้องได้อยู่
 Digoxin-specific fragment antigen binding (Fab) antibody
fragments -> ในรายที่ severe acute digitalis toxicity
 GI decontamination ->
o Activated charcoal [multiple-dose charcoal (1 g/kg/d) ]
o Gastric lavage มีประโยชน์ในรายที่ early after ingestion
o Whole-bowel irrigation ( ทากันน้อยมากๆๆ)
o Steroid-binding resins, such as cholestyramine and colestipol ก็
พอใช้ได้นะแต่ก็ได้ผลดีชดั เจนในรายที่เป็ น renal insufficiency ร่ วมด้วยอ่ะ

67. ผู้หญิงอายุ 60 ปี สูบบุหรี่ มา 30 ปี ไอมาก x-ray พบ right upper lung mass และมี Na ในเลือดต่า( urine
osmole 50) เกิดจากอะไร
a. SIADH
b. Polydipsia
เฉลยนะ a. SIADH จากโจทย์จะเห็นว่า ผู้ป่วยอายุมาก มีประวัติ Chronic smoking แล้ วพบ right upper lung
mass ทาให้ เรานึกถึงในเรื่ องของ lung cancer ซึง่ สามารถทาให้ เกิดภาวะ SIADH ได้ ซึง่ เราจะนึกถึงเป็ น CA
lung ชนิด small cell นะ แต่เค้ ายังรู้สกึ แปลกๆเรื่ อง urine osm.นะ เพราะถ้ าเป็ น SIADH เนี่ย urine osm. มัน
ต้ องมากกว่า 100 อ่ะ

โรคทาง malignancy อื่นๆ ที่ทาให้ เกิด SIADH ได้

 Lung cancer, small cell type


 Gastrointestinal cancers (pancreatic cancer, exocrine; duodenal or stomach cancer)

 Genitourinary cancer (bladder cancer, prostate cancer, ovarian cancer)

 Lymphoma, including Hodgkin's disease

 Head and neck cancers (oral cancers, laryngeal cancer, nasopharyngeal cancer)

 Thymoma

 Brain and central nervous system tumors

 Breast cancer

 Melanoma

Certain cancers produce and secrete ADH themselves. This production occurs without regard
for the needs of the body. Thus, the kidneys receive repeated signals to save water, even when the
body already has a marked excess of fluid. Of all the types of cancer that produce ADH themselves,
small-cell lung cancer is by far the most common. Small-cell cancer of the lung is the cause in 75%
of cases of SIADH caused directly by a tumor. In some cases, the appearance of SIADH may be the
first indication that a cancer exists.
ความรู้เพิ่มเติมนะจ๊ ะ
Syndrome of inappropriate ADH secretion (SIADH) เป็ นภาวะ
ผู้ป่วย SIADH จะมีลกั ษณะ hypoosmolar euvolemic hyponatremiaมี urine osmolality สูงกว่าที่ควร
จะเป็ น (สูงกว่า 100 mOsm/kg ไม่จาเป็ นต้ องมากกว่า plasma osmolality) urine Na สูงกว่า 20 mEq/L
เนื่องจากไตจะขับ Na ออกมากเพื่อป้องกันไม่ให้ ปริมาตร ECF มากเกิน (ยกเว้ นใน ผู้ป่วยที่ทาน Na น้ อยมาก
อาจพบ urine Na ต่าได้ ) การวินิจฉัยต้ องพิสจู น์วา่ ไม่มีภาวะอื่นที่มีลกั ษณะคล้ ายกัน เช่น hypothyroid,
adrenal insufficiency, renal failure และ reset osmostat เป็ นต้ น (ตารางที่ 3) อย่างไรก็ตาม บางครัง้ ผู้ป่วยที่มี
SIADH อาจมีโรคอื่นแทรกซ้ อนที่ทาให้ มี hypovolemia ในขณะนัน้ urine Na อาจน้ อยกว่า 10 mEq/L ดังนัน้
การพบว่ามี hypovolemia หรื อ urine Na ต่าอาจไม่สามารถ exclude ภาวะ SIADH ได้ เสมอไป เพียงแต่ยงั ไม่
สามารถให้ การวินิจฉัย SIADH ที่อาจแฝงอยูใ่ นขณะนันได้ ้ ในการรักษา SIADH สิ่งที่สาคัญ คือ การจากัด
ปริมาณน ้าเข้ าร่ างกายให้ เหลือ 800-1,000 ml ต่อวัน และรักษาโรคพื ้นฐานที่เป็ นสาเหตุ การให้ hypotonic หรื อ
isotonic fluid ในผู้ป่วยกลุม่ นี ้กลับยิ่งทาให้ ภาวะ hyponatremiaเป็ นมากขึ ้นเนื่องจากความเข้ มข้ นของ Na ใน
สารละลายไม่มากพอเมื่อเทียบกับในปั สสาวะ และกลับเป็ นการเพิ่มปริมาตร ECF ของผู้ป่วยซึง่ ไตจะตอบสนอง
โดยการขับ Na ออกทางปั สสาวะมากขึ ้น ถ้ าเป็ นรุนแรงควรให้ hypertonic fluid ในการแก้ ไขระดับ Na การรักษา
ระยะยาวให้ ใช้ วิธีเพิ่มปริมาณ solute intake ร่วมกับการให้ furosemide จะ
ช่วยให้ การรักษาทาได้ ง่ายขึ ้นโดยเฉพาะอย่างยิ่งในผู้ป่วยที่ มี urine osmolality สูงและ
ไม่สามารถจากัดปริมาณน ้าดื่มได้ เพียงพอหรื อไม่ตอบสนองต่อการรักษาเท่าที่ควร หรื อ
เกณฑ์ การวินิจฉัย SIADH
Essential
1. Decreased effective osmolality of the extracellular fluid (Posm < 275 mOsm/kg H2O).
2. Inappropriate urinary concentration (Uosm > 100 mOsm/kg H2O with normal
kidney function) at some level of hypo-osmolality.
3. Clinical euvolemia, as defined by the absence of signs of hypovolemia
(orthostasis, tachycardia, decreased skin turgor, dry mucous membranes)
or hypervolemia (subcutaneous edema, ascites).
4. Elevated urinary sodium excretion while on a normal salt and water intake.
5. Normal thyroid, adrenal, and kidney function.
Supplemental
6. Abnormal water load test (inability to excrete at least 80% of a 20 mL/kg water
load in 4 hours and/or failure to dilute Uosm to < 100 mOsm/kg H2O).
7. Plasma AVP level inappropriately elevated relative to plasma osmolality.
8. No significant correction of serum Na with volume expansion but improvement after fluid
restriction. อาจใช้ demeclocyclin มาใช้ รักษา chronic SIADH เนื่องจากยานี ้มีพิษต่อไตทาให้ เกิดภาวะ
nephrogenic insipidus ไตตอบสนองต่อ ADH น้ อยลง ขับ free water ได้ มากขึ ้น
Excessive water intake ได้ แก่ ผู้ป่วยที่มีปัญหา primary polydipsia พบในผู้ป่วย psychiatric
disorders ที่มีพฤติกรรมดื่มน ้ามากผิดปกติ กลไกการควบคุมการหลัง่ ADH และความสามารถในการขับ free
water ยังคงเป็ นปกติ แต่มีการดื่มน ้าในปริมาณที่มากเกินขีดความสามารถที่ไตของมนุษย์ปกติจะสามารถขับ
free water ออกได้ ทังนี
้ ้ถ้ าผู้ป่วยทานอาหาร (solute) ได้ ตามปกติจะต้ องดื่มน ้ามากกว่า 12 ลิตรต่อวันขึ ้นไปจึง
จะเกิดภาวะ hyponatremia ผู้ป่วยกลุม่ นี ้มีลกั ษณะ euvolemia เช่นเดียวกับกลุม่ low solute intake
By….Dew

68. pt. CHF…Bx: lymphocyte infiltration, most common virus?


a. Adenovirus
b. Coxsakie B
c. EBV
d. CMV
เฉลย ข้ อ b. Coxsakie B
Coxsackie B viruses are responsible for numerous cases of central nervous system infections
in infants and children, as well as heart muscle infections in both children and adults. These viruses
are the most common agent for myocarditis (inflammation of the muscular walls of the heart) and
dilated cardiomyopathy (a group of disorders where the heart muscle is weakened and cannot pump
effectively resulting in the dilation of the cardiac chambers).
Infection of the heart by the Coxsackie B virus can lead to viral myocarditis – an
inflammatory disease of the heart that can cause heart failure .The disease does not usually cause
death, but permanent heart damage can occur. There are usually no long-term complications from
the mild illnesses/symptoms caused by the Coxsackie B virus. However, some patients who have
paralysis may not fully recover. Those who develop dilated cardiomyopathy (heart failure) from
myocarditis will require long term care for their symptoms.
By…Dew

69. ชายอายุ 30 ปี น ้าท่วมบ้ าน มาด้ วยปวดกล้ ามเนื ้อ ไข้ Dx.?


a. Leptospirosis
เฉลย a. Leptospirosis
ดูจากประวัตนิ ะ ผู้ป่วยรายนี ้มีประวัตลิ ยุ น ้า และมีไข้ ปวดกล้ ามเนื ้อ(เป็ นอาการเด่นที่พบได้ ในโรคนี ้ ส่วนมากจะ
ปวดบริเวณน่อง) ซึง่ เข้ าได้ กบั โรคนี ้มากที่สดุ นะจ๊ ะ ลองอ่านรายละเอียดเกี่ยวกับโรคนี ้ดูนะ
โรคเล็ปโตสไปโรซิส (Leptospirosis) , โรคฉี่หนู หรื อ หรื อไข้ ฉ่ ีหนู เป็ นโรคติดต่อจากสัตว์สคู่ น
(zoonosis) สามารถติดโรคได้ ในสัตว์หลายชนิด เช่น สุนขั หนู โค กระบือ สุกร แพะ แกะ สัตว์เลี ้ยงในบ้ าน เป็ น
ต้ น แต่พบมากใน หนู ซึง่ เป็ นแหล่งรังโรค ส่วนมากสัตว์ที่ไวต่อการรับเชื ้อมักจะเป็ นสัตว์เลี ้ยงลูกด้ วยนมที่มีอายุ
น้ อย หรื อลูกสัตว์ที่ไม่เคยได้ รับภูมิค้ มุ กันจากแม่มาก่อน มักจะพบการระบาดในเดือนตุลาคม และพฤศจิกายน
เนื่องจากเป็ นฤดูฝนต่ อหนาว มีนา้ ขัง
สาเหตุ
โรคเล็ปโตสไปโรซิส มีสาเหตุจากการติดเชื ้อแบคทีเ รี ยรูปเกลียว (spirochete) ชื่อ เล็บโตสไปร่า อินเทอ
โรแกนส์ (Leptospira interrogans) ซึง่ มีความหลากหลายทางซีโรวิทยามากกว่า 200 ซีโรวาร์ (serovars) เชื ้อ
ชนิดนี ้อาศัยอยูใ่ นท่อหลอดไตของสัตว์ได้ หลายชนิด โดยมีหนูเป็ นแหล่งรังโรคที่สาคัญที่สดุ บางซีโรวาร์ มี
ความจาเพาะกับสัตว์บางชนิด เชื ้อสามารถมีชีวิตได้ นานหลายเดือนหลังจากถูกขับออกทางปั สสาวะจากสัตว์ ที่มี
เชื ้อ สัตว์อื่นๆที่เป็ นแหล่งรังโรค ได้ แก่ สุกร โค กระบือ สุนขั แรคคูณ โดยที่สตั ว์อาจจะไม่มีอาการแต่สามารถ
ปล่อยเชื ้อได้ เป็ นเวลาหลายสัปดาห์หรื ออาจจะตลอดชีวิตสัตว์ จากการสารวจเมื่อปี พ.ศ. 2540 โดยสถาบัน
สุขภาพสัตว์แห่งชาติ พบภูมิค้ มุ กันใน ควาย 31% โค 28.25% แพะแกะ 27.35% สุกร 2.15%
อาการและอาการแสดง
อาการและอาการแสดงของโรคเล็ปโตสไปโรซิสนันมี ้ ได้ ตงแต่
ั ้ มีอาการเล็กน้ อยจนถึงมีอาการรุนแรงถึงแก่
ชีวิต กว่า 90% ของผู้มีอาการจะมีอาการเล็กน้ อยและมักเป็ นแบบไม่เหลือง ส่วนเล็ปโตสไปโรซิสที่มีอาการ
รุนแรงนัน้ พบได้ 5-10% ของผู้ตดิ เชื ้อ
โรคเล็ปโตสไปโรซิสแบบไม่ เหลือง (anicteric leptospirosis) แบ่งเป็ น 2 ระยะ
ระยะมีเชื ้อในเลือด (leptospiremic phase) ระยะนี ้มีอาการคล้ ายไข้ หวัดใหญ่ คือ มีไข้ หนาวสัน่ ปวด
ศีรษะ คลื่นไส้ อาเจียน และปวดกล้ ามเนือ้ ซึง่ ในโรคเล็ปโตสไปโรซิสจะมีลกั ษณะเฉพาะคือ ปวดบริเวณน่ อง
หลัง และหน้ าท้ อง ผู้ป่วยส่วนมากมักมีอาการปวดศีรษะมาก โดยเฉพาะบริเวณด้ านหน้ าหรื อหลังเบ้ าตา
อาการอื่นๆที่อาจพบได้ ได้ แก่ เจ็บคอ ไอ เจ็บหน้ าอก ผื่น สับ สน ไอเป็ นเลือด อาการแสดงที่อาจตรวจพบ ได้ แก่
เยื่อบุตาแดง ต่อมน ้าเหลืองโต คอแดง กดเจ็บบริเวณกล้ ามเนื ้อ ตับม้ ามโต อาจพบอาการดีซา่ นได้ เล็กน้ อย
อาการในระยะนี ้จะหายไปได้ เองภายใน 1 สัปดาห์ ช่วงที่ไม่มีอาการจะนาน 1-3 วัน ก่อนจะเข้ าสูร่ ะยะที่สอง
ระยะมีเชื ้อในปั สสาวะ (leptospiruric phase) ระยะนี ้เป็ นผลมาจากปฏิกิริยาทางภูมิค้ มุ กันของร่างกาย
อาการและอาการแสดงมีความจาเพาะและความรุนแรงน้ อยกว่าในระยะแรก ลักษณะที่สาคัญของโรคในระยะนี ้
คือ กว่า 15% ของผู้ป่วยจะมีอาการและอาการแสดงของภาวะเยื่อหุ้มสมองอักเสบแบบไร้ เชื ้อ (aseptic
meningitis) ส่วนใหญ่พบในเด็ก ซึง่ อาจหายได้ เองภายในเวลาไม่กี่วนั หรื ออาจคงอยูน่ านเป็ นสัปดาห์ สาหรับ
ภาวะแทรกซ้ อนอื่น เช่น ม่านตาอักเสบ จอตาอักเสบ มักเกิดตามหลังอาการเริ่มแรกของโรคนานหลายเดือน และ
คงอยูไ่ ด้ นานเป็ นปี
โรคเล็ปโตสไปโรซิสรุนแรง (severe leptospirosis)
โรคเล็ปโตสไปโรซิสรุนแรง หรื อกลุม่ อาการเวล (Weil's Syndrome) กลุม่ อาการนี ้มีอตั ราการตายอยูท่ ี่
ประมาณ 5-15% พบได้ เป็ นพิเศษในการติดเชื ้อในซีโรวาร์ อิกเทอโรฮีมอราเจียอี /โคเปนเฮเกไน
(icterohaemorrhagiae/copenhageni) อาการในระยะเริ่มแรกไม่ตา่ งจากโรคเล็ปโตสไปโรซิสแบบไม่เหลือง แต่
ไม่มีลกั ษณะที่แบ่งออกเป็ นสองระยะชัดเจน มักแสดงอาการรุนแรงใน 4-9 วันหลังจากเริ่มมีอาการ ประกอบด้ วย
 อาการดีซา ่ น ที่พบในกลุม่ อาการเวล จะมีลกั ษณะเหลืองมากจนแทบเป็ นสีส้มเมื่อสังเกตทางผิวหนัง มัก
พบตับโตร่วมกับกดเจ็บ ประมาณ 20% ของผู้ตดิ เชื ้อมีอาการม้ ามโตร่วมด้ วย มีไม่มากนักที่เสียชีวิตจาก
ภาวะตับวาย
 ไตวายเฉียบพลัน
 อาการทางปอด เช่น ไอ มีเสมหะปนเลือด เจ็บหน้ าอก หอบเหนื่อย จนถึงระบบหายใจล้ มเหลว

 ความผิดปกติทางระบบการแข็งตัวของเลือด มีตงแต่ ั ้ อาการเพียงเล็กน้ อย เช่น เลือดกาเดา จ ้าเลือดตาม


ผิวหนัง ไปจนถึงอาการรุนแรง เช่น เลือดออกในทางเดินอาหาร เลือดออกในช่องเยื่อหุ้มสมอง เป็ นต้ น
 อาการอื่นๆ ได้ แก่ กล้ ามเนื ้อลายสลายตัว เม็ดเลือดแดงแตก เยื่อหุ้มหัวใจอักเสบ กล้ ามเนื ้อหัวใจอักเสบ

ตับอ่อนอักเสบรุนแรง ภาวะการทางานของอวัยวะล้ มเหลวหลายระบบ เป็ นต้ น


การดาเนินโรค
เล็บโตสไปร่าขยายด้ วยกล้ องจุลทัศน์อีเล็กตรอน
โรคเล็ปโตสไปโรซิส ติดต่อจากคนสูค่ นได้ น้อยมาก ส่วนใหญ่ตดิ ต่อกันโดยการสัมผัสกับปั ส สาวะ เลือด
หรื อเนื ้อเยื่อของสัตว์ที่มีการติดเชื ้อโดยตรง หรื อสัมผัสกับสิ่งแวดล้ อมที่มีการปนเปื อ้ นของเชื ้อ เช่น
 การกินอาหารหรื อน ้าที่ปนเปื อ ้ นเชื ้อเข้ าไป
 การหายใจเอาไอละอองของปั สสาวะ หรื อ ของเหลวที่ปนเปื อ ้ นเชื ้อเข้ าไป
 เข้ าผ่านเยื่อบุตา่ งๆ เช่น ตา และปาก
 ไชเข้ าทางผิวหนังตามรอยแผลและรอยขีดข่วน

 ไชเข้ าทางผิวหนังปกติที่เปี ยกชุม ่ จากการแช่น ้านานๆ


ระยะฟั กตัวของโรคใช้ เวลา 1-2 สัปดาห์แต่อาจนานได้ ถึง 3 สัปดาห์ แบ่งเป็ นระยะมีเชื ้อในเลือด
(leptospiremic phase) ซึง่ จะเริ่มแสดงอาการและส่วนใหญ่จะหายไปเองใน 1 สัปดาห์ หลังจากนันอี ้ ก 1-3 วัน
จะเข้ าสูร่ ะยะมีเชื ้อในปั สสาวะ (leptospiruric phase) ผู้ป่วยบางส่วนจะแสดงอาการอีกครัง้ ประมาณ 5-10%
ของผู้ตดิ เชื ้อจะมีอาการของโรคเล็ปโตสไปโรซิสรุนแรง
โรคเล็ปโตสไปโรซิสแพร่กระกายและเกิดขึ ้นได้ ง่ายในช่วงปลายฤดูฝน พบบ่อยในช่วงเดือนตุลาคม -
พฤศจิกายน เพราะช่วงนี ้พื ้นดินแฉะ มีน ้าขัง เอื ้ออานวยต่อการเจริญเติบโตและสะสมของเชื ้อในรรมชาติ พบโรค
นี ้ได้ มากตามจังหวัดที่ทาการปลูกข้ าว บริเวณที่ต้องย่าน ้า หรื อแหล่งน ้าขังที่มีพาหะนาโรคชุกชุม อาจรวมถึงบ่อ
น ้าขนาดใหญ่ด้วย
กลุม่ ประชากรบางกลุม่ ถือเป็ นกลุม่ ที่มีความเสี่ยงสูงต่อการเกิดโรค ได้ แก่
 สัตวแพทย์

 ผู้ทางานในภาคเกษตรกรรม ปศุสต ั ว์ ประมง


 การสันทนาการและกีฬาทางน ้า เช่น การว่ายน ้า การเล่นเรื อแคนู วินด์เซิร์ฟ สกีน ้า ไตรกีฬา ฯลฯ

การรักษา
โรคเล็ปโตสไปโรซิสสามารถรักษาได้ ด้วยยาปฏิชีวนะ ควรให้ ยาเร็วที่สดุ เท่าที่จะเป็ นไปได้ หรื อไม่ควรเกิน
4 วันหลังจากมีอาการเป็ นอย่างช้ า ระยะเวลาที่ให้ นานอย่างน้ อย 7 วัน โดยชนิดของยาปฏิชีวนะจะแตกต่างกัน
ไปตามความรุนแรงของอาการ ดังนี ้
ระดับความรุ นแรง สูตรยา

ด็อกซีไซคลิน (doxycycline) , 100 มิลลิกรัม รับประทาน วันละ 2 ครัง้ หรื อ

ไม่รุนแรง แอมพิซิลลิน (ampicillin) , 500-750 มิลลิกรัม รับประทาน วันละ 4 ครัง้ หรื อ

อะม็อกซิซิลลิน (amoxicillin) , 500 มิลลิกรัม รับประทาน วันละ 4 ครัง้

เพนนิซิลลิน จี (penicillin G) , 1.5 ล้ านยูนิต ทางหลอดเลือดดา วันละ 4 ครัง้ หรื อ

แอมพิซิลลิน (ampicillin) , 1 กรัม ทางหลอดเลือดดา วันละ 4 ครัง้ หรื อ


รุนแรงปานกลางถึงมาก
อะม็อกซิซิลลิน (amoxicillin) , 1 กรัม ทางหลอดเลือดดา วันละ 4 ครัง้ หรื อ

อีริโธรไมซิน (erythromycin) , 500 มิลลิกรัม ทางหลอดเลือดดา วันละ 4 ครัง้

By…Dew

70. ชาย 65 ปี U/D AAA มีจ ้าเลือดตามตัว PE: multiple ecchymoses, อื่นๆ ปกติ, CBC: Hb 10, WBC 7,200
(N 75, L 25), Plt 30,000, PT, PTT, TT: prolong, fibrinogen decrease, จะตรวจอะไรเป็ นประโยชน์ที่สดุ ใน
การบอกสาเหตุ ?
a. D-dimer
b. Euglobulin lysis time
c. Blood smear
ตอบ จากประวัตมิ ี U/D เป็ น aortic aneurysm มาด้ วย bleeding tendency พบ bicytopenia,
coagulopathy นึกถึง hemostatic disorder จาก aortic aneurysm ทาให้ เกิด turbulence flow  vascular
injury  primary and secondary hemostasis  thrombosis ในหลอดเลือด  RBC แตกเกิดภาวะ
anemia และเมื่อใช้ Plt และ coagulation factor จนลดลงมากจะพบมี clinical bleeding +
thrombocytopenia + coagulopathy ลักษณะค่อยเป็ นค่อยไป : chronic DIC
การส่ ง Lab : blood smear  MAHA blood picture (DDx: DIC, TTP, vasculitis, HUS,
prosthetic heart valve), D-dimer  สูงขึ ้นช่วยในการวินิจฉัยได้ (DDx: thrombosis (PE, DVT), คนแก่, DIC
etc.), Euglobulin lysis time  global nonspecific screen of the fibrinolytic system
สรุป นึกถึงว่ าผู้ป่วยเป็ น chronic DIC  ไม่ มี Lab ที่ตรวจยืนยันการวินิจฉัยที่จาเพาะเจาะจง
ให้ พจิ ารณากันตามข้ อมูลเบือ้ งต้ น
By…Fon
71. ลูกคนแรกเป็ น HbH disease จงหา typing ของแม่และพ่อ
Hb MCV
พ่อ 13 76
แม่ 13 87
ตอบ Thalassemia ถ่ายทอดทางพันธุกรรมแบบ Autosomal recessive ดังนันต้ ้ องมีบดิ าหรื อมารดาเป็ น a-
thal 1 gene (--) และ a-thal 2 gene (a-)
กรณีที่ไม่มีภาวะ anemia  เป็ น a-thalassemia trait, heterozygous HbCS
a-thalassemia trait แบ่งเป็ น a-thal-1 trait (aa,--) และ a-thal 2 trait (aa,-a) สามารถแยกด้ วย MCV
เนื่องจาก a-thal-1 trait มียีนขาดไป 2 ตน. ทาให้ MCV ลดลงโดยไม่ซีด ส่วน a-thal-2 traitยีนขาดหายไป 1 ตน.
ทาให้ MCV ปกติและไม่ซีด
สรุป พ่ อเป็ น a-thalassemia-1 trait (aa,--), แม่ เป็ น a-thalassemia 2 trait (aa,-a) ทาให้ ลูกมี
โอกาสเป็ น HbH disease ได้
By…Fon
72. กินเหล้ าเถื่อน แล้ วมีอาการอาเจียน หายใจหอบ Na? K?
a. Methanol
b. Ethylene glycol
c. Salicylate
d. Paraquet
ตอบ Methanol

วัตถุท่ หี ้ ามใช้ ในอาหาร พิษภัยในการบริโภค


กรดซาลิซิลิค ทาให้ การหายใจลึกถี่ผิดปกติ และทาให้ สมดุลย์ความเป็ นกรด-ด่างของร่างกายเสียไป
(Salicylic acid) ลักษณะอาการเฉียบพลัน ถ้ าอ่อน ๆ จะมีอาการปาก คอไหม้ หายใจถี่ อาเจียน หูอื ้อ
อาการปานกลางจะหายใจถี่มาก มีอาการง่วงซึม ขี ้ตกใจ เพ้ อคลัง่ เหงื่อออกมาก ถ้ ารุนแรงจะ
มีอาการฃัก หมดสติ ผิวหนังเป็ นสีเขียวเนื่องจากการขาดออกซิเจน โลหิตเป็ นพิษ
ลักษณะพิษเรื อ้ รัง ได้ แก่ หูอื ้อ มีเลือดออกในกระเพาะหรื อไต มีแผลในกระเพาะอาหาร
น ้าหนักลด จิตใจเสื่อมลง ผิวหนังพุพอง
เมธิลแอลกอฮอล์ เมื่อเข้ าสูร่ ่างกายจะถูก oxidize ได้ ช้ากว่าเอทธิลแอลกอฮอล์มาก แม้ จะผ่านไป 2 วันก็ยงั
(Methyl alcohol) พบว่า เหลือตกค้ างในร่างกายอีก 1 ใน 3 การเผาไหม้ ในร่างกายจะทาได้ ไม่สมบูรณ์จะถูก
เปลี่ยนเป็ นฟอร์ มาลดีไฮด์ และกรดฟอร์ มิคซึง่ จะมีความเป็ นพิษกว่าเมธิลแอลกอฮอล์ ถึง 6 -
60 เท่า เมธิลแอลกอฮอล์ มีความระคายเคืองสูงทาให้ เป็ นตะคริวในช่องท้ อง อาเจียน สายตา
พร่ามัว ม่านตาขยายและไม่ตอบสนองต่อแสงร่างกายมีความเป็ นกรด การหายใจลาบาก
ผิวหนังเป็ นสีเขียว เนื่องจากเลือดขาดออกซิเจน การหายใจและระบบหมุนเวียนล้ มเหลว อาจ
มีอาการเพ้ อคลัง่ หรื อหมดสติ เป็ นเวลาหลายชัว่ โมงหรื อหลายวัน และตายในที่สดุ หากโชคดี
หายก็มกั จะตาบอดถาวร
ไดเอทธิลีนไกลคอล เมื่อเข้ าสูร่ ่างกายจะถูกเปลี่ยนเป็ นกรดอ็อกซาลิก (oxalic acid) ซี่งมีพิษทาลายสมองและ
(Diethylene glycol) การทางานของไต และทาให้ เกิดโลหิตจาง
พิษเฉียบพลัน ทาให้ มีอาการอาเจียนปวดหัว หัวใจเต้ นเร็ว หายใจถี่ ความดันต่า
กล้ ามเนื ้ออ่อนกาลัง มึนงง หมดสติ ชักอาจตายภายในไม่กี่ชวั่ โมงจากระบบการหายใจ
ล้ มเหลว หรื อตายภายใน 24 ชัว่ โมง จากน ้าขังที่ปอด ผู้ป่วยที่หมดสติหรื อชักเป็ นเวลานาน
สมองอาจถูกทาลายโดยไม่สามารถฟื น้ เป็ นปกติได้

ภาวะเป็ นพิษจาก methanol (wood alcohol) ส่วนใหญ่เกิดจากการดื่มสุราที่มี methanol ปนเปื อ้ นอยู่


มักจะพบ methanol ในสุราปลอม เหล้ าเถื่อนที่ต้มกลัน่ เอง หรื ออาจจะเกิดจากความเข้ าใจผิดคิดว่า methanol
เป็ น ethanol โดยทัว่ ไป methanol ใช้ เป็ นตัวทาละลาย ซึง่ เป็ นส่วนประกอบของของใช้ ตา่ งๆ อีกมาก
ผู้ป่วยอาจมีอาการของทางเดินอาหารเกิดจาก mucosal irritaion เช่น คลื่นไส้ อาเจียน และปวดท้ อง
อาการที่คอ่ นข้ างจาเพาะได้ แก่ พิษทางตา ผู้ป่วยจะมีอาการตาพร่า ตามัว แพ้ แสง เห็นภาพขาวจ้ าไปหมด
(snowfield vision) ถ้ าดู fundi อาจเห็น retinal edema และ hyperemia ของ optic disc อาการทาง CNS
ได้ แก่ ปวดศีรษะ เวียนศีรษะ อ่อนเพลีย และ confusion ในรายที่เป็ นมากอาจมี อาการ coma และชัก
นอกจากนี ้ ผู้ป่วยยังมีอาการ hyperpnea จาก acidosis ในผู้ป่วยที่เป็ นรุนแรง acidosis จะยิ่งมาก ลักษณะเป็ น
แบบ high gap metabolic acidosis และถ้ าวัด serum osmolality อาจจะพบ osmolal gap ร่วมด้ วย
โดยสรุ ปสิ่งสาคัญในการช่ วยการวินิจฉัยโรคคือ ประวัตผิ ้ ูป่วยที่ด่ มื สุรา มีอาการพิษทางตา และ
high gap metabolic acidosis เป็ น toxic syndrome ที่จะบอกถึงภาวะเป็ นพิษจาก methanol
By…Fon

73. ผู้ชายอายุ 20 ปี ถูกสุนขั หน้ าโรงหนังกัด แผลขนาด 1*1 cm หลังจากทาแผลให้ TT แล้ วจะป้องกัน Rabies
ยังไง
a. ให้ Rabies vaccine
b. ให้ Rabies Ig
c. แนะนาให้ สงั เกตอาการสุนขั อีก 1 สัปดาห์
d. ตัดหัวสุนขั ส่งตรวจ
e. ให้ Rabies vaccine และ Rabies Ig
ตอบ e. ให้ Rabies vaccine และ Rabies Ig
การฉีด Rabies vaccine แบบ post exposure ในคนที่เคยได้ รับวัคซีนมาก่อน
 ถ้ าสัมผัสโรคภายใน 6 เดือนหลังเข็มสุดท้ ายให้ ฉีดเพิ่ม 1 เข็ม stat
 ถ้ าฉีดเข็มสุดท้ ายเกิน 6 เดือน ให้ ฉีดเพิ่ม stat และ dayที่ 3
 ไม่ต้องฉีด Ig
ข้ อบ่งชี ้ของ Rabies Immunoglobulin(RIG)
1. ถูกกัดเป็ นแผลที่ใบหน้ า ศีรษะ คอ มือ และนิ ้วมือ
2. แผลลึก หรื อแผลฉีกขาดมาก
3. ถูกกัดหลายแผลและมีเลือดออก
4. มีบาดแผลและไม่มนั่ ใจว่าสัตว์ไม่เป็ นโรคพิษสุนขั บ้ า
5. ถูกเลียที่เยื่อเมือก หรื อน ้าลายสัตว์ถกู เยื่อเมือก ตา ปาก
6. มีแผลที่ผิวหนัง และสัมผัสเนื ้อสมองสัตว์
7. ผู้ที่มีภมู ิค้ มุ กันบกพร่อง
แนวทางการให้ ภมู ิค้ มุ กันบาดทะยัก
บาดแผลสะอาด บาดแผลสะอาด บาดแผลมีโอกาสติด บาดแผลมีโอกาสติด
เชื ้อ เชื ้อ
ประวัตกิ ารได้ รับวัคซีน TT TIG TT TIG
ได้ ไม่ครบ, จาไม่ได้ ให้ ไม่ให้ ให้ ให้
ครบ 3 ครัง้ นาน < 5ปี ไม่ให้ ไม่ให้ ไม่ให้ ไม่ให้
ครบ 3 ครัง้ นาน 5-10 ไม่ให้ ไม่ให้ ให้ ไม่ให้
ปี
ครบ 3 ครัง้ นาน>10ปี ให้ ไม่ให้ ให้ ไม่ให้

74. ผู้ป่วยชายหมดสติมาที่ ER ผลการทา EKG เป็ นดังรูป


(VF) การรักษาที่เหมาะสมที่สดุ ในผู้ป่วยรายนี ้คือข้ อใด
a. Defibrillation
b. Atropine injection
c. Adenosine injection
d. Adrenaline injection
ตอบ a. Defibrillation
มี EKG ที่ shockable 3 แบบ คือ ventricular
fibrillation, pulseless ventricular tachycardia,
Torsade de Pointes

Advanced Cardiac Life Suppor


75. หญิงอายุ 35 ปี มาที่ ER ประวัตวิ า่ กินส้ มตาปูปลาร้ า พริกขี ้หนู 10 เม็ด หลังจากนัน้ 2 ชัว่ โมง มีอาการชา
รอบปาก น้ องสาวที่ทานด้ วยกันก็มีอาการเช่นเดียวกับผู้ป่วย PE: BT 37oc , BP 140/90 mmHg, HR 120/min,
RR 14/min with shallow breathing, motor grade II/V, reflex 1+ ถามว่าเกิดจากพิษอะไร
a. Saxitoxin
b. Tetrodotoxin
c. Botulinum toxin
Botulinum Tetrodotoxin/Saxitoxin
1. ระยะเวลาฟั กตัว 1-8 วัน 1/2-12 ชัว่ โมง
2. ลักษณะกล้ ามเนื ้ออ่อนแรง เริ่มจากใบหน้ า,กล้ ามเนื ้อ แขนขาไปสูก่ ล้ ามเนื ้อหายใจ
หายใจไปสูแ่ ขน ขา
3. ประสาทความรู้สกึ ไม่มีความผิดปกติ มีชาที่ลิ ้นรอบปาก และ ปลายมือ-เท้ า
4. อาหารที่เป็ นสาเหตุ อาหารกระป๋ อง,หน่อไม้ ปี๊บ ปลาปั กเป้า, ไข่แมงดาทะเล
5. ความทนต่อความร้ อนของสารพิษ ไม่ทน ทนความร้ อนได้ ดี

 Saxitoxin กิ นปลาปักเป้ า
ระยะเวลาที่เกิดอาการ : 15 นาที
กลไก : sodium channel blocker, paralytic shellfish poisoning (PSP)
อาการ : ทาให้ เกิดเวียนศรี ษะ อัมพาต ทาลายระบบประสาท การหายใจล้ มเหลว เสียชีวิตใน 2-12 hr
 Tetrodotoxin กิ นปลาปักเป้ าและ สัตว์ตา่ งๆเช่น กบ ปลาหมึกยักษ์ ปู หอย
ระยะเวลาที่เกิดอาการ : 20 นาที – 3 ชัว่ โมง
กลไก : ยับยังการขนส่
้ งโซเดียมไอออน ของระบบประสาทส่วนกลางและส่วนปลายรวมทังหั ้ วใจ
อาการ : ทาให้ เกิดการชาที่ริมฝี ปาก ลิ ้น ใบหน้ า ปวดศีรษะ ปวดท้ อง อาเจียน ท้ องเดิน เคลื่อนไหว
ลาบาก พูดไม่ชดั ความดันเลือดต่า เสียชีวิตใน 4-6 ชัว่ โมง
 Botulinum toxin เกิดจาก Clostridium botulinum
ระยะเวลาที่เกิดอาการ : 12-36 ชัว่ โมง (aerosol), 1-3 วัน (food)
กลไก : ขัดขวางการหลัง่ acetylcholine ที่เซลล์ของระบบประสาท
อาการ : - ทาง aerosol ทาให้ กล้ ามเนื ้ออ่อนแรง ปากคอแห้ ง ตาพร่า อัมพาต การหายใจล้ มเหลว
- ทาง food ทาให้ เกิด food poisoning (botulism) ปวดท้ อง ท้ องเดิน เวียนศรี ษะ ทาให้
กล้ ามเนื ้ออ่อนแรง อัมพาต การหายใจล้ มเหลว เสียชีวิตใน 2-3วัน
การรักษา :- รักษาตามอาการเช่น ให้ เครื่ องช่วยหายใจ
- ฉีด equine botulinum antitoxin (type A,B, E)
- ถ้ าถูกผิวหนังใช้ น ้าและสบูล่ ้ าง หรื อใช้ 0.5% hypochlorite solution
การป้องกัน : ฉีด vaccine
โชคดีน๊าเพื่อน ๆ^^เหมียวอ้ วนแต่นา่ รักดี
79. ผู้ป่วยชายไทยรายหนึง่ ตรวจร่างกายประจาปี มีผลเลือดดังต่ อไปนี ้ TG = 210 LDL = 200 HDL =30 ยา
ในข้ อใดน่าจะเหมาะสมกับผู้ป่วยรายนี ้มากที่สดุ
a. Gemfibrozil b. Simvastatin c.Fibric acid d. Niacin e. Cholestyramine
b. Fenofibrate
ตอบ LDL = total cholesterol – HDL – TG/5
ในผู้ป่วยรายนี ้ cholesterol = 272
มีภาวะ hypercholesterolemia and hypertriglyceridemia ซึง่ เข้ าได้ กบั Familial combined
Hyperlipidemia คือมี mixed dyslipidemia (plasma TG = 200-800 mg/dL ,cholesterol= 200-400 mg/dL
,HDL < 40 mg/dL)
หลักการรักษา คือ ควบคุมอาหาร หลีกเลี่ยงเครื่ องดื่มที่มีแอลกฮอล์และ ยากินคุมกาเนิด และมักใช้
HMG-CoA reductase inhibitor ร่วมกับ niacin ในการรักษาด้ วย
ในผู้ป่วยรายนี ้จึงพิจารณาให้ simvastatin ตามเหตุผลในตาราง
Hypolipidemic drugs lipoprotein class effected common side effects contraindications
HMG-CoA reductase inhibitors

Lovastatin 20-80 mg/d decrease(dec.) LDL 25-55 %

Pravastatin 40-80 mg qhs decrease TG 10-20 % acute or chronic liver


Simvastatin 20-80 mg qhs increase(inc.) HDL 5-10% disease of myositis
Myalgias,arthralgias ,increase
increase by impaired
Fluvastatin 20-80 mg qhs trasaminases, dyspepsia
renal function และเมื่อใช้
Atorvastatin 10-80 mg qhs ร่วมกับ fibrate

Rosuvastatin 10-40 mg qhs

Nicotinic acid dec.LDL 15-25%


flushing, hepatic dysfunction, nausea,
Peptic ulcer disease,
dec.TG 25-35% diarrhea, glucose intolerance,
hepatic disease gout
hyperuricemia
inc.15-30%

Bile acid sequestrants

Cholestyramine 4-32g qd dec.LDL 20-30%

cholestipal 5-40 g qd inc.TG 10% Constipation ,gastric discomfort biliary tract obstruction
,nausea ,gastric outlet obstruction
colesevelam 3750-4375 mg
inc.HDL 5%
qd
fibric acid derivatives

Gemfibrozil 600 mg bid inc. or dec. LDL hepatic or biliary disease,


dec.absorption of other drug ,inc.gall
renal insufficiency
Fenofibrate 160 mg qd dec.TG 25-40% stone, dyspepsia ,hepatic
associated with inc. Risk
dysfunction, myalgia
inc.HDL 5-15% of myositis
By....ว่ าน
80. ผู้ป่วยชายอายุ 56 ปี มาด้ วยเจ็บแน่นหน้ าอกด้ านซ้ าย 2 ชัว่ โมง เป็ น DM ,HT มา 10 ปี ให้ EKG มาเป็ น
STEMI in V1-V4 หลังจากให้ ASA ไปแล้ วการรักษาใดเหมาะสมที่สดุ
a. Enoxaparin b. Warfarin c. Streptokinase d. Balloon e. PCI
ตอบ หลักการรักษา STEMI
• Initial therapy
Goal คือ 1.quickly identify เพื่อให้ ผ้ ปู ่ วยได้ รับ reperfusion therapy
2. relieve pain
3. ป้องกัน, รักษา arrhythmia
 ให้ ASA 162-325 mg เคี ้ยวเมื่อมีอาการ จากนันให้ ้ กิน 162-325 mg/d
 ซักประวัติ ตรวจร่างกาย ทา EKG
 Primary PCI จะ effective มากกว่าการให้ fibrinolytic แต่จะทาได้ เฉพาะโรงพยาบาลที่มีเครื่ องมือ
เตรี ยมพร้ อมและบุคลากรที่มีความเชี่ยวชาญ
 ให้ IV fibrinolysis หากว่าทา PCI ไม่ได้ หรื อ ก่อนการเริ่ มทาPCI ใช้ เวลานานมากกว่า 1 ชัว่ โมง
fibrinolytic ให้ ภายใน 1-3 ชัว่ โมงจะดีที่สดุ และสามารถให้ ได้ ถึง 12 ชัว่ โมงหากยังมีอาการเจ็บหน้ าอก
หรื อ EKG ยังมี ST elevate อยูไ่ ม่เปลี่ยนเป็ น Q wave แต่จะมีภาวะแทรกซ้ อนตามมาได้ คือ bleeding
reperfusion arrhythmia
 หากยังคงมี chest pain or ST elevate อยู่ หลังให้ fibrinolysis ไปแล้ วมากกว่า 90 นาทีให้ refer ไปเพื่อ
ทา PCI

By…ว่ าน
81. ชาย 56 ปี เป็ นโรคตับแข็งมา 2 ปี ถ่ายเหลว 3-4 ครัง้ ต่อวันมา 3 วัน PTA วันนี ้ซึมลง BT=37.3 PR=98
RR=18 BP=104/80 stupor, mild jaundice ,mild ascites ,spider nevi positive ,palmar erythema จะส่ง
investigation ใดเพื่อช่วยในการวินิจฉัยมากที่สดุ
a. CBC b. Stool exam c. Coagulogram
d. LFT e. abdominal paracenthesis
ตอบ
จะส่งอะไรดีอะ ข้ อเนี่ย..................
ถ้ าเอาความคิดของเค้ านะ คนนี ้น่าจะเป็ น Hepatic encephalopathy อะแล้ วพอนึกถึง hepatic
encep. ก็ต้องประเมิน Child-Pugh criteria ใช่ปะ

Factor 1 2 3
serum
<2 2 ถึง 3 >3
bilirubin(mg/dL)
serum
>3.5 3 ถึง 3.5 <3
albumin(g/dL)
Ascites None Easily control poorly control
Advanced
Neuroloic disorder None Minimal
coma
Prothrombin time 0 ถึง 4 4 ถึง 6 >6

ซึง่ ถ้ าจะประเมิน class อะนะจาก choice ก็เลือกได้ 2 ตัวคือ ข้ อ c. กับ d.


By…ว่ าน
85. ผู้ป่วยหญิงอายุ 50 ปี ปวดแขนขามา 4 สัปดาห์ นัง่ แล้ วลุกขึ ้นยืนลาบาก ขึ ้นลงบันไดลาบาก ตรวจร่างกายพบ purple-red
discoloration ที่หน้ าผาก แก้ มและเปลือกตา purple nodule ที่ศอกและเข่า คิดว่าคนนี ้เป็ นอะไร
a. Psoriasis
b. Dermatomyositis
c. Discoid LE
d. SLE
e. Mixed connective tissue
ตอบ : Dermatomyositis
Case นี ้มี proximal muscle weakness (นัง่ แล้ วลุกขึ ้นยืนลาบาก ขึ ้นลงบันไดลาบาก) และมี skin rash purple-red
discoloration ที่หน้ าผาก แก้ มและเปลือกตา จึงคิดว่าน่าจะเป็ นโรคนี ้มากที่สดุ
Criteria to define Polymyositis/Dermatomyositis
(PM – เอา 3 ใน 4 ข้ อบน, DM – เอา 3 ใน 4 ข้ อบน ร่วมกับมีข้อ 5)
1. Proximal muscle weakness (insidious, symmetric, progressing over wks. to mos., ocular & facial
normal ไว้ แยกจากพวก MG)
2. Elevated serum level of skeletal muscle enzyme (Creatine kinase)
3. Myopathic change (EMG)
4. Muscle biopsy evidence of inflammation
5. Skin rash
- Heliotrope rash : reddish purple discoloration on upper eyelid
- Grotton’s sign : violaceous papules, sometimes scaly over the knuckles
Psoriasis : skin lesion (erythematous, sharply demarcated papules and rounded plaqes cover by silvery scale), มี
Auspitz’s sign (ถ้ าเอา scale ออกจะมีจดุ เลือดออก), lesionขึ ้นตามศีรษะ ลาตัว extensor area และ ที่เล็บซึง่ พบได้ 50%
(pitting nail, onycholysis เป็ นขุน่ ๆ, subungual hyperkeratosis, yellowish discoleration)

Mixed connective tissue : overlapping features of SLE, Scleroderma, and Polymyositis

Reference
- เอกสารประกอบการเรี ยน Medicine ปี 5

86. ชายอายุ 30 ปี ท้ องเสีย 3 เดือน Cachexia, muscular wasting รูปไข่พยาธิคล้ ายถัว่ ลิสง (Capillaria philippinensis) ควร
มีวิธีการป้องกันอย่างไร
a. งดกินปู
b. งดกินหอย
c. งดกินผักสด
d. งดกินปลาน ้าจืดดิบๆ
e. งดเดินเท้ าเปล่าตามทุง่ นา
f. งดดื่มน ้าที่ยงั ไม่ได้ ผา่ นการต้ มสุก
ตอบ งดกินปลานา้ จืดดิบๆ

โรคพยาธิเเคปิ ลลาเรี ย เป็ นโรคท้ องร่วงเรื อ้ รังที่เกิดจากการกินปลาดิบ นอกเหนือไปจากโรคพยาธิใบไม้ ในตับ โรคพยาธิ


ใบไม้ ในลาไส้ เเละโรคพยาธิตวั จี๊ด โดยปลาที่นามาปรุงเป็ นอาหารเป็ นพวกปลาน ้าจืด ที่ปรุงไม่สกุ เช่น ก้ อยปลา ก็จะได้ รับตัว
อ่อนพยาธิระยะติดต่อที่อยูใ่ นลาไส้ ของปลาเข้ าไป พยาธิตวั กลมที่เป็ นสาเหตุคือ Capillaria philippinensis

อาการสาคัญเเละพบได้ บอ่ ยคือ อุจจาระร่วงเรื อ้ รังเเละถ่ายครัง้ ละมากๆ วันละ 5-10 ครัง้ ส่วนอาการอย่างอื่นเช่นที่พบ
ร่วมก็คือ ท้ องลัน่ โครกคราก ปวดท้ อง ท้ องโตกดเจ็บ คลืน่ ไส้ อาเจียน เบื่ออาหาร ทาให้ เกิดภาวะขาดสารอาหาร ผู้ป่วยจะผอม
เเห้ ง ไม่มีเรี่ ยวเเรง กล้ ามเนื ้อลีบ ความดันเลือดต่า ร่างกายอ่อนเเอ ภูมิต้านทานโรคลดลง เเละอาจติดเชื ้อรุนเเรงถึงตายได้
การตรวจอุจจาระของผู้ป่วย จะพบได้ ทงพยาธิ ั้ ตวั เต็มวัย ตัวอ่อนเเละระยะไข่
การรักษา เนื่องจากผู้ป่วยมีร่างกายอ่อนเพลีย จึงต้ องให้ น ้าเกลือเเร่ เเละอาหารที่มีโปรตีนสูงๆ ยาที่ให้ ผลดีที่สดุ คือ
mebendazole 200 mg 1x2 นาน 20 วัน เเละ albendazole 400 mg 1x1 นาน 10 วัน
การป้องกัน โดยการกินปลาน ้าจืดที่ปรุงสุกเเล้ ว และการถ่ายอุจจาระลงส้ วมให้ ถกู สุขลักษณะ

พยาธิ ทีต่ ิ ดต่อทางการไชเข้าผิ วหนัง : Hookworm, Strongyloides stercoralis, Schistosoma (ใบไม้เลือด)


พยาธิ ทีต่ ิ ดต่อโดยการกิ นพืชน้า : Fasiolopsis buski (ใบไม้ลาไส้)
พยาธิ ทีม่ ี ระยะติ ดต่ออยู่ในหอย ตะกวด : Angiostrongylus cantonensis (เยือ่ หุม้ สมองอักเสบ)
พยาธิ ทีต่ ิ ดต่อโดยการกิ นหมูไม่สกุ : Trichinella spiralis (ปวดบวมกล้ามเนือ้ ), Taenia solium
พยาธิ ทีต่ ิ ดต่อทางการกิ นเนือ้ วัวไม่สกุ : Taenia saginata
พยาธิ ทีต่ ิ ดต่อโดยการกิ นไก่ ปลา นก หนู ไม่สกุ : Gnathostoma spinigerum (ตัวจี ๊ด)
พยาธิ ทีต่ ิ ดต่อทางการกิ นปลาเกล็ดขาวไม่สกุ : O. viverrini (ใบไม้ตบั )
พยาธิ ทีต่ ิ ดต่อทางการกิ นปู กุ้ง ไม่สกุ : Paragonimus (ใบไม้ปอด)

Reference :
- นิมิตร มรกต, เกตุรัตน์ สุขวัจน์. ปรสิตวิทยาทางการแพทย์โปรโตซัวและหนอนพยาธิ. เชียงใหม่:โครงการตารา คณะ
แพทยศาสตร์ มหาวิทยาลัยเชียงใหม่, 2546. พิมพ์ครัง้ ที่ 2
- วันชัย มาลีวงษ์, ผิวพรรณ มาลีวงษ์ , นิมิตร มรกต. ปรสิตวิทยาทางการแพทย์โปรโตซัวและหนอนพยาธิ. ขอนแก่น: โรงพิมพ์
คลังนานาวิทยา, 2544.
- ประยงค์ ระดมยศ. Atlas of Medical Parasitology

87. หญิงอายุ 42 ปี เดินทางมาจากเชียงใหม่ มาตรวจ FU มะเร็งเต้ านมที่ รพ.กรุงเทพฯ ขณะกาลังลงจากรถทัวร์ มีอาการเจ็บ


อกด้ ายขวา และหายใจหอบเหนื่อย PE: PR 128 /min RR 30 /min lung-clear, Rt leg-Swollen & tenderness เข้ ากับโรคใด
มากที่สดุ
a. Pleuodynia
b. Pneumonia
c. Pulmonary embolism
d. Acute coronary syndrome
e. Dissected aortic aneurism
ตอบ Pulmonary embolism
อาการและอาการแสดงใน PE
- หายใจลาบาก, เจ็บหน้ าอกเวลาหายใจลึก, ไอ, ขาบวม, ปวดขา, ไอเป็ นเลือด, ใจสัน่ , หายใจมีเสียงวี ้ด, เจ็บหน้ าอกตื ้อ-แน่น
- tachycardia, respiratory crackles, S4 (Rt sided) gallop, accentuated P2, fever, circulatory collapse
การประเมินความน่าจะเป็ นทางclinic ของการเกิด PE (ของ Wells)
- อาการและอาการแสดงของ DVT 3 คะแนน
- การ Dx. อื่นๆเป็ นไปได้ น้อยกว่า DVT 3 คะแนน
- HR > 100 bpm 1.5 คะแนน
- Sx. หรื อ มีการเคลือ่ นไหวจากัดใน 4 wk ก่อน 1 คะแนน
- เคยเป็ น DVT หรื อ PE 1 คะแนน
- ไอเป็ นเลือด 1 คะแนน
- CA (กาลังรักษาหรื อภายใน 6เดือน) 1 คะแนน
<2 ความน่าจะเป็ นต่า
2-6 ความน่าจะเป็ นปานกลาง
>6 ความน่าจะเป็ นสูง
case นี ้มีอาการและอาการแสดงของ DVT (เนื่องจากมีขาบวมข้ างเดียว), HR > 100 bpm , CA และการ Dx. อื่นๆ
เป็ นไปได้ น้อยกว่า DVT รวมเป็ น 8.5 คะแนน ความน่าจะเป็ นสูง
Dissected aortic aneurism : อาการที่พบบ่อยคือ อาการเจ็บหน้ าอกแบบ sharp stabbing pain (เหมือนของปลาย
แหลมแทง) รุนแรงมากและทันทีทนั ใด เริ่ มจากตาแหน่งที่มี dissection ร้ าวไปตามหลอดเลือดที่ถกู เซาะ ตรวจร่างกายพบ
hypertension หรื ออาจมีhypotensionถ้ ามีcardiac temponadeเป็ นต้ น และตรวจพบpulse deficit

Reference
- ตาราโรคระบบการหายใจ ของ สมาคมอุรเวชช์แห่งประเทศไทย
- อายุรศาสตร์ ฉกุ เฉิน ของรพ.รามาธิบดี

88. ชายอายุ 18 ปี ไข้ สงู เจ็บคอ 3 วัน ซีด เหลือง PE: moderate pale ,mild jaundice, liver 2 cm below
costal margin, spleen 3 cm below costal margin, Hb 7.9, Hct 28, MCV 65, WBC 9500, N 80%, L 20%,
Plt.450,000 คิดถึงโรคใดมากที่สดุ
a.AIHA
b.HbH with crisis
c.PNH
d.G6PD
e.DIC
ตอบ b. HbH with crisis คิดว่าผู้ป่วยน่าจะเป็ น HbH disease อยูแ่ ล้ ว แล้ วผู้ป่วยมีการติดเชื ้อจึงทาให้ เกิด
ภาวะนี ้ได้ เกิดhemolytic anemiaได้
AIHA ตับม้ ามไม่โต
PNH ตับม้ ามไม่โต เคยมีประวัตมิ าก่อน
G6PD ตับม้ ามไม่โต เคยมีประวัตมิ าก่อน
DIC ตับม้ ามไม่โต Plt.ต่า PT &PTT prolonged
By ……Sa-ou X.054

89. ชายอายุ 50 ปี ปวดเข่าขวามาก มีไข้ ปวดมาก เดินไม่ได้ PE: Rt. Knee swelling, tender ต้ องส่งอะไรต่อ
a.x-ray
b.synovial analysis
ตอบ b. Pt.มีอาการเป็ นแบบเฉียบพลัน เป็ นข้ างเดียว และมีไข้ น่าจะเป็ น infection (acute arthritis ) ควรส่ง
synovial analysis เพื่อแยกว่าเป็ นseptic arthritis หรื อเปล่า x-ray ไม่ได้ ชว่ ยอะไร
By ……Sa-ou X.054
90. Pt. U/D Cirrhosis มีท้องเสีย ถ่ายเหลว 3-4 ครัง้ /วัน เป็ นมานาน 3 วัน ซึม PE: พบsign Chronic liver
disease &moderate ascites จะส่ง investigationเพื่อช่วย Dx.
a. Abdominal paracentesis
b. stool exam
c. LFT
d.CBC
ตอบ a. Abdominal paracentesis คิดว่าสงสัย SBP มากที่สดุ เลยส่งอันนี ้น่าจะดีที่สดุ น่ะ (เพราะว่าSBP อาจ
มาด้ วยท้ องเสียได้ และมักพบได้ ในคนที่ hostไม่ดี)
Stool exam ก็สง่ ได้ ถ้าสงสัย strongiloid.
By ……Sa-ou X.054

91. ชายอายุ ...ปี มีไข้เจ็บคอ PE: tonsil enlargement grayish patch at pharynx and tonsil gland anterior
cervical lymphanode enlargement both side no hepatomegaly no splenomegaly ผูป้ ่ วยรายนี้ น่าจะคิดถึง
โรคใดมากที่สุด
a. Diptheria

ตอบ ข้ อ a. diphtheria เนื่องจาก ผูป้ ่ วยมีไข้ เจ็บคอ tonsil gland enlargement มีลกั ษณะ grayish patch at
pharynx and tonsil ซึ่งเป็ นลักษณะเฉพาะของ diptheria ซึ่ง patch จะดูค่อนข้าง สกปรก สามารถมีต่อมน้ าเหลืองที่ คอ
โตได้ แต่ไม่ค่อยพบว่า มี ตับม้ามโต และที่สาคัญควรมีประวัติไม่ได้รับการฉี ดวัคซีน DPT หรื อได้ไม่ครบด้วย ซึ่งไม่ได้ให้มาใน
ข้อนี้ ส่วนchoice อื่นไม่รู้ ก็ขอ DDx ว่า ถ้าเป็ น infectious mononeucleosis ควรมี generalized LN
enlargement +hepatosplenomegaly
ถ้าเป็ น scarlet fever ควรมีผื่น แบบ sand paper (groose fresh) appearance
ถ้าเป็ น Vincent ‘s angina ควรเป็ นพวก poor oral hygiene
สาเหตุของ exudative tonsillitis หรือ มี patch ที่ tonsil gland ต้อง differential ดังต่อไปนี้
1. Diphtheria :ผูป้ ่ วยจะ มีไข้ต่าๆ เจ็บคอ อ่อนเพลีย มี ตรวจพบ เยือ่ สี ขาวแกมเทาที่บริ เวณ pharynx tonsil
(grayish patch) ถ้ามีเลือดออกจะดูสกปรกสี คล้ า(dirty patch) ติดแน่นกับเนื้อเยือ่ ถ้าขูดออกจะมีเลือดซึ ม พบ
cervical lymphadenopathy ในรายที่เป็ นรุ นแรงจะมีลกั ษณะ คางและคอบวมคล้ายคอวัว เรี ยกว่า Bull neck
บางรายมีอาการ toxemia จาก exotoxin ซึ่งมีผลต่อ กล้ามเนื้อหัวใจ เกิด myocarditis (10-25%) ,heart
block , neuritis ในรายที่เป็ นรุ นแรงจะพบว่ามีupper airway obstruction จาก membrane ได้ เกิดจาการ
ติดเชื้อ Corynebacteriumdiptheria ในผูป้ ่ วยที่มีประวัติไม่ได้ฉีดวัคซีนป้ องกันคอตีบ
2. Scarlet fever: ไข้สูง เจ็บคอ เกิดจากเชื้อbeta-haemolytic streptococcus พบ erythematous tonsil
gland enlargement , erythematous pharyngeal mucosa อาจพบ exudates บน tonsil gland ร่ วมกับ
petechiae ที่ soft palate and uvula ลิ้นมีลกั ษณะ red strawberry tongue หลังไข้ลงพบ ผืน ่ นูนแดง
แบบ erythematous punctiform eruption (sand paper like appearance) ตามลาตัว แขน ขา หนาแน่น
ที่รักแร้ ขาหนีบก้น ที่ใบหน้าจะไมค่อยมีผื่น แต่จะพบ circumoral pallor
3. Infectious Mononucleosis:เกิดจาก EBV มักพบในเด็กเล็กมากกว่า มาด้วย ไข้ 38-39 C เจ็บคอ
อ่อนเพลีย ตรวจพบ clean white patch on tonsil gland หลุดลอกไปเองภายใน 5-8 วัน เด่นที่ generalized
lymphadenopathy ถ้า มี epitrochear node ยิง่ specific พบ splenomegaly ได้บ่อย มีหนังตาบวมได้
เรี ยก Hoagland sign มักพบ posterior cervical lymphadenopathy, generalized
lymphadenopathy, อาจมีมา้ มโตได้, peripheral blood smear จะcharacteristic คือมี atypical
lymphocytosis. Heterophile antibody ช่วยในการวินิจฉัย
4. Vincent's anginaหรือ trench mouth disease:ลักษณะเฉพาะ มีเยือ่ สี เทาปนเหลือง หรื อ
pseudomembraneสี เทา เขี่ยออกง่าย tissue ที่สาคัญคือจะพบulcer ที่เหงือก, ปาก,และpharynx ถ้านาไป
smear จะ พบ Vincent organism เกิดจากmixed infection ระหว่างเชื้อspirocheteชื่อ Fusobacterium
fusiforme กับ Bacteroides sp.ซึ่ งเกิดจากnecrotic พบในคนไข้poor oral hygiene

5. Group A beta hemolytic streptoccus tonsillitis เป็ น most common cause of acute
tonsillitis มีไข้สูง เจ็บคอ กลืนลาบาก อ่อนเพลีย ตรวจพบ injected pharynx enlarge tonsil gland
ลักษณะเป็ นแบบ exudative tonsillitis มักพบต่อมน้ าเหลืองที่คอโต กดเจ็บ
6. Agranulocytosis จะพบ exudate บน tonsil gland และพบที่ posterior pharyngeal wall ด้วย อาจมี
แผลลุกลามในปากถึงโคนลิน้ มีอาการกลืนลาบาก เจ็บปวดมาก รวมทั้ง มีจุดเลือดออกที่เยือ่ บุในปาก พบ cervical
lymphadenopathy

อ้างอิง: ตารากุมารฯ รามา เล่ม1 ,ปั ญหาโรคเด็กที่พบบ่อย ,ตารากุมารฯ จุฬา เล่ม ๓


เรี ยบเรี ยงโดย นิสิตแพทย์ภทั รพล คามุลตรี pattarapon_bump@hotmail.com

92. ชายอายุ 23 ปี มาโรงพยาบาลด้วย อาการเจ็บหน้าอก หอบเหนื่อยนอนราบไม่ได้ วินิจฉัย congestive heart failure


ต่อมาเสี ยชีวติ ตรวจศพ พบ lymphocyte infiltrate ที่ กล้ามเนื้อหัวใจ เกิดจากเชื้อในข้อใด
a. influenza virus
b. adenovirus
c. coxsackie B virus
d. EBV
ตอบ ข้ อ C. Coxsackie B virus
เนื่องจาก coxsackie B virus เป็ นสาเหตุหนึ่งของ dilated cardiomyopathy ที่เกิดจาก infection ที่พบได้
บ่อยที่สุด โดยพบได้ 50 % ของ viral myocarditis เมื่อตรวจทางพยาธิวทิ ยาจะพบลักษณะ Lymphocyte infiltration
ในกล้ามเนื้อหัวใจ
Cardiomyopathy เป็ นโรคของกล้ามเนื้อหัวใจที่ ทางานได้ไม่ดีเหมือนปกติ ที่เกิดจากสาเหตุต่างๆ ยกเว้น จาก
myocardial infarction ,valvular heart disease or hypertension ซึ่ งแบ่งเป็ น 3 ชนิด คือ
2. dilated cardiomyopathy: most common cardiomyopathy จะพบลักษณะ left ventricular
dilatation และ systolic dysfunction (low EF) สาเหตุได้แก่

Chronic excessive alcohol consumption


Other drugs: Anthracyclines (daunorubicin and doxorubicin) Heavy metals
Emetine Cocaine Methamphetamine

Infections

Viral endocarditis/myocarditis ได ้แก่ coxsackievirus, adenovirus,


parvovirus, human immunodeficiency virus [HIV]) Parasites

Protozoa

Chagas disease (most common cause in parts of South America)

Thiamine deficiency (beriberi) and zinc deficiency

Family : genetic

ABCDs cause of dilated cardiomyopathy: Alcohol Beriberi Coxackie B Cocaine Chagas’s


disease Doxorubicin

3. hyperthophic cardiomyopathy:พบลักษณะ LVH ทาให้การคลายตัวของหัวใจไม่ดี เกิด diastolic


dysfunction (EF ปกติหรื อเพิม่ ขึ้น ) สาเหตุ ส่ วนใหญ่เป็ น congenital ได้แก่ idiopathic hypertrophic
subaortic stenosis;IHSSซึ่งถ่ายทอดแบบ autosomal dominant และเป็ นสาเหตุของ sudden death ที่มาก
ที่สุดในเด็ก และนักกีฬา
4. restrictive cardiomyopathy :มีการสูญเสี ย elasticity ของกล้ามเนื้ อหัวใจ ทาให้ในช่วง diastolic filling
รับเลือดได้ไม่ดี EFอาจปกติ หรื อ ลดลง สาเหตุ เกิดจาก infiltrative disease เช่น sarcoidosis
hemochromatosis amloidosis นอกจากนี้ ยงั เกิดจาก scar จาการได้รับรังสี หรื อ doxurubicin

อ้างอิง : First Aid for the USMILE STEP 2 Clinical Knowladge


เรี ยบเรี ยงโดย นิสิตแพทย์ภทั รพล คามุลตรี pattarapon_bump@hotmail.com

93. ผูป้ ่ วยชายอายุ 45 ปี ปวดศีรษะมาก 1 วัน โดยาประมาณ 1-2 เดือน มีอาการใจสัน่ เป็ นพักๆ วันละ 1-2 ครั้ง เคยไปพบ
แพทย์ตรวจพบความดันโลหิ ต 200/120 mmHg ได้ยามารับประทานแต่รับประทานไม่สม่าเสมอ ครั้งนี้ตรวจวัดความดันได้
210/130 mmHg Pulse rate 120 /min Respiratory rate 22/min Eyes: fundoscopic examination
show frame shape hemorrhage with early papilledema ในผูป้ ่ วยรายนี้ นอกจาก furosemide แล้ว จะให้ยาลด
ความดันโลหิ ตในข้อใด
a. Enarapril
b. Propanolol
c. Prazosin
d. Nitroprusside

ตอบ ข้ อ d.nitroprusside เพราะผูป้ ่ วยรายนี้เป็ นภาวะ hypertensive crisis จัดอยูใ่ นกลุ่ม hypertensive
emergency เนื่องจาก มีความดันโลหิ ตสูง >=180 /120 ร่ วมกับมีภาวะที่บ่งชี้วา่ กาลังมีการเสื่ อมของอวัยวะ(target
organ damage) คือ frame shape hemorrhage with early papilledema การรักษาคือ ให้ nitroprusside
0.25-10 ug/kg/min IV drip เพื่อลดความดันโลหิ ตโดยลด mean arterial pressure ลง 25% ใน 2-3 h ซึ่งเป็ นยาที่นิยมใช้
ในภาวะ hypertensive emergency ทุกราย
Hypertensive crises
เป็ นภาวะที่ความดันโลหิ ตสูงรุ นแรงทันที โดยมี SBP >=180 mmHg ร่ วมกับ DBP >= 120-130 mmHg
(อัตราการเพิ่มขึ้นของความดันโลหิ ตสูงมีความสาคัญมากกว่าระดับความดันโลหิ ตในการบอกถึงอันตราย)
Hypertensive crises แบ่งเป็ น 2 กลุ่ม คือ
1 hypertensive emergencies (ความดันโลหิ ตสูงฉุกเฉิ น) = HTN +การทางานผิดปกติหรื อการเสื่อมของอวัยวะ
ที่เกิดขึ้นอย่างต่อเนื่อง (HTN with TODs ;target organs damage) ซึ่งจาเป็ นต้องรี บลดความดันโลหิ ตอย่าง
รวดเร็ วทันที เป็ นนาที หรื อไม่เกิน 1 ชัว่ โมง เพื่อป้ องกันไม่ให้อวัยวะเสื่ อมต่อไป ได้แก่ hypertensive
encephalopathy stroke ,unstable angina ,myocardial infartion ,left side heart failure
,pulmonary edema ,aortic dissection ,pregnancy induced hypertension ซึ่ง การรักษาให้เป็ น IV
antihypertensive drug (ไม่จาเป็ นต้องลดถึงระดับปกติ)
2 hypertensive urgencies (ความดันโลหิ ตสูงเร่ งด่วน)= HTN+ไม่มีความผิดปกติที่บ่งชี้วา่ อวัยวะนั้นกาลังเลือ่ ม
อย่างต่อเนื่อง (HTN without TODs) ได้แก่ HTN stage II (BP >=160/100 mmHg ) มีอาการปวด
ศีรษะรุ นแรง , มีหายใจลาบาก ,เลือดกาเดาไหล ,เครี ยดมาก การรักษา ให้oral antihypertensive drug ที่ ER
และ ติดตามการรักษาภายใน 24 h หรื อหลายวันหลังจากนั้น (ไม่จาเป็ นต้อง admit หรื อลดความดันอย่างรวดเร็ ว)

Management ในภาวะ hypertensive emergency

Patient presentation มีข้อใดข้ อหนึ่ง การรักษา


High risk patient hypertensive ลดความดันเป็ นนาที
encephalopathy IV anti HTN
stroke
unstable angina
myocardial infartion
left side heart failure
pulmonary edema
aortic dissection
pregnancy induced
hypertension
No high risk patient Eye ground : มี ลดความดันภายในเวลาเป็ นชัว่ โมง
-papilledema
-hemorrhage ไม่มี ลดความดันภายในเวลาเป็ นวัน
-exudates
IV antihypertensive drug ใน hypertensive crises
:
Vasodilator ข้ อบ่ งใช้ เริ่มออกฤทธิ์ ออกฤทธิ์นาน Dose Side effect/ข้ อควร
ระวัง
Nitroprusside นิยมใช้ แพร่ หลาย ใช้ได้กบั ทันที 1-2 นาที 0.25-10 N/V กล้ามเนื้ อกระตุก
ผูป้ ่ วยทุกราย ug/kg/min IV เหงื่อออก หน้าแดง พิษจาก
drip thiocyanate and
cyanide
ระวังการใช้ใน :renal
failure ,liver failure

Nitroglycerine -มีภาวะ myocardial 2-5 นาที 3-5 นาที 5-100 ug/min ปวดศีรษะ อาเจียน ดื้อยา
ischemia/infartion IVdrip (tolerance)
เนื่องจากยาจะขยาย methemoglobinuria
coronary a. ด้วย tachycardia
-HTN ระหว่างหรื อหลัง
การการผ่าตัด
Nicardipine ใช้ ได้ เกือบทุกราย ยกเว้น 5-10 นาที 30-60 นาที 5-15 mg/h IV Tachycardia ,head
heart failure /MI (อาจ > 4 drip ache .หน้าแดง ,
ชัว่ โมง) vasculitis
Fenoldopam ใช้ได้เกือบทุกราย โดยเฉพาะ < 5 นาที 30 นาที 0.1-0.3 Tachycardia,
ความดันโลหิ ตสู งที่เกิดจาด mcg/kg/min headache
ไตเสื่ อม(ขยายหลอดเลือดที่ nausea, flushing.
ระวังการใช้ใน
ไต) รวมถึง ในภาวะ
glaucoma.
intracerebral
hemorrhage

Enalapilate ภาวะ left side heart 15-30 นาที 6 ชัว่ โมง 0.625 – 1.25 mg ความดันลดมากในภาวะ
failure หรือ IV q 6 h rennin สูง
congestive heart
failure
Hydralazine Pregnacy induced 10-20 นาที 3-8 ชัว่ โมง 5-10 mg IV q 4-6 Tachycardia,
hypertension h headache, flushing,
(Alternative หรื อ เจ็บหน้าอก
drugs:MgSO4 20-30 นาที 10-50 mg IM
IV/IM ,methyl
dopa ,nicardipine
IV)
Oral antihypertensive drug for hypertensive crises

Drug ลักษณะเด่ น onset duration doses S/E


Captopril ออกฤทธิ์ส้ นั สามารถ 15 นาที 4-6 ชัว่ โมง 6.25 -25 mg ความดันโลหิ ตต่า ไตวายในกรณี
PO /SL
ใช้ได้ดี มีประสิ ทธิภาพใน หลอดเลือดไตตีบ
การลดความดันโลหิ ต

Clonidine Alpha adrenergic 0.5-2 6-8 ชัว่ โมง เริ่ มต้น 0.2 mg ความดันโลหิ ตต่า ซึม ปากแห้ง
antagonist ชัว่ โมง ตามด้วย 0.1
mg/h จนสูงสุด
0.8 mg
Labetalol Alpha and Beta 0.5-2 8-12 ชัว่ โมง 200-400 mg Bronchospasm,hypotension
adrenergic ชัว่ โมง q 2-3 h
antagonist
Prasozin Alpha adrenergic 1-2 8-12 h 1-2 mg q 1h เป็ นลม ใจสัน่ ใจเต้นเร็ ว
antagonist ได้ผลดีใน ชัว่ โมง
ระยะแรกของ
pheochromocytoma

อ้างอิง: อายุรศาสตร์ฉุกเฉิ น รามาฯ


Hypertensive emergency and urgency ของ นพ. ธนวิตต สกุลแสงประภา
Hypertension 2006 ของ พญ.วิลยั พัววิไล
เรี ยบเรี ยงโดย นิสิตแพทย์ภทั รพล คามุลตรี pattarapon_bump@hotmail.com
national license (medicine)

94. ชาย 45 ปี กินเหล้ามานาน มา รพ.ด้วยเลือดออกใน stomach ขณะนี้เลือดหยุดแล้วตอนนี้มี


อาการกระสับกระส่าย กระวนกระวาย มี restlessness ,sweating , BP =150/100 ,
PR =120 ,tremor both hands จะ treatment อย่างไร
a. chlorpromazine IM
b. Diazepam IV
c. Haloperidol IM
d. Thiamine IM
เฉลย
สาหรับข้อนี้ คิดว่าผู้ป่วยน่าจะมีอาการของ alcohol withdrawal ดังนั้น จึงตอบ diazepam IV เนื่องจาก
มี guideline( ของ รพ พุทธ ที่ออกมาเพื่อใช้ตอนปีใหม่ที่ผ่านมา) ดังนี้
(guideline อันนี้คิดว่าคงเป็นประโยชน์กับหลาย ๆ คนนะค่ะเลยเอามาให้ทั้งดุ้นเลยค่ะ)
95. ชายอายุ 60 ปี มีตุ่มน้าเป็นกลุ่มบริเวณหลังไปถึงใต้ชายโครง ปวดแสบร้อน ถามว่าเกิดจากโรคอะไร
a. cellulitis
b. contact dermatitis
c. dermatitis dermatiformis
d. Herpes simplex infection
e. vericella zoster infection

เฉลย
จากโจทย์ผู้ป่วยเป็นโรค Herpes zoster เนื่องจากมีlesion ตาม dermatome เป็นลักษณะ group of
blister on erythematous base และมี postherpetic neuralgia ซึ่งทาให้มีอาการปวดแสบปวดร้อนได้ แต่ใน
choice ไม่มี ซึ่งความจริงแล้วโรค herpes zoster นั้น ถือว่าเป็นโรคที่เกิดจาก การ recurrent ของvaricellar
zoster นั่นเอง จึงทาให้ข้อนี้ตอบ varicella zoster infection ( e ) ส่วนข้ออื่น ๆ
- cellulitis : เป็นการติดเชื้อของใต้ผิวหนัง ( subcutaneous) ซึ่งจะมีอาการ ไข้สูง ปวด
เมื่อย lesion พบ ขอบเขตไม่ชัด มีผื่น ปวด บวม แดงร้อน
- contact dermatitis : เป็น reaction การแพ้จากสารภายนอกร่างกาย ซึ่งแบ่งเป็น 2
อย่างคือ
 irritant contact dermatitis : เป็นการแพ้ของสิ่งที่สัมผัสผิวหนังโดยตรง ดังนั้นความรุนแรง
จะขึ้นกับสารที่สัมผัส และความเข้มข้นของสารที่ได้รับ ที่น่าสังเกตุคือ จะมีผื่นขึ้นแค่
บริเวณที่สัมผัส
 Allergic contact dermatitis : เป็นการแพ้ที่เป็น reaction จาก immune ในร่างกาย
โดยสาร hapten ดังนั้น ผื่นจะขึ้นโดยลามไปบริเวณอื่นได้ ( ไม่ใช่แค่บริเวณที่
สัมผัส )
- Herpes simplex infection : คือ เริม นั่นเองโดยแบ่งเป็น2 ชนิดดังนี้
 type 1 : มักเป็นในเด็ก โดย lesion จะเป็นที่ปาก ถ้าเป็นรุนแรงจะเป็น herpetic
gingivostomatitis with oral erosion
 type 2 : มักเป็นในผู้ใหญ่ เป็นลักษณะ bilateral erosive vesicular lesion
accompanied by edema and lymphadenopathy

96. ผู้ป่วยมี anaphylaxis shock ต้อง treatment อย่างไร


a. IM adrenaline ดีกว่า subcutaneous adrenaline
เฉลย
จากข้อนี้โจทย์เป็น anaphylaxis shock ซึ่งการรักษาคือ การให้ epinephrine IM ขนาด 1:1000 ใน
ขนาด 0.01 มล/กก ( max dose ไม่เกิน 0.3 มล)
100. ผู้ป่วยชายวัยกลางคน มีไข้ ไอ เหนื่อย ลมหายใจมีกลิ่นเหม็น CXR : cavitary lesion with air fluid level
ในผู้ป่วยรายนี้คิดถึงโรคใดมากที่สุด
a. Lung abscess
ตอบ Lung abscess
Cause : m/c = anaerobe; Peptostreptococcus,peptococcus, Bacteroides melaninogenicus ,
Fusovacterium nucleatum (มักมีอาการแบบค่อยเป็นค่อยไป )
- Aerobic and facultative (S.aureus, K.pneumoniae, Norcadia spp. , Gram -ve) : มักมี
การดาเนินโรคแบบ acute onset
- Fungus (Mucor,Aspergillus) , parasite
- Immunocompromise host : m/c aerobic,OIs
- DM : Mediodosis
‚ผล sputum c/s ของ lung abscess มักพบหลายเชื้อ‛
Risk factor : Impaired upper airway and oral hygeien
Impaired swallow
Impair consciousness : ทั้งหมดมีความเสี่ยงต่อการสาลัก
Symptoms: cough, purulent sputum production , pleuritic chest pain, fever , hemoptysis, Clue : ‚fetid
breath‛ (หายใจมีกลิ่นเหม็น)
Sign : evidence of consolidation , +/- clubbing finger , crepitation
CXR : classical finding
- one or two thick wall cavities in dependent area of the lung (upper lobes and
posterior segment of the lower lobes)
- มักพบ air fluid level
Treatment : ส่วนใหญ่ใช้ penicillin 1 MU iv. Q 4 hr. หรือ 500 mg oral QID
ในกรณีแพ้ penicillin หรือใช้ยาแล้วไม่ได้ผล ใช้ clindamycin แทนขนาด 150-300
mg q 6 hr (Harrison ed17 บอกให้ใช้ clindamycin เลย เพราะมีเชื้อดื้อยา
penicillin เยอะ)
ระยะเวลาในการให้ยา 4-6 wks

Ref : Harrison ed.17 ; lung abscess ; 1631


ตาราโรคระบบทางเดินหายใจ ; ฝีในปอด ;328
101. ผู้ป่วย CRF มาด้วยอาการ urine ออกน้อยและมี BP สูง Serum K = 7 และมี EKG change การรักษาที่
เหมาะสมที่สุดในผู้ป่วยรายนี้
ตอบ 10% calcium gluconate

Hyperkalemia, defined as a plasma K+ concentration >5.0 mmol/L, occurs as a result of either K+


release from cells or decreased renal loss.
Causes of Hyperkalemia
I. Renal failure
II. Decreased distal flow (i.e., decreased effective circulating arterial volume)
III. Decreased K+ secretion
A. Impaired Na+ reabsorption

1. Primary hypoaldosteronism: adrenal insufficiency, adrenal enzyme deficiency (21-


hydroxylase, 3β-hydroxysteroid dehydrogenase, corticosterone methyl oxidase)
2. Secondary hypoaldosteronism: hyporeninemia, drugs (ACE inhibitors, NSAIDs, heparin)
3. Resistance to aldosterone: pseudohypoaldosteronism, tubulointerstitial disease, drugs (K +-
sparing diuretics, trimethoprim, pentamidine)

B. Enhanced Cl- reabsorption (chloride shunt)

1. Gordon's syndrome
2. Cyclosporine

Clinical manefestation
hyperkalemia partially depolarizes the cell membrane. Prolonged depolarization impairs
membrane excitability and is manifest as weakness, which may progress to flaccid paralysis and
hypoventilation if the respiratory muscles are involved.
Hyperkalemia also inhibits renal ammoniagenesis and reabsorption of NH 4+ in the thick
ascending limb of the loop of Henle. Thus, net acid excretion is impaired and results in
metabolic acidosis, which may further exacerbate the hyperkalemia due to K+ movement out of cells.
The most serious effect of hyperkalemia is cardiac toxicity, which does not correlate well with the
plasma K+ concentration. The earliest electrocardiographic changes include increased T-wave
amplitude, or peaked T waves. More severe degrees of hyperkalemia result in a prolonged PR
interval and QRS duration, atrioventricular conduction delay, and loss of P waves. Progressive
widening of the QRS complex and merging with the T wave produces a sine wave pattern. The
terminal event is usually ventricular fibrillation or asystole.

Ref: Harrison ed. 17


102. ผู้ป่วยมีไข้ต่า ไอเรื้อรัง นน.ลด CXR: cavitary lesion + infiltration at RUL , AFB negative 3 ครั้ง การ
รักษาที่เหมาะสมที่สุดในผู้ป่วยรายนี้คือ

ตอบ impression pulmonary TB

แนวทางในการรักษาเป็นดังนี้

ที่มา : แนวทางปฏิบัติในโครงการหลักประกันสุขภาพถ้วนหน้า เรื่องวัณโรคผู้ใหญ่ (ปี 2545)

By IM คร้าบผม
103. Pt.มีอาการปวดหัวเป็นๆหายๆ มีHT ตรวจพบNa 140(137-150) K 3(3.5-5) สาเหตุของ
HT คืออะไร
a. Pheochomocytoma
b.Primary hyperaldosteronism
c. Renal a. stenosis
ตอบ b.Primary hyperaldosteronism

ในผู้ป่วยรายนี้มHี T กะ hypoK ข้อนี้ถามเรื่อง สาเหตุของ2nd HT


1. Primary renal dis. : Renal a. stenosisพบrenal bruit
2. Pheochomocytoma: เป็นก้อนเนื้องอกที่หลั่งNorepi. มากๆ ทาให้ปวดหัว ใจสั่น เหงื่อออกเป็นพักๆ
Lab: urine metanephrine
3. Primary hyperaldosteronism :มีaldoสูงทาให้เกิดการดูดกลับของNaและขับKเกิดhypoK,metabolic
alkalosis
อาการจะพบproximalweakness,rhabdomyolysis,nephrogenic DI
4. Oral contraceptive
5. Hyper/hypothyroism
6. hyperparathyroism:hyperCa hypoPO4
7. cushing
8. sleep apnea syn.
9. coaratation of aorta

104. Ptมี tenia unguium การรักษาที่เหมาะสมคืออะไร


a. Topical ketoconazole
b. Oral ketoconazole
c. Oral itraconazole
d.Topical itraconazole

ตอบ: c. Oral itraconazole ( 200 milligrams daily for a duration of 12 weeks)


อธิบาย:เชื้อราที่เล็บ(Tinea ungium)

สาเหตุ : เกิดจากเชื้อ Trichophyton มักพบที่นิ้วเท้า และพบในคนที่มีเชื้อราที่อื่นร่วมด้วยเช่น เท้า หรือมือมา


ก่อน ผู้หญิงที่ไปทาเล็บตามร้านเสริมสวยมีโอกาสเป็นราที่เล็บได้ง่าย
อาการ : เล็บที่เป็นโรคจะหนาตัว เล็บจะเปลี่ยนเป็นสีเหลือง ขาวขรุขระ เล็บจะแยกจากหนังใต้เล็บ
การรักษา: -ยารับประทานเช่น griseofulvin,itraconazole,fluconazole ,ketoconazole
-ถอดเล็บ
แนวทางการรักษา
Tinea unguium ที่ FDAยังไม่รับรองให้ใช้ ดังนี้ AMOROLFINE, BIFONAZOLE, CICLOPIROX
OLAMINE, KETOCONAZOLE, NATAMYCIN, TIOCONAZOLE, TOLNAFTATE

105 . Pt. ได้penicillin มานาน ต่อมาdiarrhea ตรวจproctoscope พบลักษณะเข้าได้กับAACการรักษาที่


เหมาะสมคือข้อใด
a. Metronidazole

อธิบาย : Antibiotic-associated colitis(AAC) เป็นการอักเสบอของintestinesโดยมีสาเหตุจากantibiotic


treatment ( caused by toxins produced by the bacterium Clostridium difficile.)

antibiotics caused :clindamycin (Cleocin), ampicillin (Omnipen), amoxicillin (Amoxil, Augmentin, or


Wymox), cephalosporin class (such as cefazolin or cephalexin).

Symptoms :มักพบหลังจากใช้ATB 4-10 daysโดยจะมี lower abdominal cramps, an increased need to


pass stool, and watery diarrhea. Fever

การรักษา:ให้antibiotic to control the growth of the Clostridium difficile, usually vancomycin (Vancocin)
or metronidazole (Flagyl or Protostat). taken orally four times a day for 10-14 days.

By:kaew^^

106. pt .post arrest มี EKG ให้ดูเป็น VT การรักษาที่เหมาะสมคือข้อใด

A . defibrillation
ตอบ cardioinversion===ก่อนอื่นต้องดูก่อนาว่า VT มีpulse หรือเปล่า คนนี้ post arrest น่าจะมี pulse การ
รักษาแบ่งเป็น
1 รักษาด้วยยา ให้ amiodarone 150 mg iv ซ้า ได้ 10นาที ไม่เกิน 2.2 gm ใน 24 ชม.
2. electrical synchonized cardioconversion ถ้าไม่ดีขึ้น หรือ unstable { chest pain, decrease level of
conscious, low blood flow , shock }
107. pt pale +jx ตับม้ามไม่โต จะส่งตวจอะไร
a. Hb typing
b. inclusion body
c. BMA
d. G-6 PD screening
e. Croomb’s test

ตอบ d. G-6 PD screening


anemia + jx คิดถึง hemolysis anemia ไม่มีตับม้ามโต ทาหัคิดถึง G-6 PD def น่าจะมีประวัติ
มากกว่านี้ < AIHA น่าจะมีม้ามโต>
Location Mechanism Example
Intrinsic Enz def G-6 PD def
Hemoglobinopathy Sikle cell anemia, thallussemia
Membrane defect Heretary sperocytosis
Immune AIHA

extrinsic Traumatic MAHA


infection Malaria
Entrapment hypersplenism

108. pt clinical DKA การรักษาที่เหมาะสมที่สุดคือ


a. iv insulin
b. iv fluid

ตอบ iv fluid
หลักการรักษาคือ expand intravascular volume แก้ไขภาวะขาดน้า และ E’lyte imbalance และแก้ไข
metabolism ด้วยการให้ insulin
- iv fluid วันแรกให้ 0.9 NSS maintenance + def ½ แรก ใน 8 ชม. อีก .½ .ใน16 ชม หลัง
จากนั้นจึงพิจจารณาเปลี่ยน iv ที่หลัง
- insulin ให้เป็น continouse low dose iv or low dose IM จะช่วยลด ระดับน้าตาลลง 80-100 mg/dl
อ่านเพิ่มเติมใน เวชศาสตร์ฉุกเฉิน, อายุรศาสตร์ฉุกเฉิน
109. ผู้ป่วย Clinical DIC การรักษาที่เหมาะสมที่สุดในผู้ป่วยรายนี้คือข้อใด
A. Antibiotic
B. platelet
C. FFP
เฉลยจ้า
อาการ และอาการแสดงในผู้ป่วยที่มีภาวะ severe DIC
ผิวหนัง purpura , เลือดออกบริเวณที่มี injury , hemorrhagic bullae , focal
necrosis มีเนื้อเน่าตายบริเวณส่วนปลาย
หัวใจและหลอดเลือด shock , acidosis , กล้ามเนื้อหัวใจตายเฉียบพลัน , thromboembolism
ไต oligouria , azotemia , hematuria , acute tubular necrosis ,
renal cortical necrosis
ตับ jaundice , parenchymal damage
ปอด ARDS , hypoxemia , pulmonary edema
GI UGIH mucosal necrosis
CNS stupor , coma , seizure , focal lesion
Adrenal gland adrenal insufficiency

การรักษา
1. การรักษาและขจัดโรคหรือตัวกระตุ้นที่เป็นสาเหตุของ DIC โดยให้ Antibiotic
2. supportive treatment
3. specific treatment จะแตกต่างกันตามอาการแสดง
- ผู้ป่วยที่ต้องทาหัตการ การให้ Plt และ plasma มีประโยชน์
- ผู้ป่วยที่มี Plt ต่า ก็ควรให้ Plt ก็จะมีประโยชน์
- ผู้ป่วยที่มี PT,PTT prolong ก็ควรให้ FFP
- ผู้ป่วยที่มี fibrinogen ต่า (acute head injury, abruptio placenta) หรือผู้ป่วยที่มี
hyperfibrinolysis (CA prostate) ก็ให้ Cryoprecipitate

ดังนั้นไม่ทราบว่าผู้ป่วยเป็น DIC จาก sepsis หรือไม่ และผลทางห้องปฏิบัติการก็ไม่บอก ก็ดูไปตาม


condition แล้วกันนะจ๊ะ
110. รูป EKG เป็น PVC 3 ตัวติดกัน ในผู้ป่วยรายนี้น่าจะคิดถึงโรคใดมากที่สุด
A. Ventricular tachycardia
เฉลยจ้า
Premature ventricular complex
QRS complex มักจะกว้างเกิน 0.12 sec (3 ช่อง) Pwave ที่ตามหลัง PVC มักจะมีทิศทางตรงข้ามกับ
ตัว PVC เช่น ถ้า QRS complex upright ก็จะมี Twave inverted หรือ ถ้า QRS complex inverted ก็จะมี T
wave upright แต่ก็ไม่เสมอไป
- ถ้ามี PVC 2 ตัวติดกัน โดยไม่มี QRS complex ที่ปกติคั่นเลย เรียกว่า couplet
- ถ้ามี PVC 3 ตัวติดกันขึ้นไป โดยไม่มี QRS complex ที่ปกติคั่นเลย เรียกว่า short run of
Ventricular tachycardia ตอบข้อนี้จ้า
- ถ้ามี PVC 1 ตัว เกิดคู่กับ QRS complex ที่ปกติทุก 1 ตัว ไปเรื่อยๆ เรียกว่า Bigeminy of PVC
- ถ้ามี PVC 1 ตัว เกิดคู่กับ QRS complex ที่ปกติทุก 2 ตัว ไปเรื่อยๆ เรียกว่า Trigeminy of PVC
- ถ้ามี PVC 1 ตัว เกิดคู่กับ QRS complex ที่ปกติทุก 3 ตัว ไปเรื่อยๆ เรียกว่า Quadrigeminy of
PVC
111. รูป EKG เป็น complete heart block การรักษาที่เหมาะสมในผู้ป่วยรายนี้คือข้อใด
A. Atropine
เฉลยจ้า
Congenital : โดยมักเกิดร่วมกับ corrected transposition of great artery ,
Ebstein’ s anomaly , ASD premum , VSD
- ระยะแรก ผู้ป่วยมักจะมี prognosisดี โดยผู้ป่วยจะมีชีพจรสูงขึ้นได้เมื่อออกกาลังกาย หรือได้รับ
ยา เช่น Atropine , Isoproterenol
- ระยะยาว prognosis มักแย่ลง PR < 40/min หรือาจเกิด prolong QT ขณะที่อัตราชีพจรช้า
หรือ PVC ที่สัมพันธ์กับ pause dependent ซึ่งการรักษา อาจต้องใช้ Pacemaker เพื่อควบคุม
อาการ
Acquired : most common มักเป็น Acute inferior wall MI เกิดจากการอุดตันที่
dominant Rt. LCA ซึ่งจะให้แขนง ไปเลี้ยง AV node โดยทั่วไปมักเกิดที่
1 ” 3 วัน หรืออาจถึง 7 วันได้
การรักษา จะดีขึ้นเมื่อให้ Atropine, Isoproterenol

แต่สาหรับผู้ป่วยที่เป็น complete heart block แบบถาวร การรักษาคือ pacemaker


ดังนั้นข้อนี้ ควรดู condition ของผู้ป่วยด้วยนะจ๊ะ

By Mai ^o^
national license (Ped)

1. เด็กอายุ 5 ปี มาด้วยไข้เจ็บคอ ตรวจร่างกายพบ cervical lymphadenopathy, splenomegaly and gray


patch at tonsil เชื้อใดต่อไปนี้เป็นเชื้อก่อโรคในผู้ป่วยรายนี้
a. EBV
b. Diptheria
c. GAS
d. Measles
ตอบ EBV
EBV ทาให้เกิด infectious mononucleosis วัยรุ่นหรือเรียกว่า 'glandular fever', 'Mono' และ 'Pfeiffer's
disease'. "The Kissing Disease,"
การติดต่อเกิดขึ้นได้โดยผ่านทางน้าลาย airborne พบในประเทศที่พัฒนาแล้ว มักพบในเด็กอายุตั้งแต่ 3 ปีขึ้น
ไป
อาการ เจ็บคอ มีไข้ อ่อนเพลีย น้าหนักลด มีการอักเสบที่ pharynx พบมี petechiae และ common signs คือ
lymphadenopathy splenomegaly hepatitis hemolysis
Diphtheria มีอาการเจ็บคอ ไข้ต่า กลืนลาบาก มี pseudomembrane ที่ tonsil pharynx อาจพบที่ nasal cavity
คลื่นไส้ อาเจียน อาจพบมีไข้สูงหนาวสั่นได้ในเด็ก พบมีคอบวมได้ 10เปอร์เซนต์ toxin ของเชื้อจะทาให้มีอาการ
ซีด หัวใจเต้นเร็วขึ้น ได้ มีความดันโลหิตต่าลง ในระยะยาว อาจทาให้เกิด cardiomyopathy, peripheral
neuropathy (sensory type) ได้
GAS การติดเชื้อ group A strep อาจทาให้มี
 Bacteremia
 Impetigo, cellulites, erysipelas
 Focal infection
- pneumonia
- tonsillitis - peritonitis
- septic arthritis - meningitis
- osteomyelitis - vaginitis
 necrotizing fasciitis
 scarlet fever
 sinusitis
 strep throat *** Cervical lymphadenopathy , not general lymphadenopathy
 toxic shock syndrome

Measles
อาการ คือ มีไข้สี่วัน มีสาม C =cough, coryza (runny nose), conjunctivitis (red eyes) ไข้อาจสูงได้ถึง 40
องศาเซลเซียส ตรวจพบ Koplik’s spots ในปาก ที่ second upper molar เป็น pathognomonic sign แต่จะ
หายไปในเวลา 1 วัน

ผื่นมีลักษณะเป็น generalized maculopapular erythematous rash ที่ขึ้นหลังจากมีไข้นานหลายวัน เริ่มขึ้น


จาก ศีรษะ และลงไปทางด้านล่าง มีอาการคันได้ ก่อนผื่นจะหายจะมีการเปลี่ยนสี จากสีแดงเป็นน้าตาลเข้ม
แล้วหายไป

2. เด็กหญิงอายุ 6 ปี มีผื่นขาวขุยเล็ก ๆ ที่ใบหน้า เห็นชัดเวลาว่ายน้า เป็นมานาน 3 เดือน ไม่คัน ตรวจร่างกาย


มี multiple ill-defined, fine scaly patch, hypopigmented macule and patch ที่หน้าและคอ KOH:
negative จงให้การวินิจฉัย
a. Vitiligo
b. Pityriasis alba
c. Pityriasis vesicolor
d. Tinea Facialis
e. Contact leukoderm
ตอบ pityriasis alba

Vitiligo
Vitiligo หรือ leukoderma เป็นโรคเรื้อรังของผิวหนัง ที่มีการ loss of pigment ทาให้มี irregular pale patches
เกิดโดย melanocyte ตาย หรือทางานไม่ได้
สาเหตุอาจเกิดจาก autoimmune genetic หรือ environmental factors.
รอยโรคมีลักษณะเป็น macular depigmentation, มักพบที่ extensor หน้า ลาคอ, พบในวัยรุ่น รอยโรคสามารถ
ขยายขนาดขึ้นได้
Pityriasis alba
Pityriasis alba พบบ่อยในเด็ก ลักษณะ เป็นสะเก็ดเรียบๆ สีซีด
พบมากที่บริเวณใบหน้า สามารถหายได้เอง รักษาได้ง่ายโดยการใช้ครีม
moisturizer
Pityriasis แปลว่า เรียบๆ alba แปลว่า ขาว
สาเหตุ ไม่ทราบแน่ชัด อาจเกิดจากการใช้ corticosteroid ในการรักษา
eczema มากเกินไป มีการลดการทางานของ melanocytes ทาให้
melanosomes เล็กลง
พบบ่อยในเด็กอายุ 3-16 ปี ผู้ชายมากกว่าผู้หญิง
อาการและอาการแสดง มี dry scaling เกิดขึ้น พบบ่อยในฤดูหนาว เนื่องจากมีอากาศที่แห้ง ส่วนในฤดูร้อน จะ
สังเกตเห็น patch ชัด เพราะสีผิวอาจเข้มขึ้น ระยะโรคมี 3 ระยะ
1. Raised and red
2. Raised and pale
3. Smooth flat pale patches
รอยโรคมีลักษณะกลม หรือรี ขนาดประมาณ 0.5-2 cm อาจพบได้ตั้งแต่ 4-5 อัน ไปถึง 20 อัน
การรักษา redness, scale และอาการคัน อาจใช้ emollients และ hydrocortisone นานประมาณ 1 อาทิตย์ ใช้
ครีมกันแดดในการทาให้ผิวหนังรอบ ๆ ที่เป็นด่างขาว สีเข้มขึ้น เพื่อไม่ให้สังเกตได้ชัด
Pityriasis vesicolor
Pityriasis versicolor เกิดจากเชื่อรา ชื่อ Malassezia
พบที่ ลาตัว คอ และแขน patches มีสีชมพู สีน้าตาลทองแดงหรือ สีซีดกว่าสีผิวที่อยู่รอบ ๆ ได้ คันได้
เล็กน้อย สีที่รอยโรคอาจจะพบว่าขาวกว่าเมื่อเทียบกับ สีผิวหนังตามปกติข้างๆ จึงมีชื่อเรียกว่า pityriasis
versicolor alba อาจพบว่ามีตกสะเก็ดได้
วิธีการยืนยันการวินิจฉัยใช้วิธี wood’s light เป็นแสงความยาวคลื่นยาว สีเขียวเหลือง ส่องไปจะเห็น
เรืองแสงขึ้นมา
Pityriasis versicolor พบบ่อยในที่อากาศ ร้อน ชื้น คนที่เหงื่อออกมาก ๆ พบบ่อยในฤดูร้อน ไม่พบในผิวหนังที่
สัมผัสกับแสงแดดบ่อย ๆ

Tinea facialis describe lesion : sharply marginated


erythematous scaling and crusted plaque
Contact leukoderm delayed type hypersensitivity
3. ผู้ป่วยเด็กชายอายุ 3 ปี สบายดีมาตลอด หลังเล่นกับพี่มีไอ lung: rhonchi right lung ควรส่ง investigation
อะไร
a. AP, lateral
b. PA, lateral
c. Inspiration and expiration
d. Right lateral decubitus
e. Left lateral decubitus

จากโจทย์ สงสัย upper airway obstruction จนกระทั่งถึง trachea and right main bronchus (มีไอ) Right
main bronchus เป็น dependent part ที่มักจะเกิด aspiration ได้ง่าย
มักพบที่ posterior segment of upper lobe และ apical segment of lower lobe
คิดว่าน่าจะตอบ inspiration and expiration film เพราะมี indication ในการดู unilateral obstruction of major
bronchus ตามที่อธิบายด้านล่าง เป็นการส่ง CXR ในแบบต่างๆ และมี indication อะไรบ้าง
Standard Views iii. Unilateral diaphragmatic paralysis
1. Standing (Upright Chest XRay) iv. Unilateral obstruction of major
4. Posteroanterior (PA) Film bronchus
5. Left Lateral XRay 2. Lordotic View Indications
a. Request right lateral film if v. Posterior Apical Disease
right-sided finding vi. Middle Lobe disease
b. More sensitive than PA for 3. Reverse Lordotic View Indications
abdominal free air vii. Anterior apical disease
2. Supine (Portable Chest XRay) 4. Oblique Film
i. Anteroposterior (AP) Film viii. Peripheral small lesions
1. Magnifies heart and 1. Separated from overlying
anterior mediastinum chest shadows
2. Emphasizes rib and 2. Lesions poorly seen on
calcium contrast lateral chest XRay
3. Lung parenchyma may ix. Rib Fractures (at axillary lines)
appear washed out 5. Lateral decubitus Film
Special Views x. Detect small areas of air at
1. Inspiration and Expiration Film Indications uppermost pleural space
***** xi. Detect small areas of dependent
ii. Pneumothorax accentuated on Pleural Fluid
expiration
1. Measure size and mobility 8. Barium Swallow
of fluid collection xxiv. Enlarged retro-mediastinal nodes
2. Accessible with sampling xxv. Define Posterior intrathoracic mass
needle (>1 cm size) xxvi. Confirm ruptured diaphragm or
xii. Uncover Lung tissue obscured by Diaphragmatic Hernia
Pleural Fluid xxvii. Impaired swallowing with
1. Place side of interest up aspiration
xiii. Mobility of mediastinal or pleural 9. Diagnostic Pneumothorax (instill air in
masses pleural space)
xiv. Assess mobility of solids and fluids xxviii. Distinguish peripheral lung mass
within cavities from pleural lesion
xv. Assist with maximizing inspiration xxix. Define Mesothelioma
of uppermost lung xxx. Parenchymal disease extending
6. High Penetration Film with moving grid towards chest wall
(Bucky Film) 6. Circumstances that decrease Chest XRay
xvi. Obesity quality
xvii. Dense pleural or pulmonary a. Semi-upright position (neither standing nor
opacities supine)
xviii. Calcified lesions i. May enlarge normal structures
xix. Lesions obscured by heart or ii. Changes air-fluid levels
diaphragms
xx. Air Bronchograms in densely b. Lordosis or vertical axis rotation
infiltrated areas i. Widens heart and mediastinum
7. Intrathoracic Pressure Maneuvers c. Inadequate sustained inspiration
xxi. Valsalva Maneuver: shrinks i. Breathing film
pulmonary vessels 1. Lung structures and
xxii. Muller Maneuver: distends diaphragm blurred
pulmonary vessels ii. Expiration film
xxiii. Indications 1. Basilar infiltrates
1. Distinguish blood vessel accentuated
from lymph node 2. Interstitial structures
2. Distinguish A-V accentuated
malformation from solid a. Vessels
lesion b. Pleural Fluid
3. Increased heart size 3. Air-Fluid levels (e.g. Lung
d. Supine Film Abscess) disappear
i. Decreases Lung Volume v. Pneumothorax signs on supine film
1. Highlights infiltrates and 1. Deep Sulcus sign
interstitium a. Costophrenic angle
ii. Increases venous return to heart sharply outlined by
1. Distends azygous vein and air
pulmonary vein b. Diaphragm-
iii. Diaphragm rises and intracardiac mediastinal junction
pressure increases sharply outlined
1. Heart and mediastinal 2. Hyperlucency
structures enlarge superimposed over liver
iv. Fluid and air migrate shadow
1. Pleural Effusions disappear
2. Small Pneumothorax References
disappears A Marini (1987) Respiratory Medicine,
Williams & Wilkins Katz (1999) Clin Chest Med
20(3):549
4. Term new born BW = 4000 g มารดาเป็น DM ชักเกร็งทั้งตัว สั่นและมือเท้าเย็น Blood sugar =26 mg%
ให้การรักษาด้วยสารน้าชนิดใดและอัตราเท่าใดจึงจะเหมาะสม
A) oral 10% D/W 10ml/kg
B) IV 10% D/W 2 ml/kg
C) IV 10% D/W 2 ml/kg
D) IV fluid with GPR 4-6 mg/kg/min
E) IV fluid with GPR 6-8 mg/kg/min
F) IV hydrocortisone 10 mg/kg/day
ตอบ
การรักษาของโรค IDM คือป้องกันภาวะน้าตาลในเลือดต่า โดยให้เจาะดูระดับน้าตาล
ถ้า > 40 ให้ early feeding ด้วยนมหรือน้าตาล และเจาะตรวจน้าตาลเป็นระยะ
ถ้า <40 ควรให้ fluid ที่มี GPR (glucose production rate) ประมาณ 6-8 mg/kg/min
ในเด็กที่ไม่สามารทานทางปากได้หรือระดับน้าตาลไม่สูงขึ้นหลังจากให้ทางปากก็ให้เป็นสารน้าเหมือนในกลุ่ม
น้าตาลน้อยกว่า 40
ข้อนี้เด็กน้าตาล <40 จึงควรให้เป็น IV fluid with GPR 6-8 mg/kg/min
5. เด็กอายุ 4 ปี มาด้วยไข้และซึม BP 90/60 mmHg, PR= 120/min ตรวจร่างกายพบจุดตามตัว stiff neck
positive เชื้อที่เป็นสาเหตุการเกิดโรคในผู้ป่วยรายนี้มากที่สุดคือ Meningococcal Meningitis เกิดจากเชื้อ
Neisseria meningitides ส่วนใหญ่ในไทยมักพบเป็น group B
โรคแพร่ติดต่อได้อย่างไร
ตอบ การติดต่อเกิดขึ้นโดยการคลุกคลีใกล้ชิดกับผู้ป่วยหรือผู้ที่เป็นพาหะ (ผู้ที่มีเชื้อในช่องปากและจมูก
แต่ไม่แสดงอาการ) โดยเชื้ออยู่ในละอองน้ามูก น้าลาย เมื่อผู้ป่วยหรือพาหะ ไอ หรือจาม ก็จะสามารถ แพร่เชื้อ
ไปสู่คนอื่นได้
โรคนี้มีระยะฟักตัวนานกี่วัน
ระยะเวลาตั้งแต่ได้รับเชื้อจนกระทั่งเริ่มปรากฏอาการป่วยของโรคนี้ (ระยะฟักตัวของโรค)
มักอยู่ในช่วง 2 ” 10 วัน
ผู้ป่วยจะแสดงอาการอะไรบ้าง
อาการเริ่มด้วยมีไข้สูงทันทีทันใด ปวดศีรษะมาก อ่อนเพลีย คลื่นไส้ อาเจียน เจ็บคอ
ปวดข้อ ปวดเมื่อยกล้ามเนื้อ โดยเฉพาะที่ขา และหลัง ต่อมาจะมีเลือดออกใต้ผิวหนัง
เป็นจุดแดงขึ้นตามตัว แขน ขา แล้วกลายเป็นจ้าเลือด สีคล้า และสะเก็ดดาในที่สุดผู้ป่วยมักมี
อาการ เยื่อหุ้มสมองอักเสบ คือ มีอาการคอแข็งร่วมด้วย ในรายที่เป็นรุนแรง ผู้ป่วยจะซึม ชัก ช็อก และ
อาจเสียชีวิตได้ภายใน 24 ชั่วโมง หลังเริ่มมีอาการ

6. เด็กหญิงไม่มีประจาเดือน ตรวจร่างกายพบ webbed neck, wide nipple distance and low hair line
ความผิดปกติของผู้ป่วยรายนี้อยู่ที่ฮอร์โมนชนิดใด
A) Decrease Insulin
B) Decrease GH
C) Increase estrogen
D) Increase GnRH
E) Decrease PTH
ตอบ Decrease GH
Turner’s syndrom
“ eye problems (drooping eyelids, ‚lazy‛ eye) “ curvature of the spine (scoliosis)
“ high palate (roof of mouth) “ arms that turn out more than usual at the
“ small jaw elbows
“ low hairline at the back of the head “ missing 4th or 5th knuckle
“ wide and short neck, sometimes with an “ puffiness of the hands , feet (lymphedema)
excess of skin that joins the neck with the “ narrow fingernails
collar bone (called ‚neck webbing‛) “ knock knees
“ broad chest “ increased numbers of moles on the skin
Skeletal
ตัวเตี้ย : ขาด GH
การหลั่ง estrogen ผิดปกติทาให้เกิด osteoporosis, increased risk of bone fractures ,spine
scoliosis
Kidney
horseshoe-shaped kidney, abnormal urine-collecting system, Poor blood flow to the kidneys
Thyroid :hypothyroid

7. เด็กหญิงไทยอายุ 14 ปี ไม่มีประจาเดือนมานาน 8 เดือน ไม่มีก้อนที่คอ ไม่มีใจสั่น เหงื่ออกมากผิดปกติ


ปฏิเสธน้านมไหลผิดปกติ ไม่ได้ลดน้าหนักหรือออกกาลังอย่างหักโหม ตรวจร่างกายไม่พบความผิดปกติใดๆ ทา
progesterone challenge test : negative และ estrogen-progesterone challenge : negative การวินิจฉัยใด
เหมาะสมที่สุด
a. Ovarian failure
b. PCOS (Polycystic Ovarian Syndrome)
c. Exercise amenorrhea
d. Asherman’s syndrome
เฉลย amenorrhea แบ่งออกได้ 2 ประเภท คือ
1. primary amenorrhea หมายถึง สตรีที่มีภาวะขาดระดูนั้นยังไม่เคยมีระดูมาก่อน โดยวินิจฉัย คือ
- สตรีอายุตั้งแต่ 14 ปีขึ้นไปที่ยังไม่มี secondary sexual characteristics และยัง
ไม่มีระดู
- สตรีอายุตั้งแต่ 16 ปีขึ้นไปทีมีการพัฒนาของ secondary sexual
characteristics แล้ว แต่ยังไม่มีระดู
2. secondary amenorrhea หมายถึง สตรีที่เคยมีระดูแล้ว ต่อมาเกิดการขาดระดูขึ้น เป็นเวลาตั้งแต่ 3
รอบระดู หรือ 6 เดือนขึ้นไป
จากโจทย์ที่ให้มาแสดงว่าเด็กหญิงคนนี้น่าจะเคยมีประจาเดือนมาแล้ว แต่ขาดประจาเดือนไป 8 เดือน
จึงถือเป็น ‚secondary amenorrhea‛
สาเหตุของ secondary amenorrhea ได้แก่
1. มดลูกและช่องคลอด 3.3 Sheehan’s syndrome
1.1 Asherman’s syndrome 3.4 Cushing ‘s syndrome
2. รังไข่2.1 การทางานของรังไข่ล้มเหลว 4. Hypothalamus and brain
2.2 เนื้องอกรังไข่ 4.1 ภาวะไข่ไม่ตกเรื้อรัง
3. ต่อมใต้สมอง 4.2 น้าหนักลด / anorexia
3.1 hyperprolactinemia 4.3 Hypothalamic suppression
3.2 empty sella syndrome 4.4 Hypothyroidism
การตรวจวินิจฉัยภาวะ secondary amenorrhea
ขั้นที่ 1
- ตรวจให้แน่ใจว่าไม่ตั้งครรภ์ เช่น human chorionic gonadotropin จาก
ปัสสาวะหรือเลือด
- TSH
- serum prolactin
- Progesterone challenge test ซึ่งเป็นการตรวจปริมาณ estrogen ของร่างกาย และช่องทาง
ออกของระดู ถ้าผู้ป่วยมี estrogen อยู่ในร่างกาย estrogen จะทาให้เยื่อบุโพรงมดลูกเป็น
proloferative phase เมื่อให้ progestin เข้าไปจะเปลี่ยนให้เป็น secretory phase และเมื่อ
หยุดให้ progestin จะมี withdrawal bleeding ออกมา
หลังหยุดยา 2-7 วัน ถ้ามีเลือดออกมาเพียงเล็กน้อย ก็ถือว่ามี withdrawal bleeding แสดงว่าผู้ป่วย มี
ภาวะไข่ไม่ตกและโพรงมดลูกปกติ

ขั้นที่ 2
หากให้ Progesterone challenge test แล้วไม่มีเลือดออก (negative) จะให้ estrogen (estrogen-
progesterone challenge test) ถ้ามีเลือดออกแสดงว่าโพรงมดลูกปกติ หากไม่มีเลือดออกแสดงว่าโพรงมดลูก
ผิดปกติ
ขั้นที่ 3
Serum FSH เพื่อแยกว่าเป็นจาก ovary , pituitary หรือ hypothalamus
ถ้าได้มากกว่า 20 IU/L แสดงว่าเป็นที่รังไข่ล้มเหลว
ถ้าได้ระดับปกติหรือตากว่า 5 IU/L แสดงว่าเป็นที่ pituitary หรือ hypothalamus

ดังนั้น ผู้ป่วยคนนี้จากการตรวจขั้นตอนที่ 2 พบว่า estrogen-progesterone challenge test : negative แสดง


ว่าโพรงมดลูกผิดปกติ  Asherman’s syndrome (ภาวะที่มีการทาลายเยื่อบุโพรงมดลูกแล้วมีพังพืดขึ้นมา
แทน)
8. เด็กอายุ 5 ปี ปวดท้อง ปวดข้อ และพบผื่นขึ้นที่ขาทั้งสองข้าง การส่งตรวจในข้อใดต่อไปนี้มีประโยชน์ในการ
ประเมิน prognosis มากที่สุด
a. CBC
b. ESR
c. skin biopsy
d. UA
เฉลย เด็กที่มาด้วยอาการ ปวดท้อง ปวดข้อ และมีผื่นที่ขาทั้ง 2 ข้าง ทาให้นึกถึง Henoch Schonlein
purpura มากที่สุด
Henoch Schonlein purpura เป็นการอักเสบของหลอดเลือดโดยม่ทราบสาเหตุ มักเกิดกับเด็กอายุต่า
กว่า 7 ปี ซึ่งพบว่ามี IgA immune complex ติดที่ post-capillary venules ทั่วร่าง และเกิดการกระตุ้น
alternative pathway ของ complement มีการติดของ C3 ที่ immune complex มีการอักเสบของหลอดเลือด
ทาให้หลอดเลือดเปราะบางและแตก เกิด palpable purpura ที่ skin และ submucosal bleeding ที่ผนังลาไส้
อาการแสดง
1. palpable purpura ที่ skin
2. ปวดท้องทั่วๆ จาก submucosal edema หรือ hematoma ที่ผนังลาไส้
3. ปวดข้อและข้ออักเสบ มักเป็นที่ข้อเข่าและข้อเท้า
4. อาการทางไต hematuria และ proteinuria
5. ปวดศีรษะ จาก CNS vasculitis
prognosis
ขึ้นอยู่กับว่าผู้ป่วยมีพยาธิสภาพทางไตหรือไม่ การพยากรณ์โรคจะไม่ดีหากมีพยาธิสภาพที่ไตรุนแรง คือ
มี elevated BUN , persistent high grade proteinuria
ดังนั้นการส่งตรวจที่มีประโยชน์ต่อ prognosis คือ UA

9. ปวดข้อและพบผื่นกดเจ็บสีแดงที่หน้าแข้งทั้งสองข้าง จงให้การวินิจฉัย
Differential diagnosis โรคที่มีอาการปวดข้อ และมีผื่น
1. SLE : Butterfly, discoid
2. Juvenile rheumatoid arthritis : Salmon-pink macules
3. Rheumatic fever : Erythema marginatum
4. Kawasaki disease : diffuse maculopapular , desquamation
5. Henoch Schonlein purpura : palpable purpura
6. Gonococcemia : palms/soles papulopustules
10. NB 35 wk, 2700 g แม่ Blood group O, Rh positive น้าเดิน 4 ชั่วโมง หลังคลอดเหลือง Hct 40% TB =
10 เหลืองจากข้อใดต่อไปนี้เป็นไปได้มากที่สุด
A) Sepsis
B) Prematurity
C) Thalassemia
D) G6PD deficiency
E) ABO incompatibility
จากโจทย์ตีความว่า เด็กpreterm, ไม่ทราบเพศ, normal birth weight, blood gr.ของลูกไม่บอกมา เหลืองหลัง
คลอด น่าจะเหลืองใน24 hr.แรกหลังคลอด บ่งบอกว่าเป็น pathologic jaundice ดังนั้น
Ans : คิดถึง ABO incompatibility มากที่สุดค่ะ เนื่องจาก อาการเหลืองมักเกิดขึ้นเร็วใน 24 hr. แรก มีซีด
รวมถึงปัจจัยเสี่ยงด้วย; แม่ gr.O เข้าได้กับโรคนี้
Sepsis clinical: ซึมลง ไม่ดูดนม หายใจเร็ว ร้องกวน BTไม่stable ไม่เหทือนในเด็กคนนี้
Risk factor: ที่เข้าได้ก็คือเป็นเด็ก preterm, น้าเดิน 4 ชม.ยังไม่เป็น risk ของsepsisนะคะ ( prolong
ROM > 18-24 hr. ) จึงไม่คิดถึง sepsis ค่ะ
Prematurity: ถ้าเป็น physiologic jaundice ของ เด็ก preterm ควรจะเหลืองประมาณวันที่ 5-6 ค่ะ จึงไม่คิดถึง
ข้อนี้
Thalassemia: คิดไม่ออกอ่ะ แต่ถ้าเค้าอยากให้เราตอบข้อนี้ก็น่าจะให้โจทย์ประวัติพันธุกรรมมาซักนิดนึง
G6PD deficiency: ถ้าเป็นเพศชาย มีประวัติ precipitating ก็จะให้น้าหนักโรคนี้มากขึ้น แต่ดันไปให้ blood gr.
มา ก็เลยbias ไป คิดถึงน้อยเนื่องจากว่า onsetของjaundice ไม่เข้า โดยที่ G6PD deficiency จะเหลืองที่ day
2-3 นานถึงหลายสัปดาห์

แถมซักนิด
Pathologic jaundiceได้แก่
1. MB>15
2. เหลืองเร็วใน24 hr.
3. direct B >20%ของ TBหรือ >2 mg/dl
4. เหลืองนานมากกว่า 2 wk
5. เจาะ MB >3
6. rate rising >5 mg/dl/day หรือ>0.2/hr.
7. มีอาการของโรคที่ทาให้เหลือง
11. เด็กอายุ 2 ปี สุขภาพแข็งแรงดี เมื่ออายุ 1 ปีอยู่สถานรับเลี้ยง และเป็นหวัดบ่อย 1-2 ครั้งต่อสัปดาห์ ครั้งละ
3-5 วัน ควรแนะนาอย่างไร
A) Pneumococcal vaccine
B) ติดเครื่องฟอกอากาศที่สถานเลี้ยงเด็ก
C) ให้กิน antihistamine ป้องกันอาการ
D) ทา skin test
E) ไม่ทาอะไรเป็น normal ของเด็ก
ตีความว่า เด็กติดเชื้อไวรัสบ่อยๆ แล้วหวัดอันนี้ก็แพร่ผ่านทางการจามรดหน้ากัน การใช้สิ่งของส่วนตัวร่วมกัน
ข้อนี้ไม่แน่ใจนะคะ แต่ก็ได้ปรึกษากับเพื่อนส้มพะเยามาแล้วว่า
Ans : ส้มข้อ A
กิ๊ฟข้อB
เพื่อนตอบ.............
ข้อEคงคิดถึงน้อยที่สุด เพราะมันไม่ใช่ความปกติ
CและD คิดถึงว่าเป็น allergic rhinitis ถ้าเค้าอยากให้เราตอบ 2ข้อนี้ก็น่าจะบอกประวัติ atopy อื่นๆด้วยว่ามั้ย
แต่คนนี้ ประวัติบอกโต้งๆว่า เป็นโรคติดต่อ ก็เลยไม่คิดถึง
เหลือข้อAและB
-ให้ vaccine ป้องกัน bacteria ถ้าตอบข้อนี้ ก็แสดงว่าเราป้องกันไม่ให้เกิดโรคปอดรุนแรง อะไรประมาณนั้น
(ref.ส้มพะเยา)
-ถ้าติดเครื่องฟอกอากาศเค้าคิดว่าน่าจะช่วยเรื่องโรคติดต่อทางเดินหายใจได้ ใช่มั้ยฮึ
------ข้อนี้เค้าสองคนคิดแบบนี้ พยายามหาแล้วนะ ยังไงก็ขอให้เพื่อนๆโปรดใช้วิจารณญานส่วนบุคคลด้วย ------
12. เด็กอายุ 2 ปี ไข้สูง 1 wk เจ็บคอ หายใจลาบาก ตรวจร่างกายพบ retraction, greenish patch at posterior
pharynx and tonsils จงให้การวินิจฉัย
A) Croup D) Streptococcus tonsillitis
B) Diptheria E) Infectious mononucleosis
C) Candidiasis
Ans : Diptheria คิดถึงเนื่องจากอาการเข้าได้ คือ ไข้ น้ามูกไหล ไอ เจ็บคอ หายใจลาบาก คอบวม กลืนลาบาก
ร่วมกับตรวจพบ greenish patch ขูดออกยาก ขูดแล้วมีเลือดออก
DDX. Tonsil with patch
- Streptococcus tonsillitis : patch เป็นจุดเป็นหย่อมๆ สีขาว
- Infectious mononucleosis: patch สีขาวสะอาด+lymphadenopathy+spleenomegaly
Croup: หายใจลาบากได้เหมือนกัน แต่ไม่มี patch ไอเป็นเสียงหมาเห่า เสียงแหบ ได้ยิน stridorตอนหายใจเข้า
มี suprasternal retraction
Candidiasis : ข้อนี้ไม่ได้เปิดหนังสือเป็นเรื่องเป็นราว แต่ จากclinical ก็ไม่ค่อยคิดถึง onsetน่าจะนานกว่านี้
reaction ไม่น่ารุนแรงอย่างนี้
Pediatric
13. เด็กชายอายุ 6 ปี U/D Asthma ไข้ ไอ น้้ามูก 2 d หอบมากขึ้น PE:ไข้ คอแดง suprasternal, intercostal, subcostal
retraction, inspiratory and expiratory wheezing, RR 40/min ให้ salbutamol 3 dose ทุก 20 min หอบน้อยลง แต่ยังมี
exp.wheezing อยู่ ในขณะนั้นให้การรักษาอะไรเพิ่มดีที่สุด
A. Oral mucolytic
B. Oral antibiotic
C. IV Aminophylline
D. IV Dexamathasone
E. NB Ipratropium bromide
ตอบ D. IV Dexamethasone
ผู้ป่วย asthma หลังให้ salbutamol 3 dose แล้วยังคงมี wheezing เหลืออยู่ ซึ่งถือว่ามีความรุนแรงปานกลาง
(RR>30/min, exp.wheezing) ต้องท้าการรักษาต่อคือ
- NB salbutamol 2.5 mg q 1-2 hr. ประเมินอาการทุก 2 hr. ถ้าดีขึ้นปรับให้ยาทุก 3-4 hr. และ 4-6 hr. ตามล้าดับ
- ให้ Corticosteroids ต่างๆ เช่น Oral prednisolone หรือ IV hydrocortisone 5 mg/kg/dose q 6 hr

14. เด็ก 10 ปี U/D Bthal/HbH ต้องรับเลือดทุกเดือน แพทย์พิจารณาท้า splenectomy ก่อนผ่าตัดควรได้รับ vaccine ใด เพื่อ
prophylaxis
A. Typhoid vaccine
B. Influenza vaccine
C. Pneumococcal vaccine
D. Meningococcal vaccine
ตอบ C. Pneumococcal vaccine + D. Meningococcal vaccine
Splenectomy เสี่ยงต่อการติดเชื้อ Encapsulated organism เช่น Sterptococcus pneumoniae, Haemophilus
influenzae type b, Neisseris meningitidis จึงควรฉีด vaccine ก่อนการท้า splenectomy 2 wk
Streptococcal pneumoniae : Polyvalent pneumococcal vaccine (pneumovax23)
Haemophylus influenzae : Haemophylus influenzae b vaccine (HibTITER)
Neisseris meningitidis
- age 16 – 55 yr : Meningococcal (group A, C, Y, W135) polysaccharide diphtheria toxiod
conjugated vaccine (Menactra)
15. ผู้ป่วยหญิง 13 ปี คอโตแบบ generalize มา 3 mo ไม่มีอาการอื่น FT4 1.25, TSH 3.5 Dx?
A. Simple goiter
B. Hashimoto thyroiditis
C. Subacute thyroiditis
D. Iodine deficiency
E. CA thyroid
ตอบ A. Simple goiter
ผู้ป่วยรายนี้มีค่า FT4 ปกติ (0.8 – 2.4 ng/dl), TSH ปกติ (2 -10 mu/l) เป็น euthyroid
Simple goiter : - มักพบในวัยรุ่น ช่วงเริ่มมีการเจริญพันธุ์ พบในเพศหญิงมากกว่าเพศชาย
- มีก้อนโตแบบทั่วๆ (Diffuse or generalize) กดไม่เจ็บ
- ผู้ป่วยมักไม่ค่อยมีอาการผิดปกติใดๆ นอกจากก้อนโตที่คอ
- TFT เป็น euthyroid หรือ hyperthyroid เล็กน้อย
Hashimoto thyroiditis : เป็น autoimmune disease ที่มี B-lymphocyte เกาะตามบริเวณต่างๆ ของต่อม thyroid พบในเพศ
หญิงมากกว่าเพศชาย
- ก้อนโตทั่วๆ (diffuse) แข็งกว่า simple goiter
- พื้นผิวไม่เรียบ (pebbly surface) ผู้ป่วยเด็กหรือวัยรุ่นจะคล้าได้พื้นผิวเรียบ
- TFT อาจพบว่าเป็น hypothyroid แต่ถ้าเป็นแบบ subacute แล้ว ผล TFT อาจปกติได้
Subacute thyroiditis : มักพบว่ามีไข้ ก้อนโตและกดเจ็บบริเวณก้อนอย่างชัดเจน
Iodine deficiency : มักพบในพื้นที่ที่ขาดสารไอโอดีน (endermic goiter)
- ผู้ป่วยจะมีคอพอกโตมาตั้งแต่เด็ก
- อาจมีการเจริญเติบโตและพัฒนาการทางสมองช้าได้
- มีอาการแบบ hypothyroid เชื่องช้า,
- TFT มี FT4 ปกติหรือต่้า , TSH ปกติหรือสูง
CA thyroid : - มีก้อนเป็น nodule ลักษณะมีทั้งนิ่มและแข็ง,
- มีประวัติเคยฉายรังสีบริเวณศีรษะและคอ,
- มีประวัติโรคมะเร็งต่อมไทรอยด์ในครอบครัว,
- TFT อาจเป็น euthyroid, ควรตรวจ FNA, Thyroid scan เพิ่มเติม

16. หญิง 2 ปี ขาอ่อนแรง เดินไม่ไหว PE : mild pale no Jx มี nodule ที่หัว มีก้อนที่ท้องขนาด 6x8 cm cross
midline hard consistency Dx
A Neuroblastoma
B Wilm’ s tumor
C ANLL
D Non-Hodgkin lymphoma
E Retroperitoneal teratoma
เฉลย A Neuroblastoma

17. เด็กหญิงอายุ 4 ปี มี RUQ pain with mass urine : RBC 10 -20 , WBC 1-2 จงให้การวินิจฉัยโรค
A Wilm’ s tumor
B Neuroblastoma
C Lymphoma
D Hydronephrosis
E ……….
เฉลย A Wilm’ s tumor
ค้าอธิบาย 2 ข้อรวดเดียวเลยนะ
NEUROBLASTOMA

- Neuroblastoma เป็น highly malignant tumor ซึ่งเกิดจาก neural crest ดังนั้นจึงสามารถพบเนื้องอกนี้ตาม sympathetic
chain ตั้งแต่คอลงมาจนถึง pelvis ร้อยละ 70 ของ neuroblastoma ทั้งหมดพบในช่องท้อง โดยที่ร้อยละ 40 พบที่ชั้น medulla
ของต่อมหมวกไต ร้อยละ 25 พบที่ paraspinal sympathetic ganglia และร้อยละ 5 พบใน pelvis นอกจากนี้ยังสามารถพบได้
ในที่อื่นซึ่งไม่ใช่ในช่องท้องได้แก่ ร้อยละ 20 พบในทรวงอก และร้อยละ 3-5 พบที่บริเวณคอ neuroblastoma เป็น malignant
tumor ที่พบบ่อยที่สุดของช่องท้อง ร้อยละ 50 ผู้ป่วยมีอายุน้อยกว่า 2 ปี และร้อยละ 85 น้อยกว่า 4 ปี เด็กมักจะซูบ (cachexia)
ลักษณะของก้อน fixed ใน retroperitoneum, แข็ง, ผิวขรุขระ, ข้ามถึงกลางล้าตัว ซึ่งแตกต่างจากลักษณะก้อนของ Wilms’
tumor สองในสามจะมี metastases ไปยังต่อมน้้าเหลือง, กระดูก (พบบ่อยใน skull, long, bone) bone marrow, ตับ,
subcutaneous nodules, ปอด, สมอง ตามล้าดับ ความถี่ที่พบ plain film จะพบ calcification ได้ร้อยละ 50-60 IVP จะมี
ลักษณะ displacement ของ calyceal system

Clinical Presentation ประมาณ 70% ของผู้ป่วยจะมาพบแพทย์ในระยะที่โรคมีการแพร่กระจายแล้ว ( disseminated disease)


ซึ่งต่างกับ Wilms' tumor ซึ่งมีผู้ป่วยเพียงประมาณ 15-20% เท่านั้นที่จะมาในระยะที่โรคแพร่กระจาย อาการน้าของผู้ป่วยจะ
ขึ้นกับต้าแหน่งของเนื้องอกและระยะของโรคเป็นส้าคัญ

1. Localized Symptoms ขึ้นกับต้าแหน่งของ primary tumor เป็นส้าคัญ อาการที่พบได้บ่อย ได้แก่ large abdominal mass,
เนื่องจากต่อมหมวกไตเป็นต้าแหน่งที่พบได้น้อยที่สุด ในผู้ป่วยที่มี paraspinal tumor อาจมาพบแพทย์ด้วยเรื่อง paralysis
เนื่องจากเนื้องอกมีการลุกลามผ่าน neural foramen เข้าไปใน spinal canal ("dumbbell tumor")

2. Systemic Symptoms เนื่องจากเด็กส่วนใหญ่จะมาพบแพทย์ในระยะที่โรคมีการแพร่กระจาย ดังนั้นจึงมักพบอาการทั่ว ๆ ไป


ร่วมด้วย ได้แก่ ไข้ , อ่อนเพลีย, น้้าหนักลด, ซีด, failure to thrive และอาจพบอาการและอาการแสดงจาก catecholamine
production เช่น flushing, headache, tachycardia

3. Metastatic Symptoms อาการและอาการแสดงขึ้นกับต้าแหน่งที่โรคมีการแพร่กระจายไป ต้าแหน่งที่พบได้บ่อย ๆ ได้แก่ ต่อม


น้้าเหลือง, ไขกระดูก , กระดูก, ตับ
WILMS' TUMOR ( NEPHROBLASTOMA )

Definition : Highly malignant embryonal tumor of the kidney

Wilms' tumor พบได้มากที่สุดในมะเร็งของระบบ Genitourinary tract ในเด็ก อุบัติการณ์ของโรค ประมาณ 7.7 ราย : 1,000,000 ต่อปี
ของเด็กที่อายุ 1-15 ปี มักพบในเด็กอายุน้อยกว่า 5 ขวบ ,

Clinical Presentation ผู้ป่วยส่วนใหญ่จะมาพบแพทย์ด้วยเรื่องคล้าได้ก้อนในท้อง ( palpable mass) โดยมักจะเป็นการคล้าได้โดย


บังเอิญ เช่น ในขณะที่อาบน้้าให้เด็ก ลักษณะก้อนจะค่อนข้างแข็ง , เรียบ, ไม่เจ็บ นอกจากนั้นส่วนใหญ่ (ร้อยละ 80) ของผู้ป่วยที่มีอายุน้อย
กว่า 6 ปี เด็กจะมาพบแพทย์ด้วยท้องโต มีก้อนแข็งผิวเรียบบริเวณด้านหลังของท้อง มักจะไม่ข้ามแนวกลางล้าตัว เด็กบางคนมีอาการปวด
ของก้อน, hematuria, ไข้ ส่วนใหญ่จะไม่มีอาการของเบื่ออาหาร, น้้าหนักลด ยกเว้นรายที่มีก้อนโตมาก ร้อยละ 5 ของเนื้องอกนี้เป็นที่ไตทั้ง
สองข้าง มีร้อยละ 5-10 ที่เนื้องอกได้ลุกลามมาเส้นเลือดด้าของไตข้างที่มีเนื้องอก ปัสสาวะสีแดงหรือน้้าล้างเนื้อ (gross hematuria), ไข้
อ่อนเพลีย การตรวจร่างกายอาจพบความดันโลหิตสูง และความผิดปกติทพันธุกรรมอย่างอื่นร่วมด้วย ( associated congenital
anomalies) โดยเฉพาะ genitourinary tract, hemihypertrophy และ aniridia

Teratoma abdominal teratoma มักมาจาก rectroperitoneum, กระเพาะและตับ ก้อนมีลักษณะ mixed cystic-solid และมี
calcification ร้อยละ 8 ของ retroperitoneal teratoma จะเป็นเนื้อร้ายโดยเฉพาะอย่างยิ่ง endodermal sinus tumor tumor marker มี
AFP, HCG, CEA
Lymphoma abdominal lymphoma จะเกิดใน GI tract ซึ่งมักพบที่ lymphoid tissue ในล้าไส้เล็กหรือล้าไส้ใหญ่ , mesertery และ
retroperitoneum lymphoma มี 2 กลุ่ม คือ Hodgkin และ non-Hodgkin lymphoma ใน Hodgkin lymphoma จะพบในเพศชายมากกว่า
หญิง 3:1 อายุ 10 ปีขึ้นไป พยาธิวิทยาจะพบ Reed-Sternburg cells แบ่งย่อยได้ 4 แบบ lymphocyte predominance (พบร้อยละ 23),
nodular sclerosis (ร้อยละ 38.8) mixed cellularity (ร้อยละ 33.5), lymphocyte depletion (ร้อยละ 4.7) ตามอัตราส่วนของ
lymphocyte และ fibrosis lymphoma ชนิดนี้มักพบอยู่เหนือต่อกระบังลม ทางศัลยกรรมมักเกี่ยวข้องในการ exploratory staging ใน
stage I หรือ II โดยการท้า nodal biopsy, wedge liver biopsy และ splenectomy เพื่อตัดสินการรักษาตามระยะของโรค Non-
Hodgkin’s lymphoma เพศชายต่อหญิง 3:1 อายุเฉลี่ย 7-11 ปี แบ่งได้ 3 แบบ lymphoblastic lymphoma (derived จาก T cells) มักพบ
เหนือต่อกระบังลม เช่น ใน mediastinum small cell lymphoma ซึ่งสามารถแบ่งย่อยเป็น Burkitt และ non Burkitt lymphoma ซึ่งเป็น
สาเหตุส่วนใหญ่ของ abdominal lymphoma ท้าให้เด็กมีอาการก้อนในช่องท้อง หรือล้าไส้กลืนกัน (intussusception) ในเด็กโต large-cell
lymphoma ซึ่งมักจะเกิดที่บริเวณใบหน้า , ปอด หรือสมอง

hydronephrosis คือ การมี dilatation ของ renal pelvis และ collecting system ซึ่งเกิดจาก mechanical obstruction หรือ functional
obstruction hydronephrosis เป็นหนึ่งในสองสาเหตุของก้อนในท้องของเด็กแรกเกิดที่พบบ่อยที่สุด โดยทั่วไป hydronephrosis ที่
สามารถคล้าได้จากหน้าท้องจะมีสาเหตุจาก mechanical obstruction สาเหตุที่พบบ่อยที่สุดของ hydronephrosis ในเด็กคือ
ureteropelvic junction (UPJ) obstruction ซึ่ง UPJ obstruction มักเป็นจาก idiopathic stenosis ส่วนน้อยเป็นจาก extrinsic pressure
จากเส้นเลือดพาดผ่านบริเวณ UPJ, aperistatic ของท่อไต, หรือ ureteral polyp สาเหตุรองลงมาของ hydronephrosis คือ ureterocele
บางครั้ง ureterocele โตจนมีการอุดกั้นของท่อไตด้านตรงข้าม หรือ ureterocele ของท่อไตท่อใดท่อหนึ่งของ complete duplex system
แล้วเกิดการอุดกั้นของท่อไตทั้งสองได้ ส้าหรับการมี hydronephrosis ของไตทั้งสองข้าง ซึ่งพบได้ร้อยละ 30 ของ hydronephrosis
ทั้งหมด เป็นเรื่องที่ต้องรับตรวจวินิจฉัยและรักษา สาเหตุที่พบบ่อยที่สุดของกรณีนี้คือ posterior urethral value ฉะนั้นเด็กเพศชายที่มี
bilateral hydronephrosis และคล้ากระเพาะปัสสาวะได้ให้คิดถึง posterior urethral value ลักษณะทางคลินิกของ hydronephrosis เป็น
ก้อนที่อยู่ในบริเวณด้านหลังของท้อง หรือสีข้างมีผิวเรียบ cystic consistency การตรวจด้วยอัลตราซาวด์จะแสดงให้เห็นการ dilatation
ของ calyceal system ระดับการ dilatation ของท่อไต สามารถแสดงถึงต้าแหน่งของการอุดตันของทางเดินปัสสาวะส่วนบนหรือล่าง โดย
หลักทั่วไปแล้วการตรวจเพิ่มเติมด้วย DTPA scan หรือ IVP ส้าหรับรายที่มีการอุดตันของท่อไตส่วนบน หรือตรวจด้วย voiding
cystourethrography ส้าหรับรายที่มีการอุดกั้นของทางเดินปัสสาวะส่วนล่างตั้งแต่ vesicoureteral junction ลงมา การรักษาขึ้นอยู่กับ
สาเหตุแต่ละอย่างของ hydronephrosis

ANLL Leukemia: Signs and Symptoms

Bone marrow infiltration

 Anemia: Pallor, lethargy , Dyspnea, murmur


  Platelets : Bleeding, petechiae, purpura
 Neutropenia :Fevers and infections
 Bone pain :Limp,  walking, irritability

Extramedullary spread

 Lymphadenopathy
 Hepatosplenomegaly
 Orthopnea, cough
 mediastinal mass
 tracheal compression
 Facial nerve palsy
 Testicular enlargement
 Skin lesions
 Gingival hypertrophy

Fever of malignancy

18. เด็กชาย 3 ปี ไข้ ซีด เหลือง reticulocyte 7 % ให้ตรวจอะไรต่อ


A G6PD screening
มี choice มาอันเดียวอ่ะ
Reticulocyte count เป็นการตรวจนับเม็ดเลือดแดงตัวอ่อน ค่านี้บ่งถึงการตอบสนองของไข
กระดูกต่อภาวะซีดในคนปกติมีค่าประมาณ 1-2% ค่าที่สูงขึ้นแสดงถึงการที่ไขกระดูกพยายามผลิตเม็ด
เลือดแดงใหม่ พบในภาวะเสียเลือด hemolysis หรือภาวะ hypoxuia ค่าที่ต่้าไม่ได้สัดส่วนกับระดับ
ฮีโมโกลบิน แสดงถึงภาวะที่ไขกระดูกท้างานผิดปกติเช่น การขาดธาตุเหล็ก หรือ bone morrow
iltrative condition ต่างๆ
G6PD อ าการและอาการแสดง

ผู้มีภาวะ G-6-PD ส่วนใหญ่ไม่มีอาการ แต่อาจมีอาการที่ส้าคัญได้ 3 อย่างคือ ภาวะซีดอย่างเฉียบพลัน (acute hemolytic anemia


หรือ hemolytic crises) ภาวะเหลืองจัดในทารกแรกเกิด และภาวะซีดเรื้อรัง คล้ายกับภาวะพร่องเอ็นซัยม์อื่นๆที่เรียกกันว่า chronic
nonspherocytic hemolytic anemia (CNSHA)

1. ภาวะซีดอย่างเฉียบพลัน ( Acute hemolytic anemia)


ในประเทศไทยผู้ที่พร่อง G-6-PD จะมีอาการปรกติดีทุกอย่าง ส่วนมากจะไม่ทราบว่าตนเองมีความผิดปรกตินี้อยู่ นอกจากบาง
คน นานๆครั้ง เมื่อมีภาวะเจ็บป่วย หรือได้รับยาหรือสารบางอย่างเข้าไป ซึ่งสิ่งเหล่านี้ก่อ oxidatie stress จะมีผลท้าให้เม็ดเลือด
แดงแตกรวดเร็วอย่างมากในหลอดเลือด ( acute intravascular hemolysis) ดังกล่าวแล้ว ผู้ป่วยจะมีอาการซีดลงอย่างรวดเร็ว
ร่วมกับถ่ายปัสสาวะด้า และอาจตามมาด้วยภาวะไตล้มเหลวเฉียบพลันได้ acute hemolytic anemia จากภาวะพร่อง G-6-PD
อาจจ้าแนกได้เป็น
1.1 Drug-induced hemolytic anemia ได้แก่กลุ่มที่มีอาการซึดที่เกิดจากยาหรือสารเคมีบางอย่างอันเป็นสาเหตุท้าให้เม็ด
เลือดแดงแตกง่าย มียามากชนิดที่มีผู้รายงานไว้ว่า อาจเป็นสาเหตุท้าให้เม็ดเลือดแตกง่าย(ตารางที่ 16.18) ยาบางชนิดอาจก่อ
ภาวะ hemolysis ในผู้ที่มีภาวะพร่อง G-6-PD บางคนหรือบางครั้งใช้ยาเท่านั้น จึงน่าจะมีปัจจัยอื่นๆมาเกี่ยวข้องด้วย
ในประเทศไทยซึ่งผู้ป่วยอาจซื้อหายามารับประทานได้เองโดยสะดวก โอกาสที่จะได้รับยาอันตรายต่างๆ รวมทั้งการได้รับยาผิด
ขนาดมีมาก อย่างไรก็ตามผู้ป่วยส่วนมากมักมีการเจ็บป่วย ไม่สบายก่อนจีงกินยาและมาโรงพยาบาลเมื่อมีอาการของเม็ดเลือด
แดงแตกแล้ว ฉะนั้นจึงไม่อาจสรุปได้ว่าอาการดังกล่าวเป็นผลจากการใช้ยาหรือแท้จริงเกิดจากการเจ็บป่วยหรือร่วมกัน
1.2 Spontaneous hemolysis without drug exposure เป็น hemolytic anemia ที่เกิดขึ้นในผู้ป่วย โดยได้รับยาหรือสารเคมี
ใดๆ มาก่อน ได้แก่ การติดเชื้อไวรัส เช่นไข้หวัด ไข้เลือดออก ตับอักเสบ หรือแบคทีเรีย เช่น ไทฟอยด์ ซึ่งคาดว่าสาเหตุส้าคัญและ
ส่วนใหญ่ของ acute hemolysis จะอยู่ในกลุ่มนี้ นอกจากนี้ยังพบได้ในภาวะอื่นๆอีก เช่น การมีน้าตาลในเลือดต่้า หรือภาวะเป็น
กรดสูง เช่นเดียวกันพบว่า ผู้ป่วยส่วนใหญ่มักได้รับยาก่อนมาโรงพยาบาลด้วย มีข้อสังเกตในภาวะตับอักเสบ นอกจากจะร่วมกับ
ภาวะซีดลงแล้วจะมีอาการเหลืองจัดร่วมด้วย แต่อาการมักไม่รุนแรงเมื่อเทียบกับความเหลือง
1.3 Favism ผู้มีภาวะพร่อง G-6-PDบางรายโดยเฉพาะอย่างยิ่งชาวผิวขาวในแถบทะเลเมดิเตอร์เรเนียน เมื่อได้รับประทานถั่ว
ปากอ้า (fava bean) ซึ่งมีสารพวก divicine และ isouramil อยู่ อาจเกิดภาวะ hemolysis เรียกว่า Favism จากประสบการณ์ของ
เราก็พบผู้ป่วยมี hemolysis จากการรับประทานถั่วปากอ้า 7 ราย เป็นชาย 5 ราย เป็น หญิง 2 ราย อาการ hemolysis ที่เกิด
ค่อนข้างรุนแรง แต่ผู้ป่วยก็สามารถฟื้นตัวได้ในระยะเวลาอันสั้น มีรายงานภาวะ favic crisis ในเด็กชายญี่ปุ่นที่พร่อง G-6-PD มี
hemoglobinuria และอาการดีขึ้นจากการรักษาด้วย haptoglobin
Acute hemolytic anemia ในภาวะพร่อง G-6-PD ที่ส้าคัญและรุนแรงคือ acute intravascular hemolysis ท้าให้เกิดอันตราย
ถึงชีวิตได้จากภาวะแทรกซ้อนภายหลังจากภาวะติดเชื้อหรือการได้รับยาบางชนิดประมาณ 48-72 ชั่วโมง ผู้ป่วยจะมีไข้ มีอาการ
อ่อนเพลีย ปัสสาวะสีแดงหรือสีน้าตาลคล้ายโคคาโคล่า หรือด้าคล้ายยาแก้ไอน้้าด้า ทั้งนี้สุดแต่ความรุนแรง และความใหม่ของ
ปัสสาวะ บางรายอาจมีอาการคลื่นไส้ ปวดท้องร่วมด้วย พร้อมกันนี้จะมีอาการซีดลงมาก หัวใจเต้นเร็ว กระสับกระส่าย ถ้ารุนแรง
มากจะมีอาการ shock หัวใจวายได้ อาจพบตับโตกดเจ็บ บางรายจะพบว่าม้ามโตและเหลืองเล็กน้อยซึ่งอาจเกิดจากมีภาวะ
extravascular hemolysis ร่วมด้วย ในภาวะ Favism อาการผิดปรกติอาจเริ่มภายหลังจากรับประทานถั่วปากอ้า ในเวลาเพียง
ไม่กี่ชั่วโมงเท่านั้น ภาวะแทรกซ้อนที่ส้าคัญคือ ซีดมาก หัวใจวาย โปแตสเซียมสูงในเลือด และไตวาย
เป็นที่น่าสังเกตว่าภาวะ acute hemolytic anemia ไม่ได้เกิดบ่อยในผู้มีภาวะพร่อง G-6-PD ทั้งที่ผู้นั้นก็เคยมีภาวะติดเชื้อ
ได้รับยา หรือรับประทานถั่วปากอ้ามาก่อน เข้าใจว่าขึ้นอยู่กับปริมาณที่ได้รับหรือความรุนแรง หรืออาจมีปัจจัยอื่นมาเกี่ยวข้อง
ด้วย ส้าหรับภาวะ Favism พบว่าขึ้นอยู่กับปริมาณถั่วที่รับประทานและพบว่าถั่วดิบก่อ Favism มากกว่าถั่วสุก ถั่วที่ผ่านการแช่
แข็ง หรือถั่วกระป๋อง รวมทั้งปริมาณของ glycosides และ aglycone ในถั่วด้วย ในขณะที่ภาวะติดเชื้อจากแบคทีเรีย กลไกที่ท้า
ให้มีการสร้าง peroxide มาก เกิดขึ้นระหว่างที่เม็ดเลือดขาวชนิด granulocyte จับกินแบคทีเรีย (phagocytosis) ส่วนใหญ่ผู้ป่วย
พร่อง G-6-PD ที่มี acute intravascular hemolysis และได้รับการรักษาจนผ่านพ้นภาวะวิกฤติไปแล้วจะมีอาการปรกติอย่าง
รวดเร็ว ทั้งนี้เพราะไขกระดูกสร้างเม็ดเลือดแดงใหม่ๆออกมาชดเชยอย่างเต็มที่ hemolysis ก็จะหยุดไปเอง ( self limited) แม้ใน
รายที่ไม่ได้ให้เลือด ระดับฮีโมโกลบินก็จะกลับสู่ปรกติได้ภายใน 3-6 สัปดาห์ ฉะนั้นการดูแลรักษาผู้ป่วยในช่วงวิกฤติจึงมี
ความส้าคัญมาก
2. ภาวะเหลืองจัดในทารกแรกเกิด สาเหตุของภาวะเหลืองในทารกแรกเกิดในประเทศไทย ที่ส้าคัญเกิดจากหมู่เลือด ABO แม่-
ลูก ไม่เข้ากัน และภาวะติดเชื้อ จากการศึกษาที่โรงพยาบาลศิริราชพบว่าในทารกเพศชาย เหลืองบ่อยกว่าทารกเพศหญิงใน
อัตราส่วน 1.6:1 และทารกเพศชายที่พร่อง G-6-PD เหลืองบ่อยกว่าทารกเพศชายปรกติในอัตราส่วน 1.7:1 แต่ความรุนแรงไม่
แตกต่างกัน ได้มีรายงานจากสองคณะในภาคเหนือของไทย พบพาวะพร่อง G-6-PD ในทารกที่เหลืองแรกเกิดในอัตราที่ต่างกัน
มาก (ร้อยละ 64 และร้อยละ 6.5) และความรุนแรงต่างกันมากด้วย ในต่างประเทศก็พบข้อมูลที่แตกต่างกันเช่นนี้ ทั้งๆที่เป็น
การศึกษาในชนกลุ่มที่ G-6-PD variant เดียวกัน มีรายงานภาวะเหลืองแรกเกิดที่รุนแรงมากจนท้าให้เกิด kernicterus เขาพบว่า
ปัจจัยแวดล้อมอื่นได้แก่ การใช้ยาในแม่ การติดเชื้อและการใช้ยาในทารก รวมทั้งสารเคมีต่างๆ เช่น ลูกเหม็น ( naphthalene)
เป็นปัจจัยที่ส้าคัญที่ส่งเสริมให้เกิดภาวะเหลืองบ่อยกว่าและรุนแรงกว่าทารกที่ไม่มีปัจจัยเสริม ซึ่งในกลุ่มหลังนี้อาการมักไม่
รุนแรงเท่า ส่วนใหญ่จะไม่มีซีด ไม่มี reticulocytosis เข้าใจว่าจะเป็นผลจากภาวะพร่อง G-6-PD ของเซลล์ตับเป็นส้าคัญ ที่
โรงพยาบาลศิริราชพบว่าร้อยละ 20 ของเด็กชายที่มีอาการเหลือง มีภาวะพร่อง G-6-PD แต่ทารกจะเหลืองไม่มาก นอกจากจะมี
สาเหตุอื่นร่วมกันอยู่ เช่นมีหมู่เลือด แม่-ลูกไม่เข้ากัน หรือมีโรคติดเชื้อร่วมด้วยจึงจะมีอาการเหลืองจัด เนื่องจากทารกที่เหลือง
มาก อาจเกิด kernicterus และสมองพิการตามมาได้ จึงต้องได้รับการรักษาโดยรีบด่วน
3. ภาวะเม็ดเลือดแดงแตกเรื้อรัง (CNSHA) ผู้ป่วยจะมีอาการซีดเรื้อรังแบบโรคธาลัสซีเมีย หรือภาวะพร่องเอ็นซัยม์อื่นๆ พบน้อย
กว่าพวกที่ 1 มาก มีรายงานเพียงประมาณ 100 ราย เช่น G-6-PD Sunderland, G-6-PD Beverly Hill เป็นต้น ส้าหรับในคนไทย
มีรายงานแล้วเพียง 1 ราย คือ G-6-PD Bangkok ปัจจุบันพบอีก 2 ราย ที่ภาควิชากุมารเวชศาสตร์ โรงพยาบาลศิริราช
นอกจากนี้ยังมีรายงานภาวะพร่อง G-6-PD ที่พบร่วมกับโรคที่ท้าให้เม็ดเลือดแดงแตกเรื้อรังอื่นๆ เช่น ธาลัสซีเมีย hereditary
spherocytosis ท้าให้เสริมอาการกันรุนแรงยิ่งขึ้น

ตารางที่ 16.18 Drugs and chemicals that should be avoided by persons with G-6-PD deficiency(2)
Acetanilid Sulfacetamide
Furazolidone (Furoxone) Sulfamethoxazole (Gantanol)
Methylene blue Sulfanilamide
Nalidixic acid (NegGram) Sulfapyridine
Naphthalene Thiazolesulfone
Niridazole (Ambilhar) Toluidine blue
Isobutyl nitrite Trinitrotoluene (TNT)
Nitrofurantoin (Furadantin) Urate oxidase
Phenazopyridine (Pyridium) Phenylhydrazine
Primaquine
การวินิจฉัยและการวินิจฉัยแยกโรค

ในทารกแรกเกิดที่มีอาการเหลืองผิดปรกติ โดยหาสาเหตุไม่ได้ หรือเหลืองมากไม่สมกับสาเหตุที่หาได้ หรือเหลืองร่วมกับมีประวัติการ


ได้รับยาที่อาจก่อภาวะเม็ดเลือดแดงแตกในมารดาหรือทารก ส้าหรับในเด็กโตหรือผู้ใหญ่ที่มีสุขภาพสมบูรณ์ดี แล้วมีอาการซีดลงอย่าง
รวดเร็ว ภายหลังจากเป็นไข้หวัด หรือการเจ็บป่วยใดๆและหรือร่วมกับการกินยา หรือ exposure ต่อสารอาหารที่อาจเป็นสาเหตุให้เม็ด
เลือดแดงแตกดังกล่าวมาแล้ว ประวัติที่เกี่ยวกับลักษณะและจ้านวนปัสสาวะที่ผิดปรกติ การเหลืองที่ตรวจไม่พบ bile ในปัสสาวะ และใน
ผู้ป่วยตับอักเสบที่ซีดลง และเหลืองจัด เหล่านี้ควรนึกถึงภาวะพร่อง G-6-PD

การวินิจฉัยแยกโรค ต้องแยกโรคที่มีภาวะซีดเฉียบพลันอย่างอื่น เช่น hemoglobin H disease ที่มี anemia crises, autoimmune


hemolytic anemia และ hereditary spherocytosis ที่มี anemic crises เป็นต้น ในทารกแรกเกิดต้องแยกโรคจากภาวะเหลืองจากสาเหตุ
อื่นๆ เช่น ภาวะหมู่เลือดไม่เข้ากัน ภาวะติดเชื้อ

การตรวจทางห้องปฎิบัติการ ขึ้นอยู่กับอาการทางคลินิกและเวลาที่ท้าการตรวจ ในผู้ป่วยที่เป็น CNSHA พบ hematocrit ต่้า,


reticulocytosis, indirect bilirubin เพิ่ม ซึ่งไม่ใช่ลักษณะของภาวะพร่อง G-6-PD ส่วนใหญ่ในประเทศไทย ซึ่งในภาวะปรกติไม่มีอาการ
และการตรวจ complete blood count ปรกติ แต่ระหว่างที่มี acute hemolysis และ ได้รับการตรวจเลือดเร็วจะพบ hematocrit ต่้า เมื่อปั่น
เลือดควรดูสีของ plasma ด้วยเสมอเพราะหากก้าลังมี intravascular hemolysis อยู่ ก็จะมี hemoglobinemia ท้าให้เห็น plasma มีสีแดง
ตามด้วย hemoglobinuria และจะตรวจพบ Heinz body จากการตรวจเลือดโดยใช้สี brilliant cresyl blue หรือ crystal violet เป็น
ตะกอนของ hemoglobin ซี่งมักจะติดอยู่กับผนังด้านในเม็ดเลือดแดงจ้านวนเม็ดจะน้อยกว่า inclusion body ที่เห็นใน hemoglobin H
disease อาจพบเม็ดเลือดแดงที่ก้าลังแตกมี Hb รั่วออกมา อย่างไรก็ตามผู้ป่วยส่วนใหญ่มักจะผ่านพ้นระยะ hemoglobinemia และ
hemoglobinuria ไปแล้ว จึงอาจตรวจไม่พบการเปลี่ยนแปลงดังกล่าว แต่จะตรวจพบ reticulocytosis เม็ดเลือดแดงติดสี polychromasia
และมีลักษณะผิดปรกติคือมี anisocytosis, poikilocytosis มาก จะพบ fragmented cell, eccentrocyte, contracted cell, bite cell,
spherocyte ซึ่งล้วนแต่เป็นร่องรอยที่แสดงว่ามีอาการรั่วของ hemoglobin ออกไปจากเม็ดเลือดแดง ตรวจไม่พบ haptoglobin เลย แต่จะ
มี unconjugated bilirubin เพิ่มได้ โดยเฉพาะยิ่งหากผู้ป่วยมีอาการเหลือง

การวินิจฉัยที่แน่นอนกว่าคือการตรวจเอ็นซัยม์ G-6-PD ในเม็ดเลือดแดง หลักการตรวจคือ activity ของ G-6-PD ในปฏิกริยาที่มี G-6-P


และ NADP โดยตรวจวัด NADPH ที่เกิดขึ้น

ข้อ 19 เด็กชาย 9 ปี เรียน ป.3 เขียนหนังสือไม่ค่อยได้ต้องสะกดทีละค้า IQ test ได้ 109 คะแนนการอ่าน 79 การเขียน 80 การ
ค้านวณ 97 ถามเด็กคนนี้เป็นอะไร
A) dyslexia
B) dyscalculia
C) mental retardation
D) Attention deficit disorder
E) Autism
ตอบ A) dyslexia Dyslexia คือเด็กไม่สามารถแยกแยะเสียงในแต่ละค้าที่ได้ยิน (phonological awareness) เด็กในกลุ่มนี้จึง
มีปัญหาในการสะกดค้า ซึ่งจากโจทย์ เด็กคนนี้เขียนหนังสือไม่ค่อยได้ ต้องสะกดทีละค้า และ IQ เด็กในกลุ่มนี้จะอยู่ในระดับปกติ
Dyscalculia ก็ไม่นึกถึง เพราะค้านวณคะแนนก็ดี (คิดว่าน่าจะคะแนนเต็มร้อยมั่งนะ)
ADD อาจพบร่วมกับอาการซนไม่อยู่นิ่ง ก็ไม่คิดถึง เพราะโจทย์ก็ไม่ได้บอกอาการที่เข้ากับ ADHD
MR ตัดทิ้งได้เลย เพราะ IQ ต้องน้อยกว่า 70
Autism จะมีปัญหาด้านสังคม พฤติกรรมและการสื่อความหมาย
ข้อ 20 เด็กอายุ 3 ขวบ ไข้มา 3 วัน แล้วซีด PE: Moderately pale , mild icteric sclera, No hepatosplenomegaly + ตรวจ
ร่างกายอย่างอื่นปกติ Lab: Hb 7.7, WBC 4,800, Lym เด่น, plt 380,000 , RC 7.5 , MCV 87 ต้องตรวจอะไรเพิ่ม
A) Coomb’s test
B) Inclusion test
C) Hemoglobin typing
D) G6PD screening
E) Bone marrow aspiration

ตอบ G6PD screening


จากโจทย์พบว่า ผู้ป่วยมีภาวะ normocytic anemia และพบว่า RC สูงขึ้น แสดงว่ามีภาวะ hemorrhage หรือ
hemolysis ซึ่งในผู้ป่วยรายนี้ก็นึกถึงกลุ่ม hemolysis มากกว่า เนื่องจากโจทย์บอกว่าตรวจร่างกายอื่นๆปกติ
G6PD ในผู้ที่เป็นโรคนี้ อาจมีอาการซีดได้ภายหลังอาการป่วยไข้ได้ 2-3 วัน โดยที่ไม่มีประวัติการได้รับยาก็ได้ เหลือง
ได้ anemia แบบ normochromic normocytic anemia
จาก choice ถ้าเป็น thalassemia ตรวจร่างกายน่าจะพบว่ามีตับม้ามโต lab น่าจะพบว่า red cell ตัวเล็กซีด จึงนึกถึง
น้อย ดั้งนั้นจึงไม่ส่ง hemoglobin typing และไม่เลือกท้า Bone marrow aspiration เนื่องจากเป็นหัตถการที่ invasive และถ้า
เป็น disease ใน Bone marrow น่าจะมีการสร้าง RC ได้น้อยลง ซีดแบบ normochromic normocytic และน่าจะมี series อื่น
ต่้าด้วย ไม่น่าเป็น red cell อย่างเดียว
AIHA เป็นกลุ่มอาการที่ร่างกายสร้าง ab ท้าลาย RBC ของตัวเอง อาการจะมี ซีด เหลือง อ่อนเพลีย มีไข้ ได้ ตรวจ
เลือดพบ RC สูงได้ Hb ต่้า ซึ่งเข้าได้กับผู้ป่วยรายนี้ แต่ AIHA จะมี ม้ามโตด้วย จึงนึกถึงน้อย ไม่ส่ง Coomb’s test
Hb H disease อาการจะมี ซีด เหลืองต่อเมื่อมีสูงจากการติดเชื้อได้ ตับม้ามโต ตรวจเลือดพบ RC สูงได้ Hb ต่้า และ
RBC ตัวเล็ก ดั้งนั้นจึงนึกถึงน้อย ไม่ส่ง Inclusion test

ข้อ 21 เด็กกินปลาทอด มีผื่นขึ้นตามตัว หายใจมีเสียง wheeze ประมาณ anaphylaxis ถามว่า first med คืออะไร
ตอบ Adrenaline(Epinephrine)

ยาที่ใช้ในการรักษา anaphylaxis
1. Epinephrine (1:1,000) ขนาด 0.01-0.03 cc/kg ให้ทาง sc หรือ IM เป็น drug of choice ซ้้าได้ทุก 20 นาที ไม่ควรให้
ทาง IV เพราะอาจเกิด arrhythmia ท้าให้เสียชีวิตหรือเกิด MI ได้ ผลของ Epinephrine ท้าให้หลอดเลือดหดตัว ลด
angioedema/urticaria และยับยั้งการหลั่ง mediators ออกจาก mast cells ในรายที่มี laryngeal edema และ
upper airway obstruction epinephrine จะเป็นยาตัวแรกที่จะต้องให้
2. Antihistamine : อาจต้องให้ทั้ง H1 & H2 antagonist ร่วมกันไป เช่น diphenhydramine (H1 antagonist) ขนาด 1
mg/kg IM หรือ IV และ cimetidine ขนาด 10 mg/kg
3. corticosteroid ให้ป้องกันการเกิด late phase ใช้ methylprednisolone ขนาด 1-2 มก./กก. ทุก 4-6 hr
4. ในรายที่มีหลอดลมตีบ ให้ salbutamol nebulization ขนาด 0.1-0.3 mg/kg ของ 5% salbutamol solution
5. vasopressor ใช้ในรายที่ไม่สามารถควบคุมระดับความดันโลหิตได้ มักใช้ dopamine ขนาด 0.01 mg/kg
22.เด็ก 1 ปี มีไข้ซึม 1วัน T 39.8 PR 140 RR 40 BP 90/60 CBC: Hb 11, WBC 20,000 N 93% Plt 50,000 coagulogram
prolong , petechiae and necrotic purpura at legs ถามว่าการ management ที่ส้าคัญ คือ
A. fluid therapy
B. antimicrobial
C. platelet concentration
D. FFP
E. vasopressin
Discussion
ค่าปกติของเด็กอายุ 1 ปี
Wt. 10 kg PR 120/min RR 20-30/ min SBP 96+/-30 DBP 66+/-25
แปลผล CBC
RBC : borderline anemia
WBC : leukocytosis , Neutrophil predominate = impression , severe bacterial infection
Decreased platelet & coagulogram prolong =bleed จากความผิดปกติของ primary & secondary hemostasis
วิเคราะห์
-ในผู้ป่วยรายนี้ PR &RR สูง BP ปกติ ท้าให้นึกถีง shock น้อยลง =ยังไม่ต้องการ fluid resuscitate เพื่อ แก้ shock
-DDx: DIC from severe infection R/O bacteria
Management of DIC
- Tx the underling cause ในที่นี้คือ severe infection R/O bacteria
-factor replacement : FFP (source of coagulation factor and protein C&S, AT III ) , cryoprecipitate(Provide
higher concentration of fibrinogen) and platelet transfusion . Aim to keep fibrinogen> 1.0 g/l, platelet>50,000 , & PT
normal
-anticoagulation therapy : Heparin (controversial)
ดังนั้นผู้ป่วยรายนี้การ management ที่ส้าคัญ คือ
Ans B. antimicrobial

23. ผู้ป่วยเด็ก มีหนังตาบวม ตัวบวมทั้งตัวมา 2 เดือน ตรวจร่างกายมี ascites UA: proteinuria 3+, RBC 0-1/HPF, Fine
granular cast 0-1, Fat oval cast 0-1 อะไรท้าให้เกิดการบวม
A. Decrease plasma oncotic pressure
B. Increase plasma hydrostatic pressure
C. Oncotic pressure ในcavity เพิ่มขึ้น
D. Hydrostatic pressure ใน cavity ลดลง
E. Increase vascular permeability
Discussion
วิเคราะห์ผล urinalysis
Proteinuria:
Dipstick Protein concentration (mg/dl)
Trace 10-20
1+ 30
2+ 100
3+ 300
4+ 1000-2000
Protein excretion normal 4 mg/m2/hr
Abnormal 4-40mg/m2/hr
Nephritic range>40 mg/m2/hr
RBC 0-1/HPF= no significant
Granular cast= non specific renal parenchymal disease
Fatty cast= heavy protein
Nephrotic syndrome
S&S periorbital edema
Anasarca
Weight gain
Pleural effusion
Ascites
Pitting edema
Hypertension (15%)
Lab 3-4+ proteinuria
Serum cholesterol สูง
Serum triglyceride สูง
Serum albumin < 2g/dl
Normal C3 level
>2 g urine protein in 24 hr

Serum Ca ลดลง secondary to low albumin


Urine protein: Cr ratio > 2.0
Microscopic hematuria (15%)
Pathophysiology
Proteinuria เป็นผลจาก การเพิ่มขึ้นของ glomerular wall permeability ส่งผลให้ capillary oncotic pressure ลดลง
fluid จาก intravascular shift มายัง interstitial space เป็นผลให้เกิด edema

Ans A. Decrease plasma oncotic pressure


24. เด็กหญิงอายุ 12 ปี เป็นลม หมดสติ 1 นาที หลังฟื้นปกติ ไม่เคยเป็นแบบนี้มาก่อน management อะไรต่อ
A. EKG
B. Blood glucose
C. 24 hr holter monitoring
D. EEG
E. no further inestigation
Discussion
Holter monitor (also called an ambulatory electrocardiography device), named after its inventor, Dr.
Norman J. Holter, is a portable device for continuously monitoring the electrical activity of the heart for 24 hours or
more. Its extended recording period is sometimes useful for observing occasional cardiac arrhythmias that would
be difficult to identify in a shorter period of time. For patients having more transient symptoms, a cardiac event
monitor which can be worn for a month or more can be used.
เนื่องจากผู้ป่วยไม่เคยมีอาการแบบนี้มาก่อนท้าให้นึกถึง cardiac cause น้อยลง(Ix:EKG, 24 hr holter
monitoring=arrhythmia) Cardiac syncope may be due to vascular disease, cardiomyopathy, arrhythmia, or valvular
dysfunction
หลังฟื้นเป็นปกติ=ไม่มี postictal period นึกถึงseizure น้อยลง(Ix:EEG)
Serum glucose level :In one study, 2 of 170 patients with syncope tested for serum glucose were found to
be hypoglycemic.แต่เนื่องจากhypoglycermia ที่ท้าให้เกิดsyncope มักมีอาการอื่นร่วมด้วย เช่น headache, mental
dullness, fatigue จึงท้าให้นึกถึงน้อยลง

ภาวะที่นึกถึงมากที่สุดคือ
Vasovagal syncope is the most common type in young adults but can occur at any age. It usually occurs in
a standing position and is precipitated by fear, emotional stress, or pain (eg, after a needlestick). Autonomic
symptoms are predominant. Classically, nausea, diaphoresis, fading or "graying out" of vision, epigastric discomfort,
and light-headedness precede syncope by a few minutes. Syncope is thought to occur secondary to efferent
vasodepressor reflexes by a number of mechanisms, resulting in decreased peripheral vascular resistance. It is not
life threatening and occurs sporadically.
Ans E. no further inestigation
28.) ผู้ป่วย newborn มีประวัติแม่ตั้งครรภ์ 41 wk with meconium stain amniotic fluid + fetal distress จะท้าอย่างไร
A) เช็ดตัว + ดีดเท้า
B) ET tube + suction
ตอบ ข้อ B
ทารกเป็น MAS (Meconium Aspiration Syndrome) การรักษาที่ดีที่สุดคือ การวินิจฉัยภาวะขาดออกซิเจนของทารกตั้งแต่
ในครรภ์ และถ้าเกิดขึ้นแล้วต้องป้องกันไม่ให้ขี้เทาเข้าไปในทางเดินหายใจส่วนล่าง โดย
1. suction เอา meconium ออกจากปากและล้าคอให้มากที่สุด หลังศีรษะโผล่ออกจากช่องคลอด โดยยังไม่คลอดไหล่
(ปัจจุบันไม่ท้ากัน เนื่องจากควรรีบน้าเด็กไปประเมิน vigorous และท้าตามข้อ 2)
2. การท้า direct tracheal suction เพื่อดูดเอา meconium ออกจาก trachea ให้ได้มากที่สุดในทารกที่เป็น non-
vigorous (heart rate >100 bpm. ,tone ไม่ด,ี ร้องไม่ด)ี ส่วนในทารกที่เกิดมาแล้ว tone ดี ร้องเองดี หายใจดี ไม่
จ้าเป็นต้องท้า tracheal suction
เพิ่มเล็กน้อย
ภาวะ MAS จะเกิดได้ต้องประกอบด้วย
1.) มีขี้เทาในน้้าคร่้า 2.) มี gasping respiration จาก Fetal asphyxia
การวินิจฉัย MAS
1. มีขี้เทาปนในน้้าคร่้า
2. มีอาการหายใจล้าบาก
3. Chest X-ray ผิดปกติ
4. พบขี้เทาในหลอดลมจากการท้า direct tracheal suction
Reference เอกสารประกอบการสอน อ.อารียา
เฉลยโดย กุ้ง ณ รพ.พุทธชินราช #47460431

29.) เด็ก term อายุ 2 เดือน คลอดที่บ้าน BW 4,600 gm PE: BT = 37, PR = 150, AF (Anterior fontanelle ) bulging, mod
pale, no jaundice, CBC: WBC 8,500 platelet 190,000 คิดว่าเป็นอะไร
A) sepsis
B) SDH
C) Idiopathic vitamin K deficiency of infancy
ตอบ Idiopathic vitamin K deficiency of infancy
เด็ก BBA (Birth before admit) มักไม่ได้รับ vitamin K หลังคลอด โดยจะเกิดภาวะ bleeding tendency ได้ 2 อย่าง
คือ
1.) The hemorrhagic disease of the newborn (HDN) มักเกิดภาวะเลือดออกง่ายใน 2-7 วันหลังคลอด มักมีเลือดออก
ทางล้าไส้มากกว่าทางอื่น ๆ มีเลือดออกทางผิวหนัง สะดือ เกิดเลือดออกในสมอง (พบน้อย) LAB : CBC มักอยู่ใน
เกณฑ์ปกติ ซีดได้ถ้ามีเลือดออกมาก platelet ปกติดี , coagulogram : APTT, PT prolong , TT ปกติ (ขาด factor
II,VII,IX,X)
2.) Acquired prothrombin complex deficiency syndrome (APCD) หรือ Idiopatic vitamin K deficiency of
infancy (IVKDI) เป็นโรคเลือดออกง่ายที่พบในเด็กอายุ ½ - 2 เดือน เป็นส่วนใหญ่ ที่ไม่ได้รับวิตามิน K แรกคลอด มัก
มาพบแพทย์ด้วยอาการเลือดออกในสมองมากกว่าอวัยวะอื่น โดยไม่มีอาการเลือดออกภายนอกหรือผิวหนังให้เห็น
ชัดเจน อาการทางสมอง ได้แก่ ซึม กระสับกระส่าย ร้องกวน ไม่ดูดนม ชัก anterior fontanelle bulging ,
hypoventilation ต้าแหน่ง ได้แก่ Subdural และ subarachnoid space LAB : มักซีดทุกราย Hb: < 5-10 g/dl มี
platelet และ WBC ปกติ coagulogram : APTT, PT prolong , TT ปกติ

ข้อ sepsis น่าจะลวงนะ เพราะเค้าน่าจะให้โจทย์ว่าแม่มี prolong PROM หรือเด็กมีลักษณะ poor feeding , ไม่ active อีก
ทั้งผล CBC with platelet ก็ไม่เข้ากับลักษณะ sepsis คือ WBC และ Platelet จะลดลง เป็นเค้าจะยังไม่คิดถึงนะ
ข้อ SDH (subdural hemorrhage) เค้าคิดว่ามันเป็นผลของ ) Idiopathic vitamin K deficiency of infancy นะ
Reference เอกสารประกอบการสอน อ.สมพร
เฉลยโดย กุ้ง ณ รพ.พุทธชินราช #47460431
30.)ผู้ป่วย newborn ตรวจร่างกายพบ heart murmur, cataract, ตับม้ามโต คิดว่า congenital infection เชื้อใด
ตอบ Rubella
Congenital Rubella syndrome อาการและอาการแสดง แบ่งเป็น 2 categories คือ
1.) ต้อกระจก (cataract) / ต้อหิน (congenital glaucoma) , congenital heart disease (PDA, pulmonary stenosis) ,
hearing impairment , จอประสาทตาผิดปกติ (pigmentation retinopathy)
2.) ผื่นแบบ purpura ตับม้ามโต ตัวเหลือง ศีรษะเล็ก พัฒนาการช้า เยื่อหุ้มสมอง และสมองอักเสบ Radiolucent
bone disease (ภาพถ่ายรังสี )

References : ปัญหาโรคเด็กที่พบบ่อย เรื่อง TORCHS Infection


เฉลยโดย กุ้ง ณ รพ.พุทธชินราช #47460431

31.ผู้ป่วยเด็กอายุ 6 เดือน มีอาการเขียวมาตั้งแต่อายุ 4 เดือน ครั้งนีมีเขียวมากขึ้นเมื่อร้องไห้ PE: tachycardia, tachypnea,


pansystolic murmur 2/6 at left parasternal border, cyanosis คิดว่าเป็นภาวะใด
A. Hypoxic spell
B. Asthma
C. Pulmonary …

เฉลย A. Hypoxic spell


Congenital heart disease สามารถแบ่งได้เป็น 2 กลุ่มใหญ่ คือ
1. โรคหัวใจพิการแต่ก้าเนิดชนิดไม่เขียว (Acyanotic congenital heart disease)

แบ่งตาม physiology ที่มีผลต่อ work load ของหัวใจได้เป็น 2 พวกได้แก่


ก. Left to right shunt lesion เป็นพวกที่เกิด volume load ต่อหัวใจ ได้แก่ VSD, PDA, ASD, AV septal
defect
ข. Obstructive lesions เป็นพวกที่ท้าให้เกิด pressure load ต่อหัวใจ ได้แก่ Pulmonary stenosis(PS),
Coarctation of aorta(COA), Aortic stenosis(AS)

สามารถแบ่งเป็น 2 กลุ่มตามอาการแสดง ได้แก่ Asymptomatic และ Symptomatic ซึ่งบ่งบอกถึงความรุนแรงของ


anatomical defect ซึ่งในกลุ่ม Symptomatic คือมีอาการของ CHF ในเด็ก ได้แก่ อาการเหนื่อยหอบ หายใจเร็ว ดูดนมได้
น้อย เหงื่อออกมาก เลี้ยงไม่โต น้้าหนักไม่ขึ้น ไม่สบายมีการติดเชื้อในระบบทางเดินหายใจบ่อยๆ ในเด็กที่โตขึ้นมาหน่อยก็
จะมีอาการเหนื่อยง่ายกว่าเด็กคนอื่นในขณะออกก้าลังกาย
2. โรคหัวใจพิการแต่ก้าเนิดชนิดเขียว (Cyanotic congenital heart disease)
แบ่งตามการไหลเวียนของเลือดไปที่ปอด ได้เป็น 3 พวก
ก. Decreased pulmonary blood flow ได้แก่ Tetralogy of fallot(TOF), Double outlet right ventricle with
PS, Single ventricle with PS, Pulmonary atresia with intact ventricular septum, Pulmonary atresia
with VSD, Tricuspid atresia with PS
ข. Increased pulmonary blood flow ได้แก่ Double outlet right ventricle, Single ventricle, Single atrium,
Total anomalous pulmonary venous connection, Truncus arteriosus
ค. d-Transposition of the great arteries(d-TGA)
สามารถแบ่งเป็น 2 กลุ่มตามอาการแสดง ได้แก่
1) Hypoxic spells
เป็นอาการที่พบในพวก Decreased pulmonary blood flow เด็กจะมีอาการเหนื่อยหอบและเขียวขึ้นทันที
สิ่งที่อาจเป็น precipitating factor เช่น การดูดนม การร้องไห้ เป็นต้น ซึ่งยังไม่เข้าใจกลไกการเกิดที่ชัดเจน
ในขณะเกิด spell อาจท้าให้เด็กมีอาการตัวแข็ง ตาเหลือง ชัก ไม่รู้สึกตัวและอาจถึงตายได้ นอกจากจะพบ
Hypoxic spell ในเด็กเล็กแล้ว ยังพบลักษณะเฉพาะในเด็กโตคือ squatting(การนั่งยองๆ) ขณะมีอาการเหนื่อย
หอบ บางรายที่เป็นมากแม้ในขณะพักก็จะนั่งยองๆ
2) Congestive heart failure

คือ Acyanotic congenital heart disease ที่มีอาการรุนแรง จะพบอาการเขียวไม่มากยกเว้นในโรค d-TGA


จะพบอาการเขียวมากร่วมด้วย

32.ผู้ป่วยเด็กอายุ 2 ปี 4 วัน PTA มีไข้ 38°C PE: drooling, กลืนล้าบาก, posterior pharynx บวม Dx
A. Acute tracheitis
B. Acute epiglottitis
C. Viral croup
D. Laryngitis
E. Retropharyngeal abscess
เฉลย E. Retropharyngeal abscess
Acute respiratory tract infection in children(ARIC) เป็นสาเหตุการตายอันดับ 1 ในเด็กอายุน้อยกว่า 5 ปีในประเทศ
ก้าลังพัฒนา (ประเทศไทย) แบ่งเป็น Lower tract กับ Upper tract ที่ระดับ larynx เหนือ larynx = upper ตั้งแต่ larynx ลงไป
เป็น lower
ดูอาการในข้อนี้กลืนล้าบาก(ความผิดปกติแถว pharynx) บอกได้เลยว่า upper แน่ๆ ซึ่ง URI ก็มีโรคดังต่อไปนี้
Diseases : Acute rhinitis : Acute sinusitis
: Acute pharyngitis : Otitis media
: Acute tonsillitis Infectious upper airway obstruction
: แล้วก็พวกผสมกัน เช่น : Viral croup
nasopharyngitis(common cold) , pharyngotonsillitis : Acute epiglottitis
Complications : Peritonsillar abscess : Spasmodic croup
: Retropharyngeal abscess : Bacterial tracheitis
Clinical presentation
Diseases อาการตรงไปตรงมาไม่กล่าวถึงนะ
Complications
Peritonsillar abscess : มี pharyngitis น้ามาก่อน ต่อมาเจ็บตอรุนแรงขึ้น มีอ้าปากไม่ได้ กลืนน้้าลายล้าบาก น้้าลายยืด อาจมี
คอเอียงไปด้านเดียวกับ lesion เพราะเกิด muscle spasm ข้างที่ inflame มีลิ้นไก่เอียงไปด้านตรงข้ามจาก tonsil เบียดบวม การ
รักษา ผ่า abscess ออก+ATB ถ้าเคยเป็นหรือเป็นบ่อยๆท้า tonsillectomy
Retropharyngeal abscess : มี pharyngitis น้ามาก่อนต่อมาไข้สูง เจ็บคอมาดขึ้น กลืนล้าบาก หายใจล้าบาก แหงนคอขึ้น มี
stridor เพราะน้้าลายมากเพราะกลืนล้าบากและมีน้าลายยืด( drooling) ตรวจร่างกาย เจอคอหอยโป่งพอง มี fluctuationจากมี
หนองสะสม X-ray เจอ soft tissue ที่ C3 หนากว่าเท่าครึ่งของ vertebral body ของ c3 และ loss of cervical lordosis การ
รักษาให้ผ่าเอาออก
Acute sinusitis : ประวัติหวักเรื้อรังนานกว่า 10 วันอาจมีรอบตาบวม ปวดใบหน้า ปวดกระบอกตา ปวดหัว น้้าหนองไหลลงคอ
ออกจมูก กลิ่นเหม็นตุๆ
AOM : เป็น conductive hearing loss ตรวจ weber = tunning fork 256Hz เคาะแล้ววางกลางหน้าผาก ข้าง lesion ได้ยินดัง
กว่า, Rinne = tunning fork 256Hz เคาะแล้ววางที่ mastoid bone เมื่อผู้ป่วยบอกว่าไม่ได้ยินรีบเอา fork มาหน้าหูข้างนั้น ข้าง
lesion ผู้ป่วยจะบอกว่าไม่ได้ยิน
Infectious upper airway obstruction
Croup = กลุ่มอาการที่มี RI แล้วอักเสบบวมของ larynx &trachea โดยเฉพาะต้าแหน่ง subglottic ท้าให้ obstruct
Viral croup : ไอเสียงก้อง เสียงแหบแห้ง หายใจเข้ามีเสียง stridor ก็จะมีการประเมิน croup score ดู 5 อย่าง คือ 1.เสียงหายใจ
เข้า 2.เสียง stridor 3.การไอ 4.ลักษณะการบุ๋มของหน้าอกและการบานของจมูก 5.ลักษณะเขียว การ diag ส่ง ดรสท lateral
neck มี pencil sign หรือ steeple sign(ยอดหอคอย) score น้อยกว่า 4 รักษาตามอาการ score 4-7 admit score > 7 severe
ให้ intubation และ admit การรักษาให้ epinephrine พ่น หรือ budesonide พ่น ถ้าอาการไม่ improve ให้ treat bacteria ด้วย
Acute epiglottitis : อาการมักจะรุนแรงในช่วงดึก โดยตื่นมากลางดึก ไข้สูง เสียงแหบ น้้าลายยืด หายใจล้าบาก มีเสียง stridor
แหงนคอขึ้น หายใจอ้าปาก ตรวจคออาจเห็น cherry-red epiglottis diag Film lateral neck เจอ Thumb sign
Spasmodic croup : อาการแสดงคล้าย viral croup แต่เบากว่า viral croup อาการมักดีขึ้นในเช้าวันรุ่นขึ้น
Bacterial tracheitis : อาการแสดงคล้าย viral croup แต่ treat แบบ viral croup แล้วไม่ดีขึ้น ควร intubation เนื่องจากเป็นการ
ติดเชื้อของ bacteria จึงต้องให้ antibiotic

33.ผู้ป่วยเด็กอายุ 3 ปี มารับ vaccine เคยรับ vaccine ครบจนถึงอายุ 1 ปี จากนั้นไม่ได้รับ vaccine อีกเลย จะให้ vaccine ใด
ใน visite นี้
A. OPV, DPT
B. HiB
C. MMR, OPV, DPT
D. HBV
E. OPV, DPT, JE
เฉลย E. OPV, DPT, JE
การให้ vaccine ห่างเกินกว่าก้าหนด ไม่ได้ท้าให้ภูมิคุ้มกันเกิดน้อยลง เพราะเมื่อร่างกายสร้างภูมิคุ้มกันจากการฉีด
vaccineเข็มก่อนๆ หากไม่ได้ใช้จะลดการสร้างลง แต่ยังเหลือเป็น memory cell เมื่อได้รับเชื้อหรือได้รับ vaccine ครั้งใหม่จะมี
การตอบสนองอย่างรวดเร็ว ในทางตรงกันข้ามการฉีด vaccine เร็วกว่าก้าหนดอาจท้าให้มีภูมิคุ้มกันเกิดขึ้นน้อยลง เด็กที่ไม่มา
ฉีดตามนัดสามารถฉีดเข็มถัดไปได้เลยโดยไม่ต้องตั้นต้นใหม่
Immunization สูตรแบบจ้าง่ายๆ
แรกเกิด BCG , HBV 1
2 mo. DPT 1 , OPV 1 , HBV 2
4 mo. DPT 2 , OPV 2
6 mo. DPT 3 , OPV 3 , HBV 3
9 mo. MMR 1
18 mo. DPT 4 , OPV 4 , JE 1 , JE 2 (ห่างกัน 1-4 wk.)
2 yr. JE 3
4 yr. DPT 5 , OPV 5 , MMR 2

34. ผู้ป่วย newborn preterm GA 32 wk น้้าหนักแรกคลอด 1,500 gm หลังคลอด 12 ชั่วโมงต่อมามีอาการหายใจล้าบาก ถาม


ว่าขาดสารตัวใดในปอด
A. Spyringomyelin
B. Lacitin
C. Phosphatidyl inositol
D. Phosphatidyl glycerol

เฉลย
ร่างกายสร้างสารลดแรงตึงผิวได้ตั้งแต่อายุครรภ์ประมาณ 20 สัปดาห์ โดย Pneumocyte type II ซึ่งเป็นเซลล์เยื่อบุผิว
ของถุงลมปอดและสร้างได้มากขึ้นเรื่อยๆจนสมบูรณ์เต็มที่ ที่อายุครรภ์ 35 สัปดาห์
โดยส่วนประกอบที่ส้าคัญของสารลดแรงตึงผิว คือ Phosphatidylcholine ซึ่งมีจ้านวนมากกว่า phospholipids ชนิด
อื่นๆ รองลงมา คือ Phosphatidylglycerol

35. ผู้ป่วยเด็กอายุ 4 ปี มีไข้ 39 C PE: milky patch at tonsil, cervical LN โต 1-2 node ขนาด 2 cm 2 ข้าง, นอนกรน หายใจ
ล้าบาก liver 2 cm BRCM, spleen 2 cm BLCM จงให้การวินิจฉัย
1) Infectious mononucleosis
2) TB lymphadenitis
3) Streptococcal pharyngitis
4) Dipthelia
5) Lymphoma

เฉลย Infectious mononucleosis


Infectious mononucleosis
มีอาการ triad ของโรค คือ ไข้ เจ็บคอมาก และต่อมน้้าเหลืองโตมากทั้งสองข้างของคอ ตรวจร่างกายพบมีต่อมทอลซิล
โต และมี exudates ลักษณะเป็นฝ้าขาว อาการที่พบร่วมด้วย คือ ม้ามโต ตับโต เหลือง มีอาการอ่อนเพลีย ปวดเมื่อย และเบื่อ
อาหารน้ามาก่อน

Streptococcal pharyngitis
มักพบในเด็กอายุเกิน 2 ปีขึ้นไป อากาส้าคัญ คือ ไข้สูง มีอาการปวดศีรษะ ปวดท้อง อาเจียน เจ็บคอ กลืนล้าบาก
ตรวจร่างกายจะพบ คอแดงทั่วๆ ไปโดยเฉพาะที่บริเวณทอนซิล อาจพบแผ่นหนองอยู่ในบริเวณคอ หรือมีจุดเลือดออกบริเวณ
เพดานอ่อนร่วมด้วย 1ใน3ของผู้ป่วยมีต่อมทอนซิลโต และมักพบต่อมน้้าเหลืองบริเวณคออักเสบด้วยเสมอ

Dipthelia
โรคคอตีบจะมีอาการหวัดและไอน้ามาก่อนประมาณ 2-3 วัน ต่อมาจะพบว่ามีแผ่น (patch) ที่บริเวณเยื่อบุในคอหอย
(pharynx) และต่อมทอนซิล ในแด็กโตจะบ่นว่าเจ็บคอ เหมือนก้างติดคอ มีต่อมน้้าเหลืองใต้คางบวม ถ้าบวมมากจะย้อยมาที่
คอเรียกว่า bull neck

Lymphoma
พบได้ในเด็กที่มาด้วยมีก้อนที่คอ มักมีขนาดใหญ่และไม่มีการอักเสบ มักพบหลายก้อน กระจายทั่วคอ มักมีB
symptoms
– Fever
– Night sweats
– Weight loss > 10% of body weight in 6 mo.

TB lymphadenitis
ในระยะแรกต่อมน้้าเหลืองจะโต แต่ไม่เจ็บ มีขนาดใหญ่เกิน 2 มม. ไม่ติดกับผิวหนัง หลายเดือนต่อมาจะมีการอักเสบ
ท้าให้ต่อมโตติดผิวหนัง ร่วมกับมีไข้ต่้าๆ น้้าหนักลด

36. ผู้ป่วยเด็กอายุ 4 ปี ปู่เป็น TB ตรวจร่างกายไม่พบ BCG scar CXR ปรกติ ท้า tuberculin test 13 mm จะท้าอย่างไรต่อไป
A) INH prophylaxis
B) start ยา TB
C) repeat tuberculin test อีกที 3 เดือนต่อมา
D) ฉีด BCG
E) AFB from gastric lavage

เฉลย A) INH prophylaxis


เนื่องจากผู้ป่วยรายนี้อายุน้อยกว่า 5 ปี และ ยังถือว่าไม่เป็น TB ไม่ว่าผล TT จะเป็นอย่างไรก็จะให้ INH 6-9
เดือน
แนวทางปฏิบัติผู้สัมผัสใกล้ชิดกับผู้ป่วยวัณโรค แนว

ผู้ป่วยวัณ โรคเสมหะบวก ให้การรักษา


ตรวจเด็กทุกคนในบ้าน
ท า TT, CXR เป็นวัณโรค ให้การรักษา

ไม่เป็นวัณโรค

อายุ < 5 ปี อายุ > 5 ปี HIV

TT +ve, -ve TT +ve TT -ve INH 9 เดือน

INH 6-9เดือน INH 6-9 เดือน observe


and F/U

Modified from สมาคมปราบวัณโรคแห่งประเทศไทย


วารสารวัณโรค โรคทรวงอกและเวชบาบัดวิกฤต 2546;24(2):87-93l
ทางการดูแลเด็กที่สัมผัสผู้ป่วยวัณโรค

1. น้าเด็กทุกคนที่< 15 ปี มาซักประวัติ ตรวจร่างกาย CXR ท้า Tuberculin skin test และ ± antiHIV
2. ให้การรักษาอย่างถูกต้อง ถ้าเป็น LTBI หรือ TB disease
3. ถ้าไม่ได้เป็น TB ให้ chemoprophylaxis ด้วย isoniazid (INH) 5-10 mk/kg/day ดังนี้
- เด็กอายุ < 5 ปี ซึ่งแข็งแรงดี โดยเฉพาะอย่างยิ่ง เด็กที่ดูดนมแม่ที่มีเสมหะเป็นบวก ถือว่ามีความเสี่ยงต่อการเกิดโรค
สูง ให้ INH อย่างน้อย 6 เดือน โดยไม่สนใจ ประวัติการฉีด BCG และ tuberculin test
เด็กอายุ > 5 ปี ซึ่งแข็งแรงดี ถ้าเคยได้ BCG และมีผล Tuberculin test ≥ 15 mm. ให้รักษาแบบ LTBI
นอกเหนือจากนี้ไม่ต้องให้ chemoprophylaxis แต่ควรติดตามอย่างใกล้ชิด
เด็กติดเชื้อ HIV ควรให้ INH นาน 9 เดือน โดยไม่สนใจประวัติ BCG หรือ ผลการท้า Tuberculin test

การแปลผล Tuberculin test positive


>10mm ในคนเด็กปกติ
>5 mm ในเด็กที่มีภูมิคุ้มกันปกติ
>15 mm ในคนที่เคยได้รับวัคซีน BCG มาก่อน
37. ผู้ป่วยเด็กอายุ 9 เดือน กินนมแม่ มีไข้ต่้าๆ ได้รับยา para, amoxy มา 3 วัน ต่อมาถ่ายเหลวเป็นน้้า PE: sunken eye ball,
dry lips, perianal redness ท่านคิดว่าเกิดจากสาเหตุใดมากที่สุด

1) Salmonella
2) Shigella
3) Rotavirus
4) Lactose deficiency
5) Antibiotic related diarrhea
เฉลย rotavirus

 Perianal erythema
o Frequent stools can cause perianal skin breakdown, particularly in young children.
o Secondary carbohydrate malabsorption often results in acidic stools.
o Secondary bile acid malabsorption can result in a severe diaper dermatitis that is often
characterized as a "burn."

อุจจาระร่วงเฉียบพลัน ส่วนใหญ่เกิดจากติดเชื้อ ข้อมูลเฉลี่ยโดยทั่วไปเกิดจากเชื้อไวรัส (rotavirus) ประมาณ 10-


50 % , Shigella 9-12%, E.coli 12%, Campylobacter jejuni 8-12% ที่เหลือเป็นอหิวาต์ถ้ามีระบาดและพวกที่เพาะเชื้อไม่ขึ้น

Rotavirus เป็นสาเหตุส่วนใหญ่ของอุจจาระร่วงในเด็กอายุต่้ากว่า 2 ปี พยาธิสรีรวิทยาที่ส้าคัญคือ rotavirus ท้าลาย


เซลล์ที่เติบโตเต็มที่และท้าหน้าที่ดูดซึมสารอาหารตรงยอดของ villi เซลล์จาก crypt เซลล์เคลื่อนที่ขึ้นมาแทนที่ตรงยอด เกิดผล
กระทบดังต่อไปนี้ 1. รบกวนเซลล์ transport ขัดขวางการดูดซึม
2. เซลล์ที่เข้ามาแทนที่เป็นเซลล์ immature น้้าย่อยที่ส้าคัญคือ แลคเตส มีจ้านวนน้อยท้าให้ย่อยน้้าตาลแลค
โตสในนมไม่ได้ เมื่อให้อาหารนมในระยะนี้ในขนาดปกติ เกิดอาการอุจจาระร่วมมากขึ้นจาก lactose malabsorption
3. เซลล์ที่เคลื่อนมาตรงยอดเป็น immature เซลล์ นอกจากดูดซึมบกพร่องแล้วยังหลั่งของเหลวเข้าสู่โพรง
ล้าไส้ด้วย
4. เกิดภยันตรายที่เยื่อบุผิวล้าไส้
ซึ่ง rotavirus ท้าให้อาการเป็นแบบ absorptive diarrhea ท้าให้การดูดซึมผิดปกติ ให้อาหารแล้วถ่ายมาก หยุดอาหาร
อาการดีขึ้น Naในอุจจาระน้อย และมักมี pH ต่้ากว่า 6 มีreducing substance > 1+ ถ้าดูดซึมน้้าตาล lactose ไม่ได้ อุจจาระ
เป็นน้้ามีฟองและกลิ่นเหม็นเปรี้ยว หรืออาจจะเรียกว่า Osmotic diarrhea ซึ่งอาจเกิดจากสาเหตุอื่นๆได้ และไวรัสเองท้าให้เกิด
ภาวการณ์หลั่งเกินด้วย (osmotic + secretory diarrhea)

Watery diarrhea หรือ non-invasive diarrhea ซึ่งมีสาเหตุจากสารพิษของแบคทีเรียและไวรัส


- สารพิษของแบคทีเรียจะ activate adenyl cyclase และหรือ guanyl cyclase ตลอดล้าไส้เล็ก และจะคงอยู่
จนกว่าเซลล์ mucosa นั้นจะหมดอายุและหลุดไป จึงท้าให้เกิดภาวะhypersecretion ของเกลือและน้้าเข้าสู่โพรงล้าไส้ เชื้อที่เป็น
สาเหตุได้แก่ Vivrio cholerae 01, O139, Enterotoxigenic E.coli (ETEC), enteropathogenic E.coli (EPEC),
Staphylococcus, Clostridium perfringens, Vibrio parahemolyticus, Bacillus cereus ลักษณะอุจจาระที่เกิดจาก
hypersecretion จากผนักล้าไส้มีภาวะเป็นด่าง pH>6, reducing substance < 1+
- เชื้อไวรัส เช่น Rotavirus, Norwalk virus เชื้อไวรัสท้าอันตรายต่อเซลล์เยื่อบุส่วน tip ของvilli ลอกตัวหลุดออก
เซลล์ที่ส่วน crypt ยังเป็นเซลล์อ่อนเคลื่อนเข้ามาคลุม ผลคือ ขาดน้้าย่อยแลคเทส ... (เหมือนตรง rotavirusข้างบน)
Mucous bloody หรือ invasive diarrhea ในกลุ่มนี้เกิดจากแบคทีเรียซึ่งเมื่อปล่อยสารพิษในช่วงที่ผ่านล้าไส้เล็ก
สารพิษนี้จะยับยั้งการดูดซึมของเกลือและน้้าแต่เมื่อผ่านมาถึงล้าไส้เล็กส่วนปลายและล้าไส้ใหญ่จะท้าให้เกิดการอักเสบเป็นแผล
โดยเฉพาะเชื้อ Salmonella ท้าให้เกิดlocal damage ท้าให้มี secretion เข้าไปในโพรงล้าไส้และรบกวนการดูดซึม ส่วนเชื้อ
shigella นอกจากจะท้าให้การดูดซึมลดลงแล้วยังปล่อย cytotoxinเกิดเซลล์ตายเป็นหย่อมๆ เป็นแผลเล็กๆ อุจจาระจึงมีมูกปน
เลือด บางครั้งปนหนอง และเกิดอาการปวดเบ่งมาก ถ้าเกิดการอักเสบตรง rectosigmoid colon กระตุ้นให้ล้าไส้ใหญ่บีบตัว และ
กล้ามเนื้อหูรูดทวารหนักเปิด เรียกว่า ปวดเบ่ง รวมเรียกกลุ่มอาการอุจจาระร่วงแบบนี้ว่า dysentery diarrhea เชื้อโรคที่เป็น
สาเหตุส้าคัญได้แก่ shigella spp. , Salmonella spp., Enteroinvasive E.coli (EIEC), campylobacter jejunia, Yersenia
enterolitica, Entamoeba histolytica ผู้ป่วยกลุ่มนี้อาจเกิดโรคแทรกซ้อน hemolytic uremic syndrome ตามมาได้ ถ้าติดเชื้อ
shigella dysenteriae 1 และ EHEC เช่น สายพันธุ์ O157:H7 เป็นต้น

38. เด็กชาย 1 ขวบ มีไอ น้้ามูก Barking cough, inspiratory stridor, expiratory rhonchi จะท้าอย่างไรนอกจากให้ IV
1) ampicillin IV
2) Cloxacillin IV
3) พ่น beta-agonist
4) พ่น adrenaline
5) ETT
เฉลย พ่น adrenaline
Croup Score
Score
0 1 2
Cough None Hoarse cry Barking
Stridor None Inspiratory Biphasic
Retraction and nasal None Flaring and suprasternal As 1 + Subcostal and
flaring retraction Intercostal retraction
Cyanosis None In room air In 40% oxygen
Inspiratory breath Normal Harsh with rhonchi delayed
sounds
Score < 4 = mild, 4-7 = moderate, >7 = severe airway obstruction

เด็กคนนี้ได้ croup score= 4 moderate


Viral Croup
�Mild ( croup score <4)
- Treat as OPD case
- ± Corticosteroid

�Moderate :Admit : อาการเปนมากขึ้น, Stridor at rest,

�Severe airway obstruction: (score > 7) -> intubations


Supportive Treatment
�Rest: รบกวนเด็กใหนอยที่สุด
�Hydration : IV fluid
�Mist therapy: no evidence suggestion
�Humidified oxygen: if SpO2 <90%
Adrenaline Nebulization
�Racemic epinephrine or L- epinephrine (1:1000)
�Dose : 0.05-0.5 mg/kg (<4yr max 2.5cc , >4yr max 5cc )
�Action in 10-30 min duration 2hr
�At ER observe at least 2-3 hr (to 8-12 hr) at ER
Corticosteroid
�Dexametasone 0.6 mg/kg IM single dose action in 6hr
�Mild case: Budesonide nebulization 2mg/dose
�Prednisolone 1mg/kg
39. เด็กอายุ 3 ปี ไอแบบ Barking cough จง management
1) adrenaline NB stat
2) Salbutamol NB stat
3) Dexamethasone IV
4) Prednisolone PO

เฉลย ไม่แน่ใจ ว่าโจทย์เหมือนข้อ 38 หรือเปล่า


โจทย์ไม่สามารถบอก score ได้
จ้า score แล้วก็ แผนภูมิ management ตามนั้น
40. เด็กอายุ 7 วัน ไข้ ซึม ร้องเวลาจับเต้านมขวา มีน้านมไหลตั้งแต่อายุ 2 เดือน มารดาบีบออกทุกวัน , PE: lethargy,
moderate jaundice, erythematous indurated mass at Rt. Breast, ควรให้ ATB ?
1. Ceftriaxone 4. Fortum + Amikacin
2. Ampicillin +Gentamicin 5. Fortum + Vancomycin
3. Cloxacillin + Gentamicin
ตอบ 2. Ampicillin +Gentamicin
common antibiotic regimen in infants with suspected sepsis is a beta-lactam antibiotic
(usually ampicillin) in combination with an aminoglycoside (usually gentamicin) or a third-
generation cephalosporin (usually cefotaxime—ceftriaxone is generally avoided in neonates due
to the theoretical risk of causing biliary stasis.) The organisms which are targeted are species
that predominate in the female genitourinary tract and to which neonates are especially
vulnerable to, specifically Group B Streptococcus, Escherichia coli, and Listeria monocytogenes

41. ผู้ป่วยเด็กชาย ได้ยา ampicillin มา 14 วัน หลังจากนั้นมีไข้ ท้องเสีย ถ่ายเหลว ท้า endoscope พบว่ามี
hyperemia Tx?
1. Metronidazole
2. Ceftriaxone

โจทย์ไม่ละเอียด แต่คิดว่าน่าจะ
เป็น Ampicillin-Associated Diarrhea.โรคท้องเสียที่เกี่ยวข้องกับการใช้ยาปฏิชีวนะ หรือที่เรียกว่า antibiotic-
associated diarrhea ยาปฏิชีวนะที่พบว่าเป็นสาเหตุ ได้แก่ clindamycin, ampicillin ,cephalosporins ,
cephalexin
การกินยาปฏิชีวนะ สมดุลของร่างกายจะเปลี่ยนไป เนื่องจากยาปฏิชีวนะ จะท้าลายแบคทีเรียที่เป็นจุล
ชีพเฉพาะถิ่นในล้าไส้ ยาปฏิชีวนะจะท้าลายเชื้อแบคทีเรียไปเป็นจ้านวนมาก และก่อให้เกิดภาวะเสียสมดุลของ
จุลชีพในล้าไส้ ส่วนใหญ่ผลที่เกิดขึ้นจะเป็นแค่อาการเพียงเล็กน้อย ผู้ป่วยมีอาการถ่ายเหลว ซึ่งอาการถ่ายเหลว
จะหายไปหลังจากหยุดยาปฏิชีวนะไม่นาน แต่ก็มีบางครั้งที่ยาปฏิชีวนะท้าลายเชื้อแบคทีเรียไปมาก จนกระทั่งท้า
ให้เชื้อแบคทีเรียที่ส้าคัญชนิดหนึ่ง มีชื่อเรียกว่า Clostridium difficile การตรวจทางห้องปฏิบัติการอาจพิจารณา
ตรวจหาเชื้อ C. difficile หรือตรวจแอนติบอดี้ต่อเชื้อ
ในกรณีที่อาการรุนแรง พิจารณาให้ยาฆ่าเชื้อแบคทีเรีย C. difficile เช่น metronidazole (Flagyl) หรือ
vancomycin (Vancocin)
42. เด็กชาย อายุ 8 ปี เมื่อก่อนแข็งแรงดี มีประวัติเป็นหวัด 1 PTA, มีจ้าเลือดตามตัวและเลือดออก
ตามไรฟัน 2 wk PTA, vital sign ปกติ, PE : Oozing per gum, petichiae & ecchymosis ที่
trunk & extremities, no lymphadenopathy, no hepatosplenomegaly, Dx?
1. Hemophilia 4. ITP
2. Acute leukemia 5. APDE
3. DHF

ตอบ 4. ITP = Idiopathic thrombocytopenic purpura


I. Idiopathic thrombocytopenic purpura แบ่งออกเป็น
- Primary autoimmune immune thrombocytopenia (ITP)
Acute childhood ITP
chronic or adult ITP
- Secondary Autoimmune thrombocytopenia
Acute post viral ITP(acute childhood ITP)
- พบในเด็กอายุน้อย - เกิดรวดเร็วภายใน2สัปดาห์
- เกิดตามหลังการติดเชื้อไวรัส หรือการ - หายได้เองภาย6เดือน
ฉีดวัคซีน
Chronic ITP
- พบในวัยเจริญพันธุ์
- พบในผู้หญิงมากกว่าผู้ชาย
- อาการค่อยเป็นค่อยไป
- มักเป็นโรคเรื้อรัง
- โอกาสหายเองน้อย
อาการทางคลินิก chronic ITP
- Skin bleeding –petechiae,ecchymoses
- Mucosal bleeding-bleeding per gum,epistaxis,hematuria
- Intra cerebral hemorrhage
- Anemia related to degree of bleeding
- No hepatosplenomegaly if splenomegaly was detected should be considered
hypersplenism or lymphoproliforative disorders
Criteria for diagnosis ITP
- Isolated thrombocytopenia
- Normal or increase megakaryocyte in bone marrow
- No splenomegaly
- Autoantibody to platelets
- No other cause of thrombocytopenia
Treatment
1. General treatment
- Bleeding precaution avoid trauma,Intramuscular injection,antiplatelets medication
- Local treatment;nasal packing,oral contraceptive drug
- Blood transfusion in active bleeding,iron replacement therapy,platelets transfusion
in life thretening situations
2. Specific treatment
Corticosteroid actions:
- Inhibit autoantibody bind to platelets
- decrease phargocytosis by RE cell,
- decrease autoantibody production,
- increase vascular suppor
Indication for splenectomy
- Steroid relaspe =steroid dependent
- Stroid partial and non response
 Diagnosis and treatment over 6 weeks with platelets less than 10,000
 Treatment for 3 months and platelets less than 30,000 with or without bleeding
- Complication of streroid
- Ememegency condition
- Difficulties in follow up
Pediatric

43. Female Infant 5 hr APGAR 8,9 ที่ 1,5 min BW 4,200 gm. คลอดมามีอาการชักเกร็ง ตัวยาว 52 cm. Hc 35
cm. The most appropriate investigation for Dx this Patient’s Disease is?
1. Plasma Glucose
2. Urine Ketone
3. Serum Ammonia
เฉลย
1. Plasma Glucose
Clinical manifestation of hypoglycemia : ไม่จาเพาะเจาะจงในทารกแรกเกิด คือ อาจมี tremor, jitteriness,
apnea, cyanosis, pallor, hypotonia, irritability, seizures, coma
Management :
***10% D/W or 25% D/W IV push 0.25-0.5 g/kg and continuous infusion rate 4-6 mg/kg/min
***Reference : pocket ped
เด็กทารกที่เสี่ยงต่อภาวะ Hypoglycemia (Blood glucose < 40 mg/dL) ดังนี้
 Infant of diabetics mother (IDM) ทารกกลุ่มนี้เสี่ยงต่อภาวะน้าตาลในเลือดต่าได้ง่าย มักเกิดเร็วระหว่าง
อายุ 1-6 ชั่วโมง และอาจเป็นอยู่หลายวัน เนื่องจากทารกมีปริมาณ Insulin ในเลือดและตับอ่อนจานวน
มากตั้งแต่อยู่ในครรภ์
 Preterm พบได้ประมาณร้อยละ 15 มักมีอาการในชั่วโมงแรก ๆ หลังเกิด ถ้าป่วยจะมีโอกาสเกิดมากขึ้น
 Small for gestational age (SGA) จะมี glycogen สะสมน้อย จึงมีอาการน้าตาลต่าเร็ว ภายใน 1-2 ชั่วโมง
หลังคลอด ถึงอายุ 2-3 วัน
 การมี perinatal stress ทาให้ร่างกายของทารกที่ขาดออกซิเจนมี anaerobic metabolism ทาให้น้าตาลใน
ร่างกายถูกใช้มากกว่าปกติ
 สาเหตุอื่น ๆ เช่น hypothermia, sepsis, polycytemia, adrenal insufficiency เป็นต้น
*** Reference : ชีทเรียน ascessment and care of newborn อ.พญ. สอางค์ หน้า 15

---------------------------------------------------------------------------------------------------------------------By เอ๋
44.(ชุด A) Case preterm 32 wk. BW 1,500 g ดูหายใจเร็วๆ ถามว่าเกิดจากภาวะอะไร
1. Respiratory distress syndrome
เฉลย
นึกถึงภาวะ Respiratory distress syndrome or Hyaline membrane disease มากที่สุดเนื่องจากมี Risk factor
คือ
1. GA < 35 wks
2. cesarean section โดยยังไม่มี labor pain
3. มารดาเป็น DM class A,B, C
4. acute perinatal asphyxia
5. antepartum hemorrhage : abruption of placenta or placenta previa
6. แฝดคนที่สอง
7. มีประวัติโรคนี้ในครรภ์ที่แล้ว
โดย RDS มีสาเหตุจากขาด surfactant มักรุนแรงมากในทารกเพศชาย GA < 28 wks
Clinical manifestation :
 หายใจลาบาก (หายใจเร็ว, หอบ, หน้าอกบุ๋ม, เสียง grunting, cyanosis) อาจพบได้ตั้งแต่แรกเกิดหรือ
ภายใน 3-4 ชั่วโมงหลังเกิด หากเกิดหลัง 6 ชั่วโมงจะไม่นึกถึงภาวะนี้เลย
 ความรุนแรงน้อยถึงเสียชีวิตได้ ภายใน 2-3 วัน
 ความรุนแรงจะเพิ่มขึ้นเรื่อย ๆ ในระยะ 24 ชั่วโมงแรก และคงที่ 24-72 ชั่วโมง หลังจากนั้นจะค่อย ๆดีขึ้น
หากไม่มีอาการแทรกซ้อน
 Preterm มักมีภาวะ RDS รุนแรง และพบภาวะแทรกซ้อนได้บ่อย คือ CNS hemorrhage, PDA, air leak,
infection
Film x-ray : hypoaeration, airbronchograms, atelectasis, diffuse reticulogranular infiltration หากรุนแรง
อาจเห็นเป็น ground glass appearance , bilateral opacity (white-out lung)
*** Reference : ชีทเรียน Respiratory distress of the newborn อ.พญ. อารยา
----------------------------------------------------------------------------------------------------------------------By เอ๋
ไม่มีหมายเลขข้อ
เด็กชายอายุ 4 ปี จมน้้าในสระว่ายน้้า หลังจาก resuscitation แล้วหายใจได้เอง แต่ยังไม่ค่อยรู้สึกตัว น้าส่ง ER
P.E. : T = 36.5 , PR = 140 , RR = 55 ,BP = 90/60 , drowsy , mild cyanosis , diffuse fine crepitation and
expiratory wheezing , capillary refill 4 sec , O2 sat 85% On O2 mask with bag 5 LPM ไอมี frothy pink จะ
ท้าอะไร
A. IV dopamine D. On O2 mask with bag 10 LPM
B. Nebulized adrenaline E. Positive pressure ventilation
C. Nebulized bronchodilator
เฉลย
อยากได้ทั้ง Positive pressure ventilation และ Nebulized bronchodilator ไม่แน่ใจว่าอันไหนอ่ะ
Near – drowning หมายถึง การจมน้าแล้วยังไม่เสียชีวิต เมื่อถูกนาขึ้นมาจากน้า
เหตุผล  คิดว่าเด็กมี non - cardiogenic pulmonary edema เนื่องจาก persistant pulmonary
hypoxemia รายนี้มี O2 sat 85% และจาก hypoxic pulmonary vasoconstriction ยิ่งทาให้ pulmonary edema เลว
ลงโดยผู้ป่วยน่าจะเป็น Category B ซึ่งให้ใช้ยาขยายหลอดลมเมื่อมีปัญหาหลอดลมตีบ แต่หากดูจากกลไกการ
เกิด อยากให้ใส่ tube (คิดเองอ่ะ) เลยเลือก Positive pressure ventilation
หมายเหตุ ถ้าขี้เกียจอ่าน อ่านแค่ พยาธิสรีรวิทยา กับ อาการของ Catergory B และ การรักษาของ
Catergory B ก็ได้
พยาธิสรีรวิทยา
เมื่อเด็กจมน้าจะมีผลต่อร่างกาย อย่างน้อย 3 อย่าง คือ hypoxia, aspiration , hypothermia โดยขณะ
จมน้าเด็กจะตกใจกลัว พยายามดิ้นรนขึ้นสู่ผิวน้า ร่างกายจะป้องกันตัวเองโดยเกิด
 laryngospasm ในกรณีที่เกิด laryngospasm รุนแรงจะทาให้น้าเข้าปอดไม่ได้ เมื่อเสียชีวิตจะไม่มีน้าใน
ปอด เรียก “dry drowning” พบได้ 10%
 ในรายที่ไม่เสียชีวิต เด็กจะกลืนน้าเข้าไปจานวนมาก มี FB aspiration and contaminated water ทาให้
เกิด airway resistance สูงขึ้น และ bacterial pneumonitis เกิด hypoxia และหมดสติกล้ามเนื้อต่างๆ จะ
อ่อนแรง มีการคลายตัวของกล้ามเนื้อกล่องเสียงทาให้น้าเข้าไปในปอดได้ เรียก “wet drowning ” พบ
ได้ประมาณ 90%
 มี Surfactant dysfunction เกิด atelectasis , decreased FRC , lung compliance ลดต่าลง และ
ventilation - perfusion mismatch ตามมา
 เกิด hypoxic pulmonary vasoconstriction
 เกิด non - cardiogenic pulmonary edema ( ARDS ) ซึ่งเป็นสาเหตุของ persistant pulmonary
hypoxemia
อาการและอาการแสดง
1. Category A ( awake )
เด็กจะไม่หมดสติ Glasgow coma score = 15 การทางานของระบบประสาทปกติ
อาจมีอุณหภูมิกายต่าเล็กน้อย CXR อาจพบการเปลี่ยนแปลงในปอดบ้าง เจาะ blood gas มี mild metabolic
acidosis และ mild hypoxemia
2. Category B ( blunted )
เด็กจะซึม ( impaired cortical function ) แต่การหายใจ และการตอบสนองต่อความเจ็บปวดปกติ (
normal brainstem function ) มี cyanosis, tachypnea , dyspnea , เสียงผิดปกติในปอด ถ้าเจาะ blood
gas จะผิดปกติ โดยพบ hypoxemia , hypercabia , metabolic acidosis ส่วน CXR ผิดปกติ และมี
hypothermia
3. Category C ( comatose )
เด็กจะหมดสติ ( cortical function ผิดปกติ ) มีการตอบสนองต่อความเจ็บปวด และการหายใจ
ผิดปกติ ( brainstem dysfunction )
การรักษาที่โรงพยาบาล
1. Ventilation ด้วย 100% O2
2. ให้ IV fluid ด้วย 5% D/NSS/3 ( ส่ง CBC, electrolyte, urine, CXR, blood gas )
3. Mornitor vital signs
4. รักษาตาม category ABC
Category A ให้ความอบอุ่นถ้ามี hypothermia ,ให้ sodium bicarbonate เข้าเส้นเลือดดาช้าๆ ถ้ามี
metabolic acidosis, ให้ยาปฏิชีวนะ ถ้ามี pulmonary infiltration หลังเฝ้าระวังอาการประมาณ 24 ชั่วโมง
ถ้าไม่มีความผิดปกติจึงให้กลับบ้านได้
Category B ต้องดูแลอย่างใกล้ชิดให้ lasix 0.5 – 1 mg/kg ถ้ามี pulmonary edema ให้ sodium
bicarbonate 1 – 2 ml/kg เข้าเส้นเลือดดาเพื่อรักษา metabolic acidosis ซึ่งมักเกิดเสมอ , ให้ยาปฏิชีวนะถ้าจมน้า
คลอง , ให้ใช้ยาขยายหลอดลมเมื่อมีปัญหาหลอดลมตีบ เด็กควรอยู่โรงพยาบาลนานหลายวัน ติดตามตรวจ
ร่างกาย และระบบประสาททุกวัน และดู CXR เป็นระยะๆ
Category C จาเป็นต้องใส่ endotrachial tube ทุกรายโดยใช้ PEEP 5 – 10 cmH2O เพื่อให้ได้ PaO2 >
100 mmHg , PH 7.35 – 7.5, PaCo2 20 – 30 mmHg ต้องให้ sodium bicarbonate ยาขยายหลอดลม ยาขับ
ปัสสาวะ ยาปฏิชีวนะและยาลดความดันในสมองทุกราย ถ้าเกิด hypotension ต้องให้ isotonic solution เช่น NSS
หรือ RLS bolus dose และให้ซ้าอีกถ้าจาเป็น ถ้าไม่ดีขึ้นอาจให้ dopamine 2 – 20 มค./กก/นาที การรักษาอื่น
โดยการให้ความอบอุ่น ให้ diazepam ถ้ามีอาการชัก และเฝ้าติดตามภาวะแทรกซ้อน เช่น ปอดอักเสบ , ARDS ,
cerebral edema
----------------------------------------------------------------------------------------------------------------------By เอ๋

45. เด็ก 11 ปี , 1 เดือนก่อน ไม่สบาย มีจ้าเลือดตามตัว, CBC : Plt ต่้า อย่างเดียว ,Dx?
1. ITP
เฉลย
1. ITP แหละ
โรคต่าง ๆ ที่นึกถึงได้มีดังนี้
1. ITP
ITP เป็นสาเหตุสาคัญที่ทาให้เกล็ดเลือดต่าในเด็กมากกว่าร้อยละ 80 พบบ่อยในเด็ก 3-5 ปี โดยมักเกิด
ตามหลังโรคติดเชื้อโดยเฉพาะโรคติดเชื้อไวรัส บางคนเรียกภาวะนี้ว่า Postinfection thrombocytopenia มักเกิด
ตามหลังการติดเชื้อประมาณ 2 สัปดาห์ อาการมักมี petechiae, ecchymosis, เลือดกาเดาไหล เลือดออกตาม
ไรฟัน เลือดออกในทางเดินอาหาร ปัสสาวะเป็นเลือด เลือดออกในสมองได้ ตรวจร่างกายปกติ ไม่มีตับหรือ
ม้ามโต ส่วนมากเกล็ดเลือดน้อยกว่า 20,000 cell/cu.mm. หายได้เอง ใน 3 เดือน
2. Acquired platelet dysfunction with eosinophilia (APDE)
APDE จะมีจ้าเลือดออกตามตัวและแขนขาเป็น ๆ หาย ๆ เป็นสัปดาห์หรือเป็นเดือน ลักษณะจ้าเลือดมักเป็น
ecchymosis อาจเกิดขึ้นเองหรือตามหลังกระทบกระแทกเพียงเบา ๆ จ้าเลือดคงอยู่ประมาณ 1-2 สัปดาห์ แล้ว
ค่อย ๆ จางหายไป สบายดีไม่มีไข้ มักไม่ซีด ตับและม้ามไม่โต ตรวจร่างกายปกติเป็น 6 เดือนถึง 1 ปี หายได้
เอง มี Eosinophil 8-69% ลักษณะ platelet ตัวใหญ่ติดสีจาง  ไม่เหมือนในรายนี้
3. von Willebrand’s disease (vWD)
พบได้ทุกเพศ ทุกวัย มักมาด้วยเลือดกาเดาไหลบ่อย ๆ มีจ้าเลือดขึ้นง่าย มีประจาเดือนมากผิดปกติ ความ
รุนแรงมีตั้งแต่น้อยถึงมาก บางครั้งมีอาการคล้าย hemophilia A ได้ จานวนเกล็ดเลือดปกติ  ไม่เหมือนใน
รายนี้
4. Hemophilia
เป็นโรคเลือดออกในข้อ ถ่ายทอดทางพันธุกรรม พบเมื่อเริ่มหัดเดิน เกิดขึ้นเองหรือหลังกระทบกระแทก มักเกิด
ที่ข้อเข่า ข้อศอก ข้อเท้า ปวดมาก มักอยู่ท่างอเข่า ข้อบวมอุ่น ผิวหนังตึงอุ่น กดเจ็บ  ไม่เหมือนในรายนี้
*** Reference : ชีท bleeding disorder อ.พญ. สมพร
----------------------------------------------------------------------------------------------------------------------By เอ๋

46. เด็กชาย อายุ 15 ปี ไปสถานกักกันมา 2 ปี ตรวจร่างกาย พบ Upper/Lower motor weak grade IV/II
DTR no impaired pain and touch, glottic stroking pattern
1. Beri Beri
2. GBS
เฉลย
คิดว่าน่าจะ ตอบ Beri-Beri
 Guillain-Barre’ syndrome
 มีอาการอ่อนแรงแบบ ascending symmetrical muscle weakness โดยเริ่มจาก lower extremities
 มี gait disturbance,
 deep tendon reflex ไม่มีหรือลดลง ,
 sensory changes : pain , proprioception , touch
 มี cranial nerve involvement + bulbar paralysis มี trancient urinary incontinence or retention
 CSF profile : normal glucose, elevated protein, lymphocyte ปกติ
เชื่อว่า Guillain-Barre’ syndrome เกิดจากภูมิคุ้มกันของร่างกายผิดปกติ จะตามหลังการติดเชื้อ 3-4
สัปดาห์ จึงเริ่มมีอาการอ่อนแรง มักเกิดตามหลังการติดเชื้อทางเดินหายใจ ทางเดินอาหารหรือเชื้ออื่น ๆ
เช่น Campylobator, mycoplasma, CMV และไวรัสอื่น ๆ เช่น EBV, chicken pox, influenza, coxsackie
virus, echovirus, คางทูม หัด
***Reference : pocket ped และ กุมาร รามา เล่ม 2
 Beri-Beri
จะมีอาการอ่อนแรงของขามากกว่าแขนได้เช่นกัน ร่วมกับมีการสูญเสียความรู้สึกที่ปลายมือปลายเท้า (glove
and stocking pattern) บางครั้งหากสัมผัสที่ขาหรือปลายเท้าจะเจ็บมาก (hyperesthesia) วินิจฉัยจากประวัติการ
กินอาหาร เด็กมีอาการอ่อนแรง มี DTR ลดลงได้
ความคิดเห็นส่วนตัว รู้สึกว่า คนออกข้อนี้ เน้นมาให้ว่าอยู่สถานกักกัน 2 ปี เลยคิดว่าน่าจะตอบ Beri-Beri
นะ แต่ตอนสอบจริงคนออกข้อสอบน่าจะให้รายละเอียดของการตรวจ sensory มามากกว่าอ่ะ ก็อย่าลืมดู
sensory ด้วยล่ะกัน
----------------------------------------------------------------------------------------------------------------------By เอ๋
47. เด็กชาย 7 ปี มีอาการคันตาเป็นๆหายๆ มา 1 ปี ตรวจตา พบ Giant papillae ลักษณะเป็น Cobble stone การ
วินิจฉัยที่เป็นไปได้มากที่สุด
1. Trachoma
2. Inclusion Conjunctivitis
3. Hay fever Conjunctivitis
4. Vernal karatoconjunctivitis
5. Epidemic Conjunctivitis
ตอบ 4. Vernal karatoconjunctivitis
ข้อนี้เหมือนเป็นเนื้อหาของเด็ก แต่จริงๆ แล้วเป็นเนื้อหาของ eye ล้วนๆ
Key word ที่ข้อนี้ให้มาคือ อาการคันตานาน 1 ปี Giant papillae และ cobble stone ซึ่งทั้ง 3 อย่างนี้เข้าได้
กับ Vernal karatoconjunctivitis เป็นโรคในกลุ่ม Allergic conjunctivitis ที่เกิดจาก immediate hypersensitivity ไม่
พบสาเหตุของการแพ้แต่สัมพันธ์กับการแพ้ละอองหญ้า มักมีประวัติแพ้ในครอบครัว พบ 3 แบบ คือ Palpebral
type, Limbal type และ Mixed type คนนี้เป็น Palpebral type (รายละเอียดอ่านเพิ่มใน ตาราจักษุวิทยา พิมพ์ครั้งที่
2 หน้า 117)
ส่วนตัวเลือกข้ออื่น
1. Trachoma (รายละเอียดอ่านเพิ่มใน ตาราจักษุวิทยา พิมพ์ครั้งที่ 2 หน้า 114)
2. Inclusion Conjunctivitis (รายละเอียดอ่านเพิ่มใน ตาราจักษุวิทยา พิมพ์ครั้งที่ 2 หน้า 113)
3. Hay fever Conjunctivitis (รายละเอียดอ่านเพิ่มใน ตาราจักษุวิทยา พิมพ์ครั้งที่ 2 หน้า 116-117)
5. Epidemic Conjunctivitis (รายละเอียดอ่านเพิ่มใน ตาราจักษุวิทยา พิมพ์ครั้งที่ 2 หน้า 115)
ปล. ข้อสอบแบบจาได้ก็ตอบได้ จาไม่ได้ก็ตอบไม่ได้ ไม่รู้จะเฉลยยังไงดี
------------------------------------------------------------------------------------------------------------------By bever

48. เด็กมาด้วยอาการ Drooling, miosis, dysphoresis, profuse sweating ถามว่าเกิดจากสารใด


1. Ethanol 4. Ephedrine
2. Organophosphate 5. Cocaine
3. Opioid
ตอบ 2. Organophosphate
จากโจทย์ บอกยากว่าเป็นอันไหนแน่ แต่เท่าที่ดูเหมือนจะใกล้เคียงกับของ Organophosphate มากที่สุด
- ก่อนอื่นมาอธิบาย Term กันก่อน
Drooling => น้าลายไหล
Miosis => รูม่านตาดาหดเล็กกว่าปกติ
Dysphoresis น่าจะเป็น diaphoresis มากกว่า => การขับเหงื่อ, การหลั่งเหงื่ออย่างมากมาย
Profuse sweating => เหงื่อออกในปริมาณมาก
- ฤทธิ์ของสารแต่ละชนิด
Ethanol : CNS depression (ระดับของ CNS depression ขึ้นกับปริมาณ ethanol ในกระแสเลือด)
Organophosphate : Miosis, cramps, salivation, urination, diaphoresis, myofibrillation, sialorrhea,
bronchorrhea, defecation, garlic odor
Opioid : nausea and vomiting, drowsiness, itching, dry mouth, miosis, constipation dose-related
respiratory depression, confusion, hallucinations, delirium, urticaria, hypothermia, bradycardia/tachycardia,
orthostatic hypotension, dizziness, headache, urinary retention, ureteric or biliary spasm, muscle rigidity,
myoclonus, and flushing
Ephedrine (Amphetamine) : Track marks, “heat stroke”, toxic psychosis, flushing, diaphoresis,
hyperthermia, hypertension, dilated pupils (reactive), active bowel sound, anxiety, hyperactivity, dysrhythmias
Cocaine : perforated nasal septum, dilated pupils (reactive), psychosis, hyperthermia, diaphoresis, skin
tracks, seizures, agitation, tremor, tactile hallucination, anxiety
------------------------------------------------------------------------------------------------------------------By bever

49. ทารกอายุ 6 ขั่วโมง แรกคลอดไม่ดูดนม หายใจหอบเหนื่อย ฟัง breath sound ได้ข้างขวาข้างเดียว เสียงหัวใจ
อยู่ทางขวา X-ray พบ heart ที่ right chest ท้องยุบ จง Dx.
1. Situs inversus
2. TE fistula
3. Diaphragmatic hernia
ตอบ 3. Diaphragmatic hernia
พิจารณาตัวเลือกแต่ละข้อ
1. Situs inversus คือ ภาวะที่มีการกลับด้านของอวัยวะภายในทั้งหมด ซึ่งมักไม่พบว่ามีอาการผิดปกติใดๆ
จึงนึกถึงน้อยใน case นี้
2. TE fistula (Tracheoesophageal fistula) เป็นความผิดปกติแต่กาเนิดที่มีการเชื่อมต่อกันของ trachea และ
esophagus ส่วนใหญ่มักมี esophageal atresia ร่วมด้วย จึงมักมีประวัติ polyhydramnios ในระหว่าง
ตั้งครรภ์ เด็กมักมีอาการไอที่เกี่ยวกับการกลืนอาหาร บางคนมีอาการสาลักเข้าหลอดลม เขียว ท้องอืดโต
3. Diaphragmatic hernia คือ มีความผิดปกติของ diaphragm ที่มีรูทาให้ bowel แทรกผ่านรูนี้เข้ามาในช่อง
อก มักพบที่ข้างซ้ายมากกว่าข้างขวา พวกนี้มักมีอาการรุนแรงแต่แรกเกิด มี dyspnea, cyanosis, decrease
breath sound ข้างหนึ่ง, อาจฟังได้ bowel sound บริเวณทรวงอก, ได้ยินเสียงหัวใจด้านตรงข้ามกับที่
decrease breath sound, ท้องมักแฟบกว่าปกติ
------------------------------------------------------------------------------------------------------------------By bever

50. เด็กหญิงอายุ 10 ปีเป็นโรคผนังกั้นหัวใจรั่วแต่ก้าเนิด เคยเข้ารับการรักษาด้วยหัวใจล้มเหลวหลายๆครั้งตอน


เล็กๆ เมื่ออายุ 3 ปี อาการเหนื่อยน้อยลง ตอนนี้มาด้วยเหนื่อยมากขึ้นมา 2 เดือน นอนราบได้PE: central
cyanosis, clubbing finger, Loud P2, diastolic rumbing murmur 3/6 at Lt upper parasternal border lung:
clear ข้อใด น่าจะเป็นสาเหตุของการเหนื่อยในเด็กรายนี้
1. Pulmonary embolism 4. Eisenmenger complex
2. Aortic regurgitation 5. Congestive heart failure
3. Hypoxic spell
จากโจทย์บอกว่าเด็กเป็นผนังหัวใจรั่วแต่กาเนิด แสดงว่าเด็กน่าจะเป็น VSD หรือ ASD ซึ่งเป็นลักษณะ
Left-right shunt lesion รักษา heart failure จนดีขึ้น
จนเด็กอายุ 10 ปี จึงหอบเหนื่อย มีอาการเขียวเป็นลักษณะของ Right-left shunt lesion , loud P2 (แสดงถึง
มี pulmonary hypertension)
ดังนั้นอาการเหนื่อยในเด็กคนนี้เกิดจาก Eisenmenger complex
Eisenmenger complex : A defect of the interventricular septum with pulmonary hypertension and a consequent
right-to-left shunt through the defect.
อาการ : เด็กจะเขียวจาก Right-left shunt เนื่องจากมีภาวะpulmonary hypertension ซึ่งตรวจร่างกายก็จะ
พบ loud P2
Hypoxic spell :พบบ่อยในโรค tetralogy of Fallot
: จะมีอาการหอบเหนื่อยและเขียวทันที สิ่งกระตุ้น ได้แก่ การร้องให้ การดูดนม การขับถ่ายหรือ
การไอหลายๆ ครั้งติดๆ กัน
: ลักษณะเฉพาะ คือ squatting เป็นการนั่งยอง ขณะที่มีการเหนื่อยหอบ
Congestive heart failure : จะมีอาการเหนื่อยหอบ หายใจแรง ดูดนมได้ช้า เหงื่อออกมาก เลี้ยงไม่โต พบในกลุ่ม
โรคหัวใจที่มี increase pulmonary blood flow
Pulmonary embolism(PE) : เกิดจากการทีม่ ีสิ่งใดสิ่งหนึ่งไหลไปตามกระแสเลือด แล้วไปอุดกั้นหลอดเลือดส่วน
ปลายที่ปอดส่วนใหญ่เกิดจาก thromboemboli จากเส้นเลือดดาบริเวณขา
Symptoms : sudden-onset dyspnea , tachypnea, chest pain , cough, hemoptysis. More severe cases can
include signs such as pleural rub, cyanosis, collapse, and circulatory instability. About 15% of all cases of
sudden death
------------------------------------------------------------------------------------------------------By Gift^zee
^
51. เด็กชายอายุ 12 ปี มาด้วยอาการเป็นลม ล้มหมดสติไป 1 นาที ตื่นมาปกติดี เกิดขณะเข้าแถวตอนเช้า ตรวจ
ร่างกายพบว่าปกติ, neurologic exam normal ต้องท้าอะไรต่อหรือไม่
1. EKG
2. Blood sugar
3. 24hr Holter monitor
4. EEG
5. ไม่ต้องทาอะไรเพิ่มเติม
Causes of Syncope and Dizziness

1. Cardiovascular syncope
1. Neurocardiogenic syncope (most commontype of syncope in childhood and adolescence)
2. Cardiac syncope
1. Congenitaland acquired heart disease
2. Hypercyanotic episodes
3. Arrhythmias in structurally normalheart
4. Arrhythmias in structurally abnormalheart
3. Vascular syncope : 1.Orthostaticsyncope 2. Cerebrovascular syncope 3.Carotid sinus syncope
2. Noncardiovascular syncope : 1.Breath-holding 2. Hyperventilation 3.Migraine 4.Metabolic( Hypoxia including
anemia , Hypoglycemia) 5. Psychologic

Neurocardiogenic Syncope (Common Faint)


 Most common type of syncope in childhood and adolescence.
 Often a response to anxiety, fear, pain , or other emotional stress.
 Dizziness, nausea, sweating, and abdominal discomfort may precede syncope. Interruption of cerebral
perfusionfor 2–3 secs produces dizziness, but lying down during presyncopal phase may prevent
syncopal episode.
 Most episodes are associated with upright position, and child becomes limp and falls to ground. Loss of
consciousness usually lasts <1 min.
ข้อนี้ขึ้นอยู่กับการตัดสินใจของแต่ละคน แต่ขอตอบว่าท้า 1. EKG เพราะท้าได้ง่าย ไม่ invasive และยังสามารถ
R/O cardiac syncope ได้
---------------------------------------------------------------------------------------------------By Gift^zee
^
52. เด็กอายุ 7 วัน น้้าหนักแรกคลอด 2,800 gm กินนมแล้วอาเจียนเป็นน้้าสีเหลือง หลังจากนั้นซึม ดูดนมน้อยลง
น้้าหนักลดเหลือ 2,400 gm จง Dx. เบื้องต้น
1. Duodenal atresia 4. Sepsis
2. Pyloric stenosis 5. Hirchsprung disease
3. Inborn error metabolism
Pyloric stenosis
 present in the first weeks to months of life with progressively worsening vomiting
 อาเจียน ไม่มีลักษณะของ bile ปน
 เด็กบางคนอาจมาด้วย poor feeding and weight loss
Duodenal atresia
 เกิดจาก duodenum (the first part of the small bowel) ไม่เจริญ
 อาเจียนหลังกินนม มีลักษณะของ bile ปน (สีเขียวปน)
 loss of appetite, failure to gain weight
Hirchsprung disease
มีอาการและอาการแสดงแตกต่างตามช่วงอายุของผู้ป่วย
 อาการที่สาคัญที่สุดของทารกแรกคลอดกลุ่มนี้คือ ความผิดปกติของการถ่ายขี้เทา ทารกไม่
ถ่ายขี้เทาในหนึ่งถึงสองวันแรก อาจมีอาการมากถึงอาเจียนและอาจอาเจียนมีน้าดีปน
 อาการและอาการแสดงในเด็กโต คือ เด็กกลุ่มนี้อาจมีปัญหาการขับถ่ายเพียงเล็กน้อย
หรือไม่มีเลยในช่วงทารก ต่อมามีปัญหาท้องอืดได้บ้าง เริ่มกินได้น้อยลง น้าหนักขึ้นน้อย ท้องหลามอืดแน่นท้อง
ง่าย อาการท้องผูกจะค่อยๆ เป็นมากขึ้นเรื่อยๆ เมื่อถ่ายออกจะมีลักษณะเหมือนท้องเสียเป็นเนื้อเหลวปนก้อน
เล็กๆสีเข้ม คล้า หรือคล้ายสีโคลน มีกลิ่นเหม็นมาก จะเป็นเช่นนี้ เป็นๆหายๆ
ทารกที่มีอาการอาเจียนเป็นน้าดีแสดงว่ามีการอุดตันของทางเดินอาหารที่ระดับต่ากว่ารูเปิดของท่อน้าดี
(sphincter of Oddi) และหากอาเจียนเป็นอาหารหรือนมโดยไม่มีน้าดีปนแสดงว่าอาจเป็น gastric outlet
obstruction
ในข้อนี้เด็กอาเจียนเป็นสีเหลือง ไม่มีลักษณะ bile ดังนั้นตอบ 2. Pyloric stenosis

อีก 2 ข้อนี้คงไม่ตอบกันน่ะ Inborn error of metabolism : เกิดจากความผิดปกติทางชีวเคมีซึ่งเกิดจากการขาด


หรือปฏิบัติหน้าที่บกพร่องของเอนไซม์ทาให้กระบวนการเมตาบอลิซึมผิดปรกติ
Sepsis : เด่นๆ เด็กต้องมีไข้ look sick
-------------------------------------------------------------------------------------------------------- Gift^zee
53. ผู้ป่วยเด็กหญิงอายุ 9 ปี มีอาการหงุดหงิดง่าย การเรียนตกต่้าลงPE: Thyroid diffuse^enlarged,
exophthalmos, smooth skin. PR120 FT4 เพิ่ม TSH ลด Antityroglobulin 1:1600 จะรักษาอย่างไร
1. Iodized oil 3. Lugol solution
2. Iodized salt 4. Methimazole
Ans. 4
ข้อนี้เด็กเป็น Grave’s Disease (hyperthyriod) ยาที่ใช้รักษาก็ต้องเป็นกลุ่มยารักษา hyperthyroid
ซึ่งจากตัวเลือกมี methimazole กับ lugol solution แต่ lugol solution เอาไว้ใช้ในกรณี thyroid strom ที่ผู้ป่วยมี
conscious change ดังนั้นข้อนี้ ผู้ป่วยแค่มี clinical hyperthyroid เฉยๆ จึงใช้ Methimazole รักษา
----------------------------------------------------------------------------------------------------------------------By เก๋
54. เด็ก 6 yr ได้ vaccine ครบ ตอน 5 yr ถูกแมวข่วนขนาด 2 cm ต้องได้รับ vaccine อะไรบ้าง
1. Rabies vaccine
2. TT
3. Rabies vaccine + Tetanus toxoid
4. Rabies vaccine และ Rabies Ig
5. TT + Rabies vaccine และ Rabies Ig
Ans. 1
ข้อนี้เด็กได้ TT vaccine ครบแล้วตอน 5 ขวบ และโดนแมวข่วนเฉยๆ ไม่ได้มีแผลเลือดออก จึง
ให้แค่ Rabies vaccine ก็พอ ส่วน Rabies Ig ให้ในกรณีที่โดนกัด เป็นแผลมีเลือดออก
----------------------------------------------------------------------------------------------------------------------By เก๋
55. เด็ก 7 ปี Dx ว่าเป็น Chickenpox ควรแนะน้าแม่ว่าจะให้เด็กไป รร ได้เมื่อไร
1. หลังจากไข้ลด
2. เมื่อแผลตกสะเก็ดหมด
3. เมื่อตุ่มน้าใสเม็ดสุดท้ายลอก
4. หลังกิน Acyclovir 48 hr
5. ไปได้เลย แต่ต้องสวมเสื้อให้มิดชิด
Ans. 2
Chicken pox ติดต่อทางสัมผัสและทางเดินหายใจ แพร่เชื้อได้ตั้งแต่ 1 วันก่อนผื่นขึ้นจนถึง ผื่น
ขึ้นแล้ว 6-7 วัน หรือจนผื่นทั้งหมดตกสะเก็ด(ไม่พบเชื้อไวรัสในสะเก็ดที่แห้ง) ดังนั้นก็รอให้พ้นระยะที่
แพร่เชื้อได้ไปก่อนแล้วค่อยให้ไป รร.
----------------------------------------------------------------------------------------------------------------------By เก๋
56. เด็ก 9 ปี มีอาการปวดหัว ปวดท้องก่อนไป รร ไม่อยากไป รร กลัวมีเหตุการณ์ลักพาตัว แม่บอกว่ามีฝันร้าย
ตอนกลางคืน ไม่ไป รร 3 wk ถาม Dx
1) Generalized anxiety
2) Separation anxiety
3) Simple phobia
เฉลย Separation anxiety
Separation anxiety เป็นความกังวลที่เกิดจากการต้องห่างจากบ้านหรือบุคคลที่เด็กติด เช่น พ่อแม่, พบ
ได้ประมาณ 4% ในเด็ก และพบในผู้ใหญ่ได้ โดยมักมีอาการ ดังนี้
- มักรู้สึกเครียดเมื่อต้องพรากจากสิ่งที่ผูกพัน
- มีความกังวลอยู่ตลอดที่จะต้องพรากจากสิ่งที่ผูกพัน
- มีความกังวลมากว่าจะเกิดเหตุการณ์ที่ทาให้ตนต้องพรากจากสิ่งที่ผูกพัน
- มีความกลัวมากเกินไปที่ต้องอยู่คนเดียว หรืออยู่โดยไม่มีสิ่งที่ผูกพัน
- มักฝันร้ายถึงเหตุการณ์ที่ต้องแยกจากสิ่งที่ผูกพันบ่อยๆ
Simple phobia หรือ specific phobia
A.มักมีความกลัวอย่างรุนแรงและคงอยู่ตลอด รวมทั้งมีมากเกินควรหรือไม่มีเหตุผล เมื่อทราบหรือคาดว่าจะต้อง
เผชิญกับบางสิ่งหรือบางสถานการณ์ (เช่น ขึ้นเครื่องบิน อยู่ที่สูง พบสัตว์ ถูกฉีดยา เห็นเลือด)
B. การเผชิญกับสิ่งที่กลัวนั้นก่อให้เกิดความวิตกกังวลขึ้นมาอย่างทันทีแทบทุกราย ซึ่งอาจเป็น Panic Attack
C. บุคคลนั้นตระหนักดีว่าความกลัวนั้นมีมากเกินควรหรือไม่มีเหตุผล
D. มีการหลีกเลี่ยงสถานการณ์ที่ทาให้กลัว
E. การหลีกเลี่ยง การกังวลว่าจะต้องเผชิญกับสิ่งที่กลัว หรือความทุกข์ใจ มีผลกระทบต่อการดารงชีวิตประจาวัน
F. ในผู้ที่อายุต่ากว่า 18 ปี ต้องมีอาการนานอย่างน้อย 6 เดือน
GAD ก็ตาม criteria GAD คือต้องเป็นอย่างน้อย 6 เดือน และมีอาการ 3 ใน 6 ข้อต่อไปนี้ คือ
กระสับกระส่าย, อ่อนเพลียง่าย, ตั้งสมาธิยาก หรือใจลอย, หงุดหงิด, กล้ามเนื้อตึงตัว, มีปัญหาการนอน
---------------------------------------------------------------------------------------------------------------------By บี
57. เด็ก หายใจหอบเหนื่อย มีกลิ่นเหม็น hyperpnea, +ve urine ketone 4+ Dx อะไร
1) DKA 3) meningitis
2) Sepsis
เฉลย DKA ถ้าต้องการละเอียดมากๆ แนะนาชี้ต pediatric emergencyปี 5 ของ อ. ณรงค์ นะจ๊ะ
Diagnostic criteria* DKA
Blood glucose : >250 mg per dL (13.9 mmol per L)
pH : <7.3
Serum bicarbonate : <15 mEq per L
Urinary ketone : >=3+ †
Serum ketone : positive at 1:2 dilutions†
Serum osmolality : variable
มักจะเกิดกับผู้ป่วย DM type I
อาการ : Often insidious , Fatigue and malaise, Nausea/vomiting, Abdominal pain, Polydipsia, Polyuria,
Polyphagia, Weight loss, Fever
PE : Tachypnea or hyperventilation (Kussmaul respirations), Acetone odor of the breath
insulin def  hyperglycemia + ร่างกายขาดพลังงาน  gluconeogenesis + glycogenolysis + lypolysis 
ketosis
hyperglycemia  osmotic diuresis  hypovolumia tissue hypoperfusion + lactic acidosis
Hypernatremic dehydration (corrected serum sodium > 150 mmol/L)
ภาวะ hyperglycemia จะทาให้ค่า serum sodium (Na) ต่ากว่าความเป็นจริง

---------------------------------------------------------------------------------------------------------------------By บี
58. ผู้ป่วยเด็กชายอายุ 9 mo มาด้วย otitis media เคยเป็นโรคติดเชื้อเป็นๆหายๆ ตั้งแต่ 3 mo ก่อน พวก
pneumonia, S.pneumo, Hib, ถามว่าน่ามี defect ที่ใด
1) B cell
2) T cell
3) Phagocytic defect
4) Complement
เฉลย น่าจะตอบ B cell นะเพื่อนๆ
ถ้าอยากรู้ละเอียดๆให้ไปเปิดชี้ต immune ปี 3 มั้ง พอดีชี้ตเค้าอยู่วงษ์พาณิชย์ เอาคร่าวๆละกันนะ ขอโทษด้วย
Primary immunodeficiency
B cell deficiency (Humoral ID)
ความผิดปกติของ B cells สงผลใหมีระดับอิมมูโนโกลบูลินผิดปกติ ทาใหติดเชื้อในกลุม
pyogenic bacteria เชน พวก pneumococcus, Haemophilus influenza และ streptococcus รวมทั้งไวรัสและพยาธิ
บางชนิด เชน polio และ Giardia ความบกพรองทางดานแอนติบอดีนี้อาจเกิดจาก ตัว B cells เอง หรือ T cells ด
วยก็ได ความผิดปกติทางดานแอนติบอดีนี้สงผลกระทบถึงระบบ ภูมิตานทานตามธรรมชาติดวยเนื่องจาก
แอนติบอดีทาหนาที่เปน opsonin ทาให phagocytes จับกินแบคทีเรียไดดีขึ้น ตลอดจนการกระตุนระบบคอมพลีเม
นท11-3
อย่างในข้อนี้ น่าจะเป็นเฉพาะ B cell เนื่องจาก B cell มันฆ่าเชื้อโดยสร้าง antibody = IgA, IgM, IgG
ประมาณนั้น ซึ่งช่วง 6 เดือนแรกยังพอได้อานิสงส์จากแม่อยู่ และในข้อนี้เด็กมันเริ่มมีอาการตอนอายุ 6 เดือน
ซึ่งเป็นช่วงที่ immunoglobulin ที่ได้รับจากแม่หมดไปพอดี นอกจากนี้ เชื้อที่ infection ก็เข้าได้ ... เค้าเลยซื้อ B
cell นะคาฟฟฟฟ ^^
ส่วน T cell def. มักจะเป็น fungal หรือ viral infection เด่น และอาจมีผลต่อ B cell ได้บ้าง เนื่องจาก
T cell มีส่วนช่วยให้ B cell เปลี่ยนเป็น plasma cell
Complement defect เมื่อมีการตรึงคอมพลีเมนทจะท าใหเซลลเชื้อโรคแตก หรือกระตุนให phagocytes มาจับ
กินเชื้อโรคไดดีขึ้น การขาดจะสงผลใหรางกายไมสามารถกาจัดสิ่งแปลกปลอมได
อยางมีประสิทธิภาพ ถาขาด components ตนๆ (C1,C4 และ C2) สงผลใหการกาจัด immune complex เกิดได
ไมดี ทาใหมี immune complex เหลือมาก และอาจท าใหเกิดโรคอื่นๆ ตามมาได เชน SLE like disease , ถาขาด
C3 มักจะมีการติดเชื้อบอย, ถาขาด C5-C9 จะทาใหมีการติดเชื้อซ้า
บอยๆ โดยเฉพาะเชื้อ Neisseria spp
---------------------------------------------------------------------------------------------------------------------By บี
59. เด็ก 4 yr ไข้สูง 1 d มีผื่นแดงตามตัว วันนี้ซึมลงจึงมา รพ. PE: BP 90/60, RR 40, PR 120, stiff neck +ve,
Purpura??? ถาม Dx
1) Salmonella 4) S. pneumonia
2) N. meningitidis 5) H. influenza
3) Listeria monocytogenes
เฉลย N. meningitides โดยสรุปอาการของแต่ละ meningitis ดังนี้ ค่ะ
- Salmonella : สัมพันธ์กับ อุจจาระร่วงในเด็กอายุ < 1ปี
- N. meningitides : intracellular gram negative diplococci, ติดต่อทางน้าลาย หรือ respiratory secretion , ส่วน
อาการมักมาด้วยมี respiratory tract infection นามาก่อน 2-3 วัน,อาจมีผื่นขึ้น และหากรุนแรงอาจพบเป็น purpura
หรือ ecchymosis ได้
- Listeria monocytogenes : ส่วนใหญ่เป็น food borne transmition, มักพบในหญิงตั้งครรภ์, เด็กแรกเกิด,
immunocompromised host, ผู้ป่วยมักมาด้วยอาการของ diarrhea or gastroenteritis, CNS มักเป็น encephalitis
บริเวณ brain stem
S. pneumonia มักมี respiratory tract infection นามาก่อน
H. influenza
---------------------------------------------------------------------------------------------------------------------By บี
60. เด็ก 10 mo มีไข้ ไอ น้้ามุก มา 3 mo วันนี้ไอมาก หายใจหอบเหนื่อย มี inspiratory stridor, chest drawning
ภายหลังให้ epinephrine มีหอบน้อยลง epinephrine มีผลอย่างไร
1. ขยายหลอดลม
2. ขับเสมหะ
3. ขับน้าที่ค้างในปอด
4. ลดบวมเยื่อบุทางเดินหายใจ
เฉลย
1. ขยายหลอดลม
Epinephrine
อยู่ในกลุ่ม selective adrenergic receptor agonist เป็น mixed alpha and beta agonist โดยออกฤทธิ์กระตุ้น
alpha 1= alpha 2 , beta 1= beta 2 ดังนี้
alpha 1  vasoconstriction
alpha 2  ลดการหลั่ง insulin
beta 1  เพิ่ม HR และการบีบตัวของหัวใจ, เพิ่ม SBP แต่ DBP ลดลงหรือไม่เปลี่ยนแปลง
beta 2  vasodilation ,เพิ่มการหลั่ง insulin
bronchial smooth muscle  relaxation  bronchodilation
ผลโดยรวมจากการกระตุ้น alpha 2 และ beta 2 จะทาให้ลด TPR ลงเล็กน้อย ทาให้ DBP ลดลงหรือไม่
เปลี่ยนแปลง
ผลการเพิ่มการหลั่ง insulin จะเด่นชัดกว่าการลดการหลั่ง ผลโดยรวมคือ เพิ่ม glucose และ triglyceride
Side effects
palpitations, tachycardia, arrhythmia, anxiety, headache, tremor, hypertension, and acute pulmonary
edema
Ref จาก sheet pharmaco เรื่อง ยาที่ออกฤทธิ์ในระบบประสาท sympathetic ปี 3 จ้า
------------------------------------------------------------------------------------------------------------------- By แฟง
61. มารดาเป็น DM เด็กเกิดมามีอาการสั่น มี sign ของ hypoglycemia DTX 30 Hct 63 ถาม Management
1. Early feeding
2. Bolus 10%D/W 2ml/kg
3. Plasma exchange
เฉลย
2. Bolus 10%D/W 2ml/kg
จากโจทย์เด็กมีภาวะ Hypoglycemia เนื่องจากมี sign ของ hypoglycemia DTX 30 (hypoglycemia ใน
new born เอาน้อยกว่า 40)
Hct 63 ยังไม่ polycythemia นะ เอา > 65
สาเหตุของ Hypoglycemia
1. Transient hypoglycemia : preterm, SGA, IDM =infant DM of mother
2. Persistent hypoglycemia : hyperinsulinism, การขาด hormone, glycogen storage disease, ความ
ผิดปกติของ fatty acid oxidation, การได้รับยาบางชนิด
การรักษา
ตาม Guide line ของโงพยาบาลพุทธเรานะเพื่อนๆเป็นแบบนี้แหละก็เลยตอบข้อ 2 นะ
Patient at risk
(IDM =infant DM of mother, LBW, SGA, LGA, preterm, asphyxia(APGAR 5 min ≤ 7))

Check DTX at 1-2 hr

DTX >45 mg/dl DTX <45 mg/dl

Early feeding asymptomatic symptomatic


(apnea, jitteriness, tremors)

Check DTX DTX 30-45 mg/dl DTX <30 mg/dl → 10%DW 2-4 ml/kg
before neck feeding IV then GIR 4-8 mg/kg/min

>45 mg/dl <45 mg/dl Early feeding check DTX 1 hr later

Routine care check DTX 1 hr later >45 mg/dl <45 mg/dl

>45 mg/dl <45 mg/dl adjust GIR + continue same IV


10%DW2ml/kg and check DTX
Feed+IV check DTX 2 hr later
GIR 4-8 mg/kg/min before neck feeding

<45 mg/dl >45 mg/dl

Check DTX q 1 hr Routine care


Until euglycemia

อาการในทารก
1. ตัวสั่น (jitteriness หรือ tremors หรือ irritable)
2. หยุดหายใจ หายใจเร็ว หรือ เขียว หรือ ซึม
3. ชัก
4. ไม่ดูดนม , hypothermia
เพิ่มเติมกันลืม
LBW หมายถึง ทารกที่คลอดมีชีวิตและน้าหนักแรกคลอดน้อยกว่า 2500 กรัม
VLBW หมายถึง ทารกที่คลอดมีชีวิตและมีน้าหนักแรกคลอดน้อยกว่า 1500 กรัม
Preterm infant หมายถึง ทารกที่คลอดก่อนกาหนด คือ คลอดก่อนอายุครรภ์ครบ 37 สัปดาห์
Post term infant หมายถึง ทารกที่คลอดเกินกาหนด คือ คลอดหลังจากอายุครรภ์ 42 สัปดาห์
Small for gestational age (SGA): คือ เด็กที่เมื่อคลอดมีน้าหนักตัวแรกคลอดต่ากว่า (SGA) 10
percentile ของเด็กที่มีอายุครรภ์เท่าเทียมกัน
Appropriate for gestational age (AGA) คือเด็กที่คลอดมีน้าหนักเหมาะสมกับอายุครรภ์คือระหว่าง
10-90 percentile
Large for gestational age (LGA) คือ เด็กที่เมื่อคลอดมีน้าหนักเกินกว่า 90 percentile ของทารกที่มีอายุ
ครรภ์เท่าเทียมกัน
------------------------------------------------------------------------------------------------------------------- By แฟง

62. เด็กชายอายุ 4 mo ซึมลง WBC 18000 (N 80%), Hct 6.9%, Dx อะไร


1. Subdural hematoma
เฉลย
ข้อนี้มันมี choice เดียวอ่ะเพื่อนๆ
Subdural hematoma
สาเหตุ
1.Head trauma
2.Coagulopathy or medical anticoagulation (eg, warfarin [Coumadin], heparin, hemophilia, liver
disease, thrombocytopenia)
3.Nontraumatic : cerebral aneurysm, arteriovenous malformation, or tumor (meningioma or dural
metastases)
4.Intracranial hypotension (eg, after lumbar puncture, lumbar CSF leak,
lumboperitoneal shunt)
5.Child abuse or shaken baby syndrome (in the pediatric age group)
6.Spontaneous
อาการ
1.Altered level of consciousness
2.A dilated or nonreactive pupil ipsilateral to the hematoma
3.Hemiparesis contralateral to the hematoma.
อาการร่วมอื่นๆ : ทารกอาจดูซีด ซึม ชัก อาเจียน กระหม่อมหน้าโป่ง
CT scan: crescent-shaped hyperdense , ไม่ข้าม midline
Treatment
1. Subdural tapping
2. Surgery
------------------------------------------------------------------------------------------------------------------- By แฟง
63. เด็กอายุ 4-5 ขวบ มีน้ามูกไหลกลิ่นเหม็น รักษาด้วย Amoxy 2 วัน อาการไม่ดีขึ้น มาพบแพทย์ตรวจพบหนอง
ในช่องจมูกมาก ท้าอะไรต่อไป
a. ส่ง X-ray PNS
b. ตรวจด้วย nasal speculum ซ้าอีกครั้ง
c. Amoxicillin/Clavulinic acid
d. ใช้ Amoxycillin เดิมต่อไป
e. Consult โสต ศอ นอสิกแพทย์
เฉลย
b. ตรวจด้วย nasal speculum ซ้้าอีกครั้ง
ในผู้ป่วยรายนี้นึกถึง foreign body เด็กจะพบบ่อย
อาการ
มีน้ามูกมาก มีหนองในจมูก มักเป็นข้างเดียว
การรักษา
1. นาสิ่งแปลกปลอมออก ห้ามใช้เครื่องมือดันสิ่งแปลกปลอมตกไปทางด้านหลังจมูกเนื่องจากเด็กอาจ
สาลักสิ่งแปลกปลอมลงไปในหลอดลมได้
2. ให้ antibiotic โดยเฉพาะในรายที่มีสิ่งแปลกปลอมติดอยู่นาน
------------------------------------------------------------------------------------------------------------------- By แฟง
64. เด็กในข้อใดมีพัฒนาช้า
A.อายุ 10 เดือน ตั้งไข่ได้ …………… (12 เดือน)
B.อายุ 12 เดือน เดินได้ ……………….. (15เดือน)
C. อายุ 2 ปี เดินก้าวขึ้นบันไดสลับข้างได้……. (3ปี)
D.อายุ 3 ปี พูดค้าโดดได้ วลี 10-20 ค้า …….. (18เดือน)
E.อายุ 5 ปี วาดรูปสามเหลี่ยมได้…………….. (5 ปี)
เฉลย ตอบ D
พัฒนาการ (อายุ) Gross motor Fine motor Languages Social
1 เดือน เริ่มชันคอ กามือแน่น ทาเสียงในคอ มองจ้องหน้า
2เดือน ชันคอได้ 45องศา มือกาหลวมๆ หันหาเสียงคุ้นเคย สบตา ยิ้มตอบ
4เดือน ท่าคว่า, ชันคอ 90 องศา มองตามทคว้าของ ส่งเสียงอ้อแอ้ ยิ้มตอบ ทาท่าดีใจ
ใกล้ตัว
6เดือน คว่าและหงายได้เอง หยิบของมือเดียว หันหาเสียงเรียก เล่น รู้จักแปลกหน้า
เปลี่ยนมือ น้าลาย
9เดือน นั่งได้มั่นคง คลาน เกาะ ใช้นิ้วหยิบของได้ เปล่งเสียงเลียน
ยืน พยัญชนะแต่ไม่มี
ความหมาย
12เดือน เกาะยืน ยืนเองได้ชั่วครู่ หยิบของใส่ถ้วย เรียกพ่อแม่ พูดคา ตบมือ เลียนท่าทาง
หรือกล่อง โดด ร่วมมือเวลาแต่งตัว
15เดือน เดินเองได้ วางของซ้อนกัน 2 พูดเป็นคาโดดที่มี ตบมือ เลียนท่าทาง
ชิ้น ความหมาย ร่วมมือเวลาแต่งตัว
18เดือน เดินคล่อง วิ่ง จูงมือเดียว วางของซ้อนกัน 3 ชี้รูปภาพตามคาบอก ถือถ้วยน้าดื่มเอง
ขึ้นบันได ชิ้น พูดเป็นคำโดดหลำย
คำ
2ปี เดินขึ้นบันได กระโดด2 ตั้งของได้ 6 ชิ้น พูด 2-3 คาต่อกัน เลียนแบบผู้ใหญ่
เท้า เตะบอล
3ปี ขึ้นบันไดสลับขำ วาดวงกลมได้ เล่าเรื่องได้ 50% รู้เพศตัวเองคุมการ
ถ่ายอุจจาระ
4ปี กระโดดเท้าเดียว วาดสี่เหลี่ยมได้ ร้องเพลง คุมการถ่าย
ปัสสาวะเวลา
กลางวัน
5ปี กระโดดสลับเท้า วำดสำมเหลี่ยม นับเลขได้ถึง 20 เล่นอย่างมีกติกา
Reference: ปัญหาโรคเด็กที่พบบ่อย หน้า 39
-------------------------------------------------------------------------------------------------------------- By ส้ม natty
65. เด็กหญิงเป็น UTI รักษาด้วย Ampicillin ครบ 14 วัน จากนั้นปวดท้อง ไข้ขึ้น ตัวเหลือง อุจจาระมี WBC 1-5,
RBC >100 ส่องกล้องดู เจอ patch ได้ยาอะไร
A.Cotrimoxazole
B.Norfloxacin
C.Metronidazole

เฉลย C
Antibiotic-associated diarrhea (ADD) เกิดในผู้ป่วยที่ได้ antibiotic เป็นเวลานาน เกิดการทาลาย normal flora
ใน intestine ทาให้ bacteria ซึ่งอยู่ในลาไส้ คือ Clostridium difficile เกิดการ growth และ ปล่อย toxin ออกมา
ทาลาย cell wall ของลาไส้เป็นสาเหตุให้เกิด inflammation เกิด diarrhea และเกิดอาการอื่นๆตามมา
อาการขึ้นอยู่กับความรุนแรง:

 ผู้ป่วยที่เป็น carriers จะพบ bacteria ในอุจจาระ แต่ไม่มีอาการของการติดเชื้อ และสามารถแพร่เชื้อสู่


ผู้อื่นได้

 อาการที่พบบ่อยที่สุด คือ watery diarrhea และ อาจมี mild abdominal cramping.

 ใน severe cases ล้าไส้จะเกิดการอักเสบ (colitis) หรือ เกิดเป็น patches ท้าให้เกิด bleed หรือ เกิด pus
(pseudomembranous colitis). อาการจะมี watery diarrhea (up to 10 to 15 times per day), blood or
pus in the stool, dehydration, abdominal tenderness and cramping, a low-grade fever, nausea,
loss of appetite, and weight loss.
 Life-threatening complications จะเกิดน้อย อาการ จะพบ abdominal distension, severe lower abdominal
pain, fever and profuse diarrhea. In rare cases, the bowels can rupture, potentially leading to a
bodywide infection (sepsis), organ failure, or even death.

TREATMENT — The most important step in treatment of C. difficile is to stop the antibiotic that allowed
the infection to develop. If an antibiotic is necessary to treat an underlying infection, the healthcare provider
may choose an antibiotic that is less likely to allow further growth of C. difficile, when possible.
Antibiotic treatment — An oral antibiotic such as metronidazole (Flagyl®) or vancomycin (Vancocin®) is
usually recommended to treat people who are infected with C. difficile. It is important to take each dose of the
antibiotic on time and to finish the entire course of treatment (usually up to 10 to 14 days).

-------------------------------------------------------------------------------------------------------------- By ส้ม natty


66.เด็กผู้ชาย มีอาการเหมือน cushing syndrome จะตรวจอะไรผิดปกติ
A. Hypocalcemia
B. Hyperkalemia
C.Increased cortisol level
D.Advance bone age
E.Increased rennin plasma activity
เฉลย C
Cushing’s syndrome (CS) คือภาวะที่มี cortisol ในเลือดสูงจากสาเหตุต่าง ๆ ในเด็ก จะ rare พบ 10-15 คน ต่อ
ประชากร 1 ล้านคน
สาเหตุ
- pre-pubertal children : adrenal tumors (e.g. adenoma, carcinoma, or bilateral
hyperplasia) are a more common
- older children : pituitary adenomas are a more common
อาการและอาการแสดง
-increase in rate of weight gain accompanied by a decrease in growth rate
-rounded face, reddened cheeks, acne, headache, excess hair growth, purplish-
pink stretch marks, darkened skin around neck and armpit areas, easy
bruising, development of pubic hair at a younger age than usual, irregular or
absent menstrual periods, and high blood pressure
Diagnosis
- elevated cortisol levels in children is a 24 hour urine collection for urinary free cortisol (UFC)
determination
- Salivary cortisol levels (ภาวะปกติ ระดับ cortisol ตอน midnight จะต่ามากและจะสูงในตอนเช้า
แต่ใน cushing syndrome ระดับ cortisol ตอน midnight จะสูง)
- low-dose dexamethasone
reference
http://www.csrf.net/page/cushings_syndrome_in_children.php
http://www.md.chula.ac.th/rcat/htdocs/previous/200448745.pdf
-------------------------------------------------------------------------------------------------------------- By ส้ม natty
67.เด็กหญิงอายุ 4 yr ซีด 2 wk ก่อนหน้านี้ปกติ PE: Pallor, petichiae บริเวณแขนขา CBC Hb 7, Hct 22, WBC
3200(N 35, E 3, L 62), Platelet 26000, MCV 90 จะส่ง Investigation อะไรเพื่อการวินิจฉัย
A. Serum ferritin
B. Bleeding time
C. Reticulocyte count
D. Screening coagulogram
E. Bone marrow aspiration
Ans E. bone marrow aspiration
เนื่องจากว่ามันต่าทุก Series เป็น Pancytopenia ซึ่งน่าจะเกิดจากการทางานที่ผิดปกติของไขกระดูก ซึ่ง สงสัย
ALL ดังนั้นเลยต้องทา Bone marrow aspiration ช่วยในการวินิจฉัย
-------------------------------------------------------------------------------------------------------------- By ตาลสวย
68. เด็กหญิง 9 เดือน กินนมแล้วส้าลักบ่อย เป็นปอดบวม 3 ครั้ง BW 7 kg PR 120 lung: wheezing both lung,
CXR: perihilar infiltration, PPD 4 mm หลังได้ brochodilator แล้วดีขึ้น Dx
A. Retian FB D. GERD
B. Diaphragmatic hernia E. Asthma
C. TB
Ans D. GERD
Retain foreign body มักจะเด็ก 9 เดือนมันก็เริ่มหยิบของเล็กๆได้แล้วแต่ว่ามีประวัติว่าน้ามูกไหลข้างเดียวมีกลิ่น
เหม็นๆ
Diaphragmatic hernia อันนี้เด็กจะต้องแย่ต้องแต่เกิด และยิ่งทาPPVแย่ลงให้สงสัยภาวะนี้และถ้าเป็นโรคนี้เมื่อ
ทา CXR แล้วมันต้องเห็นว่ามี bowel ขึ้นมาอยู่ที่ช่องอก 90% จะอยู่ที่ posterior left
Presentation มันจะเหมือน respiratory distress (from pulmonary hypoplasia and pulmonary hypertension)
TB ( Tuberculin test negative คือน้อยกว่า 5 mm) คิดถึงน้อยเพราะไม่มHี x expose และประวัติก็ไม่เหมือนอย่าง
แรง
Asthma จากประวัติที่มีเรื่องของ wheezing response ต่อ bronchodilator ก็ทาให้นึกึงได้แต่การ Dx Asthma ในเด็ก
อายุน้อยกว่า 5 ปี จะต้องใช้ Criteriaในการ Dx
Major Minor
หอบมากกว่า 3 คั้งต่อปี Eosinophil >= 4%
พ่อหรือแม่เป็น Asthma Wheezing apart from cold
เด็เป็น Atrophic dermatitis
GERD คิดถึงภาวะนี้มากที่สุดเพราะว่า จากประวัติที่มีกินแล้วสาลักบ่อย
อาการทางคลินิกของผู้ป่วยคือมีขย้อน หายใจเร็ว หอบ ฟังปอดมีหวีด ซึงภาวะนี้ก็เป็นสาเหตุของปัญหาที่เกิดกับ
ระบบหายใจ เป็นสาเหตุของการไอเรื้อรังในเด็กแรกเกิดจน 18 เดือน ทาให้เด็กการเจริญเติบโตต่ากว่าเกณฑ์
-------------------------------------------------------------------------------------------------------------- By ตาลสวย
69. เด็กมีไข้ มีผื่นตามตัว(Purpura fulminan) Stiff neck : positive ถามว่าเกิดจากเชื้ออะไร
- N.meningitidis
N.meningitidis เป้นเชื้อที่ทาให้เกิดโรค ไข้กาฬหลังแอ่น มักมาบแพทย์ด้วยเรื่องอาการไข้และมีผื่น ไข้ก็จะเป็นไข้
สูงมีการดาเนินโรคที่เร็ว และทาให้เกิดภาวะช็อกหรือเยื่อหุ้มสมองอักเสบร่วมด้วย ผื่นท่พบเป็นลักษณะของ
pustule, bullae หรือ hemorrhagic lesion with central necrosisโดยลักษณะที่พบถ้าพบเป้น stellate-shape
purpura หรือที่เรียกว่า purpura fulminan เป็นลักษณะบ่งชี้ที่ว่าน่าจะเป็นโรคนี้ Gram stain เห็นเป็น Gram
negative diplococci bean shape
-------------------------------------------------------------------------------------------------------------- By ตาลสวย
70. เด็ก asthma มีอาการเฉพาะเวลาออกก้าลังกายและตอนกลางคืน 2-3 ครั้งต่อเดือน ถามว่าต้องให้ยาอะไร
A. Oral salbutamol C. Inhale steroid
B. Inhale sulbutamol D. Inhale ipratropium + Salbutamol
ในเด็กรายนี้อาการมีอาการเฉพาะเวลาออกกาลังกายและตอนกลางคืน 2-3 ครั้งต่อเดือน อาการเข้า Step
3: Moderate persistent อาการ มีทุกวัน, noc.>1/wk , Exacerbatation.ขณะมีactivity การให้ยาจึงให้เป็น medium
dose inhaled glucocorticosteroid
ตอบ C. Inhale steroid

Step การให้ยาasthma ในเด็ก<5 ปี Gina 2004


Level of severity Daily controller medication Other Tx
Step 1:mild intermittent none
อาการ<1/wk,nocturnal<2/mo
FEV1>80%
Step 2: mild persistent - Low dose inhaled - Sustain-release theophyline
อาการ<1/wk, <1/d, noc.>2/mo glucocorticosteroid (ICS) - Leukotriene modifier
FEV1>80%
Step 3: Moderate persistent -medium dose ICS - medium dose ICS+ Sustain-
- อาการ มีทุกวัน, noc.>1/wk release theophyline
- Exacerbat.ขณะมีactivity หรือ - medium dose ICS+long acting
นอนหลับ beta-2 พ่น
- ใช้Beta-2 พ่นทุกวัน - medium dose ICS+ Leukotriene
- FEV1 60-80% modifier
Level of asthma control Gina 2007
characteristic Controller Partly controlled uncontrolled
(ต้องมีทุกข้อนะ)
Daytime symptoms None >2/wk =>3ข้อของ Partly controlled ที่มี
(=<2/wk) อาการในแต่ละ wk
Limitation of activity none Any
Nocturnal symptoms/ awakening none Any
Need for reliever/rescue Tx None >2/wk
(<2/wk)
Lung Fx Normal <80%
(PEF or FEV1) >80%
ไม่ทาในเด็ก<5ปี
Exacerbation none =>1/yr. One in any wk.
------------------------------------------------------------------------------------------------------------ By นิกแนก
71. เด็กอายุ 6 เดือนกระตุกตอนเช้าเป็นมา 2 สัปดาห์ โดยเริ่มจากผงกศีรษะก่อนแล้วท้าท่ามือโอบกอด จงวินิจฉัย
ตอบ จากลักษณะการชักมีผงกศีรษะก่อนแล้วทาท่ามือโอบกอด ลักษณะคล้ายกับการผวาเป็นชุดุดๆ
เข้าได้กับ Infantile spasm มักพบในทารกและเด็กเล็ก
กลุ่มอาการ west syndrome ประกอบด้วย
1. ชักแบบ Infantile spasm มี 3 แบบคือ งอตัว (flexion), เหยียดตัว(extension),mixed flexion and extension
2. พัฒนาการช้า
3. คลื่นไฟฟ้าสมองแบบ hypsarrthmia
------------------------------------------------------------------------------------------------------------ By นิกแนก
72. เด็กชาย อายุ 10 ปี MCA มาที่ ER ด้วยอาการปวดบวม และขยับข้อศอกขวาไม่ได้
PE: swelling at Rt elbow, normal triangle landmark of Rt elbow, notmal neurovascular จง Dx
A. Elbow dislocation D. Displaced lateral condylar fracture
B. Displaced olecranon fracture E. Displaced medial supracondylar fracture
C. Displaced supracondylar fracture
ตอบ Displaced lateral condylar fracture เป็น Fx ของ distal radius จากอายุ 10ปีร่วมกับการเจริญของ
Epiphyseal ที่จะเห็นเป็น cartilage ตามช่วงอายุดังนี้
CIRTOE=2,4,5,8,10,12 T = trochlea 8
C = capitulum 2 O= olecranon 10
I = internal (medial )epicondyle 4 E = external (lateral) epicondyle 12
R = radial head 5
การทาให้ไม่เห็น Displaced lateral condylar fracture ในเด็กอายุ 10 ปี จึงพบ Displaced lateral condylar fracture
------------------------------------------------------------------------------------------------------------ By นิกแนก
Surgery

2. ผู้ป่วยชายไทยมีอาการปวดท้องที่บริเวณ epigastrium ทันทีขณะที่พึ่งดื่มเหล้าไปได้ 2 แก้ว มาพบแพทย์ที่


โรงพยาบาล แพทย์เวรได้ทาการตรวจร่างกายพบ Abdomen : generalized guarding, absent bowel sound การ
ส่งตรวจในข้อใดเหมาะสมที่สุดในผู้ป่วยรายนี้
a. Plain abdomen
b. CT abdomen
c. Ultrasound
d. Abdomen series
e. MRI
ตอบ c.Ultrasound
จากอาการของผู้ป่วย น่าจะเป็น acute pancreatitis ซึ่งจะมีอาการ acute epigastric pain ร้าวหรือทะลุไป
หลัง พบตามหลังอาหารมื้อใหญ่ มีอาการคลื่นไส้อาเจียน มีสาเหตุหลักจากแอลกอฮอล์ ตรวจร่างกายพบว่ามีไข้
tachycardia, Abdominal tenderness, muscular guarding, distension, hypoactive bowel sound จาก gastric และ
transverse colonic ileus, บางคนมีอาการ jaundice . dyspnea จาก irritation of the diaphragm, pleural effusionIn
ในรายที่เป็นรุนแรงจะมี hemodynamic instability, hematemesis, melena, pale นอกจากนี้อาจตรวจพบ Cullen
sign, Grey-Turner, Erythematous skin, Purtscher retinopathy
การตรวจทางรังสีที่มีประโยชน์คือ
Abdominal ultrasonography
เป็น most useful initial test ในการหาสาเหตุ แต่ไม่สามารถบอกความรุนแรงได้ โดยจะตรวจพบ
edematous, swollen pancreas, peripancreatic fluid collection, หรือ pseudocyst และเป็น technique of choice
สาหรับตรวจหา gallstones แต่คนที่มี ileus มาก จะมองเห็นภาพได้ไม่ชัดเจน

Abdominal radiography
ไม่ค่อย specific อาจพบ dilate ของลาไส้ใกล้กับ pancreas โดยเฉพาะ duodenum, jejunum และ
transverse colon (รวมเรียกว่า Sentinel loop) อาจพบ colon cutoff sign ซึ่งจะมีเงาของ gas ใน ascending และ
transverse colon ตอนต้น แล้วหายไปในตอนกลาง ซึ่งอธิบายว่าเกิดจาก colon spasm ใกล้บริเวณตับอ่อนที่อักเสบ
ใน CXR อาจพบ pleural effusion ข้างซ้าย

Abdominal CT scanning
ใช้ในการช่วยวินิจฉัยในกรณีที่การวินิจฉัยยังไม่ชัดเจน ควรทาในผู้ป่วยทุกรายที่อาการไม่ดีขึ้นใน2-3 วัน
หรือเมื่อสงสัยว่าจะมีภาวะแทรกซ้อน
3. ผู้ป่วยหญิงอายุ 22 ปี คลาได้ก้อนที่เต้านมขวา 3 เดือนก่อน ตรวจร่างกายพบก้อน ขนาด 2 cm oval shape,
firm, smooth and movable การวินิจฉัยในข้อใดเป็นไปได้มากที่สุด
a. Fat necrosis
b. Fibroadenoma
c. Fibrocystic disease
d. Intraductal carcinoma
e. Intraductal papilloma

ตอบ b.Fibroadenoma
Fibroadenoma
พบบ่อยในสตรีวัย 20-30 ปี บางครั้งเกิดขึ้นพร้อมกันหลายก้อน ก้อนมีผิวเรียบแข็งและขอบเขตชัดเจน
คล้ายเม็ดถั่วลิสง ไม่เจ็บ โตช้า กลิ้งไปมาได้
Fat necrosis
เกิดหลังจากเต้านมถูกกระแทกจนช้า ทาให้เกิด chronic inflammation มีทั้ง fibrosis และ foreign body
giant cell ทาให้เป็นก้อนขรุขระใต้ผิวหนัง อาจติดกับผิวหนังเป็นบางส่วนหรือติดกับ fascia ทาให้คล้ายมะเร็งมาก
Fibrocystic disease
เป็นสาเหตุของอาการของเต้านมทีพบได้บ่อยที่สุด เนื้อของเต้านมมีการเปลี่ยนแปลงแบบ hyperplasia
และ regression ตามระดับฮอร์โมนเพศที่แปรไปในรอบเดือน เกิดขึ้นซ้าซ้อนเป็นเวลานานจนเนื้อเต้านมเปลี่ยนไป
จากเดิม โดยมี glandular hyperplasia (adenosis), connective tissue hyperplasia (fibrosis), micro and macrocyst,
ductal papillomatosis และ lymphocytic infiltration
อาการพบได้บ่อยตั้งแต่อายุ 30 ปีขึ้นไปจนหมดประจาเดือน หลังจากนั้นจะทุเลาหรือหายไปเอง ผู้ป่วยมี
อาการหลายแบบ
1. มีก้อนในเต้านม เป็นเพราะ
1.1 เนื้อเต้านมขรุขระและนูนขึ้นโดยเฉพาะ upper outer quadrant นูนชัดมากก่อนมีประจาเดือน
และยุบลงเมื่อประจาเดือนมาแล้ว อาจเจ็บตึงร่วมด้วย ไม่พบก้อนชัดเจน แต่เต้านมขรุขระ
แข็งขึ้นทั่วไป มักเป็นทั้ง 2 ข้าง
1.2 เป็น cyst บางก้อนโตเร็วและขนาดแปรไปตามรอบเดือน โตขึ้นก่อนมีประจาเดือน และยุบ
ลงเมื่อประจาเดือนมาแล้ว ก้อนมีผิวเรียบ ขอบเขตชัดเจน เคลื่อนได้บ้างและไม่ติดกับ
ผิวหนังหรือ fascia คลาได้ค่อนข้างแข็ง อาจเป็นพร้อมกันหลายก้อนและมีขนาดต่างๆกัน
2. มีน้าออกทางหัวนม เป็นน้าสีเหลืองใสหรือขุ่นหรือเหลืองเขียวหรือเทา เมื่อกดตามแนว lactiferous
duct จะมีน้าออกมา
3. ปวดหรือเจ็บเต้านม (mastalgia) เป็นทั่วทั้งเต้านมหรือเฉพาะบางส่วน อาจปวดร้าวถึงหัวไหล่ บางราย
เป็นมมากก่อนประจาเดือนจะมา อาจเป็นนานหลายปี
Intraductal carcinoma (Ductal carcinoma in situ)
เป็น non-invasive carcinoma ที่พบบ่อยที่สุด (ขอโทษที อาการเจอแต่ที่พูดถึงมะเร็งเต้านมโดยรวมอ่ะ คง
จะคล้ายๆกัน) เป็นก้อนในเต้านม ก้อนโตขึ้นช้าๆ ส่วนใหญ่ไม่มีอาการเจ็บหรือปวด และไม่สัมพันธ์กับ
ประจาเดือน บางรายมีเลือดหรือน้าออกทางหัวนม หรือหัวนมบอด บางรายมีเต้านมบิดเบี้ยวไปหรือหดลง ผิวหนัง
บุ๋มลง ก้อนคลาได้แข็งขรุขระ ขอบเขตไม่ชัดเจน หรือยึดติดกับ pectoral fascia
Intraductal papilloma
เป็นเนื้องอก epithelium ที่บุผิวท่อน้านมใกล้หัวนม โตยื่นเข้ามาในท่อ ผู้ป่วยส่วนใหญ่อายุ 40 ปี มีน้าใสๆ
หรือมีเลือดดาๆแดงๆออกทางหัวนม บางรายคลาๆได้ก้อนเล็กๆบริเวณ areola

4. ผู้ป่วยชาย 56 ปี ประวัติครอบครัวพี่ชายเป็นโรคมะเร็งลาไส้ใหญ่แล้วเสียชีวิต จึงมาตรวจร่างกาย ทา


colonoscopy พบ polyps at descending colon แพทย์ทา polypectomy การตรวจพบในข้อใดมีโอกาสพัฒนา
เป็นมะเร็งมากสุด
a. Villous adenoma
b. Tubulovillous adenoma
c. Tubulous adenoma

ตอบ a. Villous adenoma


ตารางแสดงชนิดของ polyp และโอกาสที่เป็นมะเร็ง
ชนิด โอกาสที่เป็นมะเร็ง
Tubular 5%
Tubulo-villous 20%
Villous 40%
ข้อ 2-4 เฉลยโดย X ผอมนะคับ

5. ผู้ป่วยชายเป็น Hernia ที่ testis ด้านขวาโตมา 1 ชม.ก่อนมารพ. การปฏิบัติในข้อใดเหมาะสมที่สุดในผู้ป่วยรายนี้


a. Reduction
b. Ultrasound
c. Surgery
d. Radiation
e. Reassure
ตอบ. ไม่รู้ว่าตอบอะไรนะ เพราะโจทย์ไม่ครบ แต่น่าจะ Surgery
การรักษาไส้เลื่อน
1. ผ่าตัด ความจริงผู้ป่วยที่เป็นไส้เลื่อน ถ้าเป็นไปได้ควรจะผ่าตัด โดยเฉพาะในรายที่ไส้มีการเลื่อนเข้าเลื่อน
ออก(เพราะพวกนี้ถ้าวันไหนเลื่อน เข้าแล้วไม่ออก มักเกิดเรื่อง) เมื่อผ่าไปแล้ว ทาการตัดและเย็บปิด
ช่องทางที่ผิดปกติและใช้เทคนิกการผ่าตัดเพื่อ เสริมความแข็งแรงให้บริเวณนั้น หรืออาจจะใส่วัตถุ
สังเคราะห์รูปตาข่ายไปพยุงส่วนนั้นให้แข็งแรง
2. ดันไส้เลื่อนกลับ ในบางรายที่มารพ.ด้วยก้อนมีขนาดโตขึ้นและเจ็บปวด ในเบื้องต้นยาลดปวดและจัดท่า
เพื่อดันไส้เลื่อนให้กลับเข้าไป
3. รอต่อไป ในผู้ป่วยบางรายมีโรคประจาตัวหรือสภาวะร่างกายที่ไม่เหมาะสมต่อการผ่าตัด หรือเสี่ยงเกิดที่
จะผ่าตัดไหว ก็จะไม่ได้รับการผ่าตัด แต่ก็จะมีการแนะนาวิธีปฏิบัติตัวเพื่อลดการเป็นหรือลดการเกิด
อาการ
แล้วจะทาอย่างไรขณะรอผ่าตัดไส้เลื่อน
การป้องกันไม่ให้เป็นซ้าในช่วงที่รอผ่าตัดเป็นสิ่งสาคัญ สิ่งที่เราพอจะป้องกันได้ทั้งในรายที่เป็นแล้ว
และในรายที่ยังไม่เป็นก็คือ การลดความดันในช่องท้องจากการกระทาต่างๆ คือ

1. อย่าไอ-ในที่นี้คือไม่ไปรับสิ่งที่เสี่ยงต่อการไอ เช่น หยุดการสูบบุหรี่ อย่าให้เป็นหวัด ถ้าไอควรจิบ


น้าอุ่นบ่อยๆ
2. อย่ายกของหนัก - การยกของหนักจะทาให้เกิดการเบ่ง และเกิดไส้เลื่อนซ้าได้
3. อย่าเบ่งอุจจาระ - ก็คือควรกินอาหารที่มีกากใยเพื่อช่วยในการระบาย เพราะหากท้องผูกจนต้องเบ่ง
อุจจาระ ก็สามารถเกิดไส้เลื่อนซ้าได้
4. อย่าเบ่งปัสสาวะ - ในรายที่มีความเสี่ยงต่อการเป็นต่อมลูกหมากโตหรืออายุมาก อาจจะต้องเบ่ง
ปัสสาวะบ่อย ทางแก้คือไปพบแพทย์ในเรื่องดังกล่าวเพื่อตรวจและรับการรักษาเพื่อให้ถ่าย
ปัสสาวะคล่องขึ้น
ว่าควรรักษายังไง จึงจะตอบ definite treatment ซึงก็คือการผ่าตัดนั่นเอง)

6. ผู้ป่วยชายอายุ 30 ปี มีอาการเจ็บและปวดบริเวณถุงอัณฑะด้านซ้ายมา 5 วันก่อนมารพ. การตรวจร่างกาย


เบื้องต้นพบว่า มีอาการปวด บวม แดง อาการเจ็บลดลงเมื่อยกถุงอัณฑะขึ้น แพทย์เจ้าของไข้ส่งตรวจ Ultrasound
พบ increase vascular blood flow การวินิจฉัยในข้อใดเป็นไปได้มากที่สุด
a. orchitis
b. epididymitis
c. incarcerated hernia
d. acute torsion testis
e. torsion of appendix testis
DDx: Scrotal pain
 Acute pain ( คนนี้เป็น acute scrotal pain ก็ DDx ตามนี้นะ )

Infection - Acute epididymitis; epididymoorchitis


Ischemia - Torsion of Spermatic cord or testicular appendage
Trauma - Hematocoele; Rupture testis
Complicated inguinal hernia
 Refer pain from ureter

 Chronic pain

: Dull heavy sensation from mass –Hydrocoele; Chronic epididymitis; Tumor


: Venous strangulation --Varicocoele
: Reversible ischemia --Torsion

Epididymitis
Clinical Manifestration
acute pain in the scrotum
fever and pyuria.
Physical examination
enlarged and tender epididymis
Pain ดีขึ้นเมื่อยก scrotum ไปที่ symphysis pubis (Prehn sign).
มักพบ cremasteric reflex
อาการที่มา present เหมือนกับ testicular torsion
Torsion of the appendix of the testis ก็ทาให้เกิดpain ได้เหมือนกันแต่จะมักจะไม่มี systemic symptoms
blue dot sign, พบในผู้ป่วย torsion of the appendix of the testis
Prehn sign จะ positive ในผู้ป่วย acute epididymitis แต่ negative ในผู้ป่วย testicular torsion.
cremasteric reflex พบในผู้ป่วย acute epididymitis แต่ไม่พบในผู้ป่วย testicular torsion.
Preferred Examination
ใช้การซักประวัติและตรวจร่างกายเป็นสาคัญในการวินิจฉัย acute epididymitis
แต่ถ้าไม่แน่ใจก็ใช้ Ultrasound ซึ่งมีประโยชน์มากในการdetect epididymitis and/or epididymo-orchitis. และยัง
ช่วยในการ exclude testicular torsion อีกด้วย
Differential Diagnoses
Testicle, Malignant Tumors
Testicle, Trauma
Testicular Torsion
Testicular Torsion
Clinical Manifestration
acute onset of scrotal pain, มักเป็นตอนกลางคืน; มีประวัติปวดมาก่อน ( 40% of patients);
scrotal swelling and erythema; จนคลา testis ไม่ได้
อาจมีอาการเลียนแบบ appendicitis เช่น abdominal , nausea and vomiting.
อาจมี flank pain radiating to the groin.
Physical Examination,
hemiscrotum is swollen and erythematous.
อาจคลาไม่ได้ normal separation of the testis from the epididymis
High lying testis or horizontal position (Brunzel sign) and skin pitting at the scrotal base (Ger sign)
transillumination, อาจพบ ischemic testicle (blue-dot sign).
ยก scrotum แล้ว pain ไม่ดีขึ้น (Prehn sign)
spermatic cord - thickened and tender.
Tenderness alone บ่งบอกว่าอาจเป็น acute epididymitis.
Bilaterality พบได้ 10% ของผู้ป่วย
Preferred Examination
ultrasonography

Epididymo-orchitis Torsion
Pain 2-3 days 1-2 hours
Fever Yes Absence
Urination Abnormal Normal
Skin swollen Yes No
Spermatic Cord Normal Palpable tender
Prehn’s Sign pain relieve pain constant

orchitis
Clinical Manifestration  Fever
 Discharge from penis
 Testicular swelling on one or both sides  Blood in semen
 Pain ranging from mild to severe
 Tenderness in one or both testicles
 Nausea
Physical Examination
 Testicular examination  Rectal examination
o Testicular enlargement o Soft boggy prostate (prostatitis)

o Induration of the testis often associated with epididymo-


o Tenderness orchitis
o Erythematous scrotal skin o Stool for occult blood

o Edematous scrotal skin  Other


o Enlarged epididymis associated o Parotitis

with epididymo-orchitis o Fever

สรุป ข้อนี้โจทย์ยังไม่ค่อยครบอยู่ดี
สาหรับ hernia ไม่พูดถึงนะ มันไม่เหมือนอยู่แล้ว
คนนี้มี Prehn’s Sign ดังนั้น จึงนึกถึง epididymitis ซึงอาจมี orchitis ( testicular inflammation ) ร่วมด้วยหรือเปล่า
ก็ไม่รู้นะ เพราะส่วนใหญ่ orchitis มักจะลามมาจาก epididymitis ดังนั้นตอบ b. epididymitis
ส่วน ultrasound ไม่เห็นมันช่วยอะไรเลย บอกแค่ว่ามี increase vascular blood flow ถ้าโจทย์ครบต้องบอกว่าเลือด
ไปที่ไหน ถ้าไปที่ epididymis ก็เป็น epididymitis ถ้าไปที่ testis ก็เป็น orchitis
7. ผู้ป่วยชายไทยเคยเป็นโรคตับอ่อนอักเสบ รักษาจนหายดีแล้ว แต่ยังมีอาการจุกแน่นท้อง ท้องอืด เบื่ออาหาร
น้าหนักลด ตรวจพบ pseudocyst at the front of pancreas ตั้งแต่ 4 สัปดาห์ก่อน ก้อนโตขึ้นอย่างรวดเร็ว การปฏิบัติ
ในข้อใดเหมาะสมที่สุดในผู้ป่วยรายนี้
a. partial pancreatectomy
b. total pancreatectomy
c. excision of pseudocyst
d. internal drainage
e. percutaneous drainage

ข้อนี้ใส่เนื้อหามาเยอะเพื่อให้มันครบๆ เผื่อใครอ่านแล้วงง ว่ามันคืออะไร


แต่ถ้าอยากได้คาตอบเลยก็ข้ามไปส่วนสุดท้าย management ได้เลย
ทั้งหมดนี้เอามาจาก powerpoint ของ Kashaf Sherafgan, MD Surgery IV Conference May 5th 2006

Pancreatic Pseudocyst  Similar electrolyte concentrations


 A fluid collection contained within a well- to plasma
defined capsule of fibrous or granulation tissue
or a combination of both  High concentration of amylase,
 Does not possess an epithelial lining lipase, and enterokinases such as
 Persists > 4 weeks trypsin
 May develop in the setting of acute or chronic
Pathophysiology
pancreatitis
 Pancreatic ductal disruption 2 to
 Most common cystic lesions of the pancreas,
accounting for 75-80% of such masses  Acute pancreatitis – Necrosis
 Location
Lesser peritoneal sac in proximity  Chronic pancreatitis – Elevated
to the pancreas pancreatic duct pressures from
Large pseudocysts can extend into strictures or ductal calculi
the paracolic gutters, pelvis, mediastinum,
 Trauma
neck or scrotum
May be loculated  Ductal obstruction and pancreatic
 Composition neoplasms
 Thick fibrous capsule – not a true
epithelial lining Presentation
 Symptoms
 Pseudocyst fluid
 Abdominal pain > 3 weeks (80 –  Pseudoaneurysm formation – Splenic
90%) artery (most common), GDA, PDA

 Nausea / vomiting Management of Pancreatic Pseudocysts


 Initial
 Early satiety
 NPO
 Bloating, indigestion  TPN
 Octreotide
 Signs  Antibiotics if infected
 1/3 – 1/2 resolve spontaneously
 Tenderness
 Intervention
 Abdominal fullness  Indications for drainage
 Presence of symptoms (>
Diagnosis 6 wks)
 CT scan  Enlargement of
 MRI / MRCP pseudocyst ( > 6 cm)
 Complications
 Ultrasonography  Suspicion of malignancy

 Endoscopic Ultrasonography (EUS)

 ERCP  Intervention
 Percutaneous drainage
Complications  Endoscopic drainage
 Infection  Surgical drainage
 S/S – Fever, worsening abd pain, Percutaneous Drainage
 Continuous drainage until output < 50
systemic signs of sepsis
ml/day + amylase activity ↓
 CT – Thickening of fibrous wall  Failure rate 16%
or air within the cavity  Recurrence rates 7%
 Complications
 GI obstruction  Conversion into an infected
pseudocyst (10%)
 Perforation
 Catheter-site cellulitis
 Hemorrhage  Damage to adjacent organs
 Pancreatico-cutaneous fistula
 Thrombosis – SV (most common)
 GI hemorrhage  Head of gland with strictures of
Endoscopic Management pancreatic or bile ducts –
 Indications pancreaticoduodenectomy
 Mature cyst wall < 1 cm thick  External drainage
 Adherent to the duodenum or  Internal drainage
posterior gastric wall  Cystogastrostomy
 Previous abd surgery or  Cystojejunostomy
significant comorbidities  Permanent resolution
 Contraindications confirmed in b/w 91%–
 Bleeding dyscrasias 97% of patients*
 Gastric varices  Cystoduodenostomy
 Acute inflammatory changes that Can be complicated by duodenal fistula and
may prevent cyst from adhering to bleeding at anastomotic site
the enteric wall Laparoscopic Management
 CT findings  The interface b/w the cyst and the enteric
 Thick debris lumen must be ≥ 5 cm for adequate
 Multiloculated drainage
pseudocysts  Approaches
Endoscopic Drainage  Pancreatitis 2 to biliary etiology
 Transenteric drainage  extraluminal approach w/
 Cystogastrostomy concurrent laparoscopic
 Cystoduodenostomy cholecystectomy
 Transpapillary drainage Non-biliary origin  intraluminal (combined
 40-70% of pseudocysts laparoscopic/endoscopic) approach
communicate with pancreatic Surgical management of complications a/w
duct percutaneous and/or endoscopic management of
ERCP with sphincterotomy, balloon dilatation of pseudocyst of the pancreas
pancreatic duct strictures, and stent placement
beyond strictures Indications for Operation in Patients with
Surgical Options Complications of Percutaneous or Endoscopic
 Excision management
 Tail of gland & a/w proximal
strictures – distal pancreatectomy Discussion
& splenectomy  Morbidity rates of operative management
of pseudocyst range from 4% – 30%
 Success rates  Technically challenging to operate on
patients who failed nonoperative measures
 Endoscopic/percutaneous – 60%–
90%  Necessary to completely abolish
the prior cystic structure once it
 Surgical – 94%–99%
has been decompressed and the
 Patients who failed non-operative walls have fused
measures should have a period of
 Dissection is more challenging
stabilization prior to operation
than the dissection involved in
 Important to reverse sepsis and to simply defining a pseudocyst and
improve nutritional status prior to draining it
intervention

Management Recommendations
 Without evidence of complications, simple observation x min 6 wks

 Infected pseudocysts should be managed with percutaneous drainage until the patient is stabilized

 Severe nutritional deficits, at times an indication for percutaneous drainage, should be addressed

Once the pseudocyst is established as persistent, observe truly asymptomatic patients with small cysts
 Intervention in all pseudocysts > 6 cm, symptomatic patients

 Use ductal anatomy to guide choice of modality

 Types V, VI, and VII ductal injuries are all managed operatively

 Types I and II are always managed nonoperatively

 Types III and IV are still under debate

Significant complications are likely to occur should nonoperative measures be used in patients most likely to
sustain complications
สรุป โจทย์ข้อนี้ก็ยังไม่ค่อยครบนะ
 Indications for drainage
 Presence of symptoms (> 6 wks)
 Enlargement of pseudocyst ( > 6 cm)
 Complications
 Suspicion of malignancy
คนไข้รายนี้ มีอาการมาแค่ 4 สัปดาห์
Complication ไม่มีข้อมูล
Malignancy ไม่มีข้อมูล
ในโจทย์บอกว่าก้อนโตขึ้นอย่างรวดเร็ว ก็เดาว่าคงจะโตเกิน 6 ซม.นะ เลยต้องทา intervention อะไรซักอย่าง
 Intervention มีอยู่ 3 ทางเลือกโดยเรียงลาดับความยากง่ายดังนี้คือ
 Percutaneous drainage
 Endoscopic drainage
 Surgical drainage – excision, external drainage, internal drainage
ในโจทย์ข้อนี้ไม่เห็นมันบอกรายละเอียดอะไรเลย เลยเดาว่าทุกย่างปกติหมดนะ เลยตอบ e. percutaneous drainage

แต่ถ้ามี
 Mature cyst wall < 1 cm thick
 Adherent to the duodenum or posterior gastric wall
 Previous abd surgery or significant comorbidities
ก็เลือก Endoscopic management ( ในโจทย์ไม่มีให้เลือก )
และถ้า fail non- operative ก็ค่อยไปเลือก surgical option

8. ผู้ป่วยชายโดนหินเจียรกระเด็นทิ่มคอขณะกาลังทางาน ตาแหน่งที่โดนใกล้ลูกกระเดือก 30 นาทีก่อนมา


โรงพยาบาล ช่วงแรกที่บาดแผลมีเลือดออกตามจังหวะการเต้นของหัวใจ, วัดความดันโลหิตได้ 86/70 mmHg
ขณะนี้เลือดหยุดไหลแล้ว มีแผล laceration 3 cm anterior to sternocleidomastoid การปฏิบัติในข้อใด
เหมาะสมที่สุดในผู้ป่วยรายนี้
a. Explore wound at ER
b. Explore wound at OR
c. CT
d. MRI
e. Angiography
เฉลย เพื่อน ๆ รูปใน Schwartz มันผิดอ่ะ คือบรรยายที่เขาบรรยาย
ไว้กับรูปไม่ตรงกัน จริงๆ zone 1 มันใต้ clavicle ลงมา zone 2 คือ
zone 3ในรูป และzone 3 คือ zone 2 ในรูปนะ เราแก้รูปแล้วดูตามนี้
เลยนะ
Mx

ถ้า zone 2 ถ้าคนไข้ stable ก็เอาไป Explore wound at OR เพราะ


มันไม่มีโครงสร้างสาคัญ
zone 3 ต้องทา Angiography เพราะอาจเกิดความเสียหายต่อ carotid
and vertebral artery ซึ่ง มี 3 เหตุผลที่ควรทา Angiography คือ
1 . exposure of distal internal carotid and vertebral artery is difficult
2. the internal carotid artery may have to be ligated, a maneuver associated with high risk of stroke
3. active hemorrhage from external carotid artery and vertebral artery can be controlled by selective
embolization
ตอบ E Angiography by off NUH

9. ชายอายุ 30 ปี ประสบอุบัติเหตุรถชน ต้องทา distal below knee amputation แพทย์พัน stump ไว้ หลังจาก
นั้นแพทย์ควรจะนัดผู้ป่วยมาเพื่อใส่ขาเทียมแบบถาวร นานเท่าไร
a. 7 วัน
b. 10 วัน
c. 15 วัน
d. 30 วัน
e. 45 วัน
เฉลย เนื่องจากข้อนี้นะเพื่อนๆ เราหาแล้วมันไม่มีหนังสือเล่มไหนบอกเลยว่าเวลาที่แน่นอนคือเท่าไร
แต่ อ. นพ.ชินภัทร์ จิระวรพงศ์ เขียนไว้ใน sheet ว่า estimate time of mature stump คือ เดือนครึ่งไม่
เกินสองเดือน ตอบ E. 45 วัน by off NUH

10 หญิงอายุ 16 ปี มีก้อนที่เต้านมด้านขวา ก้อนขนาดประมาณ 2 cm. ปวดสัมพันธ์กับรอบเดือน แพทย์ตรวจ


ร่างกายพบว่ามี well define 2.5 centimeter mass, firm consistency at the right breast การปฏิบัติในข้อใด
เหมาะสมที่สุดในผู้ป่วยรายนี้
f. ให้ตรวจเต้านมด้วยตนเอง
g. มาพบแพทย์ให้ตรวจเต้านม
h. Fine needle aspiration
i. Ultrasound หลังจากนั้นนัดตรวจติดตามทุกๆ 6 เดือน
j. Mammogram หลังจากนั้นนัดตรวจติดตามทุกๆ 12 เดือน
เฉลย C ข้อนี้น่าจะเป็น Fibrocystic change เพราะสัมพันธ์กับประจาเดือน ก้อนในเต้านมส่วนมากมัก
เป็น cyst แต่อาจเป็น mass หรือ cyst ร่วมกับ mass ก็ได้
- Mammogram หลังจากนั้นนัดตรวจติดตามทุกๆ 12 เดือน ข้อนี้ไม่ทาเพราะอายุน้อย เนื้อยังแน่น
แปลผลลาบากควรทาเมื่อหมดประจาเดือน
- Ultrasound หลังจากนั้นนัดตรวจติดตามทุกๆ 6 เดือน ข้อนี้ส่งไปก็ช่วยแยกได้แค่แยกว่าเป็น cyst
หรือ mass
- FNA อันนี้น่าทาสุด เพราะถ้าทาแล้วเป็น benign ค่อยแนะนาให้ตรวจเต้านมด้วยตนเอง สังเกต
อาการต่อ ถ้าเป็นเป็น malignancy ก็ค่อยทาไปตาม line
Ref. schwartzs และ อ.นพ. องอาจ เลิศขจรสิน

11. pt 45ปี สูบบุหรี่ นิ้วมี gangrene


Buerger disease, a nonatherosclerotic vascular disease also known as
thromboangiitis obliterans (TAO)
TAO is characterized by the absence or minimal presence of atheromas, segmental vascular inflammation,
vasoocclusive phenomenon, and involvement of small- and medium-sized arteries and veins of the upper and
lower extremities. The condition is strongly associated with Age younger than 45 years , heavy tobacco use,
and progression of the disease is closely linked to continued use. The typical presentations are rest pain,
unremitting ischemic ulcerations, and gangrene of the digits of hands and feet, and as the disease evolves, the
patients may require several surgical amputations.

Ob & Gyn
1.หญิงไทยอายุ 32 ปี G2P1 GA 24 wk มีประวัติบิดาป่วยเป็นโรคเบาหวาน การตรวจคัดกรองในข้อใดต่อไปนี้
เหมาะสมที่สุด
A) Fasting blood sugar D) Random glucose
B) 50 g GCT E) Urine sugar
C) 100 g OGTT
การประเมินความเสีย่ งต่อการเกิดเบาหวานขณะตัง้ ครรภ์
ความเสี่ยงสูง พบปัจจัยต่อไปนี้ >=1
 อายุมากกว่า 35 ปี
 อ้วน (น้าหนักมากกว่า ideal weight ร้อยละ 20)
 เบาหวานในญาติใกล้ชิด
 ประวัติเบาหวานขณะตั้งครรภ์ในครรภ์ที่ผ่านมา
 ประวัติการคลอดที่ผ่านมาผิดปกติ เช่น เคยคลอดทารกตัวโต (น้าหนักทารก
มากกว่า 4,000 กรัม) ทารกตายคลอด พิการแต่กาเนิด มีภาวะครรภ์เป็นพิษ
ครรภ์แฝดน้า เป็นต้น
 พบน้าตาลในปัสสาวะ (glycosuria)
 ตรวจกรองตั้งแต่เริ่มฝากครรภ์ หรือ ทันทีที่ตรวจได้ และตรวจซ้าเมื่ออายุครรภ์ 24-28 สัปดาห์ ถ้าการ
ตรวจครั้งแรกไม่พบ GDM

ความเสี่ยงปานกลางไม่จัดอยู่ในกลุ่มความเสี่ยงสูงหรือความเสี่ยงต่าตรวจกรองเมื่ออายุครรภ์ 24-28 สัปดาห์


ความเสี่ยงต่่า (พบทุกข้อต่อไปนี้)
 อายุน้อยกว่า 25 ปี  ไม่มีประวัติของระดับน้าตาลในเลือด
 เชื้อชาติหรือพื้นเพเป็นกลุ่มที่มีความเสี่ยงต่า ผิดปกติ
 ไม่พบเบาหวานในญาติใกล้ชิด  ไม่พบประวัติทางสูติศาสตร์ที่ผิดปกติ
 น้าหนักก่อนตั้งครรภ์ และน้าหนักที่เพิ่มขึ้น  อาจไม่จาเป็นต้องตรวจกรอง
 ระหว่างตั้งครรภ์ปกติ(BMI <26 kg/m2)

ผู้ป่วยรายนี้เป็นผู้ป่วยที่มีความเสี่ยงสูงต่อการเกิดเบาหวานขณะตั้งครรภ์ เนื่องจากมีประวัติบิดาเป็นเบาหวาน
โดยผู้ป่วยความเสี่ยงสูงต้องปฏิบัติดังนี้

High risk

50 g GCT
 140 mg/dL < 140 mg/dL

100 g OGTT No further test


100-gram OGTT (3-hour)
-ไม่จากัดอาหาร 3 วันก่อนทดสอบ
- คาร์โบไฮเดรตอย่างน้อยวันละ 150 กรัม
- NPO 10-12 ชั่วโมง
- ดื่มน้าตาล กลูโคส 100 กรัม
- FPG และหลังดื่มน้าตาลทุกๆ 1 ชั่วโมงเป็นเวลา 3 ชั่วโมงติดต่อกัน
- ถ้าระดับน้าตาลในเลือด >= ค่ามาตรฐานมากกว่าสองค่าขึ้นไป = GDM
-ในกรณีที่ตรวจพบความผิดปกติเพียง 1 ค่า => ตรวจซ้าอีก 1 เดือน

เวลา ค่าระดับน้าตาลในเลือด (mg/dl)


ขณะอดอาหาร 105
1 ชั่วโมง 190
2 ชั่วโมง 165
3 ชั่วโมง 145

2. เด็กคลอดติดไหล่นาน หลังคลอดตรวจร่างกายพบ internal rotation of right arm, extended arm with flex wrist
และไม่ขยับแขนข้างดังกล่าว ตาแหน่งพยาธิสภาพในผู้ป่วยรายนี้คือข้อใด
A) C1-2 D) C8-T1
B) C3-4 E) T2-3
C) C5-6
ระหว่างการทาคลอด แพทย์ได้ใช้แรงดึงศีรษะของทารกในขณะหัวไหล่ของทารกยังติดอยูใ่ นช่องคลอด
ทาให้เกิดการบาดเจ็บที่ upper limb (ventral primary rami) บริเวณแขนงของ brachial plexus ของ C5 C6
Brachial plexus Injury ท่าให้เกิด Erb ’s palsy หรือ Erb-Duchenne palsy
แขนไม่สามารถ abduct ได้โดยจะอยู่แนบชิดลาตัวตลอดเวลา แขนเหยียดตรง ไม่สามารถทาelbow flexionได้
แขนหมุนเข้าด้านใน ฝ่ามือคว่าหันไปทางด้าน posterior
Sensation ที่ dermatome C5, C6 เสีย
- Deltoid >>> Axillary nerve ( C5,C6)
- Biceps brachii
Musculocutaneous nerve(C5,C6,C7)
- Brachialis
- Brachioradialis >>> Radial Nerve ( C5,C6,C7)
- Supraspinatus
Suprascapular nerve (C5,C6)
- Infraspinatus
- Teres minor >>> Axillary nerve ( C5,C6)
- Supinator >>> Deep branch of radial nerve( C5,C6,C7)
3. ผู้ป่วย G1P0 GA 34 wk มีน้าไหลออกจากช่องคลอด กลั้นไม่ได้ ตรวจด้วย sterile speculum พบ fluid ที่
posterior fornix และ fern test: positive ข้อใดต่อไปนี้ห้ามทาในผู้ป่วยรายนี้
A) Tocolytic drug
B) Dexamethasone
C) IV prophylactic ATB
D) Observe sign of chorioamnionitis
E) PV เพื่อดู progression อย่างต่อเนื่อง

PPROM
การซักประวัติ ผู้ป่วยจะมีประวัติว่ามีน้าใสๆไหลจากช่องคลอด คล้ายกลั้นปัสสาวะไม่อยู่ โดยผู้ป่วยอาจคิด
ว่าเป็นอาการปัสสาวะราด
การตรวจร่างกาย ควรระมัดระวังการติดเชือ้ จากการตรวจภายใน ก่อนทาการตรวจภายในควรสังเกตดู
บริเวณปากช่องคลอดว่ามีลักษณะเปียกชื้นมากน้อยเพียงใด สีน้าเป็นอย่างไร แล้วจึงใส่ Speculum ที่ปราศจากเชื้อ
เข้าไปในช่องคลอด พร้อมกับสังเกตลักษณะ และปริมาณของเหลวภายในช่องคลอด โดยทั่วไปถ้าถุงน้าคร่าแตก
จริงจะพบน้าคร่าอยู่บริเวณ Posterior fornix
การวินิจฉัยต้องเก็บของเหลวจากบริเวณช่องคลอด โดยมากจะอยู่บริเวณ Posterior fornix แล้วนาน้า
ดังกล่าวมาตรวจ Fern test โดยใช้ไม้พันสาลีป้ายลงบน Slide ทิ้งไว้จนแห้ง แล้วตรวจดูด้วยกล้องจุลทรรศน์ เป็น
การตรวจผลึก (Crystallization ) ของ NaCl ที่อยู่ในน้าคร่าจะตกผลึกเป็นรูปร่างคล้ายใบเฟิร์น โดยสามารถตรวจ
Test นี้ได้ตั้งแต่อายุครรภ์ 12 สัปดาห์ขึ้นไป การทดสอบนี้อาจให้ผลบวกลวงได้จากเมือกของปากมดลูกโดยจะมี
ลักษณะเป็นใบเฟิร์นเช่นกัน แต่จะมีขนาดใหญ่กว่าปริมาณมากกว่า ส่วนผลลบลวงอาจพบจาก มีเลือดหรือ
Meconium ปนเปื้อน
การรักษา
1. Admit
2. หลีกเลี่ยงการตรวจภายใน และการตรวจทางทวารหนัก เพื่อป้องกันการนาเชื้อเข้าไปในโพรงมดลูก
3. ภาวะการติดเชื้อ
ในผูป้ ว่ ยทุกรายทีไ่ ด้รับการวินจิ ฉัยว่าถุงน้าคร่าแตก โดยไม่คานึงถึงอายุครรภ์และการเจ็บครรภ์ จะต้อง
ได้รับการตรวจโดยละเอียดเพื่อวินิจฉัยภาวะติดเชื้อในโพรงมดลูก (chorioamnionitis หรือ intraamniotic
infection) ทั้งนี้เพราะหากวินิจฉัยได้ล่าช้าหรือวินิจฉัยไม่ได้จะเพิ่มความเสี่ยงต่อทั้งมารดาและทารก อุบัติการณ์
ของการติดเชื้อในโพรงมดลูกพบได้ประมาณร้อยละ 0.5-1 หากถุงน้าคร่าแตกเกิดขึ้นเป็นเวลานานอุบัติการณ์อาจ
สูงขึ้นร้อยละ 3-15 นอกจากนี้ยังพบได้บ่อยขึ้นในครรภ์ก่อนกาหนด(preterm) การวินิจฉัยภาวะติดเชื้อในโพรง
มดลูกมักจะอาศัยการตรวจพบว่ามีไข้ > 38.C มารดามีชีพจรเต้นเร็ว เสียงการเต้นของหัวใจทารกเต้นเร็ว มดลูกกด
เจ็บ มีถุงน้าคร่าแตก น้าคร่าอาจมีกลิ่นเหม็น ไม่มีอาการในระบบอวัยวะอื่นๆ หรือตรวจไม่พบสาเหตุอื่นๆ ของไข้
เมื่อวินิจฉัยว่ามีการติดเชื้อในโพรงมดลูก แพทย์ผู้ดูแลจะต้องให้ยาปฏิชีวนะแก่ผู้ป่วยโดยเร็ว ยาปฏิชีวนะ
ที่ให้กันอย่างแพร่หลายที่สุดและครอบคลุมเชื้อได้ดีคือ Ampicilin ขนาด 2 กรัม เข้าเส้นเลือดดาทุก 4-6 ชั่วโมง
หรือ Penicellin G sodium 5 ล้านยูนิตทุก 4-6 ชั่วโมง ร่วมกับ Gentamicin ขนาด 1.5 มิลลิกรัม/กิโลกรัม ทางเส้น
เลือดดาทุก 8 ชั่วโมง ยา 2 ตัว ดังกล่าวครอบคลุมเชื้อโรคได้ดีโดยเฉพาะเชื้อ group B streptococci (GBS)และเชื้อ
E.coli ซึ่งเป็นสาเหตุหลักที่ทาให้เกิดการติดเชื้อในทารกแรกเกิด
4. อายุครรภ์ของทารก
อายุครรภ์ของผู้ป่วยรายนี้น้อยกว่า 37 สัปดาห์จึงถือเป็น Preterm ร่วมกับอายุครรภ์34 สัปดาห์อยู่ในช่วงที่
สามารถให้ Corticosteroid เพื่อกระตุ้นความสมบูรณ์ของปอดทารกได้ จึงควรมีการให้ Tocolytic drug เพื่อ
ยืดเวลาเพื่อรอให้ Corticosteroid ออกฤทธิ์ได้เต็มที่ซึ่งต้องการเวลา 24-48 ชั่วโมง

17. หญิง ไม่มีบุตร มีตกขาวกลิ่นเหม็น ส่งอะไร


A) wet smear
B) G/S
C) C/S
ตอบ A) จากลักษณะของตกขาวควรส่งตรวจเพิ่มเติม เช่นwet preparation, gram stain เพื่อทราบชนิดของเชื้อ ซึ่ง
ตกขาวกลิ่นเหม็น อาจเกิดจาก BV(กลิ่นคาว)-พบตกขาวเป็นน้าเหลวสีขาวปนเทา,pH>4.5,clue cell, whiff test
positive(หยดKOHแล้วมีกลิ่นคาว) แต่ในmucopurulent cervicitisที่มีตกขาวคล้ายหนอง จากgonorrheaหรือ
C.tracomatis หรือherpes simplex virus มักไม่มีกลิ่น และไม่คัน ส่วนTV จะทาให้มีตกขาวเป็นฟองสีเขียวเหลือง
หรือขาว,คัน,อาจมีปัสสาวะแสบขัดร่วมด้วย พบstrawberry cervixเป็นpathognominic sign แต่ถ้าเป็นcandida ตก
ขาวจะเป็นตะกอนคล้ายนมขุ่นข้น อวัยวะเพศแดง และคัน ตรวจKOHพบpsuedohyphe with budding yeast cell
18. หญิง 18 ปี U/S mass Rt. Adnexal hypoecoic without internal content
A) functional ovarion cyst
B) PCOS
ตอบ A ใกล้เคียงสุด จากAกะB แต่จริงๆควรเป็น Follicle cysts พบmost common

โจทย์ให้มาแค่ adnexal mass ไม่มีcontent ข้างใน ซึ่งน่าจะหมายความว่าเป็นhypoecoic massที่ไม่มsี olid partจาก


U/S ซึ่งที่พบได้บ่อยในวัยรุ่นจะเป็นดังนี้

Follicle cysts

Follicle cysts of the ovary are the most common cystic structures found in healthy ovaries. These cysts arise
from temporary pathologic variations of a normal physiologic process and are not neoplastic. The tumors result
from either nonrupture of the dominant mature follicle or failure of an immature follicle Solitary follicle cysts
are common and occur during all stages of life, from the fetal stage to the postmenopausal period. The cysts are
thin walled and unilocular, usually ranging from several millimeters to 8 cm in diameter (average, 2 cm)
Corpus luteum cysts

Corpus luteum cysts are less prevalent than follicular cysts. The cysts mainly result from intracystic
hemorrhage. They are hormonally inactive but may tend to rupture with intraperitoneal bleeding, especially in
patients on anticoagulant therapy. Radiographically, these cysts may have a clear region of homogenous debris
(blood) at the gravity-dependent portion of the cyst.

ซึ่งปกติไม่จาเป็นต้องทาอะไร รอobserveไป ก็จะหายไปเอง

แต่ถ้าเป็นPCOS-Ultrasonographic findings suggestive of PCOS commonly include ovary enlargement,


increased follicle count, and stromal echogenicity. The ovaries are usually bilaterally enlarged and spherical
in shape, rather than ovoid 2 of 3 criteria are required to diagnosis PCOS. The 3 criteria are (1) polycystic
ovaries (multiple small cysts, often around the periphery of the ovary, the classic "string of pearls" appearance),
(2) signs of androgen excess (acne, hirsutism, temporal balding, male pattern hair loss, clitoromegaly, etc), and
(3) menstrual irregularities (oligomenorrhea or polymenorrhea)

ส่วนFunctional cystคือ

Functional ovarian cyst (FOC)


มักเกิดในหญิงวัยเจริญพันธ์เป็นส่วนใหญ่ โดยเกิดหลังจากไข่ตก จะเหลือสิ่งที่เรียกว่า Follicular และ
Corpus luteum ซึ่งทาหน้าที่สร้างฮอร์โมนโปรเจส-เตอโรน ที่ทาให้ผนังมดลูกเหมาะกับการฝังตัวของตัวอ่อนเพื่อ
เจริญเติบโตเป็นทารกต่อไป ซึ่งปกติแล้ว cyst เหล่านี้จะหายไปเองโดยไม่ก่อให้เกิดปัญหา อย่างไรก็ตามบางครั้ง
cyst เหล่านี้มีขนาดใหญ่ขึ้น, สามารถทาให้เกิดความเจ็บปวด หรือ อาจยังคงค้างอยู่ในรังไข่ได้เป็นเวลาหลายเดือน
ซึ่งวิธีการรักษาที่ดีที่สุดคือ การรอให้ cyst หายไปเอง แต่ถ้า cyst อยู่ในรังไข่นานหรืออยู่อย่างถาวร มักจะรักษา
โดยการผ่าตัด

19. คลอดหัวแล้วไหล่ติด ห้ามทาอะไร

A) ตัด perineum มากขึ้น


B) ผู้ช่วยกดsuprapubic
C) ผู้ช่วยกดfundus
D) flex hip เข่าติดท้อง
E) ค่อยๆดึงเด็กลงช้าๆ
ตอบ C) เนื่องจากการดันยอดมดลูกผิดจังหวะ สามารถทาให้ไหล่หน้ายิงติดมากขึ้น และหากกดยอดมดลูกอย่าง
แรงจะทาให้มดลูกแตกได้ การช่วยคลอดในภาวะshoulder dystocia ควรทาดังนี้
1. ขอความช่วยเหลือตามผู้ช่วย วิสัญญีแพทย์ กุมารแพทย์
2. สวนปัสสาวะออกให้หมด
3. ให้ยาระงับปวดที่เหมาะสมและเพียงพอ
4. ตัดฝีเย็บให้ลึกพอในแนวmediolateral (deep mediolateral episiotomy) ถ้าก่อนหน้าตัดในแนวตรง ให้ทา
episioproctotomy
5. ดูดน้าคร่าออกจากปากและจมูกทารกให้หมด หลีกเลี่ยงการกระตุ้นทารก เพราะจะทาให้เกิดaspirateเมือก
ลงปอด
6. พยายามทาคลอดไหล่หน้าด้วยการทดลองดึงศีรษะทารกลงล่างอย่างนุ่มนวล พร้อมให้มารดาช่วยเบ่ง อย่า
ดึงแรงเพราะจะทาให้branchial plexus injury
7. ให้ผู้ช่วยกดบริเวณเหนือหัวหน่าว (suprapubic pressure) ขณะที่ดึงศีรษะเด็กลงมา โดยให้ทาMcRoberts’
maneuverร่วมด้วย (ให้มารดางอขาบริเวณสะโพกชิดหน้าท้องให้มากที่สุด-ให้กระดูกหัวหน่าวเลื่อนขึ้น
ด้านบนทางด้านศีรษะ)
อ้างอิง: สูติศาสตร์ จุฬาลงกรณ์มหาวิทยาลัย หน้า209-210

20. Pt หญิง 28 y. G_P_ คลอด C/S หลังคลอด day 2 มีไข้ ปวดท้อง breast engorge นมไหลดี, lochia rubra
ปริมาณลดลง, แผล c/s – no oozing PV- tender on cervix motion, tender adnexa, Cx.: lochia rubra Dx (ข้อ
สุดท้าย หน้า 10)
A. breast abscess
B. mastitis
C. pelvic infection
D. TOA
E. C/S wound infection

ตอบ B. mastitis
แนวคิดของเจ้ เจ้นึกถึง mastitis มากที่ที่สุดน่ะ เพราะว่าคนไข้มีไข้ day 2 ตรวจร่างกายมี breast
engorgement นอกจากนี้แม่คลอด c/s วันที่สองเอง ส่วนใหญ่ลูกยังไม่ได้ดูดนมหรอก หรือดูดก็ยังไม่ดีเท่าที่ควร
ส่วนประวัติที่ว่าน้านมไหลดี บอกได้แค่ว่านมไหลออกมาดี(ไม่มี obstruction) อาจจะบีบแล้วออกมาดีก็ได้น่ะ
เพราะโจทย์ไม่ได้บอกว่าเด็กดูดนมได้ดี น่ะค่ะ
ส่วน finding อื่นๆๆ อ่ะน่ะ พวก ปวดท้อง PV แล้ว มี tender on cervical motion , adnexal tenderness
(โจทย์ไม่ได้ระบุข้าง) เจ้ว่าพวกนี้ก็เป็นปกติของคนคลอด day 2 อยู่แล้วอ่ะน่ะที่ต้องปวดท้อง PV ก็ต้องเจ็บ
แน่นอน (เจ้เคย-เคย PV น่ะจ่ะ ไม่ได้เคยคลอด) ที่ cervix มี lochia rubra และปริมาณลดลง ก็ไม่แปลกเพราะมี
ได้ถึง day 3
ข้อ : pelvic infection/TOA
กลุ่มนี้ทั้งกลุ่ม มักเกิดจากการคลอดธรรมชาติมากกว่า เช่น การติดเชื้อที่แผลฝีเย็บ การติดเชื้อที่ปาก
มดลูก หรือการติดเชื้อที่มดลูก (metritis) ในผู้ป่วยรายนี้คลอด C/S ดังนี้ที่เป็นไปได้ก็เหลือแต่ metritis ผู้ป่วย
มักมีไข้(เริ่ม day 2 ) ไข้สูง(>38.5) ปวดท้องน้อยบริเวณมดลูก น้าคาวปลามีกลิ่นเหม็นโดยเฉพาะ anaerobes แต่
ถ้าเป็นอื่นๆๆ อาจจะไม่เหม็นก็ได้ ตรวจ PV พบมดลูกกดเจ็บ สรุปแล้วเจ้ว่าไม่เหมือนน่ะ ส่วนอันอื่นๆ(การติด
เชื้อที่แผลฝีเย็บ การติดเชื้อที่ปากมดลูก) มักเกิดจาคลอดธรรมชาติที่ทาสูติศาสตร์หัตการ (V/E, F/E) พวกนี้มักมี
อาการปวดเฉพาะที่มักอาการไม่รุนแรงนัก มักมีอาการปัสสาวะลาบากร่วมด้วย ประมาณนั้นค่ะ

ข้อ A breast abscess


การตรวจร่างกายจะต้องพบเต้านม บวม แดง แม่มีไข้ มี fluctuation ร่วมด้วยค่ะ อันนี้ไม่ค่อยเหมือน
เท่าไหร่

ข้อ E : C/S wound infection


ไข้น่าจะมาวันที่ 3 มีอาการปวดที่แผล แผลมี oozing ไม่แห้ง มี discharge per gauze จานวนมาก
ประมาณนนี้ ซึ่งคนนี้ไม่มีเลย จึงไม่น่านึกถึงน่ะค่ะ

21. ผู้ป่วยหญิงอายุ 52 ปี เข้าสู่วัย menopause ต้องการแคลเซียมวันละเท่าไร


A) 300 D) 1000
B) 500 E) 2000
C) 800

ตอบ D) 1000
แนวคิดของเจ้ อันนี้ไม่มีแนวคิดค่ะ จามเอาอย่างเดียว
ในหนังสือนรีเวชวิทยา ของ อ.ธีระ เชียงใหม่ที่ใครๆ ก็ใช้กันอ่ะน่ะ เค้าบอกว่า ในสตรีหมดประจาเดือน
ที่ไม่ได้รับ HRT ควรจะได้รับแคลเซียมวันละ 1.5 กรัม แต่สาหรับสตรีที่ได้รับ HRT ควรจะได้รับแคลเซียม
วันละ 1 กรัม แต่จากการรับประทามอาหารจะได้รับแคลเซียมประมาณวันละ 500 มิลลิกรัม
ดังนั้นสตรีหมดประจาเดือนที่ไม่ได้รับ HRT ควรจะได้รับแคลเซียมเสริมวันละ 1 กรัม แต่สาหรับสตรี
ที่ได้รับ HRT ควรจะได้รับแคลเซียมวันละ 500 มิลลิกรัม

เมื่อทราบดังนี้แล้วเพื่อนๆ ก็เลือกตอบเอาแล้วกันน่ะค่ะว่าจะเลือกตอบข้อไหน เพราะถ้าโจทย์ให้มาแค่นี้


จริงๆ ก็พูดยางอ่ะเพราะไม่ได้บอกว่าได้ HRT หรือเปล่าอ่ะน่ะ และก็แคลเซียมที่โจทย์ถามเป็นแคลเซียมเสริม
หรืว่าแคลเซียมรวมก็ไม่รู้น่ะ เอาเป็นว่าจาความรู้ไปก็แล้วกันน่ะ

แต่ถ้าเป็นเจ้น่ะ ตอบข้อ D แล้วกันเพราะข้ออื่นๆๆ ไม่น่าเลือกอย่างแรงว่าไหมจ่ะ


22. ผู้ป่วยหญิง มีประวัติเคยใส่ IUD ตอนนี้ขาดประจาเดือนมา 6 สัปดาห์ จะทาอย่างไรต่อไป
A) Ultrasound
B) X-ray
C) ส่ง urine pregnancy test
D) ใช้ hook เข้าไปเกี่ยวเอา IUD ออก
E) ใส่ IUD ใหม่
ตอบ C) ส่ง urine pregnancy test
แนวคิดของเจ้ เจ้ว่าน่ะ คนไข้ที่ขาดประจาเดือนมาน่ะ ต้อง R/O pregnancy ให้ได้ก่อนอ่ะน่ะ จริงไหม
การรักษาอื่นๆๆจึงจะตามมาได้ และในผู้ป่วยรายนี้ขาดมา 6 สัปดาห์แล้วถ้าท้องจริงน่ะ UPT ต้องตรวจเจอแล้ว
ดังนั้นเจ้ว่าให้ส่ง UPT ไปก่อนล่ะกันเพราะ เป็นการ investigation ที่คัดกรองการตั้งครรภ์แบบ non-invasive
ราคาถูก จึงน่าจะเลือกก่อนเป็นอันดับแรก ส่วน ultrasound ก็สามารถวินิจฉัยการตั้งครรภ์ได้เช่นกัน แต่ผู้ป่วย
ขาดประจาเดือนมา 6 wk เอง คงต้องทา TVS (Transvaginal sonography) จึงจะดีซึ่งก็ invasive กว่า แพงกว่า
และต้องใช้ความชานาญคือส่งพบสูติแพทย์ให้ทาประมาณนั้น ดังนั้นเจ้ว่าถ้า UPT positive แล้วค่อยทา TVS ก็
ไม่น่าจะเสียหายน่ะ
ข้ออื่นๆๆๆ
X-ray : ไม่ค่อยทากันน่ะ อาจจะช่วยในกรณีที่สงสัยมีมดลูกทะลุจากการใส่ห่วงคุมกาเนิด ซึ่งผู้ป่วยมัก
มาด้วยอาการปวดท้องน้อยมากขณะกาลังใส่ห่วงหรือว่าปวดท้องน้อยในสัปดาห์แรกหลังใสห่วง film อาจจะพบ
เห็นห่วงอยู่ห่างจากตาแหน่งของมดลูก หรือถ้าไม่พบห่วงแปลว่าห่วงหายไปแล้ว หรืออาจจะใส่ uterine sound
เข้าไปด้วยแล้ว film ก็จะช่วยยืนยันตาแหน่งที่ห่วงทะลุออกไปได้ค่ะ แต่ในผู้ป่วยรายนี้ ไม่ได้สงสัยมดลูกทะลุ
นอกจากนี้ยังไม่รู้จะท้องหรือเปล่า ดังนั้นไม่ควร X-ray อย่างแรง

การเอาห่วงออก : คือมันก็ไม่ค่อยมีเล่มไหนเขียนชัดเจนอ่ะน่ะ แต่เท่าที่หาให้ได้อ่ะน่ะจะเอาออกเมื่อ


- PID ชัดเจน มีการติดเชื้อที่ปีกมดลูกชัดเจน ควรถอดห่วงออกทันที และรักษาด้วยยาปฏิชีวนะ
- การติดเชื้อ actinomycoses ถ้ามีอาการ หรือตรวจพบฝีหนองในอุ้งเชิงกรานควรถอดห่วงทันที แต่
ถ้าไม่มีอาการ แต่ตรวจพบจา pap smear ของสตรีที่แข็งแรงดี แนะนาให้ถอดห่วง และรอจนกว่าจะ
เช็ค pap smear แล้วปกติจึงใส่ห่วงทองแดงใหม่
- ห่วงทุกชนิดที่ทะลุมดลูกควรเอาออก
- การตั้งครรภ์ที่มีห่วงอยู่ในโพรงมดลูก ถ้าเห็นหางห่วงควรถอดออก ในรายที่ไม่สามารถถอดห่วง
ออกได้ง่ายๆๆ (เช่น ท้อง GA เยอะๆ ไม่เห็นหางห่วงแต่ U/S ยังมีห่วงในโพรงมดลูก) ควรพิจารณา
ทาแท้งรกษา เนื่องจากมีความเสี่ยงสูงต่อการเกิด septic abortion
การใส่ IUD ใหม่ : ทาในรายที่ต้องการใส่ห่วงและไม่มีข้อห้าม
ในผู้ป่วยรายนี้คงไม่มีใครตอบข้อนี้มั้ง เพราะว่ะ ผู้ป่วยที่ตั้งครรภ์หรือสงสัยว่าจะมีการตั้งครรภ์ถือเป็นข้อ
ห้ามต่อการใส่ห่วงในผู้ป่วยรายนี้ค่ะ (เปิดดูข้อห้ามในการใส่ห่วงในหน้า 572 นรีเวชวิทยา ของอาจารย์ธี
ระ เชียงใหม่น่ะค่ะ)
26.หญิง GA 20 wk จาก LMP ที่จาได้แน่นอน ตรวจท้องพบ fundal height สูงกว่า umbilicus ¼ จะทาอย่างไร
A) ultrasound
B) ให้ void ก่อน แล้วมาตรวจใหม่
ตอบ ให้ void ก่อน แล้วมาตรวจใหม่
การประเมินอายุครรภ์
- คลาได้ที่เหนือหัวหน่าว ~ 12 week
- ระดับ 1/3 เหนือหัวหน่าว ~ 14 week
- ระดับ 2/3 เหนือหัวหน่าว ~ 16 week
- ระดับสะดือ ~ 20 week
- ระดับ 1/4 เหนือสะดือ ~ 24 week
- ระดับ 2/4 เหนือสะดือ ~ 28 week
- ระดับ 3/4 เหนือสะดือ ~ 32 week
ในผู้ป่วยรายนี้ตรวจท้องพบ fundal height สูงกว่า umbilicus 1/4 แปลว่าอายุครรภ์ประมาณ 24 week จาก
LMP ประเมินได้ 20 week แปลว่ามีขนาดมดลูกโตกว่าอายุครรภ์ ซึ่งถ้าตรวจพบว่ามีความแตกต่างกันเกิน 2 week
จะต้องหาสาเหตุ
สาเหตุที่ขนาดมดลูกโตกว่าอายุครรภ์ ( size>date )
- คานวณผิดพลาด - twin
- full bladder - polyhydramnios
- เนื้องอกมดลูกและรังไข่ - ทารกตัวใหญ่
- ครรภ์ไข่ปลาอุก
ในผู้ป่วยรายนี้มี size>date เกิน 2 week ต้องหาสาเหตุ ซึ่ง full bladder ก็เป็นสาเหตุให้เกิด size>date ได้ จึง
ควรให้ผู้ป่วยปัสสาวะก่อนแล้วค่อยมาตรวจใหม่ แต่ถ้าหลังปัสสาวะแล้วยังพบว่า size>date ก็ควรทา ultrasound
เพื่อหาสาเหตุ
สาเหตุที่ขนาดมดลูกเล็กกว่าอายุครรภ์ ( size<date )
- คานวณผิดพลาด - ทารกโตช้าในครรภ์ ( IUGR )
- ทารกเสียชีวิตในครรภ์ - oligohydramnios

27.หญิงตั้งครรภ์ severe preeclampsia จะทาอย่างไร


A) urgent C/S
B) obsererve อาการ
ตอบ urgent C/S
ผู้ป่วย severe preeclampsia มีโอกาสชักได้สูง ควรป้องกันการชักโดยการใช้ magnesium sulfate ภายหลังให้
magnesium sulfate 4-6 hr. ทาให้การตั้งครรภ์สิ้นสุดลงโดยเร็วโดยไม่คานึงถึงอายุครรภ์ ในกรณีที่ผู้ป่วยเจ็บครรภ์
และสามารถคลอดได้อย่างรวดเร็ว ควรพิจารณาให้คลอดทางช่องคลอด
ในกรณีที่ผู้ป่วยไม่เจ็บครรภ์หรือเจ็บครรภ์แต่ไม่น่าจะคลอดได้อย่างรวดเร็วหรือมีข้อบ่งชี้ในการผ่าท้องทา
คลอดอื่นๆ ควรพิจารณาให้คลอดโดยการผ่าท้องทาคลอด
ในกรณีอายุครรภ์น้อยกว่า 34 week อาจพิจารณาการให้ corticosteroid เพื่อเร่งความพร้อมของปอดทารกหาก
ไม่มีข้อบ่งห้าม และให้คลอดภายหลัง 48 hr. ได้โดยต้องมีการเฝ้าระวังภาวะแทรกซ้อนอย่างใกล้ชิดในสถาบันที่มี
ความพร้อมทั้งเครื่องมือและอุปกรณ์

28. Pt. pregnancy BP 140/90 mmHg, urine alb 1+ จงให้การวินิจฉัย


A) mild preclamsia
ตอบ mild preclamsia
ความแตกต่างระหว่าง mild และ severe preeclampsia
สิ่งที่ตรวจพบ Mild preeclampsia Severe preeclampsia
ความดันโลหิต น้อยกว่า 160/110 mmHg ตั้งแต่ 160/110 mmHgขึ้นไป
โปรตีนในปัสสาวะ น้อยกว่า 2 gm/d มากกว่า 2 gm/d
(dipstick 1+ หรือ +) (dipstick 3+ หรือ 4+)
ปวดศีรษะ ไม่มี มี
ตามัว ไม่มี มี
จุกแน่นลิ้นปี่ ไม่มี มี
Oliguria (<500ml/24hr) ไม่มี มี
ชัก ไม่มี มี (eclampsia)
Serum creatinine ปกติ สูงผิดปกติ
เกร็ดเลือด ปกติ ต่ากว่า 100,000 ต่อ มม.3
Liver enzyme ผิดปกติเล็กน้อย ผิดปกติชัดเจน
ทารกโตช้าในครรภ์ ไม่มี มี
Pulmonary edema ไม่มี มี
การแบ่งชนิดของภาวะความดันโลหิตสูงในขณะตั้งครรภ์
1. Chronic hypertension คือ ความดันโลหิตสูง ซึ่งพบก่อนการตั้งครรภ์ หรือ ก่อนอายุครรภ์ 20สัปดาห์ หรือ
ภาวะความดันโลหิตสูงยังคงอยู่นานกว่า 12 สัปดาห์หลังคลอด
2. Gestational hypertension (Transient hypertension) คือ ความดันโลหิตตั้งแต่ 140/90 mmHg ซึ่งพบเป็นครั้ง
แรกขณะตั้งครรภ์ หลังอายุครรภ์ 20 สัปดาห์ โดยไม่มีโปรตีนในปัสสาวะ (proteinuria) เมื่อติดตามจนหลัง
คลอด 12 สัปดาห์ ความดันจะต้องลดลงเป็นปกติ
3. Preeclampsia คือ ความดันโลหิตตั้งแต่ 140/90 mmHg ซึ่งพบเป็นครั้งแรกขณะตั้งครรภ์ และ มีโปรตีนใน
ปัสสาวะ (300 mg ต่อ 24 ชม. หรือ urine protein โดย random specimen ตั้งแต่ 30 มิลลิกรัมต่อเดซิลิตร หรือ
distick ตั้งแต่ 1+)
4. Eclampsia คือ ภาวะชักซึ่งหาสาเหตุอื่นไม่พบ ในผู้ป่วย preeclampsia
5. Superimposed preeclampsia คือ ภาวะ preeclampsia ซึ่งเกิดขึ้นกับผู้ป่วยที่มีภาวะ chronic hypertension การ
วินิจฉัยภาวะ superimposed preeclampsia ในผู้ป่วยที่ไม่มีโปรตีนในปัสสาวะอยู่เดิม ใช้เกณฑ์การตรวจพบ
โปรตีนในปัสสาวะ 300 mg ต่อ 24 ชั่วโมง สาหรับผู้ป่วยที่มีโปรตีนในปัสสาวะอยู่เดิม ใช้เกณฑ์การตรวจพบ
โปรตีนในปัสสาวะสูงขึ้นอย่างชัดเจน หรือความดันโลหิต systolic สูงขึ้น 30 mmHg หรือความดันโลหิต
diastolic สูงขึ้น 15 mmHg หรือเกร็ดเลือดน้อยกว่า 100,000 ลูกบาศก์มม.
29. แม่ท้อง 7 เดือน ครรภ์ที่ 3 ลูกทั้ง 2 คนแรก มีประวัติเป็น Asthma จะแนะนาแม่อย่างไรเกี่ยวกับการป้องกัน
ไม่ให้เป็น Asthma ในครรภ์นี้
1. กินนมแม่เท่านั้น
2. กิน soy formular milk
3. กิน Hydrolysated extend casein formular
4. ห้ามให้แม่กินนมหรืออาหารที่มีส่วนผสมโปรตีนในนมวัว
5. รอลูกคลอด แล้วตรวจ Serum IgE เพื่อวางแผนการรักษาต่อไป

30. แม่เป็น TB เลี้ยงลูกด้วยนมแม่ ลูกไม่มีอาการ Tuberculin test ได้ 4 mm ควรทาอย่างไร?


1. รักษา TB
2. INH prophylaxis 1 month
3. Observe chest X ray อีก 6 เดือน
4. ...
ตอบ INH prophylaxis 1 month
การดูแลทารกแรกคลอด : รายที่แม่ได้รับยามาจนควบคุมโรคได้นานอย่างน้อย 3 สัปดาห์ สามารถเลี้ยงดู
สัมผัสหรือให้นมลูกได้ แต่รายที่เป็น active หรือได้ยาไม่นานพอ ควรให้ INH Prophylaxis แก่ทารก
10 mg./kg/day และให้วัคซีนตอนแรกเกิดร่วมด้วย โจทย์ลูกไม่มีอาการ Tuberculin test ได้ 4 mm แสดงว่า ไม่เป็น
TB และไม่ได้บอกว่าแม่รักษาด้วยยาหรือไม่ จึงคิดว่าน่าจะให้ INH Prophylaxis
ถ้าไม่ได้รับการรักษาทารกที่คลอดจากมารดาที่มีโรค active มีโอกาสติ 50% ในปีแรก

31. หญิง 21 ปี LMP 6 wk. มาด้วยเลือดออกทาง vagina ปวดท้องกลางท้องน้อย PV: uterine normal size, soft, no
adnexal mass, not tender, TVS : no intrauterine sac, no mass, Mx?
A) D&C
B) Serun ß-hCG
C) Exploratory laparotomy
D) Diagnostic laparotomy
E) MTX injection
ตอบ B) Serun ß-hCG

หลักสาคัญในการวินิจฉัย
ต้องนึกถึง ectropic pregnancy ในหญิงเจริญพันธุ์ทุกรายที่มีอาการปวดท้องน้อยเฉียบพลันร่วมกับมี
เลือดออกทางช่องคลอดตลอดเวลา ไม่มีอาการจาเพาะที่ชัดเจน
Clinical triad ที่สาคัญ
- ขาดระดู
- มีเลือดออกทางช่องคลอด
- คลาได้ก้อนที่ปีกมดลูก
ดังนั้น จากอาการจึงคิดถึง ectropic pregnancy มากที่สุด
Serun ß-hCG ช่วยในการวินนิจฉัยได้ ถ้าเป็นลบจะ R/O ได้
การวัด ß-hCG เป็นระยะช่วย สองครั้งห่างกัน 48 ชั่วโมง ปกติควรเพิ่มขึ้นไม่ต่ากว่า 66 %
ถ้าผิดปกติจะเพิ่มน้อยกว่านี้

36. หญิง 30 ปี ตั้งครรภ์ 3 เดือน มีอาการปวดแสบลิ้นปี่ เคย EGD พบ DU จะให้ยาใด


1. Omeprazole 4. Ranitidine
2. Misoprostol 5. Cimetidine
3. Sucralfate
Pregnancy category A สามารถใช้ได้อย่างปลอดภัย เนื่องจากมีการศึกษาในหญิงตั้งครรภ์แล้วพบว่ายาไม่มี
อันตรายต่อทารกในครรภ์ ทั้งไตรมาสแรกและตลอดอายุครรภ์
Pregnancy category B ปลอดภัย – มีการศึกษาว่าไม่ก่อให้เกิดพิษกับทารกในครรภ์ของสัตว์ทดลอง แต่ยังไม่มี
การศึกษาทดลองในมนุษย์
Pregnancy category C พบว่ามีอันตรายต่อทารกในครรภ์ของสัตว์ทดลอง แต่ยังไม่มีการศึกษทดลองในหญิง
ตั้งครรภ์
Pregnancy category D มีรายงานว่าเป็นอันตรายกับทารกในครรภ์ของมนุษย์ แต่เป็นยาที่จาเป็นต้องใช้ เช่น ยาที่ใช้
ช่วยชีวิต หรือไม่สามารถใช้ยาตัวอื่นในการรักษาได้
Pregnancy category X พบการรายงานว่ามีอันตรายต่อทารกในครรภ์ทั้งในสัตว์ทดลองและมนุษย์
meprazole = C Ranitidine = B ( จากผลการทดลองต่างๆพบว่า
Misoprostal = X ranitidine ปลอดภัยกว่า cimetidine )
Sucralfate = B Cimetidine = B
การรักษา DU จะต้องใช้ยากลุ่ม PPI หรือ H2 blocker ดังนั้นข้อนี้ควรตอบ Ranitidine
(sucralfate ไม่ช่วยยับยั้งการหลั่งกรด)

By. Kwang’sha 47490479


37. หญิงอายุ 30 ปี GP1021 GA 38 wk in labor จึง Drip Syntocinon ไป ต่อมาวัด BP ได้ 80/50 mmHg คลา Fetal
part ได้ชัดเจนแต่ฟัง Fetal heart sound ไม่ได้ จง Dx.
1. Abruptio placentae
2. Placenta previa
3. Uterine rupture
4. Vasa previa
5. Prolapse cord

Placenta previa หมายถึง ภาวะที่รกเกาะอยู่ใกล้กับปากมดลูก หรือขวางปากมดลูก อาการ มีเลือดออกทางช่อง


คลอดเป็นพัก ๆ โดยไม่มีอาการปวดเจ็บในท้องแต่อย่างไร และมดลูกนุ่มเป็นปกติ โดยมากมักจะเกิดเมื่อครรภ์ได้
7 เดือนขึ้นไป ถ้าเป็นไม่มาก เลือดอาจออกเล็กน้อย และหยุดไปได้เอง และทารกสามารถคลอดตามปกติได้
แต่ถ้ารกเกาะต่ามาก หรือขวางปากมดลูก อาจทาให้ตกเลือดมาก อาจทาให้ผู้ป่วยช็อก หรือทารกในครรภ์ขาด
ออกซิเจนตายในท้องได้

Abruptio placentae รกลอกตัวก่อนกาหนด หมายถึง ภาวะที่รกลอกหลุดจากผนังมดลูกก่อนกาหนดการคลอด


อาการ ผู้ป่วยจะมีอาการปวดท้อง และท้องเกร็งแข็งเป็นพัก ๆ คล้ายการคลอดบุตร อาจมีเลือดออกทางช่องคลอด
เล็กน้อยหรือไม่มีเลือดออกให้เห็น แต่ผู้ป่วยจะซีด ตัวเย็น เหงื่อออก ชีพจรเร็ว ความดันตกใช้เครื่องฟังตรวจเสียง
หัวใจของทารกในท้องจะไม่ได้ยิน - GA 38 week แล้วจึงไม่คิดถึง
Uterine rupture อาการปวดและกดเจ็บบริเวณเหนือหัวหน่าว / มดลูกหยุดการหดรัดตัวโดยทันทีภายหลังจากที่
มีการหดรัดตัวอย่างรุนแรง / ไม่สามารถฟังเสียงการเต้นของหัวใจทารกได้ / ส่วนนาของทารก (ศีรษะหรือก้น
หรือส่วนแรกที่จะคลอดออกมา) ลอยสูงขึ้น / มีเลือดออกทางช่องคลอด หลังจากนั้นจะตามมาด้วยอาการแสดง
ของภาวะช็อค ได้แก่ชีพจรเต้นเร็ว ความดันต่า เนื่องจากมีการเสียเลือดมีเลือดออกในช่องท้อง  คลา fetal part
ได้ชัดเจนน่าจะเป็น key word คาตอบข้อนี้ เนื่องจาก พอมี uterine rupture ทาให้ส่วนนาของทารก ซึ่งควรจะ
engage ไปแล้ว ลอยขึ้นมา ก็เลยคลาได้ชัดเจนขึ้น

Vasa previa ภาวะที่มีเส้นเลือดขอดบนถุงน้าคร่า ถ้าถุงน้าคร่าแตก ก็ทาให้เส้นเลือดขอดนี้แตกด้วย เกิดเลือดออก


อันตรายต่อชีวิตได้
ตอบ uterine rupture By. Kwang’sha
38. ผู้หญิงอายุ 40 ปี G2P1 GA 8 wk ความเสี่ยงในการเป็น Down syndrome จะเป็นเท่าไร
1. 1/50 4. 1/300
2. 1/100 5. 1/400
3. 1/200

Incidence of Down Syndrome with Increasing Maternal Age


Maternal Age at Delivery Risk at Term
32 1/725
33 1/592
34 1/465
35 1/365
36 1/287
37 1/255
38 1/177
39 1/139
40 1/109
41 1/85
42 1/67
43 1/53
44 1/41
45 1/32
46 1/25
47 1/20
48 1/16
49 1/12

เฉลย ก็ 1/100 นะ by.kwnag’sha


38. ผู้หญิงอายุ 40 ปี G2P1 GA 8 wk ความเสี่ยงในการเป็น Down syndrome จะเป็นเท่าไร
a.1/50 d.1/300
b.1/100 e.1/400
c.1/200

ตอบ b.1/100

39. ผู้ป่วยหญิง 38 ปี G4P3 GA 39 wk, no ANC, 2hr PTA มีมูกเลือดออกจากช่องคลอด เมื่อมาถึง รพ.มีน้าเดิน,
contraction 5’ 30”, PV: 7cm,station -2 คลาได้ก้น เท้า และสายสะดือ Management อะไร
a.Observe d.Total breech extraction
b.ให้ syntocinon e.Emergency C/S
c.Partial breech extraction
ตอบ e.Emergency C/S
คลาได้สายสะดือ แสดงว่ามีภาวะสายสะดือย้อย ซึ่งพบบ่อยในทารกที่มีท่าผิดปกติ เช่น ท่าก้น ท่าขวาง
การรักษา : เนื่องจากเป็นภาวะที่วิกฤติต่อทารก หลักการคือ ทาให้ส่วนนาของทารกกดสายสะดือน้อยลง
และท่าคลอดทารกอย่างเร็วที่สุดเท่าที่ท่าได้
1. การช่วยเหลือแบบฉุกเฉิน
- จัดท่ามารดาให้อยู่ในท่าที่ช่วยป้องกันส่วนนาไม่ให้ลงมากดสายสะดือ ให้ก้นสูง เช่น ท่า trendelenberg
, knee-chest
- ใช้มือดันส่วนนาของทารกเพื่อให้กดสายสะดือน้อยลง
- oxygen 100%
- ถ้าสายสะดือยื่นออกมาภายนอกช่องคลอด ให้ใส่สายสะดือกลับเข้าไปในช่องคลอด เนื่องจากถ้าสาย
สะดือถูกความเย็นของอากาศภานนอกทันทีอาจทาให้สันเลือดสายสะดือหดตัวได้
- ทาให้กระเพาะปัสสาวะโป่งตึง โดยการใส่น้าเกลือ 500-700 มล. ทางสายสวนปัสสาวะ เชื่อจะช่วยดัน
ส่วนนาของทารกและลดความรุนแรงของการหดรัดตัวของมดลูก
2. การคลอด
- c/s เป็นวิธีที่ดีที่สุด ยกเว้นกรณีที่ทารกสียชีวิตหรือมีความผิดปกติแต่กาเนิด
- Breech extraction ในกรณีที่เป็นท่าก้น ปากมดลูกเปิดหมด ไม่มีการผิดสัดส่วนของทารกกับช่องเชิง
กราน
- ใช้คีมช่วยคลอด ถ้าปากมดลูกเปิดหมด ส๊รษะลงมาต่าพอควร
ดังนั้นในผ็ป่วยรายนี้จึงไม่ท่า Breech extraction เนื่องจากปากมดลูกยังเปิดไม่หมด
Ref สูติศาสตร์ มช. , จุฬา

40. ผู้ป่วยหญิงครรภ์ 8 wk มีโรคประจาตัวเป็น Mitral stenosis มีร่างกายแข็งแรง อาการเหนื่อยมีอยู่เฉพาะช่วง


ออกกาลังกาย จะพิจารณาต่อไปอย่างไร
a. ตั้งครรภ์ต่อและให้ คลอดตามปกติ
b. ตั้งครรภ์ต่อ …………………
c. ตั้งครรภ์ต่อ แต่ให้ไปฝากครรภ์ที่ Tertiary care
d. หยุดตั้งครรภ์เนื่องจากอันตรายต่อแม่
e.หยุดตั้งครรภ์เนื่องจากอันตรายต่อลูก

ตอบ a. ตั้งครรภ์ต่อและให้ คลอดตามปกติ


การจาแนกความรุนแรงของโรคหัวใจ ตาม NYHA
Class I ทางานได้ตามปกติโดยไม่รู้สึกเหนื่อย อาการเหนื่อยจะเป็นขณะทีออกแรงหรือออกกาลังกาย
Class II สบายดีขณะพัก แต่ถ้าทางานปกติจะรู้สึกเหนื่อย
Class III สบายดีขณะพัก แต่ถ้าทางานเล็กน้อยจะรู้สึกเหนื่อย
Class IV หอบเหนื่อยขณะพัก

ในผู้ป่วยรายนี้จัดอยู่ใน Class I
การดูแลรักษาใน Class I Class II
1. การดูแลรักษาทั่วไป
i. ควรดูแลร่วมกันระหว่างสุติแพทย์และแพย์ทางโรคหัวใจ
ii. ลดภาวะตึงเครียด
iii. มุ่งการรักษาโดยให้ความสาคัญที่มารดาเป็นหลักแต่ใรขณะที่รักษาควรหลีกเลี่ยงภัย
อันตรายที่อาจเกิดขึ้นกับทารกให้มากที่สุด
iv. ลดการออกกาลังกายรุนแรงที่อาจทาให้เกิดอาการหอบเหนื่อย
v. ลดอาหารเค็มลงบ้าง แต่ไม่ต้องจากัดเกลือมากนัก
vi. หลีกเลี่ยงการใช้ยาโดยไม่จาเป็นควรงดดื่มสุราและสูบบุหรี่
vii. พักผ่อนอยางน้อย 10 ชั่วโมงในตอนกลางคืน และนอนพักครึ่งชั่วโมงหลังอาหาร
viii. ฝากครรภ์ในหน่วยครรภ์เสี่ยงสูงบ่อยกว่าปกติ
 28 wk แรกทุก 2 wk
 ต่อไปตรวจทุก 1 wk ควรประเมินการทางานของหัวใจทุกครั้งและตรวจหา
อาการของหัวใจล้มเหลวด้วย
2. สตรีกลุ่มนี้ให้ตั้งครรภ์ได้ เน้นระวังหัวใจล้มเหลว หรือการดาเนินโรคที่รุนแรงขึ้น ซึ่งทาให้
เปลี่ยน class ที่รุนแรงมากขึ้นได้อัตราการตายของมารดา 0.4%
3. ตรวจติดตามสุขภาพทารกในครรภ์
4. การคลอด : ควรรับไว้ในโรงพยาบาลก่อนคลอด และให้คลอดทางช่องคลอด การผ่าตัดควรถือ
ตามข้อบ่งชี้ตามสูติศาสตร์

การดูแลรักษาใน Class III


ถ้าผู้ป่วยต้องการมีบุตรอย่างมาก ทั้งตัวผู้ป่วยเองและครอบครัวจะต้องเข้าใจอันตรายที่เกิดขึ้น
ประมาณ 1 ใน 3 ของผู้ป่วยกลุ่มนี้จะเกิดภาวะล้มเหลวได้ขณะตั้งครรภ์ถ้าไม่ได้รับการดูแลป้องกัน
ถ้ามาฝากครรภ์ในไครมาสแรก จะพิจารณาถึงความต้องการลูกของผู้ป่วย ถ้าไม่ต้องการแนะนา
ให้ทาแท้งรักษา ถ้าต้องการลูกจะต้องนอนพักในดรงพยาบาลตลอดการตั้งครรภ์ ซึ่งถ้าได้รับการดูแลรักษา
อย่างดีแล้วพบว่าอัตราการตายของมารดาไม่แตกต่างจากสตรีตั้งครรภ์ทั่วไป

การคลอด : ให่คลอดทางช่องคลอด ผ่าตักเมื่อมีข้อบ่งชี้ทางสูติกรรมเท่านั้น


การดูแลรักษาใน Class IV
ให้การดูแลรักษาแบบผู้ป่วยหัวใจล้มเหลวเลย เพราะอัตราการเสียชีวิตของมารดาสูงมาก ควรมุ่ง
การรักษาที่มารดาเป็นอันดับแรก
Class III IV อัตราการตาย 4-7 %

Ref สูติศาสตร์ มช.

40. เด็กอายุ 7 วัน ไข้ ซึม ร้องเวลาจับเต้านมขวา มีน้านมไหลตั้งแต่อายุ 2 เดือน มารดาบีบออกทุกวัน , PE:
lethargy, moderate jaundice, erythematous indurated mass at Rt. Breast, ควรให้ ATB ?
1. Ceftriaxone
2. Ampicillin +Gentamicin
3. Cloxacillin + Gentamicin
4. Fortum + Amikacin
5. Fortum + Vancomycin
ตอบ 2. Ampicillin +Gentamicin
common antibiotic regimen in infants with suspected sepsis is a beta-lactam antibiotic
(usually ampicillin) in combination with an aminoglycoside (usually gentamicin) or a third-
generation cephalosporin (usually cefotaxime—ceftriaxone is generally avoided in neonates
due to the theoretical risk of causing biliary stasis.) The organisms which are targeted are
species that predominate in the female genitourinary tract and to which neonates are
especially vulnerable to, specifically Group B Streptococcus, Escherichia coli, and Listeria
monocytogenes

41. ผู้ป่วยเด็กชาย ได้ยา ampicillin มา 14 วัน หลังจากนั้นมีไข้ ท้องเสีย ถ่ายเหลว ทา endoscope


พบว่ามี hyperemia Tx?
1. Metronidazole
2. Ceftriaxone

โจทย์ไม่ละเอียด แต่คิดว่าน่าจะเป็น Ampicillin-Associated Diarrhea.โรคท้องเสียที่เกี่ยวข้องกับการ


ใช้ยาปฏิชีวนะ หรือที่เรียกว่า antibiotic-associated diarrhea ยาปฏิชีวนะที่พบว่าเป็นสาเหตุ ได้แก่
clindamycin, ampicillin ,cephalosporins , cephalexin
การกินยาปฏิชีวนะ สมดุลของร่างกายจะเปลี่ยนไป เนื่องจากยาปฏิชีวนะ จะทาลาย
แบคทีเรียที่เป็นจุลชีพเฉพาะถิ่นในลาไส้ ยาปฏิชีวนะจะทาลายเชื้อแบคทีเรียไปเป็นจานวนมาก และ
ก่อให้เกิดภาวะเสียสมดุลของจุลชีพในลาไส้ ส่วนใหญ่ผลที่เกิดขึ้นจะเป็นแค่อาการเพียงเล็กน้อย
ผู้ป่วยมีอาการถ่ายเหลว ซึ่งอาการถ่ายเหลวจะหายไปหลังจากหยุดยาปฏิชีวนะไม่นาน แต่ก็มี
บางครั้งที่ยาปฏิชีวนะทาลายเชื้อแบคทีเรียไปมาก จนกระทั่งทาให้เชื้อแบคทีเรียที่สาคัญชนิดหนึ่ง มี
ชื่อเรียกว่า Clostridium difficile การตรวจทางห้องปฏิบัติการอาจพิจารณาตรวจหาเชื้อ C. difficile
หรือตรวจแอนติบอดี้ต่อเชื้อ
ในกรณีที่อาการรุนแรง พิจารณาให้ยาฆ่าเชื้อแบคทีเรีย C. difficile เช่น metronidazole
(Flagyl) หรือ vancomycin (Vancocin)
42. เด็กชาย อายุ 8 ปี เมื่อก่อนแข็งแรงดี มีประวัติเป็นหวัด 1 PTA, มีจ้าเลือดตามตัวและเลือดออก
ตามไรฟัน 2 wk PTA, vital sign ปกติ, PE : Oozing per gum, petichiae & ecchymosis ที่ trunk &
extremities, no lymphadenopathy, no hepatosplenomegaly, Dx?
1. Hemophilia
2. Acute leukemia
3. DHF
4. ITP
5. APDE
ตอบ 4. ITP = Idiopathic thrombocytopenic purpura
I. Idiopathic thrombocytopenic purpura แบ่งออกเป็น
- Primary autoimmune immune thrombocytopenia (ITP)
Acute childhood ITP
chronic or adult ITP
- Secondary Autoimmune thrombocytopenia
Acute post viral ITP(acute childhood ITP)
- พบในเด็กอายุน้อย
- เกิดตามหลังการติดเชื้อไวรัส หรือการฉีดวัคซีน
- เกิดรวดเร็วภายใน2สัปดาห์
- หายได้เองภาย6เดือน
Chronic ITP
- พบในวัยเจริญพันธุ์
- พบในผู้หญิงมากกว่าผู้ชาย
- อาการค่อยเป็นค่อยไป
- มักเป็นโรคเรื้อรัง
- โอกาสหายเองน้อย
อาการทางคลินิก chronic ITP
- Skin bleeding –petechiae,ecchymoses
- Mucosal bleeding-bleeding per gum,epistaxis,hematuria
- Intra cerebral hemorrhage
- Anemia related to degree of bleeding
- No hepatosplenomegaly if splenomegaly was detected should be considered hypersplenism
or lymphoproliforative disorders
Criteria for diagnosis ITP
- Isolated thrombocytopenia
- Normal or increase megakaryocyte in bone marrow
- No splenomegaly
- Autoantibody to platelets
- No other cause of thrombocytopenia
Treatment
1. General treatment
- Bleeding precaution avoid trauma,Intramuscular injection,antiplatelets medication
- Local treatment;nasal packing,oral contraceptive drug
- Blood transfusion in active bleeding,iron replacement therapy,platelets transfusion in life
thretening situations
2. Specific treatment
Corticosteroid actions:- Inhibit autoantibody bind to platelets
- decrease phargocytosis by RE cell,
- decrease autoantibody production,
- increase vascular support
Indication for splenectomy
- Steroid relaspe =steroid dependent
- Stroid partial and non response
 Diagnosis and treatment over 6 weeks with platelets less than 10,000
 Treatment for 3 months and platelets less than 30,000 with or without bleeding
- Complication of streroid
- Ememegency condition
- Difficulties in follow up
73. เด็กชายอายุ 6 ปี แม่พามาเนื่องจากอ้วนมี truncal obesity , Buffalo hump , moon face , acne ,
ตัวเตี้ย จะตรวจเพิ่มเติมพบอะไร
A. Hypocalcemia D. Increase serum cortisol
B. Hyperkalemia E. Increase activity of rennin
C. Advance bone age

ตอบ D. Increase serum cortisol


ผู้ป่วยเด็กรายนี้มีอาการของ Cushing’s syndrome ซึ่งเป็นโรคที่มีระดับของ serum
cortisol สูงอยู่เป็นเวลานาน
A. Hypocalcemia
- Cortisol มีผลเพิ่ม Osteoclastic activity  Hypercalcemia (mechanism นี้ไม่
แน่ใจเท่าไหร่)
B. Hyperkalemia
- Cortisol มีฤทธิ์เป็น weak aldosterone  ดูด Na กลับ ขับ K ออก 
Hypokalemia
C. Advance bone age
- Serum cortisol สูง  กด Hypothalamus, Pituitary  GH, GnRH, TRH ที่มี
ผลต่อ Bone growth ต่าลง  Delay bone age
E. Increase activity of rennin
- Cortisol มีผลให้ความดันสูงขึ้น  Decrease activity of rennin
74. เด็กหญิงอายุ 6 ปี มีจ้าเลือดตามแขนขามา 2 สัปดาห์ ตรวจร่างกายพบ petechiae & ecchymosis
at extremities CBC : Hb 12 Hct 36 WBC 6500 Plt.150,000 ผล Lab ใดจะผิดปกติ
A. Bleeding time D. Thrombin time
B. PT E. Venous clotting time
C. PTT
ตอบ A. Bleeding time
petechiae & ecchymosis เป็น sign ของ Primary hemostasis ซึ่งเป็นความผิดปกติของ
Platelet และ Vessel
ในโจทย์ข้อนี้ จานวน platelet ปกติ แต่อาจมีปัญหาที่ function ก็ได้
A. Bleeding time ตรวจ primary hemostasis , platelet function
B. PT ตรวจ extrinsic and common pathway
C. PTT ตรวจ intrinsic and common pathway
D. Thrombin time ตรวจ fibrinogen – fibrin conversion
E. Venous clotting time ตรวจ clotting factor (Prolong if clotting factor < 5%)

เฉลย by อัง Med NU X รพ.มน.รุ่น 1


75. เด็กอายุ 2 ปี ผ่าตัด VSD หลังผ่าคลา pulse ไม่ได้ วัด BP ไม่ได้ทันที ได้บีบ AMbu ผ่าน tube
และให้ Amiodalone แล้วมี EKG เป็น ventricular tachycardia จงmanagement
A. Defibilation
B. Ca gluconate
C. NaHCO3
D. Adenosine
E. Direct – current synchronized cardioversion
ตอบ E. Direct – current synchronized cardioversion
Protocol for management of haemodynamically unstable ventricular tachycardia

The steps used in the resuscitation procedure are outlined:

 provide high flow oxygen


 establish i.v. access
 consider sedation or ideally, intubation with general anaesthesia
 perform cardioversion i.e. synchronised DC shocks
o for a monophasic defibrillator, start at 100 J

o if VT persists, increase to 200 J

o if VT persists, increase to 360J

o (if using a biphasic defibrillator, use the equivalent biphasic energy levels)

 check serum potassium and correct if less than 4 mmol/l:


o give up to 60 mmol potassium chloride at maximum rate of 30 mmol per hour

 if hypokalaemic, assume patient also to be hypomagnesaemic:


o give 10 mmol magnesium sulphate over 30 minutes (5 mls of 50% solution)

 if VT persists, obtain expert help; but consider:


o amiodarone 150 mg in 20 mls 5% dextrose over 10 minutes

o further cardioversion

o other anti-arrhythmics

o overdrive pacing

Ref : http://www.gpnotebook.co.uk/simplepage.cfm?ID=-1523253221&linkID=35230&cook=no
เฉลย by อัง Med NU X รพ.มน.รุ่น 1
76. เด็กชายอายุ 19 ปีมาพบแพทย์หลังจากไปถอนฟันกับทันตแพทย์ แล้วเลือดไหลซึมไม่หยุด
ตรวจตับและม้ามไม่โต ตรวจเลือด PTT 50 sec (control 30 sec), PT 13 sec (control 14 sec), no
prolonged bleeding time สาเหตุของการเกิดโรคนี้คืออะไร
A. Lupus coagulopathy D. von Willebrand factor
B. Factor VII deficiency deficiency
C. Factor VIII deficiency E. Hereditary platelet dysfunction
เฉลย C. Factor VIII deficiency
สาเหตุที่ทาให้ prolonged PTT ได้แก่ lupus coagulopathy, factor VIII or IX deficiency
or inhibitor, Von Willebrand disease (if factor VIII is decrease), factor XI or Xii deficiency,
prekallikrein or HMW kinigen def. or inhibitor, heparin contamination of sample นะ เพราะฉะนั้น
ตอนนี้จะเหลือคาตอบที่น่าจะเป็นไปได้คือ Lupus coagulopathy, Factor VIII deficiency และ von
Willebrand factor deficiency
มาดูกันต่อว่า prolong PTT ก็จริง แต่ก็ไม่ได้ทาให้เกิด bleeding เสมอไปนะ อาจเกิด
thrombosis ก็ได้ สาหรับ prolong PTT แล้วทาให้เกิด bleeding นั้น ได้แก่ heparin, factor VIII/ IX/
XI def./ihb. และ vWD ส่วน prolong PTT แล้วทาให้เกิด thrombosis นั้น ได้แก่ factor XII
def./ihb., lupus coagulopathy และ contact factor def./inh. เพราะฉะนั้นตอนนี้จะเหลือคาตอบแค่
Factor VIII deficiency และ von Willebrand factor deficiency
คราวนี้ก็มาถึงข้อควรรู้ คือ vWF จะมีหน้าที่หลัก 2 อย่างคือ Plt. Adhesion membrane
receptor และ carrier factor VIII ดังนั้นใน vWD จะให้ผล lab เป็น prolong bleeding time (เพราะ
function plt. เสียไป), reduced factor VIII activity (ก็จะได้ prolong PTT), reduction plasma vWF
concentration และ reduction biologic activity นั่นเอง เพราะฉะนั้นตอนนี้ก็เหลือคาตอบเพียงข้อ
เดียวแล้ว
ก็ตอบ Factor VIII deficiency หรือว่า hemophilia A แหละ โดยมีข้อสังเกตก็คือ เป็นเพศ
ชาย อาการ bleeding หลังถอนฟัน lab มี PTT prolong อย่างเดียวอะนะ พวกนี้ถ้ารักษาก็ได้ factor
VIII conc. นะ

77. เด็ก 3 ปี เหนื่อยง่าย มี mild pale other WNL หนัก 13.5 kg , blood smear : hypochromic
microcytic anemia
A. Iron deficiency
สาหรับข้อนี้ ไม่รู้จะเฉลยยังไง เพราะมีมาตัวเลือกเดียว ซึ่งจากโจทย์ที่มีมาเท่านี้ จะ
เลือกตอบข้อนี้ก็ไม่
ผิดนะ
คิดว่าโจทย์ข้อนี้เค้าคงอยากให้แปลผล blood smear เป็นอ่ะ ก็เลยจะเอาลักษณะ RBC
morphology ของ anemia จากสาเหตุต่างๆ มาให้นะ
Classification of anemia based on morphology
1. normocytic normochromic anemia  miscellaneous rare disorder : Vit B6
 acute blood loss def., sideroblastic anemia
 hemolytic anemia 4. macrocytic normochromic anemia
 hemoglobin- erythrocyte mass def. :  pernicious anemia
chronic disease, toxic agent  GI abnormalities : malabsorption
2. microcytic normochromic anemia syndromes, inflammatory disease,
 hemoglobin- erythrocyte mass def. : acute and chronic liver disease
chronic disease, toxic agent,  Def. of Vit B12, folic, Vit C
malignancy, splenomegaly, endocrine  Bone marrow failure or hypoplasia :
disorders aplastic anemia, uremia
3. microcytic hypochromic anemia  Anticonvulsant drugs and
 iron deficiency anemia folic acid antimetabolites
 chronic lead poisoning
 thalassemia

2. ผู้ป่วยชายไทยอายุ 47 ปี มีก้อนยุบๆ บวมๆ ที่ขาหนีบ เป็นๆ หายๆ มา 3 ปี ครั้งนี้มาพบแพทย์ที่


โรงพยาบาล ผู้ป่วยมาด้วย ก้อนบวมและโตมากขึ้น กดเจ็บมากและ ดันก้อนกลับไม่ได้ตั้งแต่ 5
ชั่วโมงก่อนมาโรงพยาบาล การปฏิบัติในข้อใดเหมาะสมที่สุดในผู้ป่วยรายนี้
a. Analgesic d. Surgery
b. Try reducing e. Reassure and plan discharge
c. Observe and plan admit
เฉลย b. Try reducing=จากโจทย์ที่ให้มาอาการน่าจะเข้าได้กับ irreducible (incarcerated) hernia
เนื่องจากไม่สามารถยุบกลับเข้าได้หรือถูกส่วนคอดของถุงรัดอยู่ ผู้ป่วยก็จะมีการปวดได้เล็กน้อย
การรักษาก็ให้ try reducing แต่ถ้าเป็น obstructed หรือ strangulated hernia พวกนี้จะเกิดขึ้นเมื่อ
ลาไส้ส่วนคอดของถุงรัดจนเกิดการตีบตัน ทาให้มีอาการปวดมวนท้อง คลื่นไส้อาเจียน ถ้าเป็นมา
นานจะไม่ผายลมหรือไม่ถ่ายอุจจาระและท้องจะอืด ภาวะ strangulation เกิดขึ้นเมื่อเส้นเลือดใน
mesentery ถูกรัด เส้นเลือดดามักถูกรัดก่อน ทาให้ลาไส้บวมและมีสีคล้าจากเลือดที่คั่งค้างอยู่
ภายในผนังลาไส้ ถ้าไม่ได้รับการรักษาเส้นเลือดฝอยและเส้นเลือดแดงจะเกิดอุดตันในระยะต่อมา
และลาไส้จะเกิดinfact ตามด้วย gangrene ผู้ป่วยจะมีอาการ toxic ปวดตรงก้อนไส้เลื่อนมาก และ
ปวดตลอดเวลา การรักษาก็ให้ surgery เลยนะ
llllllllllllllllllllllllllllllllllll 83.333%Loading Medicineนสพ.เกรียงศักดิ์จาปาวงค์5603

DON_1 ) หญิงอายุ 45ปี มาตรวจสุขภาพประจาปี ตามหลักประกันสุขภาพแห่งชาติ ข้อใดสามารถตรวจได้โดยไม่เสีย


ค่าใช้จ่าย

a. CXA
b. CBC
c. Fasting Blood Glucose
d. Mammogram
e. Total cholesterol

สิทธิประโยชน์ในการรับบริการสาธารณสุขตามหลักประกันสุขภาพคับ
llllllllllllllllllllllllllllllllllll 83.333%Loading Medicineนสพ.เกรียงศักดิ์จาปาวงค์5603

การตรวจสุขภาพ (Periodic Health Examination )


ระดับคาแนะนา
“ก” มีหลักฐานวิชาการสนับสนุน ว่า ดี คุ้มค่า ตรวจเลยได้ประโยชน์แน่ๆ
“ข” มีหลักฐานที่พอเชื่อได้ +ผู้เชี่ยวชาญสนับสนุน ว่า ก็ทาสิ มีประโยชน์ดี
“ค” ยังหาข้อสรุปไม่ได้ ว่าทาแล้วจะเป็นไง ดี –ไม่ดี
“ง” มีหลักฐานวิชาการสนับสนุน ชัดเจนว่า ทาแล้ว มีผลเสีย ก็อย่าทานะ

การตรวจทางคลินิกและห้องปฏิบัติการสาหรับผู้ใหญ่ที่ไม่มีความเสียงใดๆ
ระดับคาแนะนา ชายไทยไม่มี risk ใดๆ หญิงไทย ไม่มี risk ใดๆ

ก. ควรทา 1. นน. / สส. q. 1ปี

2. BP age 18 -35 q. 2 y
…..age >35 q. 1ys
เหมือนชาย
3. วัดสายตา age >40 q. 1 y
เพิ่ม
4. TT q. 10 y

1.Vaccine MMR 1 ครั้ง

2. Papsmear q.1 y (เคยมีSI ) if


ปกติ 3 ครั้ง ให้ q.3 y

3. ตรวจเต้านมด้วยตนเองทุกเดือน

( age > 20 )

ข. น่าทา 1. PR (> 40y) q.3-5y

2. Stool exam q.3-5y

3. Stool occult blod (>40y) q.5y Add  ตรวจเต้านมโดยแพทย์

4. U/A q.3-5y
(20-40y ) q. 3y
ระหว่าง TC กับ FBS ตรวจ 5. Total cholsterol(>35y) q.3-5y
( >40y) q.1y
TC ก่อน FBS 6. TG,HDL(>45) q.3-5y
นอกนั้นเหมือนชาย
(TC 35 ปี) (FBS 45ปี) 7. FBS(>45) q.3y

8.HBV vaccine 1 ชุด


llllllllllllllllllllllllllllllllllll 83.333%Loading Medicineนสพ.เกรียงศักดิ์จาปาวงค์5603

ค. อาจทา หรือ ไม่ทา 1.วัดความดันลูกตา เหมือนชาย

2.Hct,Hb เพิ่มจากชาย
3.HBsAg
1. T4,TSH (menopause)
4.VDRL
2. Bone density
(menopuase)
5.สารเสพติดในปัสสาวะ

6.CXR

7.Chol age<35y

8.FBS age< 45y

9.EKG,EST

10.Conoscope age >40y

11.Hb typingก่อนแต่งงาน

12.วัคซีนอีสุกอีใส

13.วัคซีนตับอักเสบเอ

ง. ไม่ควรทา / ห้ามทา 1.TB ( CXR, tuberculin test) 1.TB ( CXR, tuberculin test)

2. T4,TSH 2. Hb typing

3.Tumor marker 3.Tumor marker

-AFP(CA liver) -AFP(CA liver)

-CA 19-9 (CA Pancrease) -CA 19-9 (CA Pancrease)

-CEA ( CA colon) -CEA ( CA colon)

-PSA ( CA Prostate) -CA 125 ( CA Ovary)

4. UGI study 4. UGI study

5.Gastro scope 5.Gastro scope

6.U/S Liver, Pancrese 6.U/S Liver, Pancrese,pelvic

7.Transrectal U/S ( CA Prostate)

8.Bone density 7.Bone density

9.EKG,EST (< 40y) 9.EKG,EST (< 40y)


llllllllllllllllllllllllllllllllllll 83.333%Loading Medicineนสพ.เกรียงศักดิ์จาปาวงค์5603

10. BCG ซ้า 9. BCG ซ้า

11.วัคซีนไข้หวัดใหญ่ 10.วัคซีนไข้หวัดใหญ่

12.วัคซีนเชื้อปอดบวม 11.วัคซีนเชื้อปอดบวม

13.กินยาป้องกัน TB,Malaria,ไข้กาฬ 12..กินยาป้องกัน TB,Malaria,ไข้


หลังแอ่น กาฬหลังแอ่น

จากโจทย์จะเห็นว่าการตรวจตามคาแนะนา ระดับ “ข” มีทั้งสองchoice คือ TC กับ FBC แต่ TC จะ early screening
ก่อน FBS ตามอายุที่ได้กล่าวไป ดังนั้นหากจะเลือกตอบอายุ 45 ปี เปะๆ น่าจะเลือก TC ก่อน FBS( >45)

แต่บอกตามตรงว่าอันไหนฟรี นั้น ไม่ทราบคับ ชออภัยด้วย แต่แต่จากหลักฐานตามสิทธิประโยชน์ในการรับบริการ


สาธารณสุขตามหลักประกันสุขภาพแล้วบอกว่า
การตรวจสุขภาพประชาชนทั่วไปและกลุ่มเสี่ยง ตามแนวทางที่กาหนดนั้น ฟรีหมด ก็ assume ว่าน่าจะฟรี ทั้ง TC และ
FBS
หากตามแนวทางนี้ อายุ 45 ปี ถือว่าการตรวจ TC ( >35y) ฟรี แน่นอนอยู่แล้ว
ข้อนี้ผมจึงขออนุญาต แสดงความคิดเห็นตอบ TC คับ

DON_2) พนักงานโรงงาน 2000 คน เป็น TB 200 คน ทา CXR ผิดปกติ 150 คน คนที่ไม่เป็น TB มี CXR ผิดปกติ 30 คน
คนที่ CXR ผิดปกติมีโอกาสเป็น TB เท่าได

A.3

B.7

C.15

D.27

E.ข้อมูลไม่เพียงพอ

คนที่ไม่เป็น TB แต่ มี CXR


ผิดปกติ 30 คน
TB Non-TB total
ทา CXR ผิดปกติ 150
+ ve CXR 120 30 150 คน
llllllllllllllllllllllllllllllllllll 83.333%Loading Medicineนสพ.เกรียงศักดิ์จาปาวงค์5603

- ve CXR 80 1770 1850

total 200 1800 2000

พนักงานโรงงาน 2000 คน
เป็น TB 200

คนที่ CXR ผิดปกติมีโอกาสเป็น TB เท่าได  ถามถึง positive predictive value (PPV)

ดังนั้นในรายนี้
llllllllllllllllllllllllllllllllllll 83.333%Loading Medicineนสพ.เกรียงศักดิ์จาปาวงค์5603

= (120 / 150) x 100 = 80% แต่ไม่ทราบว่าเหตุผลใดใน choice ไม่มี=> โจทย์ผิด / choiceผิด หรือผมผิด ?=>ไม่ทราบ
llllllllllllllllllllllllllllllllllll 83.333%Loading Medicineนสพ.เกรียงศักดิ์จาปาวงค์5603

อันนี้เอามาแถมไหนๆก้อ่านสถิติแล้วเน้อเลยให้เผื่อไว้คับ (บางท่านอาจข้ามเลยก็ได้คับไม่ว่ากัน)
llllllllllllllllllllllllllllllllllll 83.333%Loading Medicineนสพ.เกรียงศักดิ์จาปาวงค์5603

DON_3) ในหอผู้ป่วยวิกฤตแห่งหนึ่ง พบว่าผู้ป่วยมี Nosocomial infection 20% มาตราการต่อไปนี้ ข้อใดจะป้องกันได้ดี


ที่สุด

A.เปลี่ยนเสื้อเวลาตรวจผู้ป่วย
B.เปลี่ยนรองเท้าเวลาตรวจผู้ป่วย
C.ถูพื้นทุก 6 ชั่วโมง
D.ล้างมือทุกครั้งก่อน – หลังสัมผัสผู้ป่วย
E. ใส่หมวกหน้ากากทุกครั้งที่สัมผัสผู้ป่วย

เพื่อนคงตอบได้กันหมดนะคับข้อนี้ ว่าตอบ ข้อ D.ล้างมือทุกครั้งก่อน – หลังสัมผัสผู้ป่วยแต่จะให้ข้อมูลเสริมอีกนะ


คับเพื่อความมั่นใจเป็นทวีคูณ Through its program, the hospital’s infection-control committee must determine the
general and specific measures used to control infections and must review and recommend specific
antiseptics and disinfectants for hospital use. Given the prominence of cross-infection, hand hygiene is the
single most important preventive
measure in hospitals (Table 116-2).
Health care workers’ rates of adherence to
hand-hygiene recommendations are
abysmally low (_50%). Reasons cited
include inconvenience, time pressures, and
skin damage from frequent washing.
Sinkless alcohol rubs are quick and highly
effective and actually improve hand
condition since they contain emollients and
allow the retention of natural protective oils
that are removed with repeated rinsing. Use
of alcohol hand rubs between patient
contacts is now recommended for all health
care workers except when the hands are
visibly soiled, in which case washing with
soap and water is still required
คิดถึงนะ

เธอคนนั้น
llllllllllllllllllllllllllllllllllll 83.333%Loading Medicineนสพ.เกรียงศักดิ์จาปาวงค์5603

4. นักเรียนหญิงอายุ 19 ปี บิดาไปทางานค้าขายต่างจังหวัด มารดาเสียชีวิตไปแล้ว ปัจจุบันอาศัยอยู่กับพี่ชายอายุ 20 ปี


และน้าชายอายุ 30 ปี คุณครูประจาชั้นพาเด็กนักเรียนหญิงคนดังกล่าวมาด้วยเรื่องคลาได้ก้อนที่ท้องน้อย ไม่ปวด แพทย์
วินิจฉัยว่าเป็นถุงน้าที่รังไข่ จาเป็นต้องเข้ารับการรักษาด้วยการผ่าตัด อยากทราบว่าใครเป็นผู้มีอานาจตัดสินใจในการ
ยินยอมให้ผ่าตัด

A. บิดา
B. พี่ชาย
C. น้าชาย
D. ตัวผู้ป่วยเอง
E. คุณครูประจาชั้น
ตอบ ผู้ป่วย จากคาประกาศสิทธิผู้ป่วยข้อ 10

คาประกาศสิทธิผู้ป่วย

1. ประชาชนทุกคน มีสิทธิพื้นฐาน ที่จะได้รับบริการด้านสุขภาพ ตามที่บัญญัติไว้ในรัฐธรรมนูญ


2. ผู้ป่วยมีสิทธิที่จะได้รับบริการ จากผู้ประกอบวิชาชีพด้านสุขภาพ โดยไม่มีการเลือกปฏิบัติ เนื่องจากควา
แตกต่างด้านฐานะ เชื้อชาติ สัญชาติ ศาสนา สังคม ลัทธิการเมือง เพศ อายุ และลักษณะของความ
เจ็บป่วย
3. ผู้ป่วยที่มาขอรับบริการด้านสุขภาพ มีสิทธิที่จะได้รับทราบข้อมูลอย่างเพียงพอ และเข้าใจชัดเจน จากผู้
ประกอบวิชาชีพด้านสุขภาพ เพื่อให้ผู้ป่วย สามารถเลือกตัดสินใจ ในการยินยอมหรือไม่ยินยอม ให้ประกอบ
วิชาชีพด้านสุขภาพปฏิบัติต่อตน เว้นแต่เป็นการช่วยเหลือรีบด่วนหรือจาเป็น

4. ผู้ป่วยที่อยู่ในภาวะเสี่ยงอันตรายถึงชีวิต มีสิทธิที่จะได้รับการช่วยเหลือรีบด่วน จากผู้ประกอบวิชาชีพด้าน


สุขภาพโดยทันที ตามความจาเป็นแก่กรณี โดยไม่คานึงว่า ผู้ป่วยจะร้องขอความช่วยเหลือหรือไม่

5. ผู้ป่วยมีสิทธิที่จะได้รับทราบ ชื่อ สกุล และประเภท ของผู้ประกอบวิชาชีพด้านสุขภาพ ที่เป็นผู้ให้บริการแก่ตน

6. ผู้ป่วยมีสิทธิที่จะขอความเห็น จากผู้ประกอบวิชาชีพด้านสุขภาพอื่น ที่มิได้เป็นผู้ให้บริการแก่ตน และมีสิทธิ


ในการขอเปลี่ยนผู้ให้บริการ และสถานบริการได้

"บุคคลย่อมเสมอกันในกฎหมาย และได้รับความคุ้มครอง ตามกฎหมายโดยเท่าเทียมกัน ….ฯลฯ"

7. ผู้ป่วยมีสิทธิ ที่จะได้รับการปกปิดข้อมูลเกี่ยวกับตนเอง จากผู้ประกอบวิชาชีพด้านสุขภาพโดยเคร่งครัด เว้นแต่


จะได้รับความยินยอม จากผู้ป่วยหรือการปฏิบัติหน้าที่ตามกฎหมาย สาหรับผู้ประกอบวิชาชีพเวชกรรม (แพทย์)
llllllllllllllllllllllllllllllllllll 83.333%Loading Medicineนสพ.เกรียงศักดิ์จาปาวงค์5603

8. ผู้ป่วยมีสิทธิ ที่จะได้รับทราบข้อมูลอย่างครบถ้วน ในการตัดสินใจเข้าร่วม หรือถอนตัว จากการเป็นผู้ถูก


ทดลอง ในการทาวิจัย ของผู้ประกอบวิชาชีพด้านสุขภาพ

9. ผู้ป่วยมีสิทธิ ที่จะได้รับทราบข้อมูล เกี่ยวกับการรักษาพยาบาล เฉพาะของตน ที่ปรากฏในเวชระเบียน เมื่อร้อง


ขอ ทั้งนี้ข้อมูลดังกล่าว ต้องไม่เป็นการละเมิดสิทธิส่วนตัว ของบุคคลอื่น

10. บิดา มารดา หรือผู้แทนโดยชอบธรรม อาจใช้สิทธิแทนผู้ป่วย ที่เป็นเด็ก อายุยังไม่เกินสิบแปดปี


บริบูรณ์ ผู้บกพร่องทางกาย หรือจิต ซึ่งไม่สามารถใช้สิทธิด้วยตนเองได้

พิจารณา

10.1 ที่ใช้คาว่า บิดา มารดา หมายถึง ต้องทั้งบิดาและมารดา (ทั้ง 2 คนร่วมกัน) หรือหมายถึง บิดาหรือมารดา
คนหนึ่งคนใดก็เพียงพอแล้วกันแน่ ทั้งนี้เพราะในคาประกาศสิทธิผู้ป่วย มิได้มี "_" หรือ "," ระหว่าง บิดา มารดา
จึงไม่อาจทราบได้ แต่เพื่อให้สอดคล้องกับมาตรา 28 แห่งประมวลกฎหมายแพ่งและพาณิชย์แล้ว น่าจะหมายถึง
คนหนึ่งคนใดก็ได้

10.2 เนื่องจากในข้อ 10 นี้มิได้มีการกาหนดแยกประเภท ของผู้ป่วยที่มีข้อจากัด ในการแสดงเจตนาประเภท


ต่างๆ ไว้ดังเช่นในพระราชบัญญัติข้อมูลข่าวสาร

10.3 ผู้เยาว์ในข้อกาหนด ของแพทยสภาตามประกาศฉบับนี้ ถือเอาที่อายุ 18 ปีลงมา ซึ่งอาจไม่ตรงกับตาม


ประมวลกฎหมายแพ่งและพาณิชย์ ที่บัญญัติไว้ที่อายุต่ากว่า 20 ปี (เพราะบรรลุนิติภาวะเมื่ออายุ 20 ปี) นั่น
หมายความว่า องค์กรวิชาชีพด้านสุขภาพ ซึ่งรวมถึงแพทยสภา ยอมรับการใช้สิทธิ ของบุคคลในทางการแพทย์
คือ การให้ความยินยอม ในทางการแพทย์ ในบุคคลที่มีอายุมากกว่า 18 ปีขึ้นไป ซึ่งโดยความจริงแล้ว บุคคลอายุ
ที่ยังมีอายุไม่ถึง 20 ปี ยังไม่ถือว่าบรรลุนิติภาวะ จึงยังคงถือว่าเป็นผู้เยาว์ ความยินยอมให้กระทาการอันหนึ่งอัน
ใดของผู้เยาว์ อาจทาให้การนั้นไม่ชอบด้วยกฎหมายได้ เว้นเสียแต่การ ที่ตามกฎหมายแพ่งและพาณิชย์ ได้
บัญญัติยกเว้น ไว้ให้ผู้เยาว์สามารถทาได้ตามมาตรา 22 ถึง 27

มาตรา 15

"……….ให้บุคคลตามที่กาหนด ในกฎกระทรวงมีสิทธิดาเนินการ ตามมาตรา 23, มาตรา 24 และมาตรานี้แทน


ผู้เยาว์ คนไร้ความสามารถ คนเสมือนไร้ความสามารถ หรือเจ้าของข้อมูล ที่ถึงแก่กรรมแล้วได้"

แต่ในข้อ10 นี้ใช้เพียง "ผู้แทนโดยชอบธรรม" เท่านั้น ดังนั้นผู้แทนโดยชอบธรรม ในที่นี้จึงน่าจะหมายถึง ผู้แทน


โดยชอบธรรม ตามความหมายอย่างกว้าง กล่าวคือ หมายถึง

10.2.1 ผู้แทนโดยชอบธรรม ตามมาตรา 21 แห่งประมวลกฎหมายแพ่งและพาณิชย์ คือผู้แทนโดยชอบธรรม


สาหรับผู้เยาว์เท่านั้น
llllllllllllllllllllllllllllllllllll 83.333%Loading Medicineนสพ.เกรียงศักดิ์จาปาวงค์5603

10.2.2 ผู้แทนโดยชอบธรรม

ก. ตามความเป็นจริง สาหรับ "ผู้ไร้ความสามารถ" หรือ "ผู้เสมือนไร้ความสามารถ"

ข. ตามกฎหมาย สาหรับ "คนไร้ความสามารถ" หรือ "คนเสมือนไร้ความสามารถ" ซึ่งเป็นกรณี ที่ศาลได้สั่งแล้ว


จึงมี "ผู้อนุบาล" และ "ผู้พิทักษ์" ในการเป็นผู้ดูแลตามลาดับ ตามมาตรา 34 และ มาตรา 32 ตามลาดับ

รายละเอียด ในการให้ความยินยอม ในส่วนนี้ ดูได้จากบทความ ที่ผู้เขียนได้เขียนไว้แล้ว แต่ถ้าไม่ตีความ เป็น


เรื่องผู้แทนโดยชอบธรรมอย่างกว้างแล้ว ผู้แทนโดยชอบธรรม จะเป็นเพียงผู้แทนโดยชอบธรรม ตามมาตรา 21
เท่านั้น ก็จะเกิดปัญหาของบุคคลไร้ความสามารถ และบุคคลเสมือนไร้ความสามารถ ทั้งที่ศาลได้สั่งแล้ว และยัง
มิได้สั่งว่าจะดาเนินการอย่างไร ซึ่งจะเกิดปัญหากับทางสถานพยาบาลอย่างมาก ในการปฏิบัติตามข้อ 10 แห่ง
คาประกาศสิทธิผู้ป่วย

10.4 เรื่องการใช้สิทธิแทนนี้เป็นการใช้สิทธิแทน 2 ประการ

10.4.1 การใช้สิทธิ ในเรื่องการรักษาพยาบาล หรือดาเนินการทางการแพทย์ต่างๆ เช่น ผ่าตัด การ


รักษา การบาบัด การตรวจโดยวิธีพิเศษ ฯลฯ ตามข้อ 3 ของคาประกาศสิทธิผู้ป่วย

10.4.2 การใช้สิทธิในเรื่องเอกสารแทนผู้ป่วย โดยเฉพาะตามข้อ 9 ของคาประกาศสิทธิผู้ป่วย

อาจดูยาวไปหน่อยแต่จะได้อ่านทุกข้อใช้ตอบข้อสอบจริงนะ

5. การทดสอบ RCT ผลการทา chemo กับการผ่าตัด กระบวนการสุ่มแยกผู้ป่วยเป็น 2 กลุ่ม เพื่อ ?

A. เพิ่ม........
B. ลด........
C. เพิ่มอานาจในการหาความแตกต่าง
D. กระจายตัวแปรที่มีผลต่อการศึกษาเท่าๆกัน
ตอบ ลด confounding factor

วิธีดาเนินงานวิจัยจาแนกตาม
llllllllllllllllllllllllllllllllllll 83.333%Loading Medicineนสพ.เกรียงศักดิ์จาปาวงค์5603

Cross-sectional
(Prevalence)
การวิจัยเชิงทดลอง
(Experimental Research)
Longitudinal
ผู้วิจัยกาหนด Exposure (Incidence)
การวิจัยเชิงพรรณนา
(Descriptive Study)

ไม่มกี ลุม่ เปรียบเทียบ ณ จุดเวลาใดเวลาหนึ่ง


การวิจัยโดยการสังเกต
(Cross-sectional)
(Observational Research)

Exposure เกิดตามธรรมชาติ
การวิจัยเชิงวิเคราะห์ ชนิดไปข้างหน้า
(Analytic Study) (Cohort or Prospective)

มีกลุม่ เปรียบเทียบ
ชนิดย้อนหลัง
(Case control or
Retrospective)

การวิจัยโดยการทดลอง (experimental research) เป็นการวิจัยที่ผู้วิจัยมีการกาหนดปัจจัยเสี่ยงหรือกาหนดสิ่ง


แทรกแซงที่ต้องการให้กับตัวอย่างที่นามาศึกษา ผู้วิจัยติดตามดูผลที่จะเกิดขึ้นจากการได้รับปัจจัยเสี่ยงหรือสิ่ง
แทรกแซงนั้นไปข้างหน้าในอนาคต
1.การวิจัยแบบทดลองเต็มรูป (full experimental study) หรือการวิจัยแบบทดลองที่แท้จริง (true experimental
study) หรือการวิจัยแบบทดลองคลาสสิก เป็นการวิจัยที่ผู้วิจัยกาหนดสิ่งทดลองหรือสิ่งแทรกแซงที่ต้องการ
ทดสอบ แล้วจัดให้กลุ่มทดลองได้รับสิ่งที่ต้องการทดสอบนั้น แล้วติดตามดูผลและเปรียบเทียบผลที่ได้จากกลุ่ม
ทดลองกับกลุ่มควบคุม ซึ่งเป็นผลที่กลุ่มควบคุมที่มีคุณสมบัติอื่นเหมือนกลุ่มทดลอง นอกจากไม่ได้รับสิ่งที่
ต้องการทดสอบนั้น
การวิจัยโดยการทดลองเป็นรูปแบบการวิจัยที่เชื่อถือได้มากที่สุด เพราะสามารถหลีกเลี่ยงอคติ( confounding
facter)ที่อาจเกิดขึ้นให้เหลือน้อยที่สุด จึงนิยมใช้ในการประเมินประสิทธิภาพและประสิทธิผลของสิ่งที่เรา
ต้องการทดสอบ การวิจัยโดยการทดลองที่ดีที่สุดนั้น คือการวิจัยโดยการทดลองที่มีการควบคุมที่ดี มีการแบ่งตัว
อย่างโดยการสุ่ม และทั้งผู้วิจัยและกลุ่มศึกษาไม่ทราบว่าตัวอย่างอยู่ในกลุ่มไหน(กลุ่มทดลองหรือกลุ่มควบคุม)
ซึ่งเรียกการวิจัยแบบนี้ว่า การวิจัยโดยการทดลองแบบควบคุมสองชั้น ( randomized double blind control
trial = RCT )
1.1 classical experimental study มีการแบ่งกลุ่มตัวอย่างโดยวิธีสุ่มออกเป็นกลุ่มทดลองและกลุ่มควบคุม ซึ่งมี
llllllllllllllllllllllllllllllllllll 83.333%Loading Medicineนสพ.เกรียงศักดิ์จาปาวงค์5603

ลักษณะต่างๆเหมือนกันมากที่สุดก่อนการทดลอง ทาการวัดผลทั้ง 2 กลุ่มก่อนการทดลอง (pre-test) แล้ววัดผล


หลังการทดลอง (post-test) ทั้ง 2 กลุ่มอีกครั้ง วิธีนี้ทาให้ทราบถึงระดับการเปลี่ยนแปลงระหว่างก่อนและหลัง
การทดลอง บางทีเรียกว่าการวิจัยแบบทดลองที่วัดทั้งก่อนและหลังการทดลองโดยมีกลุ่มควบคุม (before–after
experimental with control group หรือ pretest–posttest control group study)
1.2after-only experimental with one control group หรือ posttest-only control group study) มีการ
แบ่งกลุ่มโดยวิธีสุ่ม (random allocation) ให้ตัวอย่างในกลุ่มทดลองและกลุ่มควบคุมมีลักษณะต่างๆเหมือนกัน
มากที่สุด แล้ววัดผลเฉพาะหลังการทดลองเท่านั้น ไม่มีการวัดผลก่อนให้สิ่งทดลอง
2.การวิจัยแบบกึ่งทดลอง (quasi-experimental study) เป็นการวิจัยที่มีการทดลอง แต่ขาดคุณสมบัติประการ
ใดประการหนึ่งของคุณสมบัติ 4 ข้อของการวิจัยแบบทดลองเต็มรูป
คุณสมบัติ 4 ข้อของการวิจัยแบบทดลองเต็มรูป ประกอบด้วย
(1) มีกลุ่มควบคุม (control group) คือกลุ่มที่มีลักษณะต่างๆที่จะมีผลต่อการทดลองเหมือนกลุ่มทดลอง
(experimental group) ยกเว้นการได้รับสิ่งทดลองหรือสิ่งแทรกแซง ( intervention) ที่ผู้วิจัยเป็นผู้กาหนดขึ้น ทั้งนี้
เพื่อให้แน่ใจว่าผลที่ต่างกันระหว่างกลุ่มทดลองและกลุ่มควบคุม เป็นผลจากการได้รับสิ่งทดลองหรือสิ่งแทรกแซง
ไม่ใช่จากลักษณะที่ต่างกันของประชากรก่อนได้รับสิ่งทดลองที่ต้องการทดสอบ
(2) มีการติดตามไปข้างหน้าเพื่อเปรียบเทียบดูผลของสิ่งที่ต้องการทดลองที่ผู้วิจัยเป็นผู้กาหนดขึ้น และกาหนด
ให้กับกลุ่มทดลอง แล้วนาไปเปรียบเทียบกับกลุ่มควบคุม
(3) การจัดกลุ่มทาโดยวิธีสุ่ม (random allocation) ทาให้ตัวอย่างที่นามาศึกษามีโอกาสเท่ากันในการ ที่จะถูก
กาหนดให้ไปอยู่ในกลุ่มทดลองหรือกลุ่มควบคุม
(4) มีการควบคุม โดยมีการเปลี่ยนแปลงหรือควบคุมธรรมชาติบางอย่าง หรือการใส่สิ่งแทรกแซง เพื่อให้สามารถ
ตอบปัญหา

(ref .Research Designผศ.(พิเศษ)น.พ.นภดล สุชาติ และ. ชีตbiases of researchของ อ. ศุภสิทธิ์)


llllllllllllllllllllllllllllllllllll 83.333%Loading Medicineนสพ.เกรียงศักดิ์จาปาวงค์5603

6. ในการทาวิจัย ทาการศึกษา BP เด็กแต่ใช้ cuff adult มาวัด ถามว่าเป็นผิดพลาดชนิดใด

A. Bias
B. Confounder
C. Contamination
D. Co-intervention
ตอบ information bias

การวิจัยจะมีความคลาดเคลื่อนได้

ผลการวิจัย = ความจริง(truth) + ความคลาดเคลื่อน(errors)

ชนิดความคลาดเคลื่อน แบ่งเป็น

- ความคลาดเคลื่อนเชิงระบบ(systematic errors) หรือ อคติ (bias)


- ความคลาดเคลื่อนแบบสุ่ม(random errors or variations)  แก้โดยใช้ sample size ที่พอเพียง
ชนิดอคติในการวิจัย

1. Selection bias = การคัดเลือกกลุ่มที่มาเปรียบเทียบที่ไม่ยุติธรรม เกิดจาก


- ความผิดพลาดที่เกิดจากขั้นตอนการคัดเลือกประชากรที่มาศึกษาที่ไม่เหมาะสม
- มีการคัดเลือกประชากรที่เข้ามาในการศึกษาที่ไม่เท่าเทียมกันของกลุ่มที่นามาเปรียบเทียบทั้งสอง ซึ่ง
การเลือกนั้นไปเกี่ยวข้องกับ สถานะของการเป็น case การเป็น control สถานะของการมี หรือไม่มี
ปัจจัยที่ต้องการศึกษา
- ข้อบกพร่องของการเลือก study design
- การ loss follow up
- non-response
- selective survival =เลือกคนที่เหลืออยู่
2. Information bias = การวัดค่าของ exposure หรือ outcome ที่ไม่ยุติธรรม เกิดจาก
- invalid measurement=เลือกเครื่องมือไม่ถูกต้องเข้าได้กันการเลือก cuff BPไม่ถูก
- incorrect diagnostic criteria
-omission/ imprecision of record data= การละเลยการบันทึกข้อมูล
-unequal diagnostic surveillance= การ follow upต่างกัน
llllllllllllllllllllllllllllllllllll 83.333%Loading Medicineนสพ.เกรียงศักดิ์จาปาวงค์5603

3.Confounding bias =ปัจจัยซ่อนเร้นหรือปัจจัยร่วม (confounding factor) หมายถึง ปัจจัยที่


เกี่ยวข้องหรือปัจจัยที่มีอิทธิพลต่อผลการศึกษา เช่น การสูบบุหรี่ เป็นปัจจัยเสี่ยงของการเกิดโรคมะเร็ง
ปอด แต่มลภาวะในอากาศ โรคปอดที่มี อายุ ก็มีผลต่อการเกิดมะเร็งปอดเช่นเดียวกัน ดังนั้นใน
การศึกษาผลของมะเร็งปอดจากการสูบบุหรี่ จะมี อายุ มลภาวะในอากาศ และ โรคปอดของผู้ถูก
ทดลอง เป็นปัจจัยร่วม
(ref. ชีตbiases of researchของ อ. ศุภสิทธิ์และการคุมอคติ (Control Bias)ดร. อรพิน กฤษณเกรียง
ไกร
คณะสาธารณสุขศาสตร์ มหาวิทยาลัยนเรศวร)
By… Toie

7. แพทย์อธิบายให้ผู้ป่วยฟังเกี่ยวกับวัตถุประสงค์และที่มาของโครงการ ผลดีผลเสีย เพื่อให้ผู้ป่วยตัดสินใจเข้าร่วม


โครงการด้วยความสมัครใจเอง แพทย์ทาตามจริยธรรมใด

A. Beneficence
B. Autonomy
C. Confidentiality
ตอบ Autonomy

เวชจริยศาสตร์ทางคลินิก

1.การทาแต่ความดี (beneficence) คือ ดูแลผู้ป่วยตามมาตรฐาน , ตัดสินใจแทนในกรณีที่ผู้ป่วยไม่สามารถตัดสินใจ


เองได้

2.การไม่ทาความชั่ว (non-maleficence) ความชั่ว ได้แก่ การละเลยผู้ป่วย,การได้ค่าตอบแทน,การโฆษณา,การทา


แท้ง,การเขียนใบรับรองแพทย์เท็จ

3.เสรีภาพส่วนบุคคล(personal autonomy)

- consent คือ การยินยอมรับการรักษาหรือการวิจัย โดยประกอบด้วย 3 ขั้นตอน คือ แพทย์ต้องให้ข้อมูลที่


เพียงพอและเหมาะสม (appropriateness) , ผู้ป่วยต้องสามารถรับรู้ข้อมูลได้ (capacity), และ ต้องเป็นการ
ตัดสินใจด้วยความสมัครใจของผู้ป่วยเอง ( voluntariness)

- confidentiality คือ การรักษาความลับของผู้ป่วย

4.ความยุติธรรม (justice)

5. การปล่อยให้ผู้ป่วยถึงแก่กรรม (Euthanasia)

Reference: หนังสือการสัมภาษณ์ประวัติและตรวจร่างกาย โครงการตาราจุฬา เล่มดา


llllllllllllllllllllllllllllllllllll 83.333%Loading Medicineนสพ.เกรียงศักดิ์จาปาวงค์5603

8. ถ้าในชุมชนมีคนติดยาจานวนมากจะมีแนวทางแก้ไขยังไง

A. ออกกฎหมายมาควบคุม
B. ให้คนในชุมชนมีส่วนร่วมออกกฎหมายควบคุม
C. ของบประมาณเพิ่มเพื่อการดูแลผู้ป่วย.
ตอบ B.

9. การตรวจร่างกายประจาปี พบว่า พนักงานทั้งหมด 2.000 คน ผล xray ผิดปกติ 80 คนเป็น CA ไป 50 คน แต่มี


พนักงาน 1,920 คน ผล xray ปกติแต่เป็น CA ไป 150 คน ถามว่าในคน x-ray ปกติ จะมีโอกาสเป็น CA กี่ % (จา
choice ไม่ได้) น่าจะเป็นตารางแบบนี้

CA ไม่เป็น CA รวม

X-ray ผิดปกติ 50 30 80

X-ray ปกติ 150 1770 1920

รวม 200 1800 2000

ตอบ ข้อนี้ถาม false negative (test - โดยที่ disease +)

จากโจทย์ sensitivity = TP/all disease = 50/200 = 0.25

False negative = 1- sensitivity = 0.75 หรือ 75%

By… PAR

10. มารดาอายุ 16 ปี ไม่ยอมให้ LP ลูก ถามว่าแพทย์ต้องทาอย่างไร

ตอบ - อธิบายให้มารดาผู้ป่วยเข้าใจว่าการตรวจน้าไขสันหลังมีความสาคัญในการให้การวินิจฉัย การวางแผนการรักษา


และ

การติดตามผลการรักษาในกรณีทม่ี กี ารเปลีย่ นแปลงของน้าไขสันหลัง

- จากโจทย์มารดาอายุ 16 ปีแพทย์อาจจะให้ยายผู้ป่วยหรือญาติใกล้ชิดที่มีวัยวุฒิเพียงพอรับฟังคาอธิบายด้วย
llllllllllllllllllllllllllllllllllll 83.333%Loading Medicineนสพ.เกรียงศักดิ์จาปาวงค์5603

เพิ่มเติม

ข้อบ่งชี้ในการเจาะน้าไขสันหลัง

1. เพื่อช่วยในการวินิจฉัยโรคทางระบบประสาท
2. เพือ่ การรักษา โดยการใส่ยาเข้าในน้าไขสันหลัง
3. เพือ่ การรักษา การเจาะน้าไขสันหลังจะช่วยลดอาการปวดศีรษะที่รุนแรงหรือเพื่อลดความดันในสมองในรายที่มี
communicating hydrocephalus
4. เพื่อทาการตรวจพิเศษทางระบบประสาท เช่น lumbar myelography
ข้อห้าม

1. ความดันในกะโหลกศีรษะสูง
2. ในรายทีส่ งสัยว่ามี space occupying lesion

11. การทาวิจัยโดยอธิบายให้ผู้ป่วยฟังโดยละเอียด ให้ผู้ป่วยเลือกด้วยความสมัครใจ → autonomy

ตอบ autonomy

หลักของจริยเวชศาสตร์

1. เสรีภาพส่วนบุคคล (personal autonomy)


การให้ความยินยอม/อนุญาต ในการดูแลรักษาหรือการวิจัย ควรมีการดาเนินการอันประกอบด้วย
- การให้ข้อมูล (appropriate information) ผู้ป่วยต้องได้รับข้อมูลที่เหมาะสม
และเพียงพอเพื่อการตัดสินใจในการยอมรับการรักษาหรือเข้าร่วมการวิจัย
- ความสามารถในการรับรู้ข้อมูลของผู้ป่วย ( capacity)
- เป็นการตัดสินใจด้วยความสมัครใจ ( voluntariness)
การรักษาความลับ (confidentiality)
การแจ้งความจริง (truth telling)
กริยามารยาทที่ดี
llllllllllllllllllllllllllllllllllll 83.333%Loading Medicineนสพ.เกรียงศักดิ์จาปาวงค์5603

2. การไม่ประพฤติชั่ว (non-maleficence)
3. การทาแต่ความดี (Beneficence)
4. การรักษาความลับ (confidentiality)
5. ความยุติธรรม (justice)

12. รณรงค์ชุมชนสิ่งที่สาคัญที่สุด → people empowerment

ตอบ ข้อนี้ไม่มี choice เลยหาความหมายของ empowerment มาให้นะ

Empowerment is the process of increasing the capacity of individuals or groups to make choices
and to transform those choices into desired actions and outcomes. Central to this process are actions which
both build individual and collective assets, and improve the efficiency and fairness of the organizational and
institutional context which govern the use of these asset.

By…Praew
ENT
1. ผู้หญิงอายุ 40 ปี มีไข้ เจ็บคอ กลืนเจ็บ และกลืนลาบาก ซื้อยามาทานเองไม่หาย ตรวจร่างกายพบ muffled voice
film lateral neck ด้วย soft tissue technique ได้ผลการตรวจดังภาพ การวินิจฉัยใดถูกต้องมากที่สุด
A. Epiglotitis
B. Ludwig’s angina
C. Laryngotracheitis
D. Retropharyngeal abscess
E. Parapharyngeal abscess
เฉลย
เนื่องจากโจทย์บอกให้ดูฟิล์มด้วย ประกอบกับผู้ป่วยรายนี้ไม่มี specific symptom ของโรคใดโรคหนึ่งอย่างชัดเจน อาการ
ของผู้ป่วยเป็นลักษณะอาการของภาวะ Deep neck infection ที่มักจะมีอาการแบบนี้แทบทุก space จึงขอนารายละเอียด
คร่าวๆของแต่ละ choice พร้อมความคิดเห็นส่วนตัวที่ไม่แน่ใจว่าจะถูกหรือไม่ มาเฉลยเพื่อนๆ ดังนี้
จากโจทย์ ลักษณะอาการของผู้ป่วยจะเข้าได้กับภาวะ Deep neck infection ซึ่งแทบทุกข้อก็เป็น Deep neck infection กัน
หมด ยกเว้นข้อ A. Epiglottitis และ ข้อ C.Laryngotracheitis(ซึ่งก็คือ Croup นั่นเอง) ซึ่งเป็นภาวะในกลุ่ม Infectious Upper
Airway Obstruction และมักเกิดในเด็ก
Epiglottitis ลักษณะอาการจะค่อนข้างรุนแรงและรวดเร็ว ผู้ป่วยจะมาด้วย ไข้สูง เจ็บคอ กลืนลาบาก น้าลายยืด เด็กโตอาจ
นั่งแหงนหน้าและเอนตัวไปด้านหน้าเพื่อช่วยในการหายใจ หายใจพบเสียงstridor
Laryngotracheitis ภาวะ Viral croup จะพบในเด็กอายุ 3 เดือน – 5 ปี ผู้ป่วยจะมีประวัติเป็นหวัดนามาก่อน 2 – 3 วัน
อาการเด่นๆก็จะเป็นตาม Croup score ที่เราท่องกัน เช่น ไอเสียงก้อง(barking cough) หายใจมีเสียง stridor เป็นต้น
คราวนี้มาพูดถึง Deep neck infection แต่ละโรคมั่ง…
Ludwig’s angina
คือ การติดเชื้อที่ Submandibular space ผู้ป่วยจะมาด้วยอาการเจ็บและบวมบริเวณด้านใดด้านหนึ่งก่อนแล้วมักจะ
ลามไปเป็นทั้งสองข้าง นอกจากนั้นก็เป็นอาการแบบในโจทย์ complicationที่สาคัญคือ Airway obstruction
Criteria ในการ diag Ludwig’s angina มี ดังนี้
- เป็น cellulitis เท่านั้น
- Involve ทั้ง submaxillary และ sublingual space (มักเป็นที่ submaxillary ก่อนแล้วค่อยลุกลามไป)
- Infection เป็นแบบ direct extension ไปตาม fascial planes เท่านั้น
- Involvement เฉพาะ muscle และ fascia ไม่รวม submandibular gland หรือ Lymph node
การ investigate ก็ส่ง film lateral เหมือนในโจทย์ จะเห็น ดังนี้
Retropharyngeal abscess คาดว่าน่าจะตอบข้อนี้
เป็นการอักเสบใน Retropharyngeal space อาการจะเหมือนโจทย์ คือ ไข้ ไอ เจ็บคอ เวลาหันศีรษะจะรู้สึกเจ็บ เกิด
เสียง muffled voice หรือ hot potato voice ได้(เสียงพูดไม่ชัดเหมือนอมของร้อนๆไว้ในปากแล้วพูด ลักษณะเสียงแบบนี้
สามารถเกิดใน Peritonsillar abscessก็ได้ด้วยนะครับ) ในผู้ใหญ่อาการ airway obstruction พบได้น้อยกว่าเด็ก most
common complication คือ Pneumonia with lung abscess จากการแตกของ abscess บริเวณspaceนี้แล้ว aspirate ลงปอด
การ investigate ก็ส่ง film lateral neck เพื่อดู soft tissue เหมือนกัน โดยในผู้ใหญ่ที่ระดับ C2 ไม่ควรหนาเกิน 7 mm
และระดับ C6 ไม่ควรหนาเกิน 22 mm. จะเห็นเป็นแบบนี้

Parapharyngeal abscess
มี 2 compartment
Anterior compartment ที่อยากให้จา คือ มี Classical triad คือ
1. Trismus (อ้าปากไม่กว้างไม่ได้)
2. Perimandibular swelling
3. Protusion หรือ Prolapse ของ tonsil fossa
Posterior compartment อาจมี Neurological deficit คือ Horner’s syndrome(เล็ก,ตก,แห้ง,ไม่งอก)
ภาวะนี้การinvestigationไม่ค่อยเป็นจุดเด่น ส่ง film lateral neck ไปก็อาจเห็นแค่ Epiglottis swelling เฉยๆ

จบข้อ 1. ถ้ามันเยอะไป,มีแต่น้า ก็ขอโทษเพื่อนๆด้วยที่ทาให้เสียเวลา

2.ผู้ปูวยหญิงอายุ 30 ปี มาด้วยหูขวาอื้อมา 1 วัน ผู้ปูวยต้องเดินทางโดยเครื่องบินบ่อยๆ และมักหูอื้อเวลาเครื่องบิน


ขึ้นหรือลง สักพักจะหายไปเอง 2 วันก่อน ขณะเครื่องบินลงจอดที่ลาน รู้สึกหูขวาอื้อและปวดหูมาก การได้ยินลดลง
เช้านี้ยังหูอื้ออยู่แต่ไม่ปวดหูแล้ว ตรวจ left ear drum: normal , right ear drum: dark blue, retracted ear drum, Weber
test: localized to right, Rinne test: left AC>BC, right BC>AC จงบอกการรักษาที่เหมาะสม
A. Loratadine
B. Pseudoephredine
C. Serrtic peptidase
D. Chloramphenical eardrop
E. Myringotomy
โจทย์ข้อนี้ ประวัติเด่นชัดมากว่าผู้ป่วยมีอาการผิดปกติทางหูเนื่องจาก Barotrauma
Barotrauma คือภาวะที่มีการบาดเจ็บของหูที่เกิดจากผู้ป่วยอยู่ในที่ๆมีการเปลี่ยนแปลงความดันอย่างรวดเร็ว เช่น การดา
น้า อยู่บนเครื่องบิน
ลักษณะอาการทางคลินิก ผู้ป่วยจะมีประวัติดังที่กล่าวไปแล้ว มีอาการปวดหู ได้ยินเสียงลดลง ตรวจหูจะพบว่า tympanic
membrane จะแดงและยุบตัวไปด้านในหูชั้นกลาง( retracted ear drum) เนื่องจากการเกิดภาวะนี้จะเกิดในขณะที่ความดัน
รอบๆตัวสูงกว่าความดันในหูชั้นกลางทาให้หูชั้นกลางยุบตัวเข้าไปด้านในและหากเป็นอย่างนี้นานๆโดยไม่ได้รับการรักษาก็จะ
พบhemotympanium หรือ dark blue ear drum อย่างผู้ป่วยรายนี้ได้
การรักษา
1. หลีกเลี่ยงปัจจัยเสี่ยงที่จะทาให้เกิดภาวะนี้
2. สอดสายยางเป่าลมเข้าหูชั้นกลางทาง Eustatian tube เพื่อปรับความดัน
3. รักษาตามอาการ คือ ยาลดปวด ลดอาการอักเสบ decongestant
4. หากอาการเป็นมากให้เจาะเยื่อแก้วหู (Myringotomy) เพื่อปรับความดันในหูชั้นกลาง
จากโจทย์โดยความเห็นส่วนตัวแล้วคิดว่าอาการเป็นค่อนข้างมาก น่าจะพิจารณาทา Myringotomy มากกว่าจะรักษาตาม
อาการอย่างเดียว
เกร็ดความรู้เล็กน้อย เผื่อมีคนลืม
Rinne test : AC > BC (Rinne positive) = normal , sensory neural hearing loss
Bc > AC (Rinne negative) = conductive hearing loss
Weber teat จะ lateralize ไปข้างที่มีพยาธิสภาพเป็น conductive hearing loss
จบข้อ 2 เริ่มสั้นลงเพราะขี้เกียจ อ่ะ! ล้อเล่งุ.

3. หลังจากฝึกซ้อมยิงปืน หูขวาอื้อมีเสียงดังอื้อในหู ตรวจร่างกายมี ear drum : intact , Weber : no lateralization ,


Rinne : Rt BC > AC , Lt AC>BC จงวินิจฉัย
A. Acoustic trauma D. Steatosis
B. Otosclerosis E. Otitis media
C. Meniere
Acoustic trauma
คือการบาดเจ็บของหูชั้นใน เนื่องจากเสียงดัง ไม่ว่าจะเป็นเสียงดังที่รุนแรงเพียงครั้งเดียว เช่น เสียงระเบิด แต่กรณีนั้นมักพบ
การบาดเจ็บที่หูด้วย หรือ จะเป็นการ exposed เสียงดังๆเป็นเวลานานๆ เช่น การยิงปืน เป็นหนึ่งในภาวะ Noise induce
hearing loss และเป็นการสูญเสียการได้ยินแบบ Sensory neural hearing loss
ผู้ป่วยมักมาด้วยอาการได้ยินเสียงลดลง ได้ยินเสียงสองข้างไม่เท่ากัน มีเสียงดังอื้อในหู ตรวจร่างกายหากเป็นอย่างโจทย์
รายนี้ที่ expose กับเสียงดังปานกลางแต่เป็นเวลานานๆก็จะไม่พบความผิดปกติอะไร(ก็มันบาดเจ็บที่หูชั้นในนี่ เรามองไม่เห็น
หรอก)
อาการทั้งหมดที่กล่าวมาเข้าได้กับผู้ป่วย แต่สิ่งที่งงและไม่สามารถอธิบายได้ของโจทย์ข้อนี้ คือ ทาไมถึงมีภาวะ BC > AC
ได้ เพราะ Acoustic trauma เป็นการสูญเสียการได้ยินแบบ SNHL ก็ควรจะ AC > BC แต่เมื่อไปพิจารณา choice ข้ออื่นแล้วก็
เห็นว่า ข้อ A. นี่แหละน่าจะใช่ที่สุดแล้ว จึงคิดว่าน่าจะตอบข้อนี้ ขออภัยกับความไม่ฉลาดของตัวเองจริงๆครับ
มาดู Choice อื่น คร่าวๆกันบ้าง
Otosclerosis
ภาวะนี้เป็นโรคที่มีการสร้างกระดูกขึ้นมาใหม่ที่บริเวณ labyrinth capsule มีการสูญเสียการได้ยินแบบ conductive
hearing loss เพราะมีการยึดติดกันมากขึ้นของกระดูก stapes’ กับ oval window สาเหตุการเกิดยังไม่ทราบแน่นอน แต่เชื่อว่า
กรรมพันธุ์มีส่วนเกี่ยวข้อง อาการก็เป็นแบบ conducive hearing loss แต่จะเป็นมากขึ้นเรื่อยๆ จริงๆผู้ป่วยอาจจะมีความ
ผิดปกติมานานแล้วแต่เนื่องจากอาการที่progressค่อนข้างช้า ทาให้ผู้ป่วยส่วนใหญ่เพิ่งสังเกตความผิดปกติได้และมักจะมาหา
หมอเมื่อเข้าสู่วัยกลางคนแล้ว
Meniere’s disease
หรือ ภาวะ idiopathic endolymphatic hydrops เป็นความผิดปกติของหูชั้นในที่มีการสะสมของ endolymph และทาให้
membranous labyrinth โป่งพอง อาการจะมาด้วย ลักษณะแบบ sensory neural hearing loss แบบเป็นๆหายๆ และมักจะมี
อาการเวียนศีรษะบ้านหมุน( vertigo) ร่วมด้วย
Steatosis
คาว่า steatosis เท่าที่ค้นมา มันมีแต่แปลว่า ภาวะ abnormal retention of lipids within a cell และโดยทั่วไปก็จะหมายถึง
ภาวะ fatty liver ซะเป็นส่วนใหญ่ไม่ค่อยหมายถึงในหู ก็เลยงงๆเหมือนกันเลยไปหาคาที่คล้ายๆกันมาให้เผื่อจริงๆแล้วมันอาจจะ
เป็นอันนี้…
Cholesteatoma
คือ สภาวะที่มีการสะสมของ keratinizing stratified squamous epithelium ที่มีการหลุดลอกของ keratin debris ในหูชั้น
กลางหรือ mastoid antrum อาการมักจะมาด้วยการได้ยินลดลงอย่างชัดเจน มีน้าหนองไหลออกจากหูเป็นๆหายๆ อาการที่
บ่งชี้ว่ามี complication คือ ไข้ ปวดหู ปวดหัว เวียนหัวอย่างรุนแรง facial nerve palsy

จบข้อ 3 ขอโทษด้วยที่เฉลยเยอะแต่ไม่รู้จะมีแต่น้ารึปูาว ที่เอามาให้ดู คือ คิดว่าไม่อยากให้เพื่อนๆต้องไปเปิดอ่าน


เองอะไรมากมาย ผิดพลาดยังไงขออภัยด้วยครับ และขอให้ปีนี้ มน. สอบผ่านทั้ง Compre&License 100%

4. ผู้ปูวยชายอายุ 29 ปี ปวดตาขวามา 5 วัน ไม่มีขี้ตา มองเห็นปกติ


PE: คลาได้ก้อนที่เปลือกตาบน ขนาด 0.5 cm. ถามว่าความผิดปกติที่ตรวจพบเกิดจากความผิดปกติใด
A. ? D. allergic rhinitis
B. entropion E. Meibomian gland dysfunction
C. ectropion
ตอบ ข้อนี้ คาตอบน่าจะนึกถึง hordeolum ซึ่งจะมาด้วยอาการปวดบริเวณหนังตา หนังตาบวม คลาได้ก้อน
กดเจ็บ กรอกตาหรือหลับตาจะทาให้ปวด บางคนหนองไหลออกจากเปลือกตา หากหนองแตกจะทาให้มีขี้ตาเป็นสีเขียว
อาจจะคิดถึง chalazion ได้ เนื่องจากจะมาด้วยอาการมีก้อนที่หนังตา บนหรือล่างก็ได้ โดยก้อนจะโตช้า ๆ
อาจไปกดกระจกตาเกิดภาวะตาเอียงได้ แต่จะไม่เจ็บ ไม่มีการอักเสบ จึงคิดถึงน้อยลง
- Entropion คือ ภาวะหนังตาม้วนเข้าใน ทาให้ขนตาม้วนเข้าไปถูกับกระตกตาและเยื่อตา เกิดอาการระคาบเคือง
- Ectropion คือ ภาวะหนังตาม้วนออกด้านนอก จะมีอาการน้าตาไหล ระคาบเคืองตา
- Allergic Rhinitis จากการเปิดหนังสือ 2 เล่ม ไม่พบว่ามีอาการทางตา
- Meibomian gland dysfunction มักมีอาการตาแห้ง หนังตาอักเสบ หรือเกิด meibomitis chalazion หรือ punctate
keratitis ตามมาได้
5. ผู้ปูวยเด็ก มีไข้ น้ามูกไหล ตรวจหูพบ erythematous with bulging at posterior TM จะให้ยาใดรักษา
A. Tetracycline
B. Amoxycillin
C. Erythromycin
D. Doxycycline
E. Amoxy-clavulonic
ตอบ B ข้อนี้คิดถึงอาการของ Acute Otitis media ซึ่งอาจเกิดตามหลังการติดเชื้อในทางเดินหายใจ
หรือมีความผิดปกติของ Eustachian tube ก็ได้ ทาให้มีอาการของไข้ ตรวจร่างกายพบ TM บวม มีหลอดเลือดแดงมากขึ้น
ในเด็กอายุต่ากว่า 6 ปี มักเกิดจากเชื้อ H.influenzae และ M.Caharrhalis ในเด็กโตและผู้ใหญ่มักเกิดจากเชื้อ S.Pneumoniae
และ H.Influenzae เบื้องต้นจะให้การรักษาด้วย Amoxycillin และ Bactrim แต่ถ้าอาการไม่ดีขึ้นใน 2 - 3 วัน อาจเกิด
จากเชื้อ H.influenzae และ M.Caharrhalis ที่สร้าง beta lactamase จะให้การรักษาด้วย Augmentin หรือ Cephalosporin

6. เด็กชายอายุ 5 ปี มีไข้ หายใจเสียงดัง น้ามูกไหลจากจมูกทั้งสองข้าง นอนกรนมาสองปี น่าจะเป็นจากอะไร


A. Choanal atresia
B. High arch palate
C. Hypertrophy of adenoid
D. Foreign body of nasal cavity
E. Congenital tumor of nasal cavity
ตอบ C ข้อนี้ค่อนข้างชัดเจนว่า เป็นอาการของ hypertrophy of adenoid ซึ่งเกิดร่วมกับ hypertrophy of tonsil
โดยจะมีอาการของการอุดกั้นทางเดินหายใจ ก่อให้เกิดการนอนกรนและมีหายใจเสียงดังได้ สามารถเกิดภาวะ chronic
sinusitis ตามมา และขนาดของต่อมที่โตขึ้นก็เป็น source ของเชื้อโรคเกิด infection ตามมา มีน้ามูกไหลได้เป็นเวลานาน
- Choanal atresia เป็นความผิดปกติแต่กาเนิดที่พบได้บ่อยสุด ถ้าเป็นข้างใดจะมีอาการของหวัดเรื้อรัง น้ามูกไหล แต่
ถ้าเป็น 2 ข้างจะมีประวัติตัวเขียวสาลักเวลาดูดนม พบอาการได้ตั้งแต่หลังคลอด ไม่น่ามาพบตอน 5 ปี
- High arch palate เจอได้ในเด็กที่เป็น AllergicRhinitis มีอาการน้ามูกไหลได้ แต่ไม่น่าเป้นสาเหตุของการนอนกรน
- Foreign body of nasal cavity เด็ก 5 ขวบก็พอรู้เรื่องบ้างแล้ว ทั่วไปมักมีน้ามูกไหล ข้างเดียว
- Congenital tumor of nasal cavity มักมาด้วยคัดจมูก เลือดปนน้ามูก ใบหน้าไม่เท่ากัน ฟันหลุด ชารอบจมูก

7. หญิง 25 ปี คัดจมูก มีน้ามูก และจามอยู่บ่อยๆ ไปพบแพทย์ ได้รับการวินิจฉัยว่าเป็น จมูกอักเสบจากภูมิแพ้ ได้ยา


หยอดจมูก 6 เดือน อาการไม่ดีขึ้น จึงซื้อยามาหยอดจมูกเอง 3 เดือน อาการแย่ลง มีน้ามูกมากขึ้น หายใจไม่ค่อย
ออก จง Dx
A. Atrophic rhinitis
B. Vasomotor rhinitis
C. Chronic irritative rhinitis
D. Rhinitis medicamentosa
E. Severe allergic rhinitis with polypoid change
ตอบ D Topical decongestants กระตุ้น Alpha 1 & 2 receptor ทาให้เกิด vasoconstriction ลดการบวมของเยื่อบุ
จมูก ใช้หยอดจมูกวันละ 2-3 ครั้ง ไม่ควรใช้ติดต่อกันนานเกิน 7 วัน เพราะจะทาให้เกิด rebound effect ยิ่งทาให้เยื่อบุบวมมาก
ขึ้น คัดจมูกมากขึ้น หายใจไม่ออก เรียก “Rhinitis medicamentosa”
8. เด็กมาด้วยน้ามูกเขียวเหม็นไหลจากจมูกซ้าย ได้ยาแล้วก็ไม่หาย คิดว่าเป็นอะไร ?
 น่าจะเป็น Foreign body of the nose
 Unilateral rhinorrhia
 เด็ก น้ามูกไหลข้างเดียว สีเขียว มีกลิ่นเหม็น เลือดกาเดาไหล เจ็บ คัดจมูก อาจมีไซนัสอักเสบ หรือหูชั้นกลาง
อักเสบ ได้ยาแล้วไม่ดีขึ้น

9. ชาย อายุ 45 ปี มีอาการคัดจมูกด้านซ้าย น้ามูกปนเลือด หูอื้อ มีก้อนที่คอ


A. CA nasopharynx
B. Otitis media ?
ตอบ A (ใช่มั้ย?)
อาการและอาการแสดงของ Nasopharyngeal CA
 90% พบต่อมน้าเหลืองที่คอโตข้างใดข้างหนึ่ง
 50% พบต่อมน้าเหลืองที่คอโตทั้ง 2 ข้าง
 อาการที่ทาให้มาพบแพทย์ขึ้นกับการลุกลามของมะเร็ง เช่น ถ้ามะเร็งลุกลามสู่ posterior nasal cavity
จะทาให้มีอาการแน่นจมูก มีน้ามูกหรือ มีเลือดกาเดาไหล มีเสียงขึ้นจมูก เหมือนผู้ป่วยในโจทย์ ^ ^

10. ผู้ปูวยมีไข้ ไอ มาพบแพทย์ ตรวจหูพบ Bullous myringitis ยาที่ใช้รักษา คือ ?


* ผู้ป่วยมีไข้ ไอ ==> นึกถึง Pneumonia ตรวจหูพบ Bullous myringitis
* Bullous myringitis เป็น Otitis externa อย่างหนึ่ง
ลักษณะทีพ่ บคือ blister on the tympanic membrane ( intact นะ ) ทาให้ผู้ป่วยมีอาการปวดหูได้
* ถ้าโจทย์พูดถึง Pneumonia ที่ตรวจเจอ Bullous myringitis ด้วย บอกได้คาเดียวว่า เป็น “ Mycoplasma
pneumonia”
* ยาที่ใช้รักษา ( ตามที่ทราบๆกัน ) ==> Macrolides ( Erythromycin,Clarithromycin,Azithromycin )
==> อันนี้ก็ได้ Doxycycline, Levofloxacin
==> continue ABO ไป 14-21 days
Eye : รพ.อุตรดิตถ์

คาแนะนา โปรดใช้วิจารณญาณในการอ่าน

1. ผู้ป่วยเด็กอายุ 5 ปี VA ข้างขวา 20/200  20/40 with pinhole, VA ซ้าย 20/100  20/30 with pinhole ต้องทาการตรวจ
เพิ่มเติมเพื่อวินิจฉัยภาวะใด
A. Cataract
B. Amblyopia
C. Strabismus
D. Refractive error
E. Color blindness
เฉลย D. Refractive error
การวัด Visual acuity 20/200 = ผู้ป่วยยืนที่ 20ฟุต เห็นตัวเลขที่คนปกติยืนที่ 200 ฟุต ยังสามารถเห็นได้
ถ้าพบว่า VA ผิดปกติ ให้ใช้ pinhole ทดสอบ ถ้า VA ดีขึ้น นึกถึงภาวะ Refractive error
ถ้า VA ลดลง พบในพยาธิสภาพใน retina หรือ optic n.
ภาวะ Amblyopia (Lazy eyes) คือ ภาวะที่ความสามารถ ในการมองเห็นลดลง ซึ่งใช้แว่นแก้ไขแล้วไม่ดีขึ้นและตรวจ
ไม่พบโรคทางตาที่เป็นสาเหตุ ซึ่งการวินิจฉัยคือ ตา 2 ข้างเมื่อแก้ไขภาวะสายตาผิดปกติ (ถ้ามี) แล้วการเห็นของตา 2 ข้างไม่
เท่ากัน ต่างกัน 2 แถวของ Snellen chart ขึ้นไป หรือระดับสายตาต่ากว่า 6/12

2. หญิงอายุ 40 ปี ใส่แว่นสายตาสั้นมานาน ต่อมาเริ่มมองเห็นไม่ชัดโดยเฉพาะตอนกลางคืน หรือแสงสว่างไม่พอ แต่พอถอดแว่น


สายตาจะเห็นได้ชัดขึ้น น่าจะเข้าได้กับภาวะใด
A. Dry eye
B. Cataract
C. Glaucoma
D. Retinitis pigmentosa
E. Loss of accommodation
เฉลย ......โปรดใช้วิจารณญาณ
ข้อนี้ไม่รู้จะตอบอะไรดี ใครรู้ช่วยบอกด้วย แต่มีข้อมูลมาให้ดังนี้
อาการมองไม่ชัดตอนกลางคืน (Night blindness) พบได้ในโรค
- Refractive error ที่ไม่ได้รับการแก้ไข โดยเฉพาะ uncorrected myopia
- Advance glaucoma : ทาให้ประสาทตาเสื่อม
- small pupil เช่น จาก miotic drops
- Retinitis pigmentosa : เป็นโรคทางพันธุกรรม เป็นความผิดปกติของ cone dystrophy อาการแรกสุดคือ มีตาบอด
ตอนกลางคืน ต่อมามีตามัว ลานสายตาแคบลง => ไม่เข้ากับ Pt.รายนี้ => ตัดทิ้งไป
ส่วน Choices อื่นๆ
- Dry eye จะมีอาการตาแห้ง แสบตา เคืองตา รู้สึกตาฝืดๆ อาจมีตาแดงได้ => ไม่น่าเกี่ยว
- Cataract ถ้าในที่มืด pupil จะ dilate ทาให้มองเห็นชัดเจนขึ้น และในกรณีที่เป็น nuclear cataract จะทาให้มี
refractive index เพิ่มขึ้น คือแสงหักเหได้มากขึ้น ทาให้มี hyperopia ลดลง หรือเป็น myopia มากขึ้น
ตัวอย่างเช่น ในคนที่ใส่แว่นสายตายาวมานานๆ (คนนี้ดันสายตาสั้น) เวลาใส่แว่นจะเห็นชัด พอเป็น nuclear
cataract จะมองเห็นชัดขึ้นโดยไม่ต้องใส่แว่น
- Glaucoma ในกรณีที่เป็นระยะท้ายๆ ที่มีประสาทตาเสียแล้ว จะมีตาบอดตอนกลางคืนได้ แต่คนนี้ไม่เห็นบอก
อาการอะไรที่จะบ่งบอกว่าเป็น glaucoma เลย
- Loss of accommodation คือ ก็อยากตอบข้อนี้เหมือนกันนะ คิดว่าเป็นจากพวกสายตาคนแก่( Presbyopia)
อาจเกิดได้จากการที่มีอายุมากขึ้น มักพบในคนที่มีอายุมากกว่า 40 ปีขึ้นไป (คนนี้อายุ 40 เป๊ะ) โดยจะมีปัญหาที่
การมองใกล้ ซึ่งเกิดจาก ciliary muscle เสื่อมลง ทาให้มองใกล้เริ่มไม่ชัด แต่การมองไกลยังปกติดีอยู่เนื่องจาก
ไม่ได้ใช้กล้ามเนื้อนี้
ซึ่งอาการที่พบ เช่น อ่านหนังสือใกล้ไม่ชัด โดยเฉพาะในที่แสงสว่างไม่พอ และโรคนี้จะคล้ายๆกับ focus
defect คือไม่สามารถปรับสายตาระยะใกล้ได้ และถ้าอยู่ในที่สว่าง pupil จะเล็ก ช่วยให้มีการหักเหแสงมากขึ้น เห็นชัดกว่าใน
ที่มืด
ดังนั้นข้อนี้ ถ้าจะต้องตอบจริงๆ ก็อยากตอบข้อ E. Loss of accommodation

3. ชายอายุ 58 ปี เจ็บตาขวา ตาแดง ตามัวมา 3 วัน ไม่มีคลื่นไส้อาเจียน


PE : Rt eye - ciliary injection, cloudy cornea, shallow anterior chamber, pupil 5 mm no reactive to light
Lt eye - deep anterior chamber, lens sclerosis
จง Dx.
A. Acute iridocyclitis
B. Phacomorphic glaucoma
C. Phacolytic glaucoma
D. Acute dry eye
E. intermittent dry eye
เฉลย B. Phacomorphic glaucoma
ผู้ป่วยที่มาด้วย painful visual loss โรคที่พบบ่อย ได้แก่
- Acute angle closure glaucoma : ปวด ตามัว เห็นไฟเป็นรุ้ง มักเป็นข้างเดียว อาจมีคลื่นไส้อาเจียน
- Optic neuritis : ปวดลึกๆหลังลูกตา โดยเฉพาะเวลากลอกตา
- Uveitis : ปวดรอบกระบอกตา ปวดมากขึ้นเมื่อโดนแสง
ในผู้ป่วยรายนี้ นึกถึง Acute angle closure glaucoma มากสุด เนื่องจากมีปวดตา ตามัว ตาแดง(ciliary injection) shallow
anterior chamber, fixed mid-dilated pupil
ซึ่งใน choices มี 2 อัน (เป็นในกลุ่ม secondary closed angle glaucoma ทั้งคู่)
- Phacomorphic glaucoma เกิดจากเลนส์บวมขึ้นอย่างรวดเร็วใน intumescent cataract จะดัน iris ไปข้างหน้า ทา
ให้การไหลเวียนของ aqueous humor ไม่สะดวก อาการคือ ปวดตา ตามัว ตรวจพบเลนส์บวมหรือขุ่นร่วมด้วย anterior
chamber ตื้น
- Phacolytic glaucoma เกิดจากเลนส์สุกเต็มที่ (hypermature cataract) มักพบในคนอายุมาก จะมีโปรตีนหลุดไปอุด
meshwork ทาให้การไหลเวียนของ aqueous humor ไม่สะดวก ผู้ป่วยจะปวดตา ตาแดง มี anterior chamber ลึก
4. ผู้ป่วยหญิงอายุ 55 ปี มีก้อนที่หนังตาบนข้างขวามา 1 สัปดาห์ 3 เดือนก่อนเคยเป็นมาแล้ว ได้รับการวินิจฉัยเป็น
chalazion รักษาโดย I&C มาแล้ว 3 ครั้ง หลังผ่าตัดแต่ละครั้งหายไประยะหนึ่ง แล้วจะมี recurrent อีก การรักษาใด
เหมาะสมที่สุด
a. Local steroid injection.
b. I&C with culture.
c. Warm compression with systemic antibiotics.
d. Biopsy and microbial investigation.
e. Aspiration and gram staine, KOH exam and culture.
Chalazion เป็นการอักเสบเรื้อรังของต่อมไขมัน (chronic lipogranulomatous inflammation) ของต่อม meibomian
ซึ่งมีการอุดตันนามาก่อนแล้วทาให้สารที่สร้างจากต่อมสะสมอยู่ภายใน พบเป็นก้อนที่หนังตาบนหรือล่าง ก้อนจะโตช้า ไม่เจ็บ ไม่
มีการอักเสบ ถ้าก้อนมีขนาดใหญ่ขึ้น อาจกดลูกตาทาให้ความโค้งของกระจกตาผิดไป เกิดสายตาเอียงได้ ถ้ามีการติดเชื้อจะมี
อาการเช่นเดียวกับ internal hordeolum
การรักษา
- ถ้ามีขนาดเล็กและไม่มีอาการไม่ต้องให้การรักษาก็ได้
- ถ้าก้อนโตขึ้นหรือมีการอักเสบติดเชื้อ ให้การรักษาแบบเดียวกับ internal hordeolum โดยการเจาะหนองร่วมกับการให้
ยาปฏิชีวนะ อาจฉีด steroid เข้าในก้อนในรายที่ไม่มีการอักเสบแล้ว แต่อาจทาให้เกิด hypopigmentation ของผิวหนัง
บริเวณนั้นได้
- ในรายที่เป็นซ้าตาแหน่งเดิมบ่อย ๆ ควรตัดชิ้นเนื้อส่งตรวจ เพราะอาจเป็นมะเร็งของต่อม meibomian ได้
อีก reference.
- Small, inconspicuous, asymptomatic chalazia may be ignored.
- Conservative treatment with lid massage, moist heat, and topical mild steroid drops usually suffices.
- Acute therapy with oral tetracycline (eg, doxycycline 100 mg or minocycline 50 mg qd for 10 d) minimizes
the infectious component and decreases the inflammation, reputedly by inhibiting polymorph
degranulation.
- Chronic therapy with low-dose tetracycline (eg, doxycycline 100 mg PO qwk for 6 mo) frequently prevents
recurrence. If tetracycline cannot be used, then metronidazole has been used in a similar fashion.
- In most cases, surgery should be performed only after a few weeks of medical therapy.
ดังนั้นข้อนี้น่าจะตอบ Biopsy and microbial investigation. เพราะนึกถึงว่ามันเป็น Recurrent chalazion ก็ควรเอาชิ้นเนื้อ
ไปตรวจตามที่หนังสือบอกใช่ป๊ะ ส่วนตัวเลือกอื่น ๆ นั้น ล้วนเป็นการรักษาของโรคนี้ทั้งสิ้น ซึ่งบางอย่างก็น่าจะเป็นการรักษาใน
ผู้ป่วยรายนี้ได้เหมือนกัน การรักษามันเป็นศิลปะ บางคนอาจคิดไม่เหมือนกัน แต่ถ้าเราไม่มั่นใจ ก็ยึดตามหนังสือน่าจะดีกว่า
จริงป๊ะ
Ans. D. Biopsy and microbial investigation.
5. ผู้ป่วย glaucoma (acute closed angle glaucoma) ควรให้ยาอะไรก่อน
a. beta blocker.
b. atropine eyes drop.
c. pilocapine eyes drop.
d. oral glycerol.
Primary glaucoma แบ่งได้ 2 ชนิดคือ
1. Primary angle closure glaucoma แบ่งได้ 2 แบบคือ
- Intermittent angle closure glaucoma.
- acute angle closure glaucoma.
2. Primary open- angle glaucoma.
acute angle closure glaucoma. เป็น ocular emergency เพราะหากรักษาไม่ทันอาจตาบอดได้ เกิดจาก iris root
เบนมาปิดหน้า trabecular meshwork โดยรอบ 360 องศา aqueous humor ไหลออกจากลูกตาไม่ได้ ทาให้ความดันลูกตา
สูงขึ้นอย่างรวดเร็ว ผู้ป่วยจะมีอาการและอาการแสดงดังนี้
1. ปวดตาและปวดศีรษะข้างที่เป็น
2. คลื่นไส้อาเจียน
3. ตามัวเห็นสีรุ้งรอบดวงไฟ จาก corneal edema
4. Ciliary or mixed injection.
5. semidilated and fixed pupil. ช่องหน้าม่านตาตื้น
6. anterior subcapsular cataract. เรียก glaukomflecken.
7. increase intraocular pressure. ~ 60 - 80 mmHg.
ในผู้ป่วยรายนี้ เป็น acute closed angle glaucoma จึงต้องให้ hyperosmotic agent เพื่อควบคุม IOP โดยเฉพาะ
ผู้ป่วยที่มี IOP สูงอย่างฉับพลัน โดยจะดึงน้าจาก extravascular เข้ามาใน intravascular component ทาให้การสร้าง aqueous
ลดลง และ vitreous volume ลดลง มียา 2 กลุ่มคือ
- 50 % Glycerol and isosorbide. ใช้กิน glycerol ต้องระวังในคนที่เป็นโรคเบาหวาน เพราะถูกดูดซึมได้จาก
ทางเดินอาหาร แต่เป็นยาที่หาได้ง่าย ราคาไม่แพง จึงนิยมใช้ แต่ isosorbide ไม่ถูกดูดซึมจากทางเดินอาหาร
- 20 % Manital เป็น parenteral hyperosmotic agent ออกฤทธิ์เร็ว แต่ต้องระวัง systemic side effect จาก
rapid intravascular volume expansion.
ส่วนยาที่น่าสนใจอีกตัวคือ cholinergic agent คือ pilocapine ซึ่งออกฤทธิ์โดยเพิ่ม aqueous outflow ทาให้ไหลผ่าน
trabecular meshwork ออกจากตาได้มากขึ้น และกระตุ้นกล้ามเนื้อ sphincter pupillae ที่ iris ทาให้ม่านตาหดตัว (miosis) แต่
มันจะออกฤทธิ์ภายใน 4 ชั่วโมง และอยู่นาน 8 ชั่วโมง บางทีมันช้าไปมั๊ง และตาราไม่ได้บอกไว้ว่าตัวไหนให้ก่อน แต่ถ้ามีความดัน
ตาสูงมาก ยาที่จะให้ตัวแรกคือ glycerol ดังนั้นข้อนี้ก็จะตอบอันนี้นะ แต่กิง ๆ แล้ว definite treatment ของ acute closed
angle glaucoma คือ emergency peripheral iridectomy หรือ LASER iridotomy. นะจ๊ะ
Ans. D. oral glycerol.
6. ชายอายุ 30 ปี ปวดตาร้าวไปศีรษะ ตามัวลง เห็นแสงสีรุ้ง PE: mild cornea and lens ขาวๆ ? , Ciliary injection, pupil 5
cm fixed จง Dx.
A. Migraine
B. Uveitis
C. Acute glaucoma
โอ๊ย อะไรกันนักกันหนา อาการร้อยแปดมะรุมมะตุ้ม เริ่มไล่แต่ละอาการและอาการแสดงนะ
- อาการปวดตาร้าวไปศีรษะ เกิดได้ในหลาย ๆ โรค รวมทั้งที่ไม่เกี่ยวกับตาก็ได้ อย่างไมเกรนก็ปวดตาได้ แต่ปวดร้าว
ไปศีรษะมักเป็นจาก angle closure glaucoma ส่วนไมเกรนน่าจะปวดศีรษะร้าวมาที่ตา
- อาการตามัว ถ้ามัวพร้อมกับอาการปวดตาจะนึกถึงโรคต่อไปนี้
o angle closure glaucoma มี pupil dilate ด้วย และอาการอื่น ๆ เหมือนข้อก่อนอ่ะ
o optic neuritis จะปวดลึก ๆ หลังลูกตา โดยเฉพาะเมื่อกรอกตา
o uveitis มักปวดรอบกระบอกตา หรือปวดมากขึ้นเมื่อโดนแดดหรือแสงสว่าง ( photophobia)
- เห็นแสงสีรุ้ง อาการนี้มักพบในผู้ป่วย glaucoma มากมาย
- Mild increase corneal and lens opacity. นึกถึงโรคหรือภาวะต่อไปนี้
o Keratitis
o Corneal ulcer
o Corneal edema ซึ่งพบได้จากหลาย ๆ โรค รวมทั้ง Uveitis และ Acute glaucoma ด้วย
- Ciliary injection มักพบอาการปวดร่วมด้วยซึ่งมักพบใน corneal abrasion, keratitis, iriditis, iridocyclitis และ
acute closed angle glaucoma
- Pupil 5mm fixed ก็พบได้บ่อยใน acute closed angle glaucoma
ผู้ป่วยรายนี้ไม่น่าเกิดจากไมเกรนเพราะตรวจพบความผิดปกติของตาร่วมด้วยและอาการต่าง ๆ ก็เข้าได้กับ acute
closed angle glaucoma มากมาย ดังนั้นก็น่าจะตอบข้อนี้ ส่วน Uveitis นั้นมีอาการเหมือนกันกับผู้ป่วยรายนี้ได้ แต่ที่
แตกต่างจาก acute closed angle glaucoma ผู้ป่วย Uveitis จะมี meiosis ไม่ Pupil 5mm fixed เหมือนผู้ป่วยรายนี้
แน่นอน ดังนั้นข้อนี้ก็ต้องตอบ acute closed angle glaucoma
Ans. C. Acute glaucoma

7. ผู้ป่วยชายไทยอายุ 20 ปี มีอาการเจ็บตาทันทีที่ถอด contact lens มีน้าตาไหล ไม่มีขี้ตา ตรวจตาพบเป็น mild conjunctival


injection, corneal ulcer 2 cm ควรปฎิบัติอย่างไร
A. Topical ATB
B. Pressure patch
C. Contact lens culture
Ans ไม่แน่ใจ
จากอาการ มีน้าตาไหล แต่ดันไม่มีขี้ตา อาการปวดหรือเคืองตาก็ไม่พูดถึง ถ้าเป็น infection มักเคืองตา มีขี้ตามาก
จากตรวจร่างกาย corneal ulcer 2 cm (มันใหญ่จัง เกิน cornea ไปแล้ว)
ถ้าเป็น corneal ulcer จะไม่ Pressure patch ไม่เหมือน abrasion ตัด ข้อ B.
ในคนที่ใส่contact lens ถ้า infection จะเสี่ยงต่อ Pseudomonas aeruginosa และถ้าเป็นเชื้อนี้ควร admit รักษา
เต็มที่มีโอกาส cornea ทะลุใน 1-2 วัน การรักษา ถ้า ulcer เล็กให้ยาหยอดได้ Ciprofloxacin หยอดตาทุก ครึ่งหรือหนึ่งชั่วโมง
ขึ้นกับความรุนแรง ถ้ารุนแรงมากควรให้ IV Ref : เอกสารประกอบการเรียน รพ. อุตรดิตถ์
8. ผู้ป่วยเด็ก ตรวจพบ miosis, diaphoriasis, BPสูง และ drooling เกิดจากพิษของสารใด
A. Opioid
B. Cocaine
C. Carbamate
Ans C Carbamate
ยาฆ่าแมลงประเภทออร์แกโนฟอสเฟต (Organophosphate) เช่น พาราไทออน (Parathion),มาลาไทออน (Malathion),คาร์
บาเมต (Carbamate) เป็นต้น มักมีอาการภายใน 2 - 3 ชั่วโมงหลังกิน ด้วยอาการปวดศีรษะ เหงื่อออก น้าลายฟูมปาก น้าตา
ไหล อาเจียน ท้องเดิน กล้ามเนื้อเต้นกระตุก ชัก หอบ ตาลาย รูม่านตาหดเล็ก และอาจตายภายในเวลารวดเร็ว
Opioids ฝิ่นและอนุพันธ์ฝิ่น สารในกลุ่มนี้มีทั้งที่สกัดได้จากธรรมชาติ เช่น morphine, กึ่งสังเคราะห์ เช่น heroin และ
สังเคราะห์ เช่น pethidine จากการที่ยาในกลุ่มนี้จับตัวกับ µ -opiate receptors ฤทธิ์ของยาจึงทาให้รู้สึกครื้นเครง
(Euphoria) ลดการเจ็บปวด กดการหายใจ ทาให้ท้องผูก ลดการอยากอาหาร และทาให้ความต้องการทางเพศน้อยลง
อาการพิษฝิ่น (Opioid intoxication): ผู้ป่วยจะมีความผิดปกติของพฤติกรรม หรือจิตใจเช่น มีอารมณ์ร่าเริงสนุกสนานใน
ระยะแรกแล้วเปลี่ยนมาเป็นเฉยเมย รู้สึกไม่สบายใจ พลุ่งพล่าน กระวนกระวายหรือเชื่องช้า ผู้ป่วยอาจเสียชีวิตจากการเสพ
มากเกินไป (opioid overdose) ซึ่งจะพบกลุ่มอาการที่สังเกตได้ง่ายและเป็นภาวะฉุกเฉิน เรียกว่า “clinical triad” ได้แก่
coma, pinpoint pupil, respiratory depression
Cocain ผลต่อร่างกาย ผู้ที่เสพโคเคนด้วยวิธีสูดดมเข้าไปทางจมูกจะทาให้ผนังจมูกขาดเลือด
ทาให้เยื่อบุโพรงจมูกฝ่อขาดหรือทะลุ นอกจากนี้ยังมีอาการมือสั่นหัวใจเต้นเร็ว ความดันโลหิตสูง ม่านตาขยาย คลื่นเหียน
นอนไม่หลับ การย่อยอาหารผิดปกติ บางรายสมองถูกกระตุ้นอย่างรุนแรงทาให้เกิดอาการชักมีเลือดออกในสมองเนื้อสมอง
ตายเป็นบางส่วนหัวใจถูกกระตุ้นอยู่เสมอกล้ามเนื้อหัวใจเสื่อมลงทีละน้อยจนหัวใจบีบตัวไม่ไหวทาให้หัวใจล้มเหลว ผลจาก
การเสพเป็นระยะเวลานานยังทาให้เกิดอาการโรคจิตซึมเศร้าอีกด้วย

9. เด็กอายุ 3 วัน มาด้วยขี้ตาเป็นหนอง หนังตาบวม ตาบวม normal cornea การรักษาที่สาคัญในเด็กคนนี้ คือ


A. ล้างตา
B. ceftriazone IM
Ans B. ceftriazone IM
ข้อนี้นึกถึงการติดเชื้อ Neisseria gonorrhea มากที่สุด
ตารางสรุป เยื่อตาอักเสบในทารก
สาเหตุ อายุที่เริ่มอาการ อาการ การย้อมพิเศษจากเยื่อตา
Silver nitrate ภายใน 24 hr หนังตาบวม น้าตาไหล อาจพบ PMN เล็กน้อย
Neisseria gonorrhea 2-4 วัน รุนแรง ขี้ตาเป็นหนองปน Gram neg diplococci
เลือด หนังตาบวม ภายในเซลล์
Chlamydia trachomatis 4-10 วัน หนังตาบวม และมีเยื่อตา ย้อม Giemsa พบ
คลุมด้านในหนังตา basophilic cytoplasmic
(pseudomembrane) inclusion bodies
แบคทีเรียอื่นๆ 4-7 วัน หนังตาบวมแดง ขี้ตาเป็น
มูกปนหนอง
Herpes simplex 7-14 วัน มักเป็นในตาเพียงข้าง Gram stain :
เดียว ตาแดง มีขี้ตา multinucleated giant
ร่วมกับกระจกตาอักเสบ cells
เยื่อตาอักเสบในทารกที่มีสาเหตุจากการติดเชื้อพบได้ร้อยละ 1-12 อย่างไรก็ตาม เนื่องจากการติดเชื้อ Neisseria
gonorrhea มักเป็นรุนแรง และการอักเสบลุกลามอย่างรวดเร็ว ดังนั้นทารกแรกเกิดทุกรายที่มีอาการเยื่อตาอักเสบจึงควร
ได้รับการรักษาอย่างรีบด่วนเสมือนว่าติดเชื้อ Neisseria gonorrhea ไม่ควรรอ Lab
การรักษา : ทาความสะอาดหนังตา และล้างตาบ่อยๆ ด้วย sterile NSS ป้ายตาด้วย erythromycin และให้ยา third-
generation cephalosporin เช่น ceftriaxone ขนาด 30-50 mg/kg ไม่เกิน 125 mg หรือ cefotaxime 100 mg/kg ทาง IV หรือ
IM เพียงครั้งเดียว

10. คนไข้เป็น mature cataract แล้วเป็น glaucoma on top ให้ยาอะไร (Phagomorphic)


เฉลย phagomorphic glaucoma จะมีลักษณะเหมือนต้อหินมุมปิดเฉียบพลัน ยาที่ใช้รักษา มีดังนี้
- Hyperosmotic agent เช่น 20%manitol หรือ 50%glycerol ยากลุ่มนี้มักนามาใช้ในรายที่ต้องการลดความดันตา
อย่างรวดเร็วและใช้ในระยะสั้น โดยเชื่อว่ายาทาให้วุ้นตามีปริมาตรลดลง จึงทาให้ความดันตาลดลง
- Carbonic anhydrase inhibitor ยานี้จะลดการสร้าง aqueous humor เช่น acetazolamide ขนาดที่ใช้ในผู้ใหญ่ 1
กรัม/วัน โดยแบ่งให้วันละ 2-4 ครั้ง
- 2%pilocarpine เพื่อช่วยดึงโคนม่านตาออกจาก trabecular meshwork โดยให้หยอดทุก 15 นาที จานวน 4 ครั้ง
หลังจากนั้นหยอดทุก 6 ชั่วโมง
- Beta-adrenergic blocking agent เช่น timolol โดยออกฤทธิ์ลดการสร้าง aqueous humor

11. เด็กชายอายุ 9 ปี มีอาการคันเคืองตามา 1 ปี ตรวจพบ cobble bone stone (papillaly) ที่เปลือกตาบนทั้ง 2 ข้าง 
Vernal keratoconjunctivitis
โอว เฉลยมาแล้วงั้นขอพูดเกี่ยวกับ Vernal keratoconjunctivitis ละกัน
Vernal keratoconjunctivitis(VKC)
เป็นเยื่อตาอักเสบเรื้อรังที่เป็นทั้ง 2 ข้าง มักพบในเด็กผู้ชายช่วงอายุก่อน 10 ปี โดยผู้ป่วยมีประวัติเป็นโรคภูมิแพ้อื่นๆ
เช่น atopic, asthma ผู้ป่วยจะมีอาการมากหรือน้อยตามฤดูกาล โดยพบอาการคันตา ตาแดง สู้แสงไม่ได้ มี mucoid discharge
โรคนี้แบ่งเป็น 2 ชนิด
1. Palpable vernal keratoconjunctivitis พบ papillae ที่ขนาดใหญ่ทางด้าน tarsal เรียกว่า cobble stone
2. Limble vernal keratoconjunctivitis พบ limbus บวมแดง มีตุ่มนูนหลายตุ่ม อาจมีการรวมตัวของ eosinophil ซึ่งจะ
เห็นเป็นจุดขาวๆ เรียก Horner-Trantus dots
การรักษา
- หลีกเลี่ยงสารที่แพ้
- การทา hyposensitization ไม่ได้ผลในโรคนี้
- การหยอดตาด้วย steroid จะใช้เป็นช่วงสั้นๆเฉพาะที่มีอาการกาเริบ และระวังเรื่องผลแทรกซ้อนจาก steroid
คือ ต้อหินและต้อกระจก
- การรักษาในระยะยาวจะใช้ยากลุ่ม mast cell stabilizer, NSAIDและ cyclosporine ช่วยควบคุมโรค
- ในรายที่มี papillae ใหญ่มากอาจฉีด steroid เข้าที่เยื่อตาบริเวณนั้นหรือผ่าตัดเอา papillae ออก

By Natt ji เรียงใหม่
นก ข้อ 1-3 (สารแน ดวงใจ ข้อ 8,10,11
แอ้ ข้อ 4-6 ขอมีส่วนร่วม) ข้อไหนงง!! ก้อสงสัยได้เต็มที่
น้าผึ้ง(แถน) ข้อ 7,9 ข้อไหนตอบไม่ถูกใจ จิกด่าได้ตามสบาย
FORENSIC

1.พบศพบนรถทัวร์ เลือดตกลงที่ปลายเท้า กดจาง มือแข็ง ขาไม่แข็ง จงประมารระยะเวลาการตาย


A. 2 hr
B. 2-4 hr
C. 4-6 hr
D. 6-8 hr
E. 8-10 hr

ANSWER: B. 2-4 hr
คำอธิบำย :
Postmortem Rigidity (rigor mortis) กล้ามเนื้อต่างๆจะเริ่มมีการแข็งตัวขึ้น ซึ่งจะเริ่มปรากฏที่ 2-4 ชม.
และแข็งเต็มที่เวลา 6-12 ชม. ดังนั้น น่าจะตอบ 2-4 ชม. เพราะมือแข็ง แต่ขายังไม่แข็ง
เพิ่มเติมคือ การแข็งตัวจะเกิดในอัตราเดียวกันในกล้ามเนื้อทุกมัด แต่จะสามารถตรวจได้จากกล้ามเนื้อบรเวณข้อต่อ
ข้อเล็กๆก่อน คือที่บริเวณใบหน้า กราม ข้อนิ้วมือนิ้เท้า หลังจากนั้นจึงตรวจได้จากข้อที่มีขนาดใหญ่ขึ้น ได้แก่ ข้อมือ
ข้อเท้า ข้อศอก ไหล่ เข่าและสะโพก ตามลาดับ
Postmortem hypostasis จะเห็นว่าศพนี้ กดจาง แสดงว่ายังไม่ fix จึงอยู่ใน early stage คือ ไม่เกิน 12 ชม.
ถ้าเกิน 12 ชม. Fix แล้ว กดจะไม่จาง (ชีทอาจารย์ เขียนว่า over 8-12 hours: Unblanched by compression and
not displace)

..................................................................................................................................................................................................

2. ตรวจศพพบรอยรัดที่คอ ฟกช้าและมีรอยครูดสั้น ๆ ยาวๆ ผ่าศพพบรอยช้าที่หลอดเลือดแดง และมีเลือดออกในชั้นกล้ามเนื้อ


กระดูก Hyoid bone แตกทางมุมซ้าย ท่านคิดว่าตายจากสาเหตุใด
A. Mothering (คิดว่าน่าจะเป็น Smothering มากกว่านะ)
B. Complete hanging
C. Incomplete hanging
D. Ligature strangulation
E. Manual strangulation

ANSWER: E. Manual strangulation


คำอธิบำย :
Smothering
มีการขัดขวางอุดกั้นทางเดินหายใจภายนอกโดยเฉพาะจมูกและปาก มักใช้มือหรือผ้าอุด อาจไม่มีร่องรอยการ
ต่อสู้ได้ โดยเฉพาะในเด็กและคนชรา อาจพบพียงรอยช้้าหรือถลอกเล็กน้อยบริเวณใบหน้าหรือจมูก ตรวจ
ศพไม่ค่อยพบลักษณะจุดเลือดออกหรือใบหน้าแดงคล้้าหรือเขียว
Hanging
หมายถึงการใช้วัตถุพันรอบคอโดยให้ปลายของวัตถุนั้นผูกติดอยู่ที่สูง แล้วใช้น้าหนักตัวถ่วงห้อยให้วัตถุนั้น
กดหรือรัดบริเวณคอ แบ่ง 2 ชนิด
Complete hanging
เท้าลอยจากพื้น หน้าซีด อาจพบเพียง petechial hemorrhage จาก direct trauma
Incomplete hanging
แขวนโดยใช้น้าหนักตัวบางส่วนถ่วงเชือก เช่นการแขวนคอตายในท่ายืน ท่านั่ง ท่านอน หน้าเป็นสีม่วง
(occlude only vein)
Ligature strangulation
hyoid bone and/or thyroid cartilage มักจะหัก cricoid ก็อาจจะหักได้บางครั้ง
ซึ่งต่างจาก hanging คือ hanging นั้น hyoid กับ thyroid มักจะ intact
ดังนั้นจึงตัด B. กับ C. ออก
Manual strangulation
มักจะหักที่ hyoid (usually fractures at superior horn), thyroid, cricoid มีunderlying
soft tissues bruises, abrasion of the chin ดังนั้น จึงน่าจะตอบข้อ E. มากสุด
..................................................................................................................................................................................................

3. ผู้ป่วยชายนักธุรกิจอายุ 57 ปี ไปทานอาหารเย็นที่ห้างสรรพสินค้า มีอาการแน่นหน้าอก หายใจไม่ออก ได้โทรไปปรึกษาแพทย์


บอกว่าให้รีบมารพ.ทันที ผู้ป่วยมาถึงที่ ER ทาEKG ไม่มีความผิดปกติ ต่อมาผู้ป่วยเสียชีวิตใน 20 นาทีหลังนาส่ง ผู้ป่วยไม่ฟื้น
หลังจากการช่วยชีวิตกู้ชีพเต็มที่แล้ว แพทย์ต้องการชันสูตรศพ แต่เนื่องจากผู้ป่วยนับถือศาสนาอิสลามและญาติไม่ติดใจใน
สาเหตุการตาย จะขอนาศพไปทาพิธีเลยท่า นจะให้การสันนิษฐานเรื่องสาเหตุการตายว่าอย่างไร
A. Acute poisoning
B. Rupture aneurysm
C. Acute heart failure
D. Cardiopulmonary failure
E. Coronary heart disease

ANSWER : E. Coronary heart disease


คำอธิบำย :
การตายโดยธรรมชาติ คือการเสียชีวิตจากการเจ็บป่วยด้วยโรคต่างๆ
การตายอย่างกระทันหันและไม่ทันคาดคิด ไม่ได้หมายถึงการตายในทันทีทันใดเท่านั้น แต่รวมไปถึงการ
ตายภายในระยะเวลา 24 ชม.นักจากเริ่มมีอาการผิดปกติด้วย
โรคที่เป็นสาเหตุการตายโดยธรรมชาติที่พบบ่อย อาจแบ่งได้ตามระบบต่างๆของร่างกาย จากโจทย์ คิดถึงโรค
ของระบบหัวใจและหลอดเลือดมากที่สุด ซึ่งโรคกลุ่มนี้เป็นสาเหตุการตายโดยธรรมชาติอย่างกระทันหันและไม่ทัน
คาดคิดมากที่สุด มักพบในเพศชาย ช่วงอายุ 20-65 ปี
โรค coronary artherosclerosis เป็นสาเหตุการตายมากที่สุด การตายส่วนใหญ่มักไม่มีอาการแสดง
น้ามาก่อน
โรคกล้ามเนื้อหัวใจตาย สาเหตุหลักเกิดจาก coronary artherosclerosis มักเสียชีวิตในเวลาอันสั้น ราว
½- 1 ชม.
................................................................................................................................................................................................
4. หากท่านเป็นผู้อานวยการโรงพยาบาลชุมชน พบว่ามีผู้ป่วยโรคจิตเวชจากยาเสพติดจานวนมาก ท่า นจะมีแนวทางจัดการเพื่อ
ลดปัญหาอย่างไรสร้างเงื่อนไข เช่น ไม่รักษาคนที่เสพยา
a) ขออานาจรัฐ เพื่อจับกุมคนที่มีสารเสพติดในเลือด
b) พัฒนาให้ รพ.สามารถรักษาโรคทางจิตเวชได้ดีขึ้น
c) ให้ความรู้และข่าวสารเกี่ยวกับปัญหาให้ประชากรกลุ่มเสี่ยง
d) ร่วมมือกับคนในชุมชน ผลักดันกฏหมายให้ครอบคลุม
ANS D. ให้ความรู้และข่าวสารเกี่ยวกับปัญหาให้ประชากรกลุ่มเสี่ยง (เป็นไปได้มากที่สุด)
แนวคิดของจิตเวชชุมชนจะแตกต่างจากจิตเวชทั่วไป โดยเน้นในการ ป้องกันทุกระดับ โดยให้บริการทางจิตเวชศาสตร์
แก่ประชาชนในชุมชน มากกว่าจะให้บริการที่สถานบริการของรัฐ
ความพยายามที่จะทาให้คนทั่วไปเข้าใจถึงสาเหตุของโรคทางจิตเวช แล้วหาแนวทางป้องกัน รักษาหรือฟื้นฟู
สมรรถภาพผู้ป่วยจิตเวชนั้น เราได้นาหลักการของวิชาสาธารณสุขศาสตร์มาใช้ บางครั้งมีผู้ใช้คาว่า Preventive Psychiatry ซึ่ง
หมายถึงการป้องกัน โดยหลักการของ จิตเวชศาสตร์ชุมชน คือ บริการที่ชุมชนเป็นพื้นฐาน บริการที่เข้าหาผู้ป่วยและกลุ่มเสี่ยง
เน้นการป้องกันมากกว่าการแก้ไข เป็นต้น ( Ref. เอกสารประกอบการเรียนจิตเวช รพ. สวนปรุง )
...................................................................(ข้อ5 ไม่ม)ี .....................................................................................
6. ผู้ตายถูกยิงด้วยปืนที่ใบหน้า 3 นัด ลาคอ 1 นัด ชันสูตรพบ severe brain laceration, fracture base of skull มีกระสุนฝังใน
และ rupture trachea จะเขียนใบชันสูตร สำเหตุกำรตำย อย่างไร
A. severe brain laceration with gun short wound ,fracture base of skull
B. ถูกยิง …..(เป็นภาษาไทยสั้นๆ)
C. ถูกยิง และ…..(สาเหตุยาว ๆ)
D. severe brain laceration with fracture base of skull with tracheal rupture
ANS ต้องเขียนเป็นภาษาไทยเพราะ key อยู่ตรงใบชันสูตร เพราะเราต้องเขียนให้ตารวจอ่าน ซึ่งต้องใช้ภาษาที่เข้าใจง่าย
เพราะว่าตารวจไม่รู้เรื่องศัพท์ทางการแพทย์
แต่ถ้าถามว่าสาเหตุการตาย (ที่ไม่ได้เขียนในใบชันสูตร) ข้อที่เป็นไปได้มากสุดคือ
ข้อ A. severe brain laceration with gun short wound ,fracture base of skull
เพราะมีทั้ง primary cause คือ gun short wound
และ มี secondary cause คือ fracture base of skull และ severe brain laceration
ส่วนข้อดี มีแต่ secondary cause อย่างเดียว
การออกหนังสือรับรองการตาย
แพทย์ผู้รักษา ผู้ชันสูตรพลิกศพ ต้องออกหนังสือรับรองการตายให้แก่ญาติเจ้าบ้าน หรือผู้พบศพ
โดยเขียนลงในหนังสือรับรองการตายตามแบบของสานักงานทะเบียนราษฎร โดยเฉพาะการเขียนสาเหตุการตายเป็น
ภาษาไทย (ตามข้อ 2.5) และโรคที่เป็นสาเหตุการตายเป็นภาษาอังกฤษ (ตามข้อ 2.3 , 2.4)
สำเหตุกำรตำย
- the disease , injury ,or combication of disease and injury responsible for the fatality. (ซึ่งเป็นสาเหตุของ
การตายที่เกิดขึ้น)
- cause of death should be etiologically specific disease or injury
- the underlying or proximate cause of death,natural and continuous sequence
- Immediate cause of death are complication of the underlying cause.
ปล. ข้อนี้ไม่แน่ใจว่าจะตามสาเหตุการตายหรือว่าโรคกันแน่

....................................................................................................................................................................................................
7. ข้อใดต่อไปนี้ ช่วยยืนยันว่าผู้ป่วยถูกกระทาชาเรา
A. พบ sperm ในช่องคลอด
B. พบ sperm & acid phosphatase
C. พบบาดแผลฟกช้าตามร่างกาย แผลที่ vagina และพบ sperm
D. พบ sperm ในช่องปาก
เฉลย : ไม่แน่ใจว่าจะตอบข้อไหน
ลองพิจารณาจากข้อมูลดูนะ
ตามประมวลกฎหมายอาญา พ.ศ. 2499 ภาค 2 ลักษณะ 9 มาตรา 276 อธิบายความหมายของคาว่า “กระทาชาเรา”
ว่า การกระทาเพื่อสนองความใคร่ ของผู้กระทาโดยการใช้อวัยวะเพศของผู้กระทากระทากับ อวัยวะเพศ ทวารหนัก หรือ ช่อง
ปากของผู้อื่น หรือ การใช้สิ่งอื่นใดกระทากับอวัยวะเพศหรือทวารหนักของผู้อื่น
อาจารย์อธิบายว่าเน้น Keyword ตรง “เพื่อสนองความใคร่ ” ไม่จาเป็นต้องตรวจพบอสุจิก็ได้โดยเรื่องการสาเร็จความ
ใคร่แล้วหรือไม่เป็นอีกส่วนหนึ่ง แค่แหย่เข้าไปนิดเดียวก็ถือเป็นการกระทาชาเราแล้ว
Ref.เอกสารประกอบการเรียน forensic เรื่อง Sex crime
..............................................................................................................................................................................

8. สามารถทาการขีดเขียนบนกระสุนที่พบเป็นของกลาง เพื่อบอกว่าเป็นวัตถุพยานได้หรือไม่
A. ไม่ได้เลย
B. ได้ ที่ปลอกกระสุน
C. ได้ ที่ฐานกระสุน
เฉลย : C. ได้ ที่ฐานกระสุน
กระสุนปืนจะมีรอยเกลียวซึ่งเกิดจากเกลียวในลากล้องปืน เทียบได้กับ DNA สามารถนามาเป็นวัตถุพยานได้ เรา
สามารถเขียนสัญลักษณ์บนกระสุนปืนได้ แต่ต้องระมัดระวังไม่ทาลายบริเวณรอยเกลียว
ดังนั้นจึงสามารถเขียนที่บริเวณฐานกระสุนได้
.................................................................................................................................................................................
9. ผู้ป่วยหญิงได้รับอุบัติเหตุถูกรถชน จากนั้นมีเลือดออกทางช่องคลอด ตรวจพบเป็น inevitable abortion จะเขียนใบชันสูตร
บาดแผลว่าอย่างไร
A. ได้รับบาดเจ็บจนแท้งลูก
เฉลย : A. ได้รับบาดเจ็บจนแท้งลูก
การเขียนรายงานการชันสูตรบาดแผล แบ่งเป็น 2 parts คือ
1) สิ่งที่ตรวจพบ : wounds and injuries
2) ความเห็น
a. ระยะเวลาในการรักษา / ทาให้เกิดทุพพลภาพ
b. มีการสูญเสียอวัยวะ ?
c. การติดตามการประเมินผลที่เกิดขึ้น / การรักษา
d. โรค / ภาวะแทรกซ้อน
e. ระดับความรุนแรง
f. ทาให้ถึงแก่ความตายได้หรือไม่

.......................................................................................END...............................................................................................
Ortho&Rehab
2. โครงกระดูก orbit กลมเป็นชนชาติใด
A) Caucasian
B) Mongolian
C) Nigroid

ANS. B) Mongolian
ลักษณะกระดูกใบหน้า และการบ่งบอกเชื้อชาติ

Item caucasoid negroid mongoloid

1. skull height high low medium


2. skull length long long long
3. skull width narrow narrow broad
4. face width narrow narrow wide
5. face height high low high
6. sagittal contour rounded flat arches
7. nasal opening narrow wide arrow
8. orbital opening angular rectang rounded
9. lower nasal margin sharp troughed sharp
10. nasal profile straight slant straight
11. palatal plate narrow wide medium

6. C myelopathy ได้รับการรักษาโดยการดึงคอ 1 เดือน 5-8 ปอนด์ ต่อมามีขาชา อ่อนแรง ควรทาอย่างไร


A) หยุด traction + MRI
B) เปลี่ยนไป traction หลัง
C) เพิ่มน้าหนัก

ANS. A) หยุด traction + MRI


เนื่องจากผู้ป่วยรายนี้ เป็น cervical myeopathy ได้ทา traction ไปแล้ว อาการไม่ดีขึ้น
และมี sign of radiculopathy ซึ่งถ้ามีลักษณะดังกล่าว เป็น contraindication ในการทา traction ดังนั้นการ
management ที่ควรทาต่อจากนี้คือ ควรหยุดการทา traction และทา MRI เพื่อดูภาวะลักษณะของการบาดเจ็บต่อเนื้อ
ไขสันหลังว่ามีความรุนแรงขนาดไหน เพื่อใช้ในการประเมิน ในการรักษาขั้นต่อไป
เพิ่มเติมในส่วนของการรักษาดูแลรักษาผู้ป่วย cervical myelopathy มีดังนี้
1. Neck immobilization with a soft collar, Philadelphia collar, rigid orthoses,
Minerva jacket, or a molded cervical pillow for support
2. Pharmacologic treatment :NSAIDs, Tricyclic antidepressants (TCAs),Steroid
3. Lifestyle modifications :ergonomics and/or workplace modifications,relaxation techniques
postural awareness
4. Physical modalities :used for spine-related disorders
-Cervical mechanical traction 10 lb
-Manipulation : low-velocity, high-amplitude manipulation; high-velocity,
low-amplitude manipulation
5. Exercises designed for cervical pain : isometric neck strengthening ,neck and shoulder
stretching and flexibility exercises,back strengthening exercises,aerobic exercises
6. Other modalities for pain include heat, cold, acupuncture, massage, trigger-point
injection, transcutaneous electrical nerve stimulation, and low-power cold laser
7.Surgical Tx : indication For Surgical decompression
-intractable pain
-progressive neurologic deficits
-documented compression of nerve roots or of the spinal cord that
leads to progressive symptoms

7. ผู้ป่วย RA ข้อมือบวมเล็กน้อย มี swan neck ควรทา modality ใด


A) block exercise
B) tension-gliding exercise
C) low-load, high repetitive resistant exercise
D) high load, high repetitive resistant exercise

ANS. A) block exercise


ในผู้ป่วย RA ที่มี swan neck (hyperextension of PIP joint, flexion of DIP joint) จะมี deformities ของนิ้ว
มือ และจะมี stiffness ซึ่งการออกกาลังกายในผู้ป่วย RA มีหลักการคือ
1. stabilization of the joint
2. increase in range of movement
3. relaxation of antagonists
4. reduction of atrophy
ซึ่งการออกกาลังกายที่เหมาะสม และสามารถ stabilize function of finger ได้มากที่สุดคือ การทา block exercise(
อาจารย์ rehab บอกว่า จริงๆการออกกาลังกายที่ใช้กับ RA มันชื่อว่า Bunnell exercise แหละ ไม่รู้ว่าจา choice มาผิดป่าว
นะ) ซึ่งมีหลักคือ fixed นิ้วอื่นๆ ไว้ แล้วทา active motion ของนิ้วที่เราต้องการให้เต็มที่ range of motion เท่าที่จะทาได้ เพื่อ
ไม่ให้เกิดการติดของข้อ และเพิ่ม ROM ได้อีก ส่วน choice อื่นๆนะ Tension gliding exercise เหมาะกับกรณีของ hand injury
ซึ่ง involve nerve,tendon ได้ ซึ่งเหมาะกับ
Tension gliding exercise ส่วน low-load, high repetitive resistant exercise และ high load, high repetitive resistant
exercise มันจะมีการใช้น้าหนักในการ load (อาจารย์บอก คล้ายๆ traction ) แต่ไม่ค่อยนิยมใช้กับ hand (คาตอบได้จาก
อาจารย์สอน rehab ซึ่งพยายามหาเองแล้ว แต่หาไม่ได้จริงๆครับ)
8. ผู้ป่วยชายอายุ 22 ปี อุบัติเหตุ มีอาการปวดข้อสะโพกขวา ตรวจพบ hip flexion, adduction, internal rotation ถาม
diagnosis
A) Fx pelvic D) Intertrochanteric Fx
B) Fx acetabulum E) Posterior hip dislocation
C) Fx neck of femur

ANS. E) Posterior hip dislocation


พบได้บ่อย 80 % ของ hip dislocation เกิดจากการกระแทกบริเวณเข่า ในท่างอเข่า
งอสะโพกและหุบขา ซึ่งอาจมีแค่ข้อเคลื่อนหลุดอย่างเดียว( simple dislocation) หรืออาจพบร่วมกับ การ
แตกหักของกระดูกด้วย( fracture dislocation)
การวินิจฉัย
1.ปวดรอบสะโพกมาก และปวดมากขึ้นเมื่อมีการเคลื่อนไหว ทาให้มี limit range of motion
2.กดเจ็บบริเวณสะโพกมาก
3.บวม คลาหัวกระดูกต้นขาได้บริเวณใต้กล้ามเนื้อ gluteus maximus
4.ผู้ป่วยจะนอนหงายในท่า flextion, adductionและ internal rotation ของhip ที่โดนinjury
ซึ่งในบางครั้งอาจมีการบาดเจ็บต่อเส้นประสาท Sciatic nerve ร่วมด้วยซึ่งควรตรวจด้วย เสมอ โดย
ตรวจ sensation ของ common peroneal nerve บริเวณหลังเท้า และ Tibial nerve บริเวณ ฝ่าเท้า ตรวจ motor ของ
common peroneal nerve โดยทา dorsiflexion และ Tibial nerve โดยทา
plantar flexion
ตรวจทางรังสีวิทยา
พบว่า head of femur หลุดออกมานอก acetabulum และ head of femur จะมีขนาดเล็กกว่าข้างที่ ปกติ
การรักษา
ทา close reduction ซึ่งมีหลายวิธี ดังนี้
1. Gavity Method ของ Stimson จัดให้ผู้ป่วยนอนคว่า โดยให้ข้อตะโพกออกนอกเตียง ให้ผู้ช่วยกด กระดูก
pelvis บริเวณ sacrum ผู้ป่วยงอเข่า 90 องศา แล้วใช้แรงกดลงตรง ๆ ใต้เข่า อาจขยับ femur Internal หรือ external rotate
เพื่อที่จะให้กระดูกเข้าที่

2. The Allis Manever ให้ผู้ป่วยนอนหงาย ผู้ช่วยกดตะโพกบริเวณ Anterior superior illiac spine ในขณะที่
ผู้ทาดึงขาตามแนว ของกระดูก หลังจากนั้นให้ flex hip 90 องศา อาจขยับ internal external rotate ให้กระดูกเข้าที่ได้

3. The Bigelow Manever จัดทาให้ผู้ป่วยนอนหงายผู้ช่วยกดตะโพกบริเวณ anterior superior

iliac spine ผู้ทาจับข้อเท้าของผู้ป่วย ดึงขาไปตามแนวของ defformity หลังจากนั้นจึงทา

flex hip 90 องศา จนหัวกระดูกเข้า acetabulum


5. ผู้ป่วยเด็กชายอายุ 10 ปี ประสบอุบัติเหตุรถจักรยานยนต์ มีอาการปวดข้อศอก บวมข้างขวา ขยับข้อศอกไม่ได้ PE:
swelling at Rt elbow, triangle landmark at Rt elbow จงให้การวินิจฉัย
A) Dislocation elbow
B) Displaced olecranon fracture
C) Displaced supracondylar fracture
D) Displaced lateral condylar fracture
E) Displaced medial condylar fracture
ตอบ C) Displaced supracondylar fracture
เหตุผล : ข้อนี้เป็น case เด็ก มี swelling and triangle landmark at Rt elbow เป็นเรื่องของการบาดเจ็บที่รอบข้อศอก ซึ่ง
triangle landmark หรืออีกชื่อหนึ่ง Heuter’s triangle มันคือสามเหลี่ยมที่เกิดจาก olecranon process , medial และ lateral
epicondyle โดยในท่าเหยียดตรงปุ่มกระดูกทั้งสามจะเรียงตัวกันเป็นเส้นตรง แต่ถ้างอข้อศอกก็จะเรียงตัวเป็นรูปสามเหลี่ยมหน้า
จั่ว
ซึ่งจากโจทย์มันบอกแค่ว่ามี triangle landmark ซึ่งก็ไม่ได้บอกว่า
ผิดปกติไป ก็เลยคิดว่าเค้าน่าจะหมายถึงว่ายังมี normal triangle
landmark อยู่ ดังนั้นการบาดเจ็บที่จะไม่โดนปุ่มกระดูกทั้งสามคือ
olecranon process , medial และ lateral epicondyle ก็เหลือแค่ข้อ
เดียวคือ Displaced supracondylar fracture

ซึ่ง supracondylar fracture of humerus นี้มันพบได้ไม่บ่อยใน


ผู้ใหญ่ แต่พบว่าเป็นตำแหน่งที่เกิดบ่อยที่สุดบริเวณข้อศอกในเด็ก และพบบ่อยเป็นอันดับสองของกระดูกหักในเด็ก ซึ่งมัน
ก็เข้าได้กับโจทย์ที่เป็น case เด็ก
เลยตอบข้อ C Displaced supracondylar fracture

เพิ่มเติมเกี่ยวกับ supracondylar fracture


- มันแบ่งเป็น extension (95%) เกิดจากหกล้มเอามือยันพื้นในท่าศอกเหยียด และ flexion (5%) เกิดจากหกล้ม
แล้วมีแรงกระแทกโดยตรงด้านหลังต่อศอกที่งออยู่
- อาการจะมีอาการปวด บวม ซึ่งคล้าย Dislocation elbow แต่ถ้าเป็น Dislocation elbow จะทาให้ triangle
มันเสียไป
- Inveatigate : ก็ส่ง film AP , Lat
- Treatment :
ในรายที่ข้อเคลื่อนไม่มากก็ให้ใส่เฝือกในท่า flex elbow 90o , pronation ใส่นาน 3-4 wk
ในรายที่ข้อเคลื่อนมาก แนะนาให้ reduction under GA และดูการจัดกระดูกจาก fluoroscope และ
รักษาแนวกระดูกที่หักด้วย K-wire fixation ซึ่ง indication ของ K-wire fixation คือ
1. open fx
2. reduction แล้วไม่สาเร็จ
3. มี vascular injury ร่วมด้วย
Ref: ออร์โธปิดิกส์ รามาธิบดี 2550
6. หญิง 65 ปี ปวดเข่า มีเสียง crepitation Dx อะไร
a) OA
b) RA
c) gout
d) pseudogout

ตอบ a) OA
เหตุผล : ข้อนี้เป็นผู้หญิง แก่แล้ว (อายุ 65 ปี ^,^) ปวดเข่า มี crep ดูทีละ choice ละกันนะ

OA  มันเป็นได้ในกลุ่มวัยกลางคนและวัยชรา ยิ่งแก่ก็ยิ่งพบได้มาก และเป็นได้ทั้งหญิงและชาย (หญิง > ชาย) จะเป็นมากที่


ข้อที่รับน้าหนักและแรงกระแทกคือ ข้อเข่า ข้อสะโพก ข้อต่อกระดูกสันหลัง หรือข้อที่ใช้งานมากเช่น DIP ก็ได้ ตรวจ
ร่างกายมี joint crepitation ได้จากการเสียดสีของผิวหน้ากระดูกอ่อนที่เกิดการสึกกร่อน จะเห็นว่ามันเข้าได้กับโจทย์
ทุกอย่างเลยตั้งแต่เพศ อายุ อาการ และตรวจร่างกาย จึงตอบข้อนี้

RA  พบในหญิงมากกว่าชาย แต่มักเป็นในวัยกลางคน 30-50 ปี และเป็น symmetrical poly arthritis มากกว่าเป็นข้อเดียว


มักเป็นข้อเล็กๆ เช่น PIP , MCP ข้อนี้จึงตัดไป

Gout  พบในชายมากกว่าหญิง ในชายพบได้มากที่อายุ 35 ปีขั้นไป ส่วนในหญิงจะพบในวัยหลังหมดประจาเดือน มักเป็น


acute monoarthritis ที่มี abruptness of onset อาการจะปวดมากจนเดินไม่ได้เลยก็ได้ ส่วนใหญ่เป็นที่ข้อเท้า หรือนิ้ว
หัวแม่เท้า และมักไม่มี crepitation ข้อนี้ก็ตัดทิ้งไปนะ

Pseudogout  คือ calcium pyrophosphate dihydrate deposition disease พบได้ในเพศหญิงมากกว่าชายและจะมา


ด้วยอาการคล้าย OA ได้ ต่างกันที่ตาแหน่งของ Pseudogout พบบ่อยที่ข้อมือ , MCP ฉะนั้นข้อนี้ก็ตัดทิ้งซะ

แถมๆ เกณฑ์ในการวินิจฉัยเข่าเสื่อม OA knee


1. ปวดเข่าเกือบทุกวันในเดือนที่ผ่านมา
2. ขยับข้อแล้วมีเสียงกรอบแกรบ (crepitation)
3. มีข้อยึดหรือข้อฝืดในตอนเช้าน้อยกว่า 30 นาที
4. อายุตั้งแต่ 38 ปี
5. เข่าบวมขึ้น
จะวินิจฉัยเป็น OA knee เมื่อ มีข้อ 1-4 หรือ มีข้อ 1,2,5 Sensitivity 89% , Specificity 88 %

Ref : ออร์โธปิดิกส์ รามาธิบดี 2550 + short note in medicine ขอนแก่นฯ


7. เด็กชาย 10 ปี MCA มีอาการปวดข้อศอกบวมข้างขวา ขยับข้อศอกไม่ได้ PE:swelling at elbow, triangle landmark
at Rt. Elbow, Dx?
1. Dislocation elbow
2. Displaced olecranon Fx
3. Displaced supracondylar Fx
4. Displaced lateral condylar Fx
5. Displaced medial epicondyla Fx
ตอบ 3) Displaced supracondylar fracture
เหตุผล : ข้อนี้มันซ้้ากะข้อ 5 นะ เป็น case เด็ก มี swelling and triangle landmark at Rt elbow เป็นเรื่องของการบาดเจ็บที่
รอบข้อศอก ซึ่ง triangle landmark หรืออีกชื่อหนึ่ง Heuter’s triangle มันคือสามเหลี่ยมที่เกิดจาก olecranon process ,
medial และ lateral epicondyle โดยในท่าเหยียดตรงปุ่มกระดูกทั้งสามจะเรียงตัวกันเป็นเส้นตรง แต่ถ้างอข้อศอกก็จะเรียงตัว
เป็นรูปสามเหลี่ยมหน้าจั่ว
ซึ่งจากโจทย์มันบอกแค่ว่ามี triangle landmark ซึ่งก็ไม่ได้บอกว่า
ผิดปกติไป ก็เลยคิดว่าเค้าน่าจะหมายถึงว่ายังมี normal triangle
landmark อยู่ ดังนั้นการบาดเจ็บที่จะไม่โดนปุ่มกระดูกทั้งสามคือ
olecranon process , medial และ lateral epicondyle ก็เหลือแค่ข้อ
เดียวคือ Displaced supracondylar fracture

ซึ่ง supracondylar fracture of humerus นี้มันพบได้ไม่บ่อยใน


ผู้ใหญ่ แต่พบว่าเป็นตำแหน่งที่เกิดบ่อยที่สุดบริเวณข้อศอกในเด็ก และพบบ่อยเป็นอันดับสองของกระดูกหักในเด็ก ซึ่งมัน
ก็เข้าได้กับโจทย์ที่เป็น case เด็ก
เลยตอบข้อ C Displaced supracondylar fracture

เพิ่มเติมเกี่ยวกับ supracondylar fracture


- มันแบ่งเป็น extension (95%) เกิดจากหกล้มเอามือยันพื้นในท่าศอกเหยียด และ flexion (5%) เกิดจากหกล้ม
แล้วมีแรงกระแทกโดยตรงด้านหลังต่อศอกที่งออยู่
- อาการจะมีอาการปวด บวม ซึ่งคล้าย Dislocation elbow แต่ถ้าเป็น Dislocation elbow จะทาให้ triangle
มันเสียไป
- Inveatigate : ก็ส่ง film AP , Lat
- Treatment :
ในรายที่ข้อเคลื่อนไม่มากก็ให้ใส่เฝือกในท่า flex elbow 90o , pronation ใส่นาน 3-4 wk
ในรายที่ข้อเคลื่อนมาก แนะนาให้ reduction under GA และดูการจัดกระดูกจาก fluoroscope และ
รักษาแนวกระดูกที่หักด้วย K-wire fixation ซึ่ง indication ของ K-wire fixation คือ
4. open fx
5. reduction แล้วไม่สาเร็จ
6. มี vascular injury ร่วมด้วย
Ref: ออร์โธปิดิกส์ รามาธิบดี 2550
8. ผู้ป่วยชาย 25 ปี มาด้วย Motorcycle accident
PE: Leg - internal rotation, adduction, hip flexion จง Dx.
a. Posterior hip dislocation

ตอบ a. posterior hip dislocation


เหตุผล : Posterior hip dislocation เป็นข้อสะโพกเคลื่อนชนิดที่พบได้บ่อยที่สุด โดยจะมีลักษณะของขาที่ผิดรูปไปคือ flexion-
adduction-internal rotation ตามรูป

flexion-adduction-
posterior hip
internal rotation
dislocation

กลไกการบาดเจ็บ มักเกิดจากการกระแทกบริเวณเข่าในท่างอเข่า งอสะโพก และหุบขาโดยถ้าสะโพกงอมากขาหุบเข้าในก็จะ


เกิด simple dislocation แต่ถ้าสะโพกงอไม่มาก ขากางออก femur head ก็จะกระแทกกับขอบเบ้าทาให้ acetabulur rim แตก
ร่วมด้วย

เพิ่มเติมเกี่ยวกับ hip dislocation แบบต่างๆ

posterior hip dislocation  flexion-adduction-internal rotation


anterior hip dislocation  1. Obturator tupe : flexion –abduction-external rotation
 2. iliac type : extension-abduction-external rotation
Central dislocation  ขาจะสั้นลงเนื่องจากส่วน anterior and/or posterior column ของเบ้าเตก
ร่วมกับ head of femur ถูกดันทะลุเข้าไปในอุ้งเชิงกราน และอาจพบรอย
ฟกช้าที่ greater trochanter

Ref: ออร์โธปิดิกส์ รามาธิบดี 2550


9. ผู้ป่วยชายอายุ 59 ปี ปวดคอ ตรวจพบ Biceps อ่อนแรง และ reflex ลด น่าจะมีพยาธิสภาพที่รากประสาทใด
1. C5 4. C8
2. C6 5. T1
3. C7
ตอบข้อ 1. C5 อันนี้ทุกคนก็คงจะทราบกันแล้วเกี่ยวกับการตรวจ deep tendon reflex ว่า
Biceps C5-C6 Knee L3-L4
Triceps C7-C8 Ankle S1-S2
Wrist C6-C7
11. ผู้ป่วยชายอายุ 20 ปี หกล้มขณะเล่นฟุตบอล ไหล่ขวาบวม ทา X-ray พบ fracture neck of humerus กล้ามเนื้อใดที่อาจเกิด
อาการอ่อนแรงได้
1. Deltoid 4. Triceps
2. Brachialis 5. Coracobrachialis
3. Biceps
ตอบข้อ 1. deltoid เพราะการ injury ของ neck of humerus จะทาให้เกิดการ injury ต่อ axillary nerve ซึ่งไปเลี้ยง deltoid
นั่นเอง

12. ผู้ป่วยหญิง 52 yr ปวดมือขวาตอนกลางคืน ลุกมาสะบัดแล้วดีขึ้น PE: sensory intact, no thenar atrophy, Phalen test +ve
Tx อะไร
1. Observation 4. Long arm cast
2. NSAIDS 5. A1 pulley release
3. Epidural steroid injection
จากที่โจทย์บอกมาน่าจะเป็นอาการของ Carpal tunnel syndrome เกิดจากการกดทับ median nerve ในบริเวณข้อมือเป็น
compression neuropathy ที่พบบ่อยที่สุดใน upper extremity
Clinical picture คือ ปวด-ชา มือทางด้าน palmar radial ในบริเวณที่เลี้ยงด้วย median nerve ในบางครั้งผู้ป่วยอาจมี
อาการปวดในส่วนสะบักและแขนได้ ผู้ป่วยมักมีอาการมากในช่วงตอนกลางคืน หรือในช่วงที่ทางานหนักซ้าๆ ซากๆ
ในรายที่เป็นรุนแรงจะมีการอ่อนแรงและลีบเล็กของ Thenar muscle
การรักษา มีหลายแบบโดย NSAID ในกรณีที่มี synovitis, steroid injection ซึ่งเค้าเข้าใจว่าโจทย์ที่ให้มาไม่น่าครบ
เนื่องจากควรบอกระยะเวลาว่าเป็นมานานเท่าไหร่และบ่อยแค่ไหน ซึ่งแน่ๆ ที่น่าจะตัดได้คือ choice ข้อ 3 เพราะการฉีด
steroid ไม่ได้ฉีด epidural แต่ฉีดที่บริเวณ carpal tunnel เลย (บริเวณข้างๆ Palmaris longus น่ะ) ข้อ 4 ก็ตัดได้เพราะการ
ใส่ long arm cast จะดูเวอร์ไปนะ ส่วนข้อ 5 ตัดได้เพราะ A1 pulley release นี่ไว้รักษา trigger finger ไม่ใช่ carpal tunnel
นะจ๊ะ ดังนั้นถ้าจะให้เดาเค้าก็คงจะตอบ ข้อ 2 NSAID นะ...

13. ผู้ป่วยชายอายุ 22 ปี มีประวัติว่า 10 เดือนก่อนมา รพ. ประสบอุบัติเหตุรถจักรยานยนต์ชนกัน มีอาการบาดเจ็บที่ไขสันหลัง


ระดับ T2 ลงมา ได้คาสายสวนปัสสาวะไว้ตลอดเวลา ขณะทากายภาพพบว่า ผู้ป่วยมีอาการปวดศีรษะ อึดอัด แน่น เหงื่อ
ออกมาก วัด BP 180/110, PR 56 bpm จง Mx
1. เอาสาย Foley catheter ออก
2. ให้ผู้ป่วยนั่งและปลดเสื้อผ้าออก
3. ให้ยา Hydralazine
4. นอนหัวต่าและทาการวัด BP ซ้า
5. ให้กาลังใจกับผู้ป่วยว่าอาการที่เกิดขึ้นเป็นปกติ
จากที่โจทย์บอกมาเป็นอาการของ Autonomic dysreflexia ซึ่งเป็นอาการของ over-activity of the Autonomic Nervous
System จะเกิดในคนไข้ที่บาดเจ็บที่ไขสันหลังในระดับ T6 ขึ้นไปสาเหตุเกิดจาก irritating stimulus ที่ระดับต่ากว่าที่ spinal
cord injury เช่น

Bladder (most common) - from overstretch or irritation of bladder wall


A. Urinary tract infection D. Overfilled collection bag
B. Urinary retention E. Non-compliance with intermittent
catheterization program
C. Blocked catheter
และอีกหลายๆ สาเหตุ

Signs & Symptoms

 Pounding headache  Flushed (reddened) face


(caused by the elevation in blood pressure)
 Red blotches on the skin above level of
 Goose Pimples spinal injury

 Sweating above the level of injury  Sweating above level of spinal injury

 Nasal Congestion  Nausea

 Slow Pulse  Slow pulse (< 60 beats per minute)

 Blotching of the Skin  Cold, clammy skin below level of spinal


injury
 Restlessness

 Hypertension (blood pressure greater than


200/100)
การรักษาแบ่งเป็น
 Immediate คือ sitting position, but do a pressure release immediately. You may transfer yourself to bed, but always
keep your head elevated
 หาสาเหตุซึ่งก็มีหลายอย่างมากมายแต่ที่ต้อง check และเป็น most common คือที่บอกไว้ข้างบน
 กรณีที่หาสาเหตุเพื่อที่จะแก้ไขไม่ได้ก็ medication คือ Immediate/emergent

o Procardia - 10 mg. p.o./sublingual

o Nitroglycerine - 1/150 sublingual or 1/2 inch Nitropaste topically

o Clonidine - 0.1 to 0.2 mg. p.o.

o Hydralazine - 10 to 20 mg. IM/IV

 Chronic (recurrent episode prevention)

o Prazosin ("Minipress") - 0.5 to 1.0 mg. daily

o Clonidine ("Catapres") - 0.2 mg. p.o. b.i.d.


ดังนั้นจึงคิดว่าข้อนี้อันดับแรกควรตอบข้อ 2 คือให้ผู้ป่วยนั่งและปลดเสื้อผ้าออกก่อน
14. Pt หญิง 69 yr อ้วน ,เป็น OA ปวดเข่าข้างขวา มี genu varus แนะนาอย่างไร
1. ออกกาลังแบบ isotonic
2. ใช้ไม้เท้าข้างซ้าย
3. ออกกาลัง isometric เพิ่ม Hamstring strength
4. เสริมรองเท้าด้าน medial

Ans. 2. ใช้ไม้เท้าข้างซ้าย
เกณฑ์ในการวินิจฉัยโรคข้อเข่าเสื่อม ตาม American College of Rheumatology(15)
- มีอาการปวดเข่า
- ภาพรังสีแสดง osteophyte
- มีข้อสนับสนุน 1 ข้อดังต่อไปนี้
1. อายุเกิน 50 ปี
2. อาการฝืดแข็งในตอนเช้า ประมาณ 30 นาที
3. มีเสียงกรอบแกรบขณะเคลื่อนไหวเข่า
แนวทางการรักษา OA
1. การออกกาลังและการบริหารกล้ามเนื้อรอบเข่า
- การบริหารเพื่อเพิ่มพิสัยการเคลื่อนไหวข้อเข่า (range of motion/ flexibility exercise)
- การบริหารเพื่อเพิ่มความแข็งแรงของกล้ามเนื้อหน้าขาและกล้ามเนื้อท้องขา (ทา isometric ขณะมีอาการปวดมาก แต่ถ้าไม่มี
อาการปวดแนะนาให้ทา isotonic) โดยเน้น strengthening quadriceps , stretching hamstring
- การออกกาลังแบบแอโรบิค ได้แก่ การเดินช้าๆ การปั่นจักรยาน หรือการออกกาลังกายในน้าซึ่งจะดีมาก
3. การใช้อุปกรณ์เครื่องช่วยต่างๆ
- การใช้ไม้เท้าหรือร่มควรถือไม้เท้าหรือร่มในมือด้านตรงข้ามกับข้างที่ปวด
- การเสริมรองเท้าเป็นลิ่มทางด้านนอก (heel wedging) ในผู้ที่เริ่มมีขาโก่งน้อยๆ จะช่วยลด external varus moment และ
medial compartment load
4. การลดน้าหนัก
5. การใช้วิธีการอื่นๆ ได้แก่ เลเซอร์ Transcutaneus electrical nerve stimulation (TENS) การ
ฝังเข็ม การใช้ความร้อน
6. การรักษาด้วยยา

14. ผู้ป่วยชาย มีอาการปวดคอและร้าวไปยังสะบัก มีอาการชาไปตามแขนข้างซ้าย มีอาการอ่อนแรงไป ทา Cervical traction


แล้วมีอาการเท่าเดิม ไม่ดีขึ้นท่านจะทาอย่างไรต่อ
1. MRI
2. Cervical traction เพิ่มนน. เป็น 8 kg
3. หยุด Cervical traction
4. Heat
Ans. cervical traction เพิ่มน้าหนักเป็น 8 kg.
เนื่องจากในผู้ป่วยรายนี้มีภาวะ spondylosis ซึ่งการ management ที่ควรทา ก็คือ ส่ง film c-spine และอาจจะ
รักษาโดย cervical traction ซึ่งในผู้ป่วยรายนี้ไม่มี contraindication ในการทา โดยใช้น้าหนักอยู่ที่ประมาณ 5 – 10 kg.
ถ้าหากอาการไม่ดีขึ้นอาจจะพิจารณาเพิ่มน้าหนักที่ถ่วงก่อน หรือถ้าหากอาการยังไม่ดีควรส่ง investigate เพิ่มเติม เป็น CT
โรคที่เป็นสาเหตุของอาการปวดคอที่พบได้บ่อย ได้แก่
1. whiplash injury การบาดเจ็บเนื่องจากคอสะบัด ผู้ป่วยมักจะมีอาการปวดคอและศีรษะ คอแข็งปวดเวลาขยับ
คอ ถ้ารากประสาทได้รับบาดเจ็บร่วมด้วยจะมีอาการปวดร้าวชนิด radicular pain ลงไปที่แขนหรือสะบัก ถ้า x-ray พบ
lordotic curve หายไป
2. cervical disc syndrome อาจจะเกิดจากการเคลื่อนหรือการเสื่อมก็ได้ พบว่าผู้ป่วยมักจะมีอาการปวดคอ จะ
สัมพันธ์กับภาวะที่มีแรงดันในหมอนรองกระดูกสูงขึ้น และปวดร้าวไปที่แขนร่วมด้วย แต่อาการปวดร้าวมักจะไม่เป็นไหตาม
dermatome
3. cervical spondylosis ผู้ป่วยจะมีอาการปวดคอ อาจมีอาการชาที่แขนได้
4. myofascial pain syndrome
Traction เป็นเทคนิคในการใช้แรงดึงกระทาต่อส่วนหนึ่งของร่างกายเพื่อยึดเนื้อเยื่อ ข้อต่อ และกระดูกเพื่อ
วัตถุประสงค์ในการบาบัดรักษา
Contraindication to traction
- osteomyelitis or discitis , infection
- primary bone tumor or spinal cord tumor
- unstable fracture , myelopathy
- severe osteoporosis
- hypertension
- cardiovascular disease

15. ผู้ป่วยชายอายุ 25 ปี ตรวจร่างกายพบว่ามี Fracture head of humerus ถามว่าจะมีผลต่อกล้ามเนื้อมัดใดมากที่สุด


a. Brachioradialis m.
b. Deltoid m.
c. Brachialis m.
d. Coracobrachialis m.
e. Biceps m.

Ans. b. Deltoid m.
fracture of head of humerus มักจะเกิดจากการหกล้มในท่าแขนเหยียด จะมีอาการ ปวด บวม กดเจ็บตรงหัวไหล่
ควรตรวจดูเส้นเลือดและเส้นประสาท โดยเฉพาะ axillary nerve เพราะอาจเกิด injury ต่อ axillary nerve มากที่สุด การ
รักษา คือ ORIF
กล้ามเนื้อและ nerve innervation ของแต่ละข้อ
a. Brachioradialis m. innervated by radial nerve
b. Deltoid m. innervated by axillary nerve
c. Brachialis m. innervated by musculocutaneous nerve
d. Coracobrachialis m. innervated by musculocutaneous nerve
e. Biceps m. innervated by musculocutaneous nerve

ดังนั้นข้อนี้จึงตอบ deltoid muscle


เสริม : ถ้าโจทย์ถาม fracture shaft of humerus จะต้องระวัง radial nerve ถูกกระแทกหรือถูกหนีบจากปลายกระดูกที่
หัก มักจะเป็นการหักตรงตาแหน่งปลายกระดูกต้นแขน หักแบบเฉียงขึ้นและออกไปด้านนอก (Holdstein – Lewis fracture)
ซึ่งจะ innervation muscles in the posterior compartment of the arm and forearm

16. ผู้ป่วยหญิงอายุ 90 ปี ปวดคอร้าวลงแขนซ้ายมาประมาณ 1 เดือน ได้ทา cervical traction วันละครั้ง 30 นาที หนัก 5-8 kg
มา 1 wk ไม่ดีขึ้น และขาทั้งสองข้างแข็งเกร็งเดินลาบาก ทาอย่างไรดี
a. เพิ่มน้าหนัก traction
b. เปลี่ยน traction ไปที่เอว
c. เพิ่ม physical modality อื่น
d. หยุดการใช้ traction then MRI
e. ให้ยาลดเกร็ง + ยาแก้ปวด

Ans d. หยุดการใช้ traction then MRI


เนื่องจากผู้ป่วยรายนี้มีอาการปวดคอร้าวลงแขนซ้าย และมีอาการขาทั้งสองข้างแข็งเกร็งเดินลาบาก จึงทาให้นึกถึง
ภาวะ cervical spondylotic myelopathy ที่สุด สาหรับการรักษาที่เหมาะสมนั้นควรจะเลือกเป็น MRI เนื่องจากจะสามารถ
เห็นพวกเนื้อเยื่อต่าง ๆ รวมถึงไขสันหลัง หมอนรองกระดูก ได้ชัดเจน และสาหรับการทา cervical traction นั้น ภาวะ
cervical spondylotic myelopathy จัดเป็น contraindication ในการทา cervical traction
cervical spondylotic myelopathy
พบได้บ่อยในผู้สูงอายุ สาเหตุมาจากการเสื่อมของกระดูกสันหลังส่วนคอ และมีการกดทับรากประสาทไขสันหลัง
จนเกิดการสูญเสียการทางานของระบบประสาทไขสันหลัง จะเริ่มมีอาการปวดร้าวจากคอไปที่แขนตามราก
ประสาทที่ถูกกด นอกจากนี้จะพบอาการแสดงจากการที่ประสาทไขสันหลังถูกกด การตรวจพบ long tract
sign (แขนขาเกร็ง อ่อนแรง)
17. ชายประสบอุบัติเหตุรถชน ได้รับการทา Rt. Below knee amputation แพทย์พัน stump ไว้ ควรนัดผู้ป่วยมาใส่ขาเทียม
ถาวรหลังผ่าตัดเมื่อใด
A. 7 วัน D. 30 วัน
B. 10 วัน E. 45 วันs
C. 15 วัน
Ans 7 วัน - รับความรู้สึกได้ดี
Post operative rehab. - ความยาวเหมาะสม
Stump ที่พึงประสงค์ - ไม่ปวดจาก neuroma หรือ phantom pain
- แผลหายตามกาหนด โดย Above knee amputation ใส่ขาเทียม
- กล้ามเนื้อและไขมันพอดี ต้องมี หลังผ่าตัด 2 สัปดาห์
- กล้ามเนื้อแข็งแรง ไม่ลีบ Below knee amputation ใส่ขาเทียม
- ข้อไม่ติด หลังผ่าตัด 1 สัปดาห์
- ระบบไหลเวียนดี
18. Erb’s palsy ถามหาตาแหน่ง Lesion
Ans. C5-7
Erb’s palsy (brachial plexus palsy, Erb-Duchenne paralysis )
- paralysis of the arm caused by injury to the upper group of the arm's main nerves
(specifically, C5-C6), almost always occurring during birth.
-
พบมากในเด็กที่มีน้าหนักตัวมาก ทาให้คลอดยากและติดอยู่นาน
- Clinical : อาการจะเกิดทันทีหลังเกิด ทารกจะมีแขนอ่อนปวกเปียก ไม่ขยับแม้เวลาที่ร้อง ในกรณีที่ทารกมีการ
ขากออกซิเจนร่วมด้วยหรือซึมมาก อาจเห็นอาการไม่ชัดเจน
อาการจาเพาะ : ทารกจะมี adduction และ internal rotation ของไหล่ ร่วมกับมี
extension และ pronation ของข้อศอก ข้อมือจะงอพับเข้า ทารกบางคนอาจมีการบาดเจ็บของ phrenic nerve
เกิด paralysis of diaphragm ได้
- Investigation : chest x-ray เพื่อดูว่ามีกระดูกหักร่วมด้วยหรือไม่
Fluoroscope ดูการ movement ของ diaphragm
- Diagnosis : อาศัยอาการทางคลินิก
- Treatment : 80-90% สามารถ recovery ได้ โดยทาร่วมกับการกายภาพบาบัด เพื่อป้องกันการยืดงอของข้อแขน
โดย Passive movement exercise ควรเริ่มหลังจากทารกอายุได้ 1 wk. กลุ่มที่อาการน้อยจะดีขึ้นเองภายใน 1-2
wk. กลุ่มที่มีอาการรุนแรงอาจใช้เวลาเป็นเดือน หรือไม่ดีขี้นเลย อาจต้องทาการผ่าตัดแก้ไข

19. Rheumatoid arthritis ออกกาลังกายที่ข้อนิ้วมืออย่างไร


Ans. การออกกาลังกายเพื่อการรักษา จะเน้นเพิ่มความแข็งแรงของ finger extensor , wrist extensor , foot intrinsic,
quadriceps muscle และ เพิ่มความยืดหยุ่นของกล้ามเนื้อกลุ่ม toe extensor, peroneus, Gastrocnemius,
Hamstring, Hip flexor และ Hand intrinsic m.
- ท่าแรก ท่าพนมมือ โดยให้ผู้ป่วยพนมมือ ใช้แรงพอควรดันเข้าหากัน จะช่วยทาให้ข้อไม่ยึดในท่างอนิ้ว
และช่วยยืดข้อออก ทาให้ทางานได้ นอกจากนี้การออกแรงพอควร ทาให้ช่วยบริหารข้อไหล่ , ศอก และข้อมือด้วย เมื่อ
พนมมือสักครู่ ให้ยกมือขึ้น บนเหนือศีรษะ และยืดให้สุด แล้วยืดแขนออกมาด้านหน้า
- ท่าที่สอง (ถ้ามีข้อเข่า ข้อเท้า อักเสบด้วย) ให้นั่งเก้าอี้ห้อยเท้า ยกขาขึ้นให้เข่าตรง และกระดกหลังเท้าขึ้น ทาท่านี้ค้าง
ไว้ นับ 1-10 (ประมาณ 10 วินาที) แล้ววางลง ทาสลับข้าง ซ้าย-ขวา ทาทั้งสองท่าบ่อยๆ ทุกวัน (อย่างน้อย วันละ 10-
20 ครั้ง) จะสามารถป้องกันความพิการได้
20. ชายอายุ 59 ปี มาด้วยอากาอ่อนแรงของกล้ามเนื้อ Biceps,Bicep reflex ลดลง ถามว่าเสียที่รากประสาทใด
A. C5 D. C8

B. C6 E. T1

C. C7

Ans. C5
Key muscle : C5 - Biceps brachii L2 - Hip flexor
C6 - ECR L3 - Knee extensor
C7 - Tricep brachii L4 - Tibialis anterior
C8 - FDP L5 - EHL
T1 - ADM S1 - Gastrocnemius

17. carpal tunnel syndrome ถาม tx :


1. observe ต่อ
2. tendon release
3. NSAIDs
เฉลย การรักษา carpal tunnel syndromeแบ่งการักษาออกเป็น (ข้อนี้ไม่บอกอะไรอันจะไดไหนเลย ไม่รู้จะเลือกข้อไหน )
 Non – operative therapy : ใส่ splint ในท่า neutral position o องศา
NSAID กรณีมี synovitis
Diuretic กรณ๊มี edema
รักษา underling disease
 Steroid injection : จะมีผลดีกรณีที่  Operative therapy : กรณีที่รักษาตามข้างต้นอาการ
o อาการน้อยกว่า 1 ปี ไม่ดีขึ้น การผ่าตัดสามารถทาได้หลายวิธี คือ
o อาการชาไม่เป็นตลอดเวลา o Classic open carpal tunnel release
o ไม่พบ thenar atrophy o Limited incision technique
o ไม่มีอาการอ่อนแรง o Endoscopic carpal tunnel release
o Two-point discrimination ปกติ
22.ผู้ป่วยชายปวดคอ ชาตามหัวไหล่ มีนิ้วอ่อนแรง แพทย์รักษาโดย C-spine traction ต่อมามีแขน-
ขาอ่อนแรง Mx อย่างไรต่อ : หยุดทาแล้วส่ง MRI
เฉลย จากอาการของผู้ป่วยแสดงลักษณะของ Cervical spine disease (ไม่แน่ใจจาก trauma, stenosis หรือ disc
herniation) แต่ทาการรักษาเป็น conservative treatment
ต่อมาพบ progressive neurological deficit ที่ชัดเจนโดยข้อบ่งชี้ที่ควรจะ investigateเพิ่มเติม หรือ conservative
treatment ไม่ได้ผลคือ
 predominantly brachial pain failure to respond to four weeks of conservative therapy ควรพิจารณา
ทา MRI for evaluation
 Worsening of brachial pain during adequate conservative therapy
 neurologic deficit in the upper or lower extremities
23. ผู้ป่วยเด็ก แขนหัก triangular landmark ปกติ จงวินิจฉัย
a) supracondylar fracture
b) lateral condylar fracture
c) olecranon fracture
d)...
เฉลย triangular landmark หรือ isosceles triangle
ประกอบด้วย olecranon process และ medial and
Lateral epocondye
ดังนั้น fracture ที่มีผลทาให้ไม่ให้เกิด triangle
ผิดปกติต้องไม่มีผลต่อ bony landmark ดังกล่าว

a) Supracondylar fracture
พบบ่อยที่สุดในกระดูกหักบริเวณข้อศอก isosceles triangle มักปกติวินิจฉัยจาก Baumann angle 70 – 780 ลาก
จาก long axis of humerus และ physis of lateral condyle

b) Lateral condylar fracture


2nd most common fracture of elbow มีผลต่อ isosceles triangle โดยตรง
c) Olecranon fracture
ล้มข้อศอกกระแทกหรือ triceps กระชาก มีผลต่อ isosceles triangle โดยตรง
Psychiatry
1. ผู้ป่วยอายุ 35 ปี รู้สึกอันตราย แน่นหน้าอก เหนื่อย ใจสั่น เหงื่อออก กลัวตาย ยาตัวใดเหมาะสมที่สุดในผู้ป่วยรายนี้
A. Midazolam
B. Lorazepam
C. Clonazepam
D. Temazepam
E. CDZ

ตอบ C. Clonazepam
ข้อนี้ จากอาการของผู้ป่วย น่าจะเป็น panic disorder คือ มีอาการ panic attack
การรักษา ก็นิยมใช้ ยากลุ่ม SSRI โดยให้ fluoxetine 10 mg/day เพิ่มได้ 20-40 mg/day กินหลังอาหารเช้า
หรืออาจให้ antidepressant กลุ่ม Tricyclic โดยให้ imipramine 25 mg ในสัปดาห์แรก แลัวค่อยปรับยาเพิ่มขึ้น 25 mg/wk
แล้วก็จะให้ยากลุ่ม BZD ร่วมไปด้วยในตอนแรก เพื่อควบคุมอาการและให้ยาต้านโรคซึมเศร้าออกฤทธิ์เต็มที่ โดยมักให้กันก็คือ
Alprazolam 2-4 mg/day นาน 4-6 wk หรือให้ Clonazepam 0.25 mg วันละ 2 ครั้ง แล้วค่อยๆปรับเพิ่มขนาด
ข้อดีของ Clonazepam คือ มีค่าครึ่งชีวิตยาว ทาให้ไม่มี interdose rebound

2. ผู้ป่วยหญิงหงุดหงิดง่าย นั่งไม่อยู่กับที่ มีประวัติใช้ยาจิตเวชใดมากที่สุด


A. Thioridazine
B. Risperidone
C. Haloperidol
D. Chlorpromazine
E. Clozapine

ตอบ C. Haloperidol
ข้อนี้ไม่แน่ใจนะไม่เข้าใจโจทย์ เลยคิดว่า คงเป็น side effect ของยาตัวใด
อาการหงุดหงิดง่าย นั่งไม่อยู่กับที่ น่าจะเป็น อาการ EPS เป็น Akathisia คือ ผู้ป่วย รู้สึกกระวนกระวายในใจ อยู่นิ่งไม่ได้ นั่งไม่
ติด ต้องขยับแขนขา ลุกเดินไปมา
ดังนั้นเลยคิดว่า ยาตัวที่เกิด EPS มากสุดน่าจะถูกต้อง เลยคิดว่าเป็น Haloperidol

3. ผู้ป่วยเด็กหญิงอายุ 9 ปี ไม่ยอมแยกจากพ่อแม่ กลัวแม่ทิ้ง ปวดหัวและปวดท้องเป็นประจา ไม่ไปโรงเรียน 3 สัปดาห์ กลัว


เหตุร้ายพลัดพรากจากแม่ ฝันร้ายบ่อยๆ กลัวตาย ผลการวินิจฉัย คือ
A. sibling rivalry
B. Schizophrenia
C. simple phobia
D. simple reaction
E. separation anxiety disorder

ตอบ E. separation anxiety disorder


การวินิจฉัย
A. มีความกังวลเกินควรและไม่เท่ากับระดับพัฒนาการในเรื่องของการต้องห่างจากบ้าน หรือจากคนที่ผู้นั้นรู้สึกผูกพัน โดย
เห็นได้จากอาการ 3 ข้อ หรือมากกว่า
1. รู้สึกไม่สบายใจอย่างเกินควรเสมอเมื่อต้อง หรือคาดว่าต้องห่างจากบ้านหรือคนที่ผูกพันสนิทด้วย
2. กังวลอย่างมากเสมอกับการสูญเสียหรือการเกิดอันตรายกับคนที่ผูกพันสนิทด้วย
3. กังวลอย่างมากเสมอกับเหตุการณ์ร้ายที่อาจนาไปสู่การพลัดพรากกับคนที่ผูกพันสนิทด้วย
4. ไม่เต็มใจหรือไม่ยอมไปโรงเรียนหรือที่อื่นเนื่องจากกลัวพลัดพราก
5. กลัวอย่างมากเสมอหรือไม่เต็มใจที่จะอยู่คนเดียว หรืหากไม่มีคนที่ผูกพันสนิทด้วยอยูที่บ้าน หรือหากไม่มี
ผู้ใหญ่ที่วางใจอยู่ด้วยในสถานที่แห่งอื่น
6. ลังเลหรือปฎิเสธการเข้านอนโดยไม่มีคนที่ผูกพันสนิทอยู่ด้วย หรือหากไปนอนค้างนอกบ้าน
7. ฝันร้ายซ้าๆเรื่องการพลัดพราก
8. บ่นอาการทางร่างกายบ่อยๆ (เช่น ปวดหัว ปวดท้อง คลื่นไส้ อาเจียน )เมื่อต้องหรือคาดว่าต้องห่างจากคนที่
ผูกพันอย่างมาก
4. ผู้ป่วยหญิง อายุ 45 ปี มีอาการซึมเศร้า ตื่นตัว ไม่อยากทาอะไร เบื่ออาหาร ข้อใดช่วยวินิจฉัยภาวะซึมเศร้าได้ดีที่สุด
A. ปวดหัวบ่อย ๆ
B. ฝันร้ายบ่อย ๆ
C. ตกใจง่าย
D. ทาแล้วกลัวผิด
E. รู้สึกว่าตนเองทาผิด
เฉลย ข้อ E

จากคาถามภาวะซึมเศร้า น่าจะหมายถึง Major Depressive Disorder


เกณฑ์การวินิจฉัย Major Depressive Disorder

A. มีอาการต่อไปนี้อย่างน้อย 5 ข้อ โดยอย่างน้อยต้องมีข้อ 1. หรือ ข้อ 2. หนึ่งข้อ


1. ซึมเศร้า
2. ความสนใจหรือความเพลินใจในสิ่งต่างๆ ลดลงอย่างมาก
3. เบื่ออาหาร หรือน้าหนักลงลง มากกว่าร้อยละ 5 ใน 1 เดือน
4. นอนไม่หลับ หรือนอนมากกว่าปกติ
5. Psychomotor agitation OR retardation
6. อ่อนเพลีย ไม่มีแรง
7. รู้สึกตนเองไร้ค่า หรือรู้สึกผิด ------- ข้อ E
8. สมาธิลดลง ลังเลใจ
9. คิดเรื่องการตาย หรือการฆ่าตัวตาย
B. อาการเหล่านี้ทาให้ผู้ป่วยทุกข์ทรมาน หรือทาให้การประกอบอาชีพ การเข้าสังคม หรือหน้าที่ด้านอื่น
ที่สาคัญบกพร่องลงอย่างชัดเจน
5. หญิงไทย อายุ 25 ปี เข้ารับการผ่าตัดไส้ติ่งอักเสบ หลังผ่าตัด 3 วัน ได้ข่าวว่าสามีประสบอุบัติเหตุเสียชีวิต
ผู้ป่วยมีอาการซึมเศร้า ไม่พูด เบื่ออาหาร ร้องไห้ทั้งคืน นอนไม่หลับ เห็นสามีมานั่งข้างเตียงทุกวันอาการเป็นคงที่
ตลอด 2 สัปดาห์ ตรวจร่างกายปกติ จงให้การวินิจฉัย
A. Normal grief reaction
B. Delirium
C. Brief psychosis
D. Adjustment disorder with depress mood
E. MDD with psychotic feature
เฉลย
Normal grief reaction คือ The normal process of reacting to a loss. The loss may be physical
(such as a death), social (such as divorce), or occupational (such as a job).
ลักษณะทางคลินิก ได้แก่ Loss of appetite, Changes in weight, Sleep disturbance, Fatigue
Headache, Palpitations , Gastrointestinal distress, hallucination, Depression
Delirium ลักษณะอาการ ได้แก่ Disturbance of consciousness, Disorientation, Attention deficit,
Sleep-wake disturbance, Illusion, Disorganized thinking สาเหตุมักเกิดจาก การติดเชื้อ ขาดการใช้สารเสพติด
ขาดสุราหรือยานอนหลับ ได้รับสารพิษ โรคหัวใจหลอดเลือด
โรคสมองเสื่อม
Brief psychotic disorder มีอาการโรคจิตเกิดขึ้นอย่างรวดเร็ว มักเป็นหลังจากประสบเหตุการณ์กดดัน
รุนแรง
ผู้ป่วยจะมีอาการพูดจาสับสน พบบ่อยว่ามีประสาทหลอน ระยะเวลาดาเนินโรคอย่างน้อย 1 วัน ไม่เกิด 1 เดือน
เมื่อหายจะกลับเป็นปกติ

Adjustment disorder with depress mood มีสาเหตุจากภาวะความกดดันที่ทาให้ผู้ป่วยเกิดความตึงเครียดจนไม่


สามารถปรับตัวได้อย่างเหมาะสม อาการที่เด่นเป็น อารมณ์เศร้า เสียใจ สิ้นหลวง ไม่มีอาการหลงผิด
หรือประสาทหลอน

6. ชายอายุ 24 ปี underlying Schizophrenia ช่วยเหลือตนเอง ทางานบ้านได้ เมื่อวานไปตากผ้าที่กันสาด


กลิ้งตกลงมาศรีษะกระแทกพื้น สลบไป 20 นาที วันนี้มีอาการไม่ยอมพูด เดินไปเดินมา ผุดลุกผุดนั่ง
ปัสสาวะใส่อ่างน้า ไม่กิน ไม่นอน จงให้การวินิจฉัย
A. Delirium
B. Akathisia
C. Acute stress disorder
D. Relapsed schizophrenia
E. Brief psychotic disorder
เฉลย A. Delirium
Delirium ลักษณะอาการ ได้แก่ Disturbance of consciousness, Disorientation, Attention deficit,
Sleep-wake disturbance, Illusion, Disorganized thinking
สาเหตุมักเกิดจาก การติดเชื้อ ขาดการใช้สารเสพติด ขาดสุราหรือยานอนหลับ ได้รับสารพิษ โรคหัวใจหลอดเลือด
โรคสมองเสื่อม
Akathisia เป็นผลข้างเคียงจากการใช้ยา Dopamine antagonist ซึ่งเป็นยา Antipsychotic drug
ลักษณะอาการ คือ รู้สึกกระวนกระวายใจ รู้สึกไม่สบายกล้ามเนื้อ โดยเฉพาะขา ทาให้ผู้ป่วยนั่งนิ่งไม่ได้
ต้องเดินไปมา หรือโยกตัว เขย่าขาจนอาจคล้ายผู้ป่วยมีอาการทางจิตเลวลงได้

Acute stress disorder เป็นโรควิตกกังวล (Anxiety disorder) ที่เกิดขึ้นภายหลังจากที่ผู้ป่วยเผชิญกับ


เหตุการณ์ที่รุนแรง เช่น ภาวะสงคราม อุทกภัย วินาศภัย ถูกทาร้ายร่างกาย อุบัติเหตุร้ายแรง หรือข่มขืน
ลักษณะทางคลินิก ได้แก่ รู้สึกสิ้นหวัง มีความกลัวที่รุนแรง คิดคานึงถึงเหตุการณ์นั้นซ้าๆ (Re-experienced)
ทั้งขณะตื่น และหลับ มีอาการตื่นเต้นตกใจง่ายทั้งๆ ที่ไม่เคยเป็นมาก่อน (Hyperarousal) หลีกเลี่ยงเหตุการณ์ หรือสิ่งที่
เกี่ยวพันกับเหตุการณ์นั้น (Avoidance)

Relapsed schizophrenia พบได้ในผู้ป่วยที่ทานยาไม่สม่าเสมอหลังการรักษา ลักษณะอาการพบ อาการหลงผิด


ประสาทหลอน , Disorganized speech, Grossly disorganized behavior, Sleep disturbance

Brief psychotic disorder มีอาการโรคจิตเกิดขึ้นอย่างรวดเร็ว มักเป็นหลังจากประสบเหตุการณ์กดดันรุนแรง


ผู้ป่วยจะมีอาการพูดจาสับสน พบบ่อยว่ามีประสาทหลอน ระยะเวลาดาเนินโรคอย่างน้อย 1 วัน ไม่เกิด 1 เดือน
เมื่อหายจะกลับเป็นปกติ
7. ชายอายุ 22 ปี อาชีพขับรถบรรทุก ตารวจพามาส่ง ER เนื่องจากพบว่าผู้ป่วยโวยวายอยู่กลางถนน เอะอะโวยวาย บอกว่า
ตนเองโดนกลั่นแกล้ง ได้คิวขับรถไม่เท่ากับคนอื่นๆ ขณะตรวจพบว่า ผู้ป่วยมีลักษณะก้าวร้าว เอะอะโวยวาย ไม่ค่อยมีสมาธิ อยู่
ไม่นิ่ง ไม่สามารถจาแนกของ 3 อย่างภายใน 5 นาที คิดว่าจะนะเป็นผลมาจากสารใด
A. กัญชา D. โคเคน
B. ใบกระท่อม E. แอมเฟตามีน
C. เฮโรอีน
Ans. ข้อ E (paranoid type และจากประวัติ )
A. กัญชา
ฤทธิ์ของกัญชาต่อระบบประสาทส่วนกลาง กระตุ้นประสาททาให้ผู้เสพตื่นเต้น ร่าเริง ช่างพูด หัวเราะตลอดเวลา
ต่อมาจะกดประสาทซึ่งผู้เสพมีอาการคล้ายเมาเหล้าอย่างอ่อนๆ มีอาการง่วงนอน เซื่องซึม และเมื่อเสพมากจะ
หลอนประสาท อาจเห็นภาพลวงตา หูแว่ว หรือมีอาการหวาดระแวง ความคิดสับสนควบคุมไม่ได้ บางรายอาจ
ไม่รู้จักตนเอง หรือไม่เข้าใจสิ่งต่างๆ รอบตัว
อาการทางจิตประสาทที่พบได้บ่อย ได้แก่ ความจาแย่ลง สมาธิน้อยลง มีปัญหาในการตัดสินใจ และบางคนอาจ
มีปัญหาการทรงตัว

B. ใบกระท่อม (mitragynine) การออกฤทธิ์


- การกินใบกระท่อมทาให้อารมณ์ดี คึกคัก ทางานกลางแดดได้ดี แต่จะกลัวเวลาครึ้มฟ้าครึ้มฝนและมีอาการ
หนาวสั่น
- การกินเป็นระยะเวลานานๆ จะทาให้หน้าสีดาคล้า โดยเฉพาะโหนกแก้ม
- อาการเสพติดจะทาให้หงุดหงิดปวดเมื่อยตามกล้ามเนื้ออย่างรุนแรง อ่อนเพลีย
- คนที่ไม่เคยเสพมาก่อนจะมีอาการมึนงง วิงเวียนศีรษะ คอแห้งคลื่นไส้และอาเจียน
C. เฮโรอีน การออกฤทธิ์
ออกฤทธิ์รุนแรงในทางกดประสาท ได้แก่ ศูนย์ประสาทส่วนการหายใจ สมองส่วนหน้า กดประสาทส่วนไขสันหลัง
และออกฤทธิ์แสดงปฏิกิริยาต่อระบบประสาทส่วนต่างๆของร่างกาย เช่น ระบบทางเดินอาหาร ระบบขับถ่าย
ปัสสาวะ และระบบการไหลเวียนของโลหิตในร่างกาย
ผู้เสพเฮโรอีนมักจะมีอาการปวดกล้ามเนื้อ ปวดกระดูก ปวดตามข้อ ปวดสันหลัง ปวดบั้นเอว และปวดหัวอย่าง
รุนแรง มีอาการจุกแน่นในอก คล้ายใจจะขาด อ่อนเพลียอย่างหนัก หมดเรี่ยวแรง มีอาการหนาวๆ ร้อนๆ อึดอัด
ทุรนทุราย นอนไม่หลับ กระสับกระส่าย บางรายมีอาหารชัก ตาตั้ง น้าลายฟูมปาก ม่านตาดาหดเล็กลง ใจคอ
หงุดหงิด ฟุ้งซ่าน มึนงง หายใจไม่ออก ประสาทเสื่อม และความจาเสื่อม
ผู้ที่ไม่เคยเสพเฮโรอีน เมื่อเสพเข้าไปจึงอาจติดได้ง่ายกว่าฝิ่นหรือมอร์ฟีน เพราะมีฤทธิ์เข้มข้นรุนแรงกว่ามากมาย
ถ้าเสพเข้าไปมากเกินกาลังความต้านทานของร่างกายก็อาจทาให้หัวใจหยุดเต้นถึงตายได้

D. โคเคน มักพบการใช้ในผู้มีฐานะดี การออกฤทธิ์


เป็นยาเสพติดที่ออกฤทธิ์กระตุ้นประสาท โดยจะกระตุ้นการทางานของระบบประสาทส่วนกลาง ดังนี้
1. กระตุ้นประสาทอย่างแรง ทาให้อารมณ์ทางจิตใจครึกครื้น มีอาการตื่นเต้นหวาดกลัว มือไม้สั่น
2. ทาให้หัวใจเต้นเร็ว ความดันเลือดสูง และม่านตาขยาย ถ้าเสพมากเกินขนาดทาให้ผู้เสพเสียชีวิตเพราะหัวใจ
หยุดเต้น
3. การใช้โคเคนเรื้อรังทาให้มีอาการคลื่นเหียน นอนไม่หลับ การย่อยอาหารผิดปกติ น้าหนักตัวลด อาจถึงขั้น
ประสาทหลอน มีอารมณ์หลงผิดแบบหวาดระแวงเห็นคนอื่นเป็นศัตรู มีจิตใจปั้นป่วน เกิดความหุนหันพลันแล่น
กลายเป็นคนดุร้าย ก่อนเหตุรุนแรงที่มีลักษณะเป็นการต่อต้าน

E. แอมเฟตามีน
อาการที่มักจะตรวจพบ ได้แก่ เบื่ออาหาร ตื่นเต้นง่าย อยู่ไม่สุข มือสั่น ตัวสั่น เหงื่อออกมาก คลื่นไส้อาเจียน ผะ
อืดผะอมได้ ความดันโลหิตสูง หัวใจเต้นเร็วและแรง อยู่ได้นานโดยไม่ต้องนอน ท้องเสียหรือท้องผูก ปาก และ
จมูกแห้ง ริมฝีปากแตก ทางานเกินปกติ หงุดหงิด ชอบทะเลาะวิวาท รูม่านตาเบิกกว้าง สูบบุหรี่จัด มวนต่อมวน
ผลทางด้านจิตใจจะเห็นได้ชัดเมื่อเสพเป็นจานวนมาก จะเกิดอาการทางจิตเฉียบพลัน หรือเป็นบ้าขึ้นได้ชั่วระยะ
หนึ่ง อาการจะคล้ายผู้ป่วยโรคจิตหวาดระแวงเกิดอาการหลงผิด คิดว่ามีคนมาทาร้ายตนเอะอะคว้าอาวุธมา
ป้องกันตัวเอง หรือพยายามจะหนีซุกซ่อนตัวเอง พูดไม่รู้เรื่อง มักเห็นภาพหลอน ต่างๆนานา ซึ่งนาไปสู่อันตราย
ต่อตัวเองหรือผู้อื่น เช่นตกใจกลัวปีนตึกหรือเสา ถูกรถชน หรือหลงผิดว่า มีคนมาทาร้าย จึงทาร้ายผู้อื่นก่อน
8. ข้อในไม่ใช่สิ่งที่เกิดขึ้นใน REM sleep
A. Dream
B. Penile erection in male
C. Decrease respiratory rate
D. Change in body temperature
E. Near-total paralysis in skeleton muscle
Ans. ข้อ E
REM atonia, a state in which the motor neurons are not stimulated and thus the body's muscles don't move.
Heart rate and breathing rate are irregular during REM sleep.
Body temperature is not well regulated during REM.
Erections of the penis (Nocturnal Penile Tumescence or NPT)
Clitoral enlargement, with accompanying vaginal blood flow and transudation (i.e. lubrication)
Dreams are strongly associated REM sleep (Always dream)

9. ผู้ป่วยหญิงไทยอายุ 18 ปี มีความวิตกกังวลเรื่องน้าหนักเกินอย่างมาก พยายามอาเจียนเอาอาหารออกมา หลังจากกิน


อาหาร ร่วมกับกินยาระบายและออกกาลังกายอย่างหนัก แต่พบว่าน้าหนักอยู่ในเกณฑ์ปกติ จะให้การรักษาอย่างไร
A. Fluoxitine
B. Paroxetine
C. Lithium
D. Imipramine
E. Amitrytyline

Ans. ข้อ A.
ข้อนี้เป็นโรคในกลุ่มของ eating-disorder (Anorexia & Bulimia nervosa) ยาที่แนะนาให้ใช้ได้แก่ antidepressants
กลุ่ม SSRIs สาหรับผู้ป่วยที่ยังมี depression คงอยู่นานแม้ว่าน้าหนักขึ้นแล้ว หรือยังไม่ขึ้นก็ตาม , มีรายงานว่า
fluoxetine สามารถเพิ่มและคงน้าหนักไว้ได้ในผู้ป่วยบางราย แม้ว่า fluoxetine ขนาดสูง(มากกว่า 60 mg/day) จะมีผลต่อ
appetite และทาให้น้าหนักลด แต่ไม่พบผลข้างเคียงนี้ในผู้ป่วย anorexia nervosa ถ้าได้รับยาขนาดต่าๆ การใช้
antidepressants ในผู้ป่วย anorexia nerrosa มีข้อระวังคือ จะมีผลข้างเคียงมากกว่า และตอบสนองต่อยาน้อยกว่าผู้ป่วย
อื่นๆ และเพิ่มโอกาสเสี่ยง ในการเกิด hypotension และ arrhythmia เพิ่มขึ้น อาการ depression ในผู้ป่วยมักดีขึ้นเอง
หลังจากน้าหนักเพิ่มขึ้น (Paroxetine ห้ามใช้ในเด็ก และ อายุน้อยกว่า 18 ปี)
10. ชาย 40 ปี ดื่มเหล้าเป็นประจา มา admit ด้วยเลือดออกในกระเพาะอาหาร ขณะรักษาเลือดหยุดแล้ว เริ่มมีอาการอยู่ไม่นิ่ง
กระวนกระวาย เห็นคนจะมาทาร้าย BP 150/110 HR 80 จงให้การรักษา
A. IM Haloperidol
B. IV Diazepam
C. IM Clopixel adepine
D. IM Clopomazine
E. IM Thiamine
--------------------------------------------------------------------------------------------------------------------------------------

Impression: Alcohol withdrawal

Clinical presentation
Classic signs
 Tremor: 6-8 hr
 Delusion & hallucination: 8-12 hr
 Seizure: generalized tonic clonic เท่านั้น
 Delirium tremens: 72 hr
 Sympathetic autonomic hyperactivity: วิตกกังวล, tachycardia, เหงื่อออก, หน้าแดง, ความดันโลหิตสูงเล็กน้อย,
mydriasis, alert

Treatment
Benzodiazepines ทุกตัวมีประสิทธิภาพในการลด signs & symptoms เหมือนกัน แต่ long acting เช่น Diazepam,
Chlordiazepoxide มีประสิทธิภาพมากกว่าในการป้องกัน seizure แต่ถ้ามี liver disease จะใช้ short acting เช่น Lorozepam
Antipsychotic drugs: Haloperidol ใช้ในผู้ป่วยที่มี agitation มาก
Thiamine: ลดการเกิด Wernike’s encephalopathy และ Korsakoff’s syndrome
Folic acid
Electrolytes

เฉลย B. IV Diazepam
11. ชาย 50 ปี หลังแพทย์แจ้งว่าเป็นมะเร็งของระบบทางเดินน้าดี ผู้ป่วยนิ่งอึ้งไป จากนั้นผู้ป่วยปฏิเสธไม่เชื่อและบอกว่าแพทย์ไม่
เก่ง ขอเปลี่ยนไปตรวจที่โรงพยาบาลอื่น ผู้ป่วยอยู่ในการตอบสนองต่อข่าวร้ายระยะใด
A. Denial
B. Depair
C. Depression
D. Bargaining
E. Anger
----------------------------------------------------------------------------------------------------------------------------- ---------
เฉลย Denial เห็นๆ
Defense mechanism
Level 1 Defense Mechanisms
The mechanisms on this level, when predominating, almost always are severely pathological. These three defences,
in conjunction, permit one to effectively rearrange external reality and eliminate the need to cope with reality. The
pathological users of these mechanisms frequently appear crazy or insane to others. These are the "psychotic"
defences, common in overt psychosis. However, they are found in dreams and throughout childhood as healthy
mechanisms.
They include:
Denial: Refusal to accept external reality because it is too threatening; arguing against an anxiety-provoking stimulus
by stating it doesn't exist; resolution of emotional conflict and reduction of anxiety by refusing to perceive or
consciously acknowledge the more unpleasant aspects of external reality.
Distortion: A gross reshaping of external reality to meet internal needs.
Delusional Projection: Grossly frank delusions about external reality, usually of a persecutory nature.
Level 2 Defence Mechanisms
These mechanisms are often present in adults and more commonly present in adolescence. These mechanisms
lessen distress and anxiety provoked by threatening people or by uncomfortable reality. People who excessively use
such defences are seen as socially undesirable in that they are immature, difficult to deal with and seriously out of
touch with reality. These are the so-called "immature" defences and overuse almost always lead to serious problems
in a person's ability to cope effectively. These defences are often seen in severe depression and personality
disorders. In adolescence, the occurrence of all of these defences is normal.
These include:
Fantasy: Tendency to retreat into fantasy in order to resolve inner and outer conflicts.
Projection: Projection is a primitive form of paranoia. Projection also reduces anxiety by allowing the expression of the
undesirable impulses or desires without becoming consciously aware of them; attributing one's own unacknowledged
unacceptable/unwanted thoughts and emotions to another; includes severe prejudice, severe jealousy,
hypervigilance to external danger, and "injustice collecting". It is shifting one's unacceptable thoughts, feelings and
impulses within oneself onto someone else, such that those same thoughts, feelings, beliefs and motivations are
perceived as being possessed by the other.
Hypochondriasis: The transformation of negative feelings towards others into negative feelings toward self, pain,
illness, and anxiety.
Passive aggression: Aggression towards others expressed indirectly or passively.
Acting out: Direct expression of an unconscious wish or impulse without conscious awareness of the emotion that
drives that expressive behavior.
Idealization: Subconsciously choosing to perceive another individual as having more positive qualities than he or she
may actually have.[2]
Level 3 Defence Mechanisms
These mechanisms are considered neurotic, but fairly common in adults. Such defences have short-term advantages
in coping, but can often cause long-term problems in relationships, work and in enjoying life when used as one's
primary style of coping with the world.
These include:
Displacement: Defense mechanism that shifts sexual or aggressive impulses to a more acceptable or less
threatening target; redirecting emotion to a safer outlet; separation of emotion from its real object and redirection of
the intense emotion toward someone or something that is less offensive or threatening in order to avoid dealing
directly with what is frightening or threatening. For example, a mother may yell at her child because she is angry with
her husband.
Dissociation: Temporary drastic modification of one's personal identity or character to avoid emotional distress;
separation or postponement of a feeling that normally would accompany a situation or thought.
Isolation: Separation of feelings from ideas and events, for example, describing a murder with graphic details with no
emotional response.
Intellectualization: A form of isolation; concentrating on the intellectual components of a situation so as to distance
oneself from the associated anxiety-provoking emotions; separation of emotion from ideas; thinking about wishes in
formal, affectively bland terms and not acting on them; avoiding unacceptable emotions by focusing on the
intellectual aspects (e.g. Isolation, Rationalization, Ritual, Undoing, Compensation, Magical thinking).
Reaction Formation: Converting unconscious wishes or impulses that are perceived to be dangerous into their
opposites; behavior that is completely the opposite of what one really wants or feels; taking the opposite belief
because the true belief causes anxiety. This defence can work effectively for coping in the short term, but will
eventually break down.
Repression: Process of pulling thoughts into the unconscious and preventing painful or dangerous thoughts from
entering consciousness; seemingly unexplainable naivety, memory lapse or lack of awareness of one's own situation
and condition; the emotion is conscious, but the idea behind it is absent.
Regression: Temporary reversion of the ego to an earlier stage of development rather than handling unacceptable
impulses in a more adult way.
Level 4 Defence Mechanisms
These are commonly found among emotionally healthy adults and are considered the most mature, even though
many have their origins in the immature level. However, these have been adapted through the years so as to optimize
success in life and relationships. The use of these defences enhances user pleasure and feelings of mastery. These
defences help the users to integrate conflicting emotions and thoughts while still remaining effective. Persons who
use these mechanisms are viewed as having virtues.
These include:
Altruism: Constructive service to others that brings pleasure and personal satisfaction
Anticipation: Realistic planning for future discomfort
Humor: Overt expression of ideas and feelings (especially those that are unpleasant to focus on or too terrible to talk
about) that gives pleasure to others. Humor, which explores the absurdity inherent in any event, enables someone to
call a spade a spade, while "wit" is a form of displacement (see above under Category 3). Wit refers to the serious or
distressing in a humorous way, rather than disarming it; the thoughts remain distressing, but they are 'skirted round'
by the witticism.
Identification: The unconscious modeling of one's self upon another person's character and behavior
Introjection: Identifying with some idea or object so deeply that it becomes a part of that person
Sublimation: Transformation of negative emotions or instincts into positive actions, behavior, or emotion
Suppression: The conscious process of pushing thoughts into the preconscious; the conscious decision to delay
paying attention to an emotion or need in order to cope with the present reality; able to later access uncomfortable or
distressing emotions and accept them

Oh !!!!!!
NO
12. ชายอายุ 40 ปี วินิจฉัย Schizophrenia รักษาด้วยยา อาการดีขึ้น 1 สัปดาห์ ที่ผ่านมารู้สึกใจสั่น ตรวจพบ PR 120 /min,
other: WNL, EKG: sinus tachycardia อาการในผู้ป่วยรายนี้เป็นผลจากยาในข้อใด
A. Haloperidol
B. Thioridazine
C. Fluphennazine
D. Perphenazine
E. ?
----------------------------------------------------------------------------------------------------------------------------- ---------

Haloperidol
CNS: seizure, extrapyramidal reaction, confusion, drowsiness, restlessness, tardive dyskinesia
EENT: blurred vision, dry eyes
Respiratory: respiratory depression
CVS: hypotension, tachycardia
GI: constipation, drymouth, anorexia, drug-induced hepatitis, ileus, weight gain
GU: urinary retention
Derm: diaphoresis, photosensitivity, reshes
Endo: galactorrhea, amenorrhea
Hemat: anemia, leukopenia
Metab: hyperpyrexia
Misc: neuroleptic malignant syndrome, hypersensitivity reaction
Thioridazine
CNS: neuroleptic malignant syndrome, sedation, extrapyramidal reaction, tardive dyskinesia
EENT: blurred vision, dry eyes, lens opacities, pigmentary retinopathy (high dose)
CVS: arrhythmia, qtC prolongation, hypotension, tachycardia
GI: constipation, drymouth, anorexia, drug-induced hepatitis, ileus, weight gain
GU: urinary retention, priaprism
Derm: , photosensitivity, pigment change, rashes
Endo: galactorrhea, amenorrhea
Hemat: agranulocytosis, leucopenia
Metab: hyperthermia
Misc: allergic reactions
Fluphennazine
CNS: neuroleptic malignant syndrome, sedation, extrapyramidal reaction, tardive dyskinesia
EENT: blurred vision, dry eyes
CVS: hypertension, hypotension, tachycardia
GI: anorexia, constipation, drymouth, drug-induced hepatitis, ileus, nausea, weight gain
GU: urinary retention
Derm: , photosensitivity, pigment change, rashes
Endo: galactorrhea
Hemat: agranulocytosis, leukopenia, thrombocytopenia
Misc:allergic reactions
Perphenazine
CNS: neuroleptic malignant syndrome, sedation, extrapyramidal reaction, tardive dyskinesia
EENT: blurred vision, dry eyes, lens opacities
CVS: hypotension, tachycardia
GI: constipation, drymouth, anorexia, ileus, weight gain
GU: discoloration of urine, urinary retention
Derm: photosensitivity, pigment change, rashes
Endo: galactorrhea, amenorrhea
Hemat: agranulocytosis, leukopenia
Metab: hyperthermia
Misc:allergic reactions
เฉลย อารายดีอ่า

13. ผู้ป่วย Schizophrenia ได้ยา Haloperidal 20 mg/d , Trihexyphenidyl 6 mg/d อาการหูแว่วดีขึ้น แต่ผู้ป่วยไม่อยากกินยาต่อ
เพราะมีอาการกระวนกระวาน ผุดลุกผุดนั้ง ท่านจะทาอย่างไรต่อไป
A. ลดยา Haloperidal
B. เปลี่ยน Haloperidal เป็น.................
C. หยุดยาทั้งหมดแล้วทา Psychotherapy
D. เปลี่ยนจากยากินเป็นยาฉีดแบบ long acting
E. ?
เฉลย A (ไม่แน่ใจข้อ A กับ B)
คนไข้มีอาการผุดลุกผุดนั้ง เป็นอาการของ extrapyramidal system(EPS) ซึ่งเป็น side effect จาก Haloperidal การให้
Trihexyphenidyl(Artane) ก็ช่วยเรื่อง ลด EPS
โดย EPS แบ่งได้หลายรูปแบบ คนนี้เป็นแบบ Akathisia (ผุดลุกผุดนั้ง กระวนกระวาย) ซึ่งอาการ Akatisia นี้จะ
ตอบสนองต่อ propranolol และ diazepam (ไม่มี choice)
คนไข้ Schizophrenia เริ่มการรักษาโดยใช้ Haloperidal 6-10 mg/d หลังจากนั้น 2 wk ถ้าอาการไม่ดีขึ้นจึงพิจารณา
เพิ่มยาโดยจะให้ไม่เกิน 20 mg/d (คนนี้น่าจะได้ยามากเกินไป)
การเปลี่ยนยาไปใช้ยาในกลุ่ม serotonin – dopamine antagonist ซึ่งมี EPS น้อยกว่า ก็น่าจะถูก แต่ควรลดขนาดยา
ก่อนมั้ง (ไม่แน่ใจนะเพื่อนๆ ถ้าผิดก็ขอโทษด้วย)
ยาฉีดนี่ใช้ในกลุ่มคนไข้ที่ไม่ร่วมมือในการกินยา
14. หญิงวัยกลางคนมีอาการท้อแท้ คิดช้า ไม่มีสมาธิ เบื่ออาหาร น้าหนักลดมา 3 ปี จงให้การวินิจฉัย
A. Schizophrenia
B. Bipolar disorder
C. Anxiety disorder
D. Depressive disorder
E.?
เฉลย D
อาการเข้าได้ กับ Depressive (อาการท้อแท้ คง คล้ายๆกับ ซึมเศร้าหดหู่มั้ง)

15. ชายอายุ 18 ปี กลัวการไปเที่ยวนอกบ้าน เพราะกลัวตนและครอบครัวเป็นอันตราย ก้าวเท้าขวาออกจากประตูทุกครั้งเพราะ


กลัวเกิดเหตุร้าย(โจทย์แปลกๆว่ะ) ชอบคิดถึงสิ่งที่ไม่ดี จงให้การรักษา
A. Buspirone
B. Imipramine
C. Chlorpromazine
D. Clomipramine
E.?
เฉลย B.
คนนี้น่าจะเป็น Agoraphobia without history of panic disorder
การรักษา ปัจจุบันนิยมให้ SSRI เป็นยาขนานแรก โดยให้ Fluoxetine (ไม่มีในchioce)
ยาแก้ซึมเศร้า กลุ่ม TCA ที่นิยมให้คือ Imipramine
Buspirone เป็นยากลุ่ม Antianxiety
Chlorpromazine เป็นยากลุ่ม DA (Antipsychotic drug)
Clomipramine เป็นกลุ่ม TCA (Antidepressant

16. ผู้ป่วยหญิง 35 ปี มาด้วยใจสั่น เจ็บหน้าอก หายใจไม่ออก กลัวตาย เป็นมา 2 เดือน กลัวมาก ไปหาหมอให้
คาแนะนา แล้วให้ยาอะไร
A. Lorazepam B. Midazolam C. Tenazepam D. Chlordiazepoxide E. ???
จาก Clinical ผู้ป่วยมีอาการ panic attack
Criteria : มีอาการในหัวข้อต่อไปนี้ ตั้งแต่ 4 อาการขึ้นไป อาการเกิดขึ้นอย่างรวดเร็ว และถึงระดับสูงสุดในระยะเวลา 10 นาที
1. ใจสั่น ใจเต้นแรง หรือหัวใจเต้นเร็วมาก 2. เหงื่อแตก
3. ตัวสั่น 4. หายใจไม่อิ่ม หรือหายใจขัด
5. รู้สึกอึดอัด หรือแน่นอยู่ข้างใน 6. เจ็บหน้าอก หรือแน่นหน้าอก
7. คลื่นไส้ ท้องไส้ปั่นป่วน 8. มึนงง วิงเวียน ปวดหัว หรือเป็นลม
9. Derealization หรือ depersonalization 10. กลัวคุมตัวเองไม่ได้ หรือกลัวเป็นบ้า
11. กลัวว่าตนเองกาลังจะตาย
Treatment : durg + cognitive behavioral therapy
- First line drug : SSRI นิยมใช้ Fluoxetine : start 10 mg/d เพิ่มได้ถึง 20-40 mg/d oral pc เช้า
- TCA นิยมใช้ Imipramine : start 25 mgx1 wk แล้วค่อยๆเพิ่ม dose 25 mg/wk โดยทั่วไปให้ ~ 50-75 mg
- Benzodiazepine นิยมใช้ Alprazolam 2-4 mg/d
*** ในทางปฏิบัติ ส่วนใหญ่จะให้ antidepressant + benzodiazepine X 4-6 wk ต่อมาเมื่อคุมอาการได้ดี และ antidepressant
ออกฤทธิ์เต็มที่แล้ว จึงค่อยๆลด dose ของ benzodiazepine ลง เหลือ antidepressant อย่างเดียวไว้ควบคุมอาการ
ข้อนี้ไม่ตรงซักแม่ง choice เลย สันนิษฐานว่า ยา 3 ตัว (Fluoxetine, Imipramine, Alprazolam) ตัวใดตัวหนึ่ง น่าจะเป็นข้อ E.

17. ชายอายุ 25 ปี กินเหล้า 2 ปี มี SI กับเกย์เป็นประจา 6 เดือนก่อนคิดว่าตัวเองเป็น HIV เลยไปตรวจผล พบผล


negative 3 ครั้ง ไม่เชื่อผลที่หมอบอก 3 เดือนก่อนได้ยินเสียงคนบอกว่าตัวเองเป็นเกย์ จงวินิจฉัยตาม DSM IV
A. Conversion disorder B. Chronic schizophrenia C. Schizotypal personality disorder
D. Delusional disorder : somatic type E.?
อาการของผู้ป่วย คือ มี non-bizarre delusion มา 6 เดือน และมี auditory hallucination มา 3 เดือน
ซึ่งโรคที่นึกถึงได้ คือ Delusional disorder กับ Schizophrenia มี criteria ดังนี้

Schizophrenia Delusional disorder


A. มีอาการตั้งแต่ 2 อาการขึ้นไป นาน 1 เดือน A. non-bizarre delusion นานอย่างน้อย 1 เดือน
- delusion B. ไม่เข้ากับเกณฑ์การวินิจฉัยข้อ A. ของ schizophrenia
- hallucination C. กิจวัตรโดยทั่วไปไม่เสื่อมจากปกติมากนัก และพฤติกรรม
- disorganized speech ไม่เป็นที่เห็นได้ชัดว่าแปลกหรือพิกล
- grossly disorganized behavior D. หากมี mood episode ขณะที่มีอาการหลงผิด ระยะเวลา
- negative symptom : flat affect, alogia, avolition โดยรวมจะต้องสั้นเมื่อเทียบกับระยะเวลาที่มีอาการหลงผิด
B. มีความเสื่อมหรือปัญหาด้าน social/occupation function E. อาการมิได้เป็นผลโดยตรงด้ารสรีรวิทยาจากสาร หรือจาก
C. อาการต่อเนื่องกันนาน 6 เดือนขึ้นไป โดยต้องมี active ภาวะความเจ็บป่วยทางกาย
phase อย่างน้อย 1 เดือน และระยะที่เหลืออาจเป็น
prodromal หรือ residual phase
D. อาการมิได้เป็นผลโดยตรงด้ารสรีรวิทยาจากสาร หรือจาก
ภาวะความเจ็บป่วยทางกาย

Delusional disorder ลักษณะของอาการหลงผิดจะไม่ประหลาดเหมือนใน schizophrenia เนื้อหาของความหลงผิดจะเกี่ยวโยง


กันเป็นเรื่องราว (systematized delusion) ไม่มีอาการหูแว่วเป็นเรื่องราวชัดเจน และโดยทั่วไป ผู้ป่วย schizophrenia จะมี
บุคลิกภาพและพฤติกรรมเสื่อมกว่า
Schizophrenia (paranoid type) อาจมีหลงผิดได้คล้ายกันกับ delusional disorder แต่ schizophrenia จะพบอาการอื่นๆร่วม
เช่น hallucination หรือ bizarre delusion นอกจากนี้เมื่อเทียบกันโดยรวมแล้ว ผู้ป่วย schizophrenia จะมีปัญหาความบกพร่อง
ในหน้าที่การงานหรือการเข้าสังคมมากกว่า
18. ผู้ป่วย emphysema ระยะสุดท้าย BP ดี , conscious ดี แต่ไม่พูดคุยกับญาติ หรือคนที่มาเยี่ยม
A. withdraw B. develop dementia C. develop personality disorder
ข้อนี้น่าจะเป็น develop personality disorder cluster A (ได้แก่ paranoid, schizoid, schizotypal)
คนนี้นึกถึง schizoid personality disorder มี criteria ดังนี้
A. ปลีกตัวจากการมีสัมพันธภาพทางสังคม และการแสดงออกของอารมณ์ขณะมีปฏิสัมพันธ์กับผู้อื่นนั้นมีน้อย รูปแบบดังกล่าว
เป็นไปแทบทุกเรื่อง เริ่มเป็นตั้งแต่วัยผู้ใหญ่ตอนต้นและพบได้ในภาวะแวดล้อมหลายๆรูปแบบ โดยมีลักษณะดังต่อไปนี้ตั้งแต่ 4 ข้อ
ขึ้นไป
1. ไม่ต้องการหรือไม่มีความเพลินใจกับการมีความสัมพันธ์ใกล้ชิดกับใคร รวมถึงการเป็นส่วนหนึ่งของครอบครัว
2. เลือกกิจกรรมที่ทาตามลาพังแทบจะทุกครั้ง
3. ความสนใจน้อยต่อการมีความสัมพันธ์ทางเพศกับผู้อื่น
4. มีความสุขกับกิจกรรมเพียงไม่กี่อย่าง
5. ขาดเพื่อนสนิทหรือคนที่ไว้วางใจได้ นอกเหนือไปจากญาติใกล้ชิด
6. เฉยชาต่อการชมเชยหรือการตาหนิของผู้อื่น
7. แสดงอารมณ์เย็นชา เหินห่าง หรือไร้อารมณ์
B. ลักษณะนี้มิได้เกิดขึ้นเฉพาะในช่วงของ schizophrenia, mood disorder with psychotic feature หรือ psychotic disorder
อื่นๆ และมิได้เป็นผลโดยตรงด้านสรีรวิทยาจากภาวะความเจ็ยป่วยทางกาย

19. หลังเหตุการณ์อุบัติเหตุรถชน พบว่าผู้ป่วยมีอาการเป็น PTSD ถามว่า อาการใดไม่ใช่อาการของ PTSD


A. จดจาเหตุการณ์ไม่ได้เลย B. นอนฝันร้าย C. ตื่นตระหนกเวลามีเสียงรถยนต์ D. ? E. ?
Criteria PTSD
1. ผู้ป่วยเคยเผชิญกับเหตุการณ์รุนแรงที่เป็นอันตรายต่อชีวิต หรือทาให้รู้สึกไม่ปลอดภัยต่อตัวเองและผู้อื่น
2. เหตุการณ์นั้นทาให้เกิดความกลัวที่รุนแรง หรือรู้สึกสิ้นหวัง
3. คิดคานึงถึงเหตุการณ์นั้นซ้าๆ (re-experienced) ทั้งขณะตื่น และหลับ (โดยการฝัน)
4. มีพฤติกรรมหลีกเลี่ยงเหตุการณ์นั้น หรือสิ่งที่เกี่ยวพันกับเหตุการณ์นั้น ( avoidance)
5. มีอาการตื่นเต้นตกใจง่าย ทั้งๆที่ไม่เคยเป็นมาก่อน (hyperarousal)
6. อาการรุนแรงจนเกิดปัญหากับหน้าที่การงาน ความสัมพันธ์ระหว่างบุคคล และชีวิตประจาวัน
7. ระยะของความผิดปกติ นานกว่า 1 เดือน (ถ้าน้อยกว่า 1 เดือน Dx. Acute stress disorder)
ข้อนี้ ตอบ A. จดจาเหตุการณ์ไม่ได้เลย
20. ผู้ชายต้มเหล้าเถื่อนกินมีอาการ ……(จาไม่ได้ )สาเหตุของอาการดังกล่าว
A. Methanal
B. Ethalene glycol
เฉลย:
อืมม..ไม่มีอาการมาให้ แต่รู้ว่ากินเหล้าเถื่อน แต่คาว่าเหล้าเถื่อนเราไม่รู้ว่าเค้าเป็นแบบไหน เพราะปัจจุบันมีตั้งแต่แค่
ส่าเหล้า(Alcohol) จนถึงใส่อะไรมากมายลงไปเช่นๆ
กรรมวิธีในการผลิตสุราเถื่อนที่มีการผสมหรือเติมวัตถุมีพิษลงไปนั้น เท่าที่ทราบว่านิยมทากันมีอยู่ 2 วิธี คือ
วิธีที่ 1 เติมยาฆ่าหญ้า โดยเฉพาะพวกพาราควอท หรือที่รู้จักกันดีในนามของ กรัมม๊อกโซน (Gramoxone) หรือ คิงโซน
(Kingzone) เป็นต้น ลงในส่าสุรา เพื่อให้ข้าวเหนียวและน้าตาลสลายตัวและเกิดแอลกอฮอล์เร็วขึ้น โดยจะใช้ระยะเวลาในการมัก
เพียง 4-5 วัน ทาให้ลดระยะเวลาในการมักส่าสุราได้ถึง 3-4 วัน
วิธีที่ 2 ใช้ยาฆ่าหญ้าหรือผงซักฟอกข้นๆ ทาก้นกะทะ ซึ่งใช้เป็นตัวทาให้เกิดการควบแน่นในช่วงการกลั่น โดยเชื่อกันว่าจะทา
ให้สุราเถื่อนที่กลั่นได้มีปริมาณมากขึ้น มีดีกรีสูง ใส มีกลิ่นฉุนและรสซ่าน่าดื่ม
นอกจากนี้ในปัจจุบันยังพบว่าอาจจะมีการผสมหรือเติมยาฆ่าแมลงบางชนิด เช่น โฟลิดอล (Folidol) หรือแลนแนท (Lannate)
เป็นต้น ลงในสุราเถื่อนที่ได้จากการต้มกลั่นแล้วอีกด้วย

สาหรับวัตถุมีพิษที่พบในสุราเถื่อนโดยทั่วไป แบ่งออกเป็น 2 ประเภท ดังนี้ คือ


ประเภทแรกเป็นวัตถุมีพิษที่เกิดขึ้นเองตามธรรมชาติในกระบวนการผลิตสุรา ซึ่งโดยปกติแล้ว ในการหมักธัญญพืช แล้ว
นามาต้มกลั่น นอกจากจะได้เอธิลแอลกอฮอล์ หรือสุราแล้วยังได้แอลกอฮอล์ชนิดอื่นและสารอื่นๆ อีกมากมายผสมออกมาด้วย
เช่น ฟูเซลออย (fusel oil), ไนโตรซามีน (nitrosamine), ไอโซเอมิลแอลกอฮอล์ (iso amyl alcohol), ไอโซบิวธิล แอลกอฮอล์ (iso
butyl alcohol) และเอ็นโปรปิล แอลกอฮอล์ (n-propyl alcohol) เป็นต้น ซึ่งสารต่างๆ เหล่านี้บางชนิดพบว่าเป็นสารเร่งให้เกิด
มะเร็ง (co-carcinogen) และเป็นสารก่อมะเร็ง (carcinogen) ในสัตว์ทดลองและในมนุษย์ ส่วนวัตถุมีพิษ
ประเภทที่สองที่อาจพบในสุราเถื่อนได้แก่ วัตถุมีพิษที่ผู้ผลิตจงใจผสมหรือเติมลงไปในกระบวนการผลิต โดยหวังแต่
ผลประโยชน์ มิได้ตระหนักถึงอันตรายที่จะเกิดแก่ผู้บริโภค เช่า ยาฆ่าหญ้า (กรัมม๊อกโซน หรือ พาราควอท) ยาฆ่าแมลง (โฟลิดอล,
แลนแนท) และผงซักฟอก
เพราะฉะนั้น ถ้าเรารู้อาการเราอาจคาดเดาได้ว่าอาจเป็นได้จาก paraquat poisoning ได้แต่จาก choice 2 ข้อนี้ เลิกเอาเน้อ
http://webdb.dmsc.moph.go.th/Hazardous/content1.asp?info_id=17
21. รับน้อง กินเหล้า แล้วซึม NS intact มารพ. จะให้อะไรอันดับแรก
A. B1 IV B. 50% glucose IV
เฉลย:
มาวิเคราะห์กันก่อน จากโจทย์ รับน้องแล้วกินเหล้า น่าสงสัยอาการของ Alcohol intoxication มากสุด
อาการแสดงมีแค่ซึม Neuro sign intact
การรักษา
1. จัดให้อยู่ในสิ่งแวดล้อมที่สงบ 4. ในกรณีที่ให้ glucose iv อย่าลืมให้ thiamine
2. อาการมักจะค่อยๆ สงบลงเอง 100 mg iv ด้วย
3. ถ้า agitate มากควรให้ diazepam 5-10 mg
iv แต่ระวังกาเสริมฤทธิ์กับ alcohol
เพราะฉะนั้นในกรณีของเรา เราคงไม่ให้ Valium เนื่องจากกลัวการเสริมฤทธิ์กับ alcohol นอกนั้นลองพิจารณาตาม choice ที่ไม่
ครบ ก็เลือกพิจารณาตามชอบแล้วกัน จิตเวชศาสตร์รามา หน้า 103
22. อาการคล้าย panic, เมื่อไม่นานมานี้เคยโดนจี้ จงให้การวินิจฉัย
A. Panic attack
B. Acute stress disorder
เฉลย:
จากข้อนี้ ทุกคนควรไป Review เรื่อง panic attack มาเน้อ แต่ตอนนี้เค้าบอกมาแล้ว่ามีอาการของ Panic attack ซึ่งเรา
ควร DDX คือ
Phobia แต่ผู้ป่วยจะมีอาการของการกลัวของที่ specific
Acute stress disorder/Posttraumatic stress disorder มีอาการคล้าย panic ได้แต่มักมีอาการหลังจากที่เผชิญ
เหตุการณ์รุนแรงและคุกคามต่อชีวิต (Acute stress disorder/Posttraumatic stress disorder แยกกันที่ 4 wk ได้ป่าว)
เพราะฉะนั้นข้อนี่ควรตอบ ข้อ..... B นะคร้าบบบบบ

จิตเวชศาสตร์รามา หน้า 165,177


23. ผู้ป่วยมีอาการ diaphoresis, miosis, drooling คิดว่าได้สารพิษใด
A. opioid
B. organophosphate

เฉลย:
อาการ Diaphoresis, miosis, drooling เข้าได้กับ orgnophosphate เพราะถ้าเป็น Opioid จะมีไม่มีอาการ drooling
ทบทวนเรื่องอาการของ organophosphate/carbamate poisioning
อาการของ muscarinic: SLUDGE
อาการของ Nicotinic: Miosis and Fasciculation
ส่วนอาการของ Opiate
nausea and vomiting, drowsiness, itching, dry mouth, miosis, and constipation
By Fiat
24. ผู้ป่วยเด็ก learning diaorder ที่อ่านสะกดคาไม่ได้
ตอบ Reading disorder หรือ Dyslexia
เฉลย : เป็นโรคที่มีประวัติพัฒนาการด้านการพูดล่าช้า เรียนรู้และจดจาตัวอักษรต่างๆได้ช้า โดยมักเริ่มแสดงอาการชัดเจนเมื่อเข้า
เรียนชั้นประถมศึกษาแล้ว คือ มีปัญหาในการสะกดคา ทาให้มีปัญหาการอ่านหนังสือและผลการเรียนไม่ดีตามมา
เพิ่มเติม Learning disorder จาแนกได้ 3 กลุ่ม
1. Reading disorder หรือ Dyslexia
2. Mathemathic disorder
3. Disorder of written expression (ผู้ป่วยมีปัญหาเรื่อง การสะกดคา การใช้ไวยกรณ์ และลายมือไม่ดี มักพบร่วมกับ
reading disorder)
25. ยาที่ใช้ในการรักษา ADHD คือยาอะไร
ตอบ Methylphenidate
เฉลย :โรค Attention deficit hyperactivity DisorderADHD เป็นโรคสมาธิสั้นมีอาการเด่น 3 ประการ คือ
1. Inattentiveness คือ สมาธิสั้นกว่าปกติ
2. Hyperactivity คือ อยู่ไม่นิ่ง อยู่ไม่สุข ซุกซนผิดปกติ
3. Impulsivity คือ หุนหันพลันแล่น ขาดการยับยั้งชั่งใจในการกระทาต่างๆ

การรักษาด้วยยา
Psychostimulant เป็นยาที่มีคุณสมบัติเฉพาะสาหรับอาการ ADHDโดยเฉพาะ
เนื่องจากมีประสิทธิภาพสูง ปลอดภัย ผลข้างเคียงน้อย และมีหลักฐานที่แน่ชัดว่าทาให้ผลการเรียนดีขึ้น ปัจจุบันยากลุ่มนี้ที่ใช้ใน
ประเทศไทย คือ methylphenidate ผลข้างเคียงของยาที่ต้องทราบคือ เบื่ออาหาร คลื่นไส้ ใจสั่น นอนไม่หลับ หงุดหงิด
น้าหนักลดหรือไม่เพิ่มขึ้นตามอายุ
26. ศพสงสัย amphetamine overdose จะส่งตรวจอะไรเพื่อ comfirm
ตอบ Whole Blood 100 ml (น่าจะตอบอันนี้มากสุดนะ) , urine 100 ml
เฉลย reference จากฉีดเรียนอาจารย์ เชาวกิจ นะ
Unknown case
สิ่งส่งตรวจต่างที่
Blood ( WB จากศพ 100 ml, Clotted blood จากผู้ป่วย 10 ml )
- ChE , paraquat , drugs , สารเสพติด (ศพ)

Blood (WB)
- Alcohol , toluene

Gastric content 100 ml


- Insecticides , herbicide , rodenticides , drugs

Urine 100 ml
- ChE , paraquat , drugs , สารเสพติด

Bile : opiates
Liver : opiates , drugs
Vitreous humor : alcohol
27. หญิงหม้ายอายุ 65 ปี หลงลืม ซึม ไม่ค่อยกินอะไร นอนไม่หลับ ควรได้รับการรักษาด้วยยาอะไร
ตอบ ไม่รู้ว่าเป็นโรคอะไรอ่ะ จะรักษาไงว่ะ choice ก็ไม่มี
แต่ผมว่าอาการเป็นได้ สองกลุ่ม พวก mood disorder หรือไม่ก็พวก neuro disorder (พวก Dementia นะ แต่ว่าไม่ค่อย
เหมือนนะ เหมือนแค่อายุมาก หลงลืม แต่ไม่รู้ onset และพวกนี้อาการเป็นมากกลางคืนนะ )
คิดว่าคงเป็นแนว mood disorder พวก MDD หลงลืม ซึม นอนไม่หลับนะ เป็นมาก 2 wk ก็ไม่มีบอกนะ รักษา ก็พวก
fluoxitine oral od,pc เช้า และหากนอนไม่หลับก็ให้ amitryptyline หรือ diazepam กินก่อนนอน

También podría gustarte